You are on page 1of 97

TOPNOTCH MEDICAL BOARD PREP PEDIATRICS SUPEREXAM

For inquiries visit www.topnotchboardprep.com.ph or email us at topnotchmedicalboardprep@gmail.com


DEAR TOPNOTCH FRIENDS:

PLEASE FOLLOW THESE INSTRUCTIONS:

1. These questions are previous diagnostic, midterm, and finals exams of Topnotch, almost all of them made by Topnotch Board Exam Topnotchers.
2. Answer this Topnotch Superexam seriously 100-items at a time. Cover the “Explanations” Column. Do not immediately look at the answers from the
answer key. That’s not the correct way of answering sample exams. You need to treat these MCQs as exercises and not as handouts.
3. Time yourself. 1.5 hours per 100-item block.
4. After answering each 100-item block, refer to the Topnotch Answer Key for the correct answers. Please be careful of “frameshift mutations” when
checking your answers – check every 10 items. (the format of the answer key was designed for you to practice against “frameshift mutations”)
5. The Topnotch Superexams are EXERCISES for the actual med boards. They will not appear verbatim in your future exams. More than knowing what’s
the correct answer, it’s more important for you to:
a. Know why the other choices are wrong
b. Know why the other choices were included in the first place
c. Know the explanation to the correct answer
6. Sharpen your mind by answering the Topnotch Superexams. Most of these questions based on past feedback are more difficult than the actual questions
in the med boards. In these exams made by Board Exam Topnotchers, if you’re getting a score of 60/100 , that’s already a good score. More than 80/100
is outstanding.

Item QUESTION EXPLANATION AUTHOR TOPNOTCH
# EXAM
1 Which phase of Kawasaki disease is associated with Kawasaki disease presents with characteristically KRISTEL TANHUI DIAGNOSTIC
coronary aneurysms? high, unremitting fever and 4 out of the 5 principal (TOP 3 - AUG EXAM -
A. Acute febrile phase features: 2015 MED MARCH 2016
B. Subacute phase Bilateral nonexudative bulbar conjunctival injection BOARDS;
C. Convalescent phase with limbal sparing; erythema of the oral and TOPNOTCH MD
D. Phase of complications pharyngeal mucosa with strawberry tongue and dry, FROM LA SALLE)
E. All of the above cracked lips; edema and erythema of the hands and
feet; rash of various forms (scarlatiniform,
maculopapular, erythema multiforme);
nonsuppurative cervical lymphadenopathy (usually
unilateral, with node size >1.5cm).

Acute febrile phase – fever and the acute signs of
illness and usually lasts 1-2wks
Subacute febrile phase – desquamation,
thrombocytosis, coronary aneurysms and highest risk
of sudden death usually lasts 2 wks
Convalescent phase – All clinical signs have
disappeared until ESR normalizes typically 6-8wks
after onset on illness

Source: Nelson’s Textbook of Pediatrics 19th edition
p864
2 A four year old presents with low-grade fever, HSP is a common vasculitis among children and KRISTEL TANHUI DIAGNOSTIC
intermittent crampy abdominal pain with emesis presents with the classic findings of abdominal pain (TOP 3 - AUG EXAM -
and swollen knees of 3 days duration. A purpuric with or without rectal bleeding, vasculitis rash, 2015 MED MARCH 2016
rash distributed below the knees of both lower arthritis and nephritis. The platelet count is normal. BOARDS;
extremities is noted on physical examination. This Gastrointestinal involvement may progress to TOPNOTCH MD
patient most likely has: intussusception. FROM LA SALLE)
A. Meningococcemia
B. Idiopathic thrombocytopenic purpura Source: Nelson’s Textbook of Pediatrics 19th edition
C. Henoch-Scholein purpura p868
D. SLE
E. Juvenile Rheumatoid arthritis
3 A 3 year old female presents with 1 week history of Don’t forget to study rheumatic diseases of childhood KRISTEL TANHUI DIAGNOSTIC
daily (“quotidian”) fever associated with arthritis of even if it’s not your favorite topic ☺. Familiarize (TOP 3 - AUG EXAM -
both ankles and her right knee. Her mother notes yourself with the relatively common ones like JRA, SLE 2015 MED MARCH 2016
that she has decreased activity and also notes and dermatomyositosis, HSP and scleroderma. Pay BOARDS;
rashes that come and go in different parts of her attention to the characteristics of the rash and other TOPNOTCH MD
body but not on the face. On physical examination, features. (Don’t fall for the trap of rash + arthritis = FROM LA SALLE)
salmon colored patches were noted on the trunk SLE.. It’s not always SLE!!)
and extremities. Palpation revealed splenomegaly. JRA – Salmon colored patch, evanescent, spares the
What is the diagnosis? face
A. Henoch Schoenlein Purpura SLE – Malar rash, photosensitivie
B. Systemic Lupus Erythematosus Dermatomyositis – heliotrope (periorbital rash),
C. Juvenile Rheumatoid arthritis gottron papules (rash on the knuckles)
D. Juvenile Dermatomyositis HSP – purpura in dependent portions
E. Dengue fever
For JRA, know the differences between subtypes
(oligoarticular vs polyarticular vs systemic onset)
This case in particular is a case of systemic onset
Juvenile Rheumatoid arthritis (also called Still’s
Disease)

Source: Nelson’s Textbook of Pediatrics 19th edition
p830

TOPNOTCH MEDICAL BOARD PREP PEDIATRICS SUPEREXAM Page 1 of 97


For inquiries visit www.topnotchboardprep.com.ph or email us at topnotchmedicalboardprep@gmail.com
TOPNOTCH MEDICAL BOARD PREP PEDIATRICS SUPEREXAM
For inquiries visit www.topnotchboardprep.com.ph or email us at topnotchmedicalboardprep@gmail.com
Item QUESTION EXPLANATION AUTHOR TOPNOTCH
# EXAM
4 A 4 yr old boy presents with his third episode of This is a case of Chronic granulomatous disease KRISTEL TANHUI DIAGNOSTIC
painful cervical lymphadenitis. Each was treated wherein patients are susceptible to catalase positive (TOP 3 - AUG EXAM -
with incision and drainage and grew S. aureus. He organisms like S. aureus. The nitroblue tetrazolium 2015 MED MARCH 2016
also experiences recurrent skin infections. A year tests the neutrophils ability to generate superoxide BOARDS;
ago, he was hospitalized for osteomyelitis. The anion and thus kill ingested bacteria. TOPNOTCH MD
most important laboratory test is: FROM LA SALLE)
A. PCR for ADA deficiency Source: Nelson’s Textbook of Pediatrics 19th edition
B. Nitroblue tetrazolium test p746
C. MAC-I assay
D. Neutrophil count
E. Genetic chromosomal analysis
5 Which among the following is not a prominent Note: The prominent immunologic impairment in KRISTEL TANHUI DIAGNOSTIC
feature of Wiskott-Aldrich syndrome? Wiskott Aldrich Syndrome is against polysaccharides (TOP 3 - AUG EXAM -
A. X-linked recessive inheritance of encapsulated organisms 2015 MED MARCH 2016
B. Atopic dermatitis BOARDS;
C. Thrombocytopenia Source: Nelson’s Textbook of Pediatrics 19th edition TOPNOTCH MD
D. Recurrent infections with encapsulated bacteria p734 FROM LA SALLE)
E. Leukopenia

6 A 12 year old seeks consult for 1 day history of Allergic rhinitis is often seasonal and associated with KRISTEL TANHUI DIAGNOSTIC
sneezing, clear rhinorrhea and nasal itching not allergic conjunctivitis. Eosinophils predominate in the (TOP 3 - AUG EXAM -
associated with fever or any other systemic nasal secretions. 2015 MED MARCH 2016
symptoms. PE reveals boggy, pale nasal edema with BOARDS;
a clear discharge. The most likely diagnosis is? It is classified as TOPNOTCH MD
A. Foreign body - Seasonal (cyclical) or perinneal (all year) FROM LA SALLE)
B. Vasomotor rhinitis - Intermittent (symptoms occur <4 days per week or
C. Allergic rhinitis <4 consecutive weeks) or persistent (symptoms occur
D. Neutrophilic rhinitis >4 days per week or >4 consecutive weeks)
E. Rhinitis medicamentosa - Mild-moderate (no impairment) or severe (with
impairment of daily living/sleep)

Source: Nelson’s Textbook of Pediatrics 19th edition
p775
7 The Kasai procedure is indicated for which of the Despite initial success of the Kasai operation, patients KRISTEL TANHUI DIAGNOSTIC
following? with biliary atresia eventually get liver (TOP 3 - AUG EXAM -
A. Neonatal hepatitis transplantation. 2015 MED MARCH 2016
B. Biliary atresia BOARDS;
C. Metabolic liver disease Source: Nelson’s Textbook of Pediatrics 19th edition TOPNOTCH MD
D. Fulminant hepatic failure 1387 FROM LA SALLE)
E. None of the above

8 An 8 month old manifests with fussiness and Aside from that, intussusception may present with KRISTEL TANHUI DIAGNOSTIC
emesis and refuses to eat. 1 week ago, he was lethargy out of proportion to the intestinal signs and (TOP 3 - AUG EXAM -
brought for consult and was diagnosed with URTI. symptoms. Intussusception in children is usually due 2015 MED MARCH 2016
On the morning of consult, his mother noted to lymphoid hyperplasia in the intestines. This BOARDS;
currant jelly like material on his diaper. On PE, a becomes the lead point. The most common location is TOPNOTCH MD
sausage shaped mass was palpable in the abdomen. ileocolic. FROM LA SALLE)
There is likewise abdominal distention and rectal
exam reveals blood in the stool. The most likely Source: Nelson’s Textbook of Pediatrics 19th edition
diagnosis is: p1288
A. Viral diarrhea
B. Duodenal atresia
C. Intussusception
D. Hypertrophic pyloric stenosis
E. Intestinal adenoma
9 A 3 year old female presents to your office with Source: Nelson’s Textbook of Pediatrics 19th edition p KRISTEL TANHUI DIAGNOSTIC
unilateral nasal discharge. The discharge is 1431 (TOP 3 - AUG EXAM -
described by the caretaker to be malodorous and 2015 MED MARCH 2016
lately has been blood tinged. The most likely BOARDS;
diagnosis is: TOPNOTCH MD
A. Tertiary syphilis FROM LA SALLE)
B. Unilateral choanal atresia
C. Nasopharyngeal carcinoma
D. Foreign body
E. Angiosarcoma of the nose
10 A 3 year old is brought to the ER. On PE, inspiratory This is a case of croup and is common in this age KRISTEL TANHUI DIAGNOSTIC
stridor and a barking cough were noted. There is group. (TOP 3 - AUG EXAM -
mild respiratory distress, tachypnea, and fever. The Barking cough = croup 2015 MED MARCH 2016
signs and symptoms are aggravated by agitation Epiglotitis and bacterial tracheitis patients are more BOARDS;
and crying. The most likely diagnosis is toxic appearing and not in merely mild respi distress TOPNOTCH MD
A. Trachiomalacia FROM LA SALLE)
B. Laryngotracheobronchitis Source: Nelson’s Textbook of Pediatrics 19th edition
C. Epiglotittis p1446
D. Bacterial tracheitis
E. Peritonsillar abscess

TOPNOTCH MEDICAL BOARD PREP PEDIATRICS SUPEREXAM Page 2 of 97


For inquiries visit www.topnotchboardprep.com.ph or email us at topnotchmedicalboardprep@gmail.com
TOPNOTCH MEDICAL BOARD PREP PEDIATRICS SUPEREXAM
For inquiries visit www.topnotchboardprep.com.ph or email us at topnotchmedicalboardprep@gmail.com
Item QUESTION EXPLANATION AUTHOR TOPNOTCH
# EXAM
11 A newborn was noted to be cyanotic. Unfortunately The hyperoxia test is used to differentiate whether the KRISTEL TANHUI DIAGNOSTIC
the imaging facilities are unavailable for the day. A etiology of cyanosis is cardiac or not. PaO2 does not (TOP 3 - AUG EXAM -
hyperoxia test was thus done which showed no improve in cardiac etiologies. 2015 MED MARCH 2016
improvement of PaO2 after administration of 100% In the absence of imaging for definitive diagnosis of BOARDS;
O2. The next step in the management of this the congenital heart disease, the clinician should not TOPNOTCH MD
neonate is: hesistate to administer prostaglandin in case the FROM LA SALLE)
A. Low tidal volume mechanical ventilation patient has a ductus dependent congenital heart
B. Positive pressure ventilation disease. Prostaglandin prevents the ductus from
C. Digoxin closing.
D. Emergency laparotomy
E. Prostaglandin E Source: Nelson’s Textbook of Pediatrics 19th edition
p1572
12 A patient was diagnosed with tetralogy of fallot. His The typical configuration on AP view consists of a KRISTEL TANHUI DIAGNOSTIC
chest xray reveals the classic “Le Couer en Sabot”. narrow base, concavity of the left heart border in the (TOP 3 - AUG EXAM -
Which of the following components of TOF is area usually occupied by the pulmonary artery and 2015 MED MARCH 2016
responsible for this shape? normal overall heart size. The hypertrophied right BOARDS;
A. Pulmonary stenosis ventricle causes the rounded apical shadow to be TOPNOTCH MD
B. VSD uptilted so that it is situated higher above the FROM LA SALLE)
C. RVH diaphragm than normal and pointing horizontally to
D. Overriding of the aorta the left of the chest wall. The cardiac shadow has been
E. Right sided aorta likened to that of a boot or a wooden shoe.

Source: Nelson’s Textbook of Pediatrics 19th edition
p1575
13 A 5 year old presents with multiple petichiae on her CBC with PC would reveal decreased platelets and KRISTEL TANHUI DIAGNOSTIC
lower extremities and oral-mucosal bleeding of 3 normal other parameters suggesting that this is a case (TOP 3 - AUG EXAM -
days duration. 2 weeks prior, she had a mild of ITP. 2015 MED MARCH 2016
respiratory tract infection but other than that, her BOARDS;
caretakers describe her to be active and generally Source: Nelson’s Textbook of Pediatrics 19th edition TOPNOTCH MD
fine. On PE, she is afebrile. No lymphadenopathy p1715 FROM LA SALLE)
nor hepatosplenomegaly is noted. Which is the best
next diagnostic step to confirm the diagnosis?
A. CBC with platelet count
B. PT
C. PTT
D. Bleeding time
E. Clotting time
14 An infant has a hemangioma that grows rapidly in McCune Albright syndrome – polyostotic fibrous KRISTEL TANHUI DIAGNOSTIC
size, leading to thrombocytopenia and dysplasia, café-au-lait skin pigmentation, autonomous (TOP 3 - AUG EXAM -
microangiopathic hemolytic anemia. Which term endocrine hyperfunction 2015 MED MARCH 2016
describes this condition? BOARDS;
A. Kaposi like form of infantile hemangioma Mafucci syndrome – Multiple enchondromas and TOPNOTCH MD
B. McCune Albright syndrome hemangiomas FROM LA SALLE)
C. Kasabach-Merritt syndrome
D. Maffucci syndrome Evan’s syndrome – autoimmune anemia +
E. Evan’s syndrome thrombocytopenia

Source: Nelson’s Textbook of Pediatrics 19th edition
p1719
15 A previously healthy 8 year old male presents with Rhabdomyosarcoma is the most common sarcoma of KRISTEL TANHUI DIAGNOSTIC
progressive painless proptosis and decreased childhood. Orbital rhabdomyosarcoma is a common (TOP 3 - AUG EXAM -
visual acuity of the left eye during the past 10 site for rhadomyosarcoma which produces local signs 2015 MED MARCH 2016
weeks. The most likely diagnosis is: as it grows and displaces normal tissues. BOARDS;
A. Orbital cellulitis TOPNOTCH MD
B. Trichinosis Retinoblastoma may be endophytic or exophytic from FROM LA SALLE)
C. Retinoblastoma the retina. They present with leukocoria or
D. Rhabdomyosarcoma strabismus.
E. None of the above
Source: Nelson’s Textbook of Pediatrics 19th edition
p1761, 1768
16 A 4 year old male experienced an upper respiratory Patient is presenting with nephrotic syndrome. The KRISTEL TANHUI DIAGNOSTIC
tract infection that was followed in 2 weeks by most common of which is minimal change disease in (TOP 3 - AUG EXAM -
generalized edema. His blood pressure is normal. children. 2015 MED MARCH 2016
Urinalysis reveals 2-5 rbc/hpf and 4+ protein. His BOARDS;
BUN is 19mg/dl. Serum albumin 0.9g/dl and C3 Hypoalbuminemia, proteinuria, edema and TOPNOTCH MD
was 93mg/dl. The most likey diagnosis is: hyperlipidemia constitute the nephrotic syndrome. FROM LA SALLE)
A. Post streptococcal glomerulonephritis
B. Membranous glomerulonephritis Source: Nelson’s Textbook of Pediatrics 19th edition
C. Minimal change disease p1804
D. Focal sclerosis
E. IgA nephropathy
17 The following listed phrases are characteristics of There is never a loss of consciousness in simple partial KRISTEL TANHUI DIAGNOSTIC
simple partial seizures EXCEPT: seizures. Some patients may actually be conversant (TOP 3 - AUG EXAM -
A. Loss of consciousness and talk to you during the event. 2015 MED MARCH 2016
B. Duration of 10-20 sec BOARDS;
C. Versive seizures Source: Nelson’s Textbook of Pediatrics 19th edition TOPNOTCH MD
D. May have secondary generalization p2021 FROM LA SALLE)
E. Abnormal EEG

TOPNOTCH MEDICAL BOARD PREP PEDIATRICS SUPEREXAM Page 3 of 97


For inquiries visit www.topnotchboardprep.com.ph or email us at topnotchmedicalboardprep@gmail.com
TOPNOTCH MEDICAL BOARD PREP PEDIATRICS SUPEREXAM
For inquiries visit www.topnotchboardprep.com.ph or email us at topnotchmedicalboardprep@gmail.com
Item QUESTION EXPLANATION AUTHOR TOPNOTCH
# EXAM
18 Pincer grasp is usually noted at what age? Pincer grasp/thumb finger grasp KRISTEL TANHUI DIAGNOSTIC
A. 2 mos (TOP 3 - AUG EXAM -
B. 4 mos Source: Nelson’s Textbook of Pediatrics 19th edition 2015 MED MARCH 2016
C. 6 mos p27 BOARDS;
D. 8 mos TOPNOTCH MD
E. 10 mos FROM LA SALLE)

19 A 12 year old male presents with complaints of The antibiotic treatment for eradication of KRISTEL TANHUI DIAGNOSTIC
polyarthritis and an evanescent rash which spares streptococcus can be either: (TOP 3 - AUG EXAM -
the face. On PE, patient appears to be in respiratory Penicillin VK x 10days (not just one week) 2015 MED MARCH 2016
distress and auscultation revealed tachycardia and Benzathine PCN 0.6-1.2 MU IM BOARDS;
a gallop rhythm, and some hard painless freely Erythromycin 250mg TID x 10 days TOPNOTCH MD
movable swellings over the extensor surfaces like FROM LA SALLE)
the elbows. Chest xray reveals cardiomegaly and Source: Topnotch handout in pediatrics
ASOT is 350 Todd units. The following are
appropriate in the management of this patient
except:
A. Penicillin V 200-500mg QID x 1 week
B. Complete bed rest
C. Morphine
D. Digoxin
E. 02 supplementation

20 A 5 year old presents with complaint of chronic Equal to or > 5 – positive if with history of close KRISTEL TANHUI DIAGNOSTIC
cough of 1 month duration with associated weight contact, with suggestive clinical findings, suggestive (TOP 3 - AUG EXAM -
loss and recurrent fever. PPD was done. Which of CXR, immunocompromised 2015 MED MARCH 2016
the following is true. Equal to or > 10 = positive BOARDS;
A. Equal to or > 5 is positive. Anything below this TOPNOTCH MD
value is negative. Source: Topnotch handouts in pediatrics FROM LA SALLE)
B. Equal to or > 10 is positive. Anything below this
value is negative.
C. Equal to or > 15 is positive. Anything below this
value is negative.
D. Equal to or > 20 is positive. Anything below this
value is negative.
E. Equal to or > 25 is positive. Anything below this
value is negative.
21 Which one of the following forms of juvenile Patients with oligoarticular arthritis without axial LESTER BRYAN MIDTERM 1
idiopathic arthritis is most likely to be associated spine involvement are most likely to develop chronic CO (TOP 10 - AUG EXAM -
with serious eye complications? and potentially severe anterior uveitis, which can be 2015 MED MARCH 2016
A. Polyarticular arthritis that is seropositive for clinically quite subtle even as it leads to progressive BOARDS;
rheumatoid factor visual loss. Up to 25% of patients in this subset may TOPNOTCH MD
B. Polyarticular arthritis that is seronegative for develop anterior uveitis, and the group that is ANA FROM UST)
rheumatoid factor positive appears to be at highest risk. Patients with
C. Oligoarticular arthritis without axial spine axial spine involvement can also develop anterior
involvement uveitis, but this tends to be acute, self-limited, and
D. Oligoarticular arthritis with axial spine easily treatable.
involvement
E. Systemic-onset juvenile rheumatoid arthritis
22 A 7-year-old boy presents with palpable purpura SIMILAR TO PREVIOUS BOARD EXAM LESTER BRYAN MIDTERM 1
on the buttocks and legs, fever, abdominal pain and CONCEPT/PRINCIPLE. The clinical description is that CO (TOP 10 - AUG EXAM -
vomiting, arthritis in his knees and ankles, melena, of Henoch-Schönlein purpura, a form of 2015 MED MARCH 2016
and hematuria. His mother states that he had an leukocytoclastic angiitis (hypersensitivity vasculitis) BOARDS;
upper respiratory illness approximately 1 week resulting from an immune reaction that damages the TOPNOTCH MD
ago, but has otherwise been well. Blood tests reveal vascular endothelium. Henoch-Schönlein purpura is FROM UST)
mild renal insufficiency. The most likely cause of closely related to IgA nephropathy, a glomerulopathy
the bleeding into the skin observed in this patient is resulting in nephritic syndrome, and may represent a
A. coagulation factor deficiency systemic version of this disease.
B. qualitative platelet dysfunction
C. quantitative platelet dysfunction
D. vasculitis
E. vitamin deficiency
23 A mother calls you frantic because she has just been SIMILAR TO PREVIOUS BOARD EXAM LESTER BRYAN MIDTERM 1
diagnosed with varicella (chicken pox). She CONCEPT/PRINCIPLE. Per CDC recommendations, CO (TOP 10 - AUG EXAM -
delivered a term infant 7 days ago that appears to varicella-zoster immunoglobulin (VZIG) should be 2015 MED MARCH 2016
be eating, stooling, and urinating without difficulty. administered to the infant immediately after delivery BOARDS;
The child has been afebrile and seems to be doing if the other had the onset of varicella within 5 days TOPNOTCH MD
well. Which of the following is the most appropriate prior to delivery, and immediately upon diagnosis if FROM UST)
step in management? her chicken pox started within 2 days after delivery. If
a. Isolate the infant from the mother. untreated, about half of these infants will develop
b. Hospitalize the infant in the isolation ward. serious varicella as early as 1 day of age. If a normal
c. Administer acyclovir to the infant. full-term newborn is exposed to chicken pox 2 or
d. Administer varicella-zoster immunoglobulin to more days postnatally, VZIG and isolation are not
the infant. necessary because these babies appear to be at no
e. Advise the mother to continue regular well-baby greater risk for complications than older children.
care for the infant. Acyclovir may be used in infants at risk for severe
varicella, such as those infants exposed perinatally.

TOPNOTCH MEDICAL BOARD PREP PEDIATRICS SUPEREXAM Page 4 of 97


For inquiries visit www.topnotchboardprep.com.ph or email us at topnotchmedicalboardprep@gmail.com
TOPNOTCH MEDICAL BOARD PREP PEDIATRICS SUPEREXAM
For inquiries visit www.topnotchboardprep.com.ph or email us at topnotchmedicalboardprep@gmail.com
Item QUESTION EXPLANATION AUTHOR TOPNOTCH
# EXAM
24 A 10-month-old baby boy has a 5-hour history of The usual presentation of intussusception is that of an LESTER BRYAN MIDTERM 1
intermittent crying, and with intermittent drawing infant between 4 and 10 months of age who has a CO (TOP 10 - AUG EXAM -
up of his knees to his chest in between crying sudden onset of intermittent colicky abdominal pain. 2015 MED MARCH 2016
episodes. On the way to the emergency room he The child can appear normal when the pain abates, BOARDS;
passes a loose, bloody stool. He has had no vomiting but as it recurs with increasing frequency, the child TOPNOTCH MD
and has refused his bottle since the crying began. can begin to vomit and become progressively more FROM UST)
Physical examination is noteworthy for an irritable obtunded. The passage of stool containing blood and
infant whose abdomen is very difficult to examine mucus, frequently described as resembling currant
because of constant crying. His temperature is jelly, is often observed. Early examination of the
38.8°C (101.8°F). The rectal ampulla is empty, but abdomen can be unremarkable, but as the problem
there is some gross blood on the examining finger. persists, a sausage-shaped mass in the right upper
Which of the following studies would be most quadrant is frequently palpated. An air, barium, or
helpful in the immediate management of this saline enema examination under fluoroscopic or
patient? ultrasound control can be therapeutic as well as
a. Stool culture diagnostic when the hydrostatic effects of the contrast
b. Examination of the stool for ova and parasites serve to reduce the intussusception, but should be
c. Air contrast enema performed with surgical backup, as a complication of
d. Examination of the blood smear attempted reduction is intestinal perforation. Rates of
e. Coagulation studies intestinal perforation are lowest with air reduction.
Early diagnosis prevents bowel ischemia. The cause of
most intussusceptions is unknown, but a Meckel
diverticulum or polyp can serve as a lead point. None
of the other choices would result in a correct diagnosis
(and potential therapy) for the child with a classic
presentation for intussusception.
25 A 13-year-old boy presents to the pediatrician with Wilson disease is a hereditary condition associated LESTER BRYAN MIDTERM 1
extrapyramidal signs, including a resting and with the accumulation of copper in the liver, brain, CO (TOP 10 - AUG EXAM -
kinetic tremor. An ophthalmologic examination and eye. Accumulation of copper in the Descemet 2015 MED MARCH 2016
demonstrates the presence of dark rings that membrane of the eye results in the pathognomonic BOARDS;
appear to encircle the iris of the eye, and his lesion known as the Kayser-Fleischer ring. TOPNOTCH MD
laboratory studies demonstrate elevated liver Accumulation in the liver results in cirrhosis. FROM UST)
enzymes. It is likely that the patient’s condition is Accumulation in the brain, specifically in the basal
associated with the accumulation of ganglia, results in motor symptoms. Councilman
A. copper bodies are apoptotic hepatocytes that were first
B. Councilman bodies identified in yellow fever. Eosinophilic hyaline
C. eosinophilic hyaline inclusions inclusions, Mallory bodies, are seen in alcoholic liver
D. glycogen disease. Glycogen accumulates in the liver in
E. iron numerous glycogen storage diseases. Iron
accumulates in hemochromatosis.
26 A 10-month-old infant on long-term aspirin therapy Reye syndrome is an acquired mitochondrial LESTER BRYAN MIDTERM 1
for Kawasaki disease develops sudden onset of high hepatopathy that results from the interaction of an CO (TOP 10 - AUG EXAM -
fever, chills, diarrhea, and irritability. A rapid swab influenza (or varicella) infection and aspirin use. 2015 MED MARCH 2016
in your office dentifies influenza A, adding her to While prevalence has decreased over the last few BOARDS;
the long list of influenza patients you have seen this decades and it is now a rare disease, mortality TOPNOTCH MD
December. Over the next few days, she slowly remains the same at more than 40% of cases. Liver FROM UST)
improves and becomes afebrile. However, 5 days enzymes and ammonia are elevated, but total bilirubin
after your last encounter you hear from the hospital is not. Patients initially present toward the end of a
that she has presented to the emergency center viral infection with sleepiness, emesis, and abnormal
obtunded and posturing with evidence of liver liver functions. As the disease progresses, the patient
dysfunction. Which of the following statements may develop seizures, coma, hyperventilation, and
about her current condition is correct? decorticate posturing. Ultimately they may develop
A. With proper supportive care, the overall respiratory arrest, loss of deep tendon reflexes
mortality rate is low. (DTRs), and fixed and dilated pupils. Death is usually
B. With her progressive liver dysfunction, increased from cerebral edema and subsequent herniation.
total serum bilirubin is anticipated. While aspirin is no longer routinely used in children
C. Administration of N-acetylcysteine is first-line as an antipyretic or pain reliever, the increase in the
therapy use of aspirin in adults with heart disease requires
D. Seizures are uncommon with this condition. specific counseling for parents of children with
E. Death is usually associated with increased influenza and varicella to avoid aspirin use. In
intracranial pressures and herniation. addition, both of these infections are preventable with
proper immunization. N-acetylcysteine is protective of
hepatocytes in acetaminophen overdose.
27 A 4-year-old boy in your practice has been having SIMILAR TO PREVIOUS BOARD EXAM LESTER BRYAN MIDTERM 1
many infections. You suspect that he may have an CONCEPT/PRINCIPLE. Although reduced levels of CO (TOP 10 - AUG EXAM -
immune dysfunction. Quantitative immunoglobulin A (IgA) or IgE may be seen in 2015 MED MARCH 2016
immunoglobulins were sent and were normal. patients with DiGeorge syndrome—a T cell deficiency BOARDS;
Which of the following immunodeficiency disorders disorder—the total serum immunoglobulin level TOPNOTCH MD
is associated with normal immunoglobulin G (IgG) usually is normal and IgG levels are normal. In X- FROM UST)
levels? linked agammaglobulinemia (Bruton’s) and late-onset
A. X-linked agammaglobulinemia hypogammaglobulinemia, IgG, IgM, and IgA levels are
B. DiGeorge syndrome all reduced, and the total immunoglobulin level is less
C. Late-onset hypogammaglobulinemia than 100 mg/dL. Patients with ataxia-telangiectasia
D. Ataxia-telangiectasia have a defect in their DNA repair mechanism. One of
E. Severe combined immunodeficiency the clinical features is low IgA and IgG. Severe
combined immunodefi ciency results in a decrease in
all immunoglobulins.

TOPNOTCH MEDICAL BOARD PREP PEDIATRICS SUPEREXAM Page 5 of 97


For inquiries visit www.topnotchboardprep.com.ph or email us at topnotchmedicalboardprep@gmail.com
TOPNOTCH MEDICAL BOARD PREP PEDIATRICS SUPEREXAM
For inquiries visit www.topnotchboardprep.com.ph or email us at topnotchmedicalboardprep@gmail.com
Item QUESTION EXPLANATION AUTHOR TOPNOTCH
# EXAM
28 A 5-month-old child was normal at birth, but the SIMILAR TO PREVIOUS BOARD EXAM LESTER BRYAN MIDTERM 1
family reports that the child does not seem to look CONCEPT/PRINCIPLE. There were 2 questions about CO (TOP 10 - AUG EXAM -
at them any longer. They also report the child Tay-Sachs in our Pediatric Exam. Children who have 2015 MED MARCH 2016
seems to “startle” more easily than he had before. Tay-Sachs disease are characterized by progressive BOARDS;
Testing of his white blood cells (WBCs) identifies developmental deterioration; physical signs include TOPNOTCH MD
the absence of β-hexosaminidase A activity, macular cherry-red spots and exquisite and FROM UST)
confirming the diagnosis of which of the following? characteristic sensitivity to noise. Diagnosis of this
a. Niemann-Pick disease, type A disorder can be confirmed biochemically by the
b. Infantile Gaucher disease absence of β-hexosaminidase A activity in WBCs.
c. Tay-Sachs disease The other GM2 gangliosidosis, Sandhoff disease,
d. Krabbe disease results from a deficiency of both β-hexosaminidase A
e. Fabry disease and B. Tay-Sachs disease is inherited as an autosomal
recessive trait; frequently, affected children are of
Eastern European Jewish ancestry. The other
disorders listed in the question are associated with
enzyme deficiencies as follows: Niemann-Pick disease
(type A), sphingomyelinase, which results in a normal-
appearing child at birth who then develops hep-
atosplenomegaly, lymphadenopathy, and
psychomotor retardation in the first 6 months,
followed by regression after that; infantile Gaucher
disease, β-glucosidase, which presents in infancy with
increased tone, strabismus, organomegaly, failure to
thrive, strider, and several years of psychomotor
regression before death; Krabbe disease (globoid cell
leukodystrophy), galactocerebroside β-galactosidase,
which presents early in infancy with irritability,
seizures, hypertonia, and optic atrophy, with severe
delay and death usually occurring in the first 3 years
of life; and Fabry disease, α-galactosidase, which
presents in childhood with angiokeratomas in the
“bathing trunk area,” ultimately resulting in severe
pain episodes.
29 The parents of a 2-week-old infant who was born at The patient has classic findings of galactosemia. LESTER BRYAN MIDTERM 1
home bring him to the emergency center in your Galactose is a component of lactose, found in breast CO (TOP 10 - AUG EXAM -
local hospital for emesis and listlessness. On milk and most infant formulas. Symptoms of 2015 MED MARCH 2016
examination, you find a dehydrated, listless, and galactosemia occur in the first weeks of life. While BOARDS;
irritable infant. Although you do not have a birth screening for classic galactosemia typically is part of TOPNOTCH MD
weight, the parents do not feel that he has gained the newborn metabolic panel, patients fitting the FROM UST)
much weight. He has significant jaundice. His clinical presentation as outlined in the question must
abdominal examination is significant for both be evaluated promptly. Signs and symptoms in
hepatomegaly and splenomegaly. Laboratory addition to those presented in the vignette include
values include a total bilirubin of 15.8 mg/dL and a cataracts and ascites. While three different errors in
direct galactose metabolism are known, most cases result
bilirubin of 5.5 mg/dL. His liver function tests are from the deficiency in galactose-1-phosphate uridyl
elevated and his serum glucose is 38 mg/dL. His transferase. Urine-reducing substances can be
admit urinalysis is negative for glucose but positive positive, but a routine urinalysis will be negative, as
for Gram-negative rods; his urine and his blood the urine strips do not react with galactose. Patients
ultimately grow E. coli. Which of the following are at increased risk for E coli sepsis, and this
nutritional considerations should be considered in infection may precede the diagnosis of galactosemia.
this child? Prompt removal of galactose from the diet usually
A. Administration of high doses of vitamin B6 reverses the symptoms, including cataracts. Use of a
B. Initial diet free of branched-chain amino acids phenylalanine-free diet would be appropriate for a
C. Lactose free formula patient with phenylketonuria (PKU), protein
D. Protein restriction and supplementation with restriction and supplementation with citrulline might
citrulline be used for treating ornithine transcarbamylase
E. Initiation of a diet low or free of phenylalanine deficiency, a diet free from branched-chain amino
acids would be appropriate for maple syrup urine
disease, and part of the treatment for a patient with
homocystinuria is high doses of vitamin B6.
30 A 2-year-old child is admitted to your hospital. The The two conditions in consideration are Kawasaki LESTER BRYAN MIDTERM 1
child’s pediatrician has been following the child for disease and scarlet fever caused by a group A β- CO (TOP 10 - AUG EXAM -
several days and has noted her to have had high hemolytic streptococci. Kawasaki disease is an acute 2015 MED MARCH 2016
fever, peeling skin, abdominal pain, and a bright red febrile illness of unknown etiology and shares many of BOARDS;
throat. You are concerned because two common its clinical manifestations with scarlet fever. TOPNOTCH MD
pediatric problems that could explain this child’s Scarlatiniform rash, desquamation, erythema of the FROM UST)
condition have overlapping presenting signs and mucous membranes that produces an injected
symptoms. Which of the following statements pharynx and strawberry tongue, and cervical
comparing these two diseases in your differential is lymphadenopathy are prominent findings in both. The
true? most serious complication of Kawasaki disease and
A. Neither has a cardiac complication. scarlet fever is cardiac
B. Serologic tests are helpful in diagnosing both. involvement. Erythrogenic toxin-producing group A β-
C. Only one of the diseases has mucocutaneous and hemolytic streptococci is the agent responsible for
lymph node involvement. scarlet fever. Isolation of the organism from the
D. Pharyngeal culture aids in the diagnosis of one of nasopharynx and a rise in antistreptolysin titers will
the conditions. confirm the diagnosis. Serologic tests for a variety of
E. A specific antibiotic therapy is recommended for infectious agents, both viral and bacterial, have been
one of the conditions, but only supportive care is negative in Kawasaki disease. Rheumatic heart disease
recommended for the other. is a serious sequela of streptococcal pharyngitis,
which can be prevented by appropriate treatment
with penicillin. Coronary artery aneurysm and
thrombosis are the most serious complications of
Kawasaki disease. The current approach to treatment
of Kawasaki disease, which includes specific therapy
with aspirin and γ-globulin administered within a
week of the onset of fever, appears to lower the

TOPNOTCH MEDICAL BOARD PREP PEDIATRICS SUPEREXAM Page 6 of 97


For inquiries visit www.topnotchboardprep.com.ph or email us at topnotchmedicalboardprep@gmail.com
TOPNOTCH MEDICAL BOARD PREP PEDIATRICS SUPEREXAM
For inquiries visit www.topnotchboardprep.com.ph or email us at topnotchmedicalboardprep@gmail.com
Item QUESTION EXPLANATION AUTHOR TOPNOTCH
# EXAM
prevalence of coronary artery dilatation and
aneurysm and to shorten the acute phase of the
illness.

31 A 3-year-old girl presents with generalized edema SIMILAR TO PREVIOUS BOARD EXAM LESTER BRYAN MIDTERM 1
shortly after recovery from an upper respiratory CONCEPT/PRINCIPLE. Our Pediatrics exam featured CO (TOP 10 - AUG EXAM -
infection. Laboratory studies reveal marked numerous renal pathology questions as the stem case. 2015 MED MARCH 2016
albuminuria, as well as hypoalbuminemia and It is therefore essential to nail the diagnosis at the BOARDS;
hyperlipidemia. Prior similar episodes responded start to keep you on track for the next questions. The TOPNOTCH MD
to steroids. The most likely diagnosis is combination of generalized edema, massive FROM UST)
A. FSGS proteinuria, hypoalbuminemia, and hyperlipidemia
B. membranous GN constitutes the nephrotic syndrome, the prototype of
C. lipoid nephrosis which is minimal change disease (lipoid nephrosis).
D. PSGN This disorder characteristically occurs in young
E. RPGN children and demonstrates intracytoplasmic lipid in
the proximal convoluted tubules, a paucity of
glomerular abnormalities by light microscopy, and
“fusing” (absence) of the epithelial foot processes by
electron microscopy.
32 4-year-old boy develops weakness of proximal The clinical picture is that of Duchenne muscular LESTER BRYAN MIDTERM 1
lower back and extremity muscles, manifested by dystrophy, the most common and most severe of the CO (TOP 10 - AUG EXAM -
lordosis, a waddling gait, and the need to push on muscular dystrophies. This X-linked disorder is 2015 MED MARCH 2016
his knees in order to stand. Examination reveals characterized by failure of synthesis of dystrophin, BOARDS;
proximal muscle weakness and bilateral most often because of deletion of one or many exons TOPNOTCH MD
enlargement of the calves. His younger brother has in the DMD gene. Patients manifest with proximal FROM UST)
begun to display similar findings, as has his older muscle weakness, progressing to muscle necrosis.
half-brother, who has the same mother. Serum CK Serum CK is markedly increased. Compensatory
is markedly elevated. Which of the following is hypertrophy is followed by pseudohypertrophy, in
characteristic of this disorder? which necrotic muscle is replaced by fat and
A. Aberrant protein coded by a very small gene connective tissue. Most patients become wheelchair-
sequence on the Y chromosome bound and die of respiratory or cardiac failure in their
B. Autosomal dominant mode of inheritance late teenage years or in their early twenties.
C. Mitochondrial inheritance
D. Regression of findings in late adolescence and
adult life
E. Total absence or marked decrease of an
important gene product
33 A 3-year-old boy, an inner city resident, has The child has rickets, which is caused by failure of LESTER BRYAN MIDTERM 1
multiple bony abnormalities, including bowlegs and action of calcitriol (1,25-dihydroxycholecalciferol), the CO (TOP 10 - AUG EXAM -
knock-knees, thickening of the skull with frontal active form of vitamin D. The bony abnormalities in 2015 MED MARCH 2016
bossing, knobby deformities of the costochondral rickets are caused by failure of osteoid matrix to BOARDS;
junctions and, at the ends of the long bones, calcify, leading to excess accumulation of osteoid, TOPNOTCH MD
distortion of the rib cage with flaring over the increased thickness of the epiphyseal growth plates, FROM UST)
diaphragm, and pigeon-breast deformity. A and other skeletal deformities. Many of the effects of
decrease in which of the following is characteristic calcitriol deficiency are mediated by increased release
of this condition? of parathyroid hormone (PTH). PTH stimulates bony
A. Bony osteoblastic activity osteoblastic activity, which is mirrored by an increase
B. Calcification of osteoid in serum alkaline phosphatase.
C. Release of parathyroid hormone
D. Serum alkaline phosphatase activity
E. Synthesis of osteoid
34 A 10-year-old boy presents with a large abdominal Expect many Pathology-type questions in Pediatrics. LESTER BRYAN MIDTERM 1
mass. Computed tomography of the abdomen The typical cytogenetic change associated with Burkitt CO (TOP 10 - AUG EXAM -
reveals enlarged retroperitoneal and mesenteric lymphoma is t(8;14) with increased expression of the 2015 MED MARCH 2016
lymph nodes. Biopsy of one of the involved lymph c-myc gene. This disorder is an aggressive B-cell non- BOARDS;
nodes shows a “starry-sky” appearance, with Hodgkin ymphoma most commonly affecting children. TOPNOTCH MD
prominent debris-containing macrophages. Which The endemic (African) form is characterized by the FROM UST)
of the following statements about this disorder is involvement of the maxilla or mandible, whereas the
correct? sporadic
A. The sporadic form is most frequently associated (Western) form usually involves the abdominal
with EBV. organs. Burkitt lymphoma is generally a rapidly
B. The disorder is considered to be a derivative of growing neoplasm, and the endemic form has a
Hodgkin lymphoma, lymphocyte depletion subtype. frequent association with EBV.
C. The most common cytogenetic change is t(8;14),
with increased expression of c-myc.
D. The tumor cells are derivatives of T lymphocytes.
E. The tumor most often has an indolent clinical
course.

TOPNOTCH MEDICAL BOARD PREP PEDIATRICS SUPEREXAM Page 7 of 97


For inquiries visit www.topnotchboardprep.com.ph or email us at topnotchmedicalboardprep@gmail.com
TOPNOTCH MEDICAL BOARD PREP PEDIATRICS SUPEREXAM
For inquiries visit www.topnotchboardprep.com.ph or email us at topnotchmedicalboardprep@gmail.com
Item QUESTION EXPLANATION AUTHOR TOPNOTCH
# EXAM
35 A 2-year-old boy presents with his third bone Osteogenesis imperfecta (or brittle bone disease) is an LESTER BRYAN MIDTERM 1
fracture within the past several months. There is no autosomal dominant disorder characterized by CO (TOP 10 - AUG EXAM -
history or evidence of trauma. Several close family multiple fractures with minimal trauma. It is caused 2015 MED MARCH 2016
members have been similarly affected. The child is by mutations in either of the genes that code for type I BOARDS;
small for his age, and the sclerae are tinged a bluish collagen and is manifested by connective tissue TOPNOTCH MD
color. Radiographs reveal generalized osteopenia abnormalities affecting the bones, teeth, skin, and FROM UST)
and evidence of multiple fractures, both old and eyes. The sclerae can appear blue due to translucency
new. Which of the following is the usual mode of of the thin connective tissue overlying the choroid.
inheritance of this disorder? The disorder occurs in several variants defined by the
A. Autosomal dominant age of onset and the severity of the fractures. In the
B. Autosomal recessive less obvious cases, the principal differential diagnostic
C. X-linked dominant consideration is child abuse.
D. X-linked recessive
E. Mitochondrial
36 A fully immunized 2-year-old presents to the Bacterial tracheitis is an uncommon but severe and LESTER BRYAN MIDTERM 1
emergency room with several days of low-grade life-threatening sequela of viral CO (TOP 10 - AUG EXAM -
fever, barking cough, and noisy breathing. Over the laryngotracheobronchitis. The typical story is that 2015 MED MARCH 2016
past few hours he has developed a fever of 40°C presented in the case, with several days of viral upper BOARDS;
(104°F) and looks toxic. He has inspiratory and respiratory symptoms, followed by an acute elevation TOPNOTCH MD
expiratory stridor. The family has not noticed of temperature and an increase in respiratory distress. FROM UST)
drooling, and he seems to be drinking without pain. Inspiratory stridor is typical in croup; the biphasic
Direct laryngoscopy reveals a normal epiglottis. stridor and high fever in this patient should be a clue
The management of this disease process includes to consider alternative diagnoses. Children may also
which of the following? present acutely and without the initial viral
A. Intubation and intravenous antibiotics symptoms. The differential must include epiglottitis;
B. Inhaled epinephrine and oral steroids the lack of drooling and dysphagia (and the rarity of
C. Intravenous methylprednisolone epiglottitis since the introduction of the Haemophilus
D. Observation in a cool mist tent influenza B vaccine) help make this a case of
E. Oral antibiotics and outpatient follow-up tracheitis. Management for tracheitis includes
establishing an airway with endotracheal intubation
and IV antibiotics. Special attention is focused on
preservation of the airway, as even intubated children
with tracheitis can have secretions thick and copious
enough to occlude the airway.
Oral antibiotics and outpatient follow-up for a patient
with respiratory distress and toxic appearance are
never appropriate. Inhaled epinephrine and oral
steroids as well as observation in a cool mist tent
suggest a diagnosis of croup, a disease that presents
without high fever but with inspiratory stridor a few
days after an upper respiratory infection. In the case
presentation, the fever and toxic appearance
differentiate this condition from viral croup.
Intravenous methylprednisolone is a component of
treatment for asthma, a diagnosis unlikely in a patient
with high fever as outlined.
37 A 15-year-old athlete with knee pain and severe Osgood-Schlatter disease is a very common problem LESTER BRYAN MIDTERM 1
point tenderness at the tibial tubercle most likely most often seen in athletically active adolescents. This CO (TOP 10 - AUG EXAM -
has disorder is characterized by ossification in the distal 2015 MED MARCH 2016
A. Osgood-Schlatter disease patellar tendon at the point of its insertion onto the BOARDS;
B. Legg-Calvé-Perthes disease tibial apophysis. This disorder is thought to result TOPNOTCH MD
C. Slipped capital femoral epiphysis (SCFE) from mechanical stress on the tendinous insertional FROM UST)
D. Tibial torsion area. X-ray views of the involved knee show a
E. Charcot-Marie-Tooth Disease characteristic irregularity in the insertional area and
often show separately discrete ossicles within the
tendon itself. The disease will present with severe
local pain and exquisite tenderness in the area of the
tibial tubercle. In almost every case, symptoms do
regress after skeletal maturity or the discontinuance
of active athletic participation. In rare cases, persistive
symptoms into adulthood can occur. Legg-Calvé-
Perthes disease, also known as coxa plana, is a
condition of the pediatric hip characterized by a
flattened, misshapen femoral head. The etiology of the
problem is related to osteonecrosis of the proximal
femoral epiphysis and is thought to result from
vascular compromise. Legg-Calvé-Perthes disease
generally presents in children, usually males, between
the ages of 4 and 8 years old. Presenting symptoms
generally include groin or knee pain, decreased hip
range of motion, and a limp. A slipped capital femoral
epiphysis (SCFE) is an acquired disorder of the
epiphysis thought to be associated with weakness in
the perichondrial ring of the growth plate. Children
within the ages of 10 to 16 years old are noted to have
the displacement of the epiphysis on the femoral neck.
In most cases, there is no identifiable trauma history.
It is associated with African American heritage and
obesity and is somewhat more common in boys than
in girls.
Tibial torsion is the most common cause of an intoeing
gait. This is most frequently noted in 1- and 2-year-old
children. This is often bilateral. Although occasionally
intoeing can be marked, pediatric tibial torsion will
completely resolve without treatment in almost every
case.

TOPNOTCH MEDICAL BOARD PREP PEDIATRICS SUPEREXAM Page 8 of 97


For inquiries visit www.topnotchboardprep.com.ph or email us at topnotchmedicalboardprep@gmail.com
TOPNOTCH MEDICAL BOARD PREP PEDIATRICS SUPEREXAM
For inquiries visit www.topnotchboardprep.com.ph or email us at topnotchmedicalboardprep@gmail.com
Item QUESTION EXPLANATION AUTHOR TOPNOTCH
# EXAM
38 Which of the following indicates a poor prognosis SIMILAR TO PREVIOUS BOARD EXAM LESTER BRYAN MIDTERM 1
in a patient with neuroblastoma? CONCEPT/PRINCIPLE. A number of biologic variables CO (TOP 10 - AUG EXAM -
A. Hyperdiploid DNA have been studied in children with neuroblastoma. An 2015 MED MARCH 2016
B. Absence of N-myc amplification open biopsy is often required to provide sufficient BOARDS;
C. Age 1 year tissue for analysis. The presence of hyper-diploid TOPNOTCH MD
D. Age >13 years tumor DNA is associated with a favorable prognosis, FROM UST)
E. Low serum ferritin levels whereas N-myc amplification is associated with a poor
prognosis regardless of patient age. The Shimada
classification describes tumors as having either
favorable or unfavorable histologic features based on
the degree of differentiation, the mitosis-karyorrhexis
index, and the presence or absence of schwannian
stroma. In general, children of any age with localized
neuroblastoma and infants 1 year of age with
advanced disease and favorable disease
characteristics have a high likelihood of disease free
survival. By contrast, older children with advanced
disease have a significantly decreased chance for cure
even with intensive therapy. For example, aggressive
multiagent chemotherapy has resulted in a 2-year
survival rate of approximately 20% in older children
with stage IV disease. Neuroblastoma in the
adolescent has a worse long-term prognosis
regardless of stage or site and, in many cases, a more
prolonged course. High serum ferritin levels portend a
worse prognosis.
39 A 5-year-old boy diagnosed with poststreptococcal SIMILAR TO PREVIOUS BOARD EXAM LESTER BRYAN MIDTERM 1
glomerulonephritis was admitted to the hospital CONCEPT/PRINCIPLE. The clinical description is that CO (TOP 10 - AUG EXAM -
several weeks ago. Over the last several weeks, his of RPGN, which is defined as the nephritic syndrome 2015 MED MARCH 2016
clinical state has not improved. Severe oliguria has that progresses rapidly to fulminant renal failure BOARDS;
developed, his serum creatinine has continued to within weeks or months. RPGN is classified into three TOPNOTCH MD
rise, and his glomerular filtration rate has categories: antiglomerular basement membrane (anti- FROM UST)
decreased by 50% since his admission to the GBM) antibody disease, immune complex deposit
hospital. Which of the following is the most likely disease, and pauciimmune disease (ANCApositive).
diagnosis? Characteristic histologic findings in RPGN include
A. Alport syndrome crescents between the Bowman capsule and the
B. Membranoproliferative glomerulonephritis glomerular tuft. In approximately 50% of cases of
C. Membranous glomerulonephritis RPGN, the disease is of poststreptococcal etiology. It
D. Rapidly progressive glomerulonephritis should be noted, however, that the most common
E. Renal papillary necrosis outcome of poststreptococcal glomerulonephritis is
complete recovery, and only a small minority of
patients progress to RPGN.
40 A 9-year-old child has developed headaches that Brain tumors are the most common solid tumor in LESTER BRYAN MIDTERM 1
are more frequent in the morning and are followed childhood, and account for 25% to 30% of all pediatric CO (TOP 10 - AUG EXAM -
by vomiting. Over the previous few months, his malignancies. While supratentorial tumors 2015 MED MARCH 2016
family has noted a change in his behavior (generally predominate in the first year of life (including choroid BOARDS;
more irritable than usual) and his school plexus tumors and teratomas), brain tumors in TOPNOTCH MD
performance has begun to drop. Imaging of this children 1 to 10 years old are more frequently FROM UST)
child is most likely to reveal a finding in which of infratentorial (posterior fossa) and include cerebellar
the following regions? and brainstem tumors such as medulloblastoma or
A. Infratentorial cerebellar astrocytoma. After 10 years of age,
B. Supratentorial supratentorial tumors (eg, diffuse astrocytoma) are
C. Intraventricular again more common.
D. Spinal canal
E. Peripheral nervous system
41 What is the treatment of choice in vaso-occlusive For vaso-occlusive crisis - Hydration and pain GEORGE MIDTERM 2
crisis of sickle cell anemia? medications; For CVA - Exchange transfusion with MICHAEL EXAM -
A. Hydration and pain medications chronic transfusion protocol to keep Hgb S <30%; For SOSUAN (TOP 5 - MARCH 2016
B. Exchange transfusion splenic sequestratrion - Emergent transfusion with AUG 2015 MED
C. Splenectomy subsequent splenectomy; For aplastic crisis - BOARDS;
D. Antibiotic prophylaxis Supportive care; transfusion if necessary; For TOPNOTCH MD
E. NOTA infections -Appropriate antibiotics, penicillin FROM UST)
prophylaxis, vaccines for S. pneumoniae, H. influenzae,
N. meningitidis
42 Inheritance of glucose-6-phosphate dehydrogenase Mutation in the G6PD gene on the X chromosome. GEORGE MIDTERM 2
deficiency G6PD deficiency is an X-linked recessive disorder. MICHAEL EXAM -
A. Autosomal dominant SOSUAN (TOP 5 - MARCH 2016
B. Autosomal recessive AUG 2015 MED
C. X-liked dominant BOARDS;
D. X-recessive TOPNOTCH MD
E. Non-mendelian FROM UST)

43 Gaucher disease is deficiency of which enzyme Inherited storage disease with deficiency of the GEORGE MIDTERM 2
A. Sphingomyelinase enzyme glucocerebroside beta -glucosidase MICHAEL EXAM -
B. Hexosaminidase A SOSUAN (TOP 5 - MARCH 2016
C. Alpha-galactosidase AUG 2015 MED
D. Beta-glucuronidase BOARDS;
E. Glucocerebrosidase TOPNOTCH MD
FROM UST)

TOPNOTCH MEDICAL BOARD PREP PEDIATRICS SUPEREXAM Page 9 of 97


For inquiries visit www.topnotchboardprep.com.ph or email us at topnotchmedicalboardprep@gmail.com
TOPNOTCH MEDICAL BOARD PREP PEDIATRICS SUPEREXAM
For inquiries visit www.topnotchboardprep.com.ph or email us at topnotchmedicalboardprep@gmail.com
Item QUESTION EXPLANATION AUTHOR TOPNOTCH
# EXAM
44 Parahemophilia is deficiency of what coagulation Factor 5 - Parahemophilia GEORGE MIDTERM 2
factor? Factor 8 - Hemophilia A MICHAEL EXAM -
A. Factor 5 Factor 9 - Hemophilia B SOSUAN (TOP 5 - MARCH 2016
B. Factor 8 Factor 11 - Hemophilia C AUG 2015 MED
C. Factor 9 BOARDS;
D. Factor 11 TOPNOTCH MD
E. NOTA FROM UST)

45 What 4 elements comprise the tetralogy of Fallot, GEORGE MIDTERM 2


except? MICHAEL EXAM -
A. VSD SOSUAN (TOP 5 - MARCH 2016
B. Pulmonic stenosis AUG 2015 MED
C. Overriding of the aorta BOARDS;
D. RVH TOPNOTCH MD
E. NOTA FROM UST)

46 Gross Motor development: Sits without support GEORGE MIDTERM 2


A. 3 months MICHAEL EXAM -
B. 6 months SOSUAN (TOP 5 - MARCH 2016
C. 4 months AUG 2015 MED
D. 8 months BOARDS;
E. NOTA TOPNOTCH MD
FROM UST)

47 A 5 y/o M came into the ER because of marked tet spell or hypercyanotic spell GEORGE MIDTERM 2
increase in cyanosis followed by loss of MICHAEL EXAM -
consciousness. He was previously diagnosed with SOSUAN (TOP 5 - MARCH 2016
Tetralogy of Fallot. Which mamong the following AUG 2015 MED
physical examination finding will support the BOARDS;
diagnosis of a hypercyanotic spell? TOPNOTCH MD
A. Loss of murmur FROM UST)
B. Bounding pulses
C. Arrhythmia
D. Retractions
E. NOTA
48 What is the duration of antibiotic treatment for GEORGE MIDTERM 2
meningitis caused by penicillin-sensitive MICHAEL EXAM -
pneumococcus? SOSUAN (TOP 5 - MARCH 2016
A. 3-5 days AUG 2015 MED
B. 5-7 days BOARDS;
C. 7-10 days TOPNOTCH MD
D. 10-14 days FROM UST)
E. NOTA
49 The anterior fontanel usually closes by: GEORGE MIDTERM 2
A. 10-12 months MICHAEL EXAM -
B. 13-15 months SOSUAN (TOP 5 - MARCH 2016
C. 16-18 months AUG 2015 MED
D. 19-21 months BOARDS;
E. NOTA TOPNOTCH MD
FROM UST)

50 The plasticity of the brain continues into The plasticity of the brain continues into adolescence. GEORGE MIDTERM 2
A. Infancy MICHAEL EXAM -
B. Childhood SOSUAN (TOP 5 - MARCH 2016
C. Adolescence AUG 2015 MED
D. Adulthood BOARDS;
E. NOTA TOPNOTCH MD
FROM UST)

51 True of object permanence (constancy) except: A major milestone is the achievement by 9 mo of GEORGE MIDTERM 2
A. Defined as the understanding that objects object permanence (constancy). Stranger anxiety MICHAEL EXAM -
continue to exist even when not seen almost same time. Introduction of a transitional object SOSUAN (TOP 5 - MARCH 2016
B. Infants persist in searching may allow the infant to self comfort in the parents’ AUG 2015 MED
C. Peek-a-boo absence. The object cannot have any potential for BOARDS;
D. Achieved at 10 months old asphyxiation or strangulation. TOPNOTCH MD
E. NOTA FROM UST)

52 A 9 month M with a purpuric rash is unresponsive GEORGE MIDTERM 2


when brought to the ER. The pulse is barely MICHAEL EXAM -
palpable. After several attempts, you are unable to SOSUAN (TOP 5 - MARCH 2016
establish peripheral venous access. Of the ff, which AUG 2015 MED
is the BEST site for the next attempt to provide fluid BOARDS;
resuscitation for this patient? TOPNOTCH MD
A. Femoral artery FROM UST)
B. Internal jugular vein
C. Intraosseous space
D. Saphenous vein
E. Superficial temporal artery
53 Major criteria for diagnosis of acute rheumatic B should be erythema marginatum. GEORGE MIDTERM 2
fever includes the following, except MICHAEL EXAM -
A. Polyarthritis SOSUAN (TOP 5 - MARCH 2016
B. Erythema nodosum AUG 2015 MED
C. Subcutaneous nodules BOARDS;
D. Carditis TOPNOTCH MD
E. NOTA FROM UST)

TOPNOTCH MEDICAL BOARD PREP PEDIATRICS SUPEREXAM Page 10 of 97


For inquiries visit www.topnotchboardprep.com.ph or email us at topnotchmedicalboardprep@gmail.com
TOPNOTCH MEDICAL BOARD PREP PEDIATRICS SUPEREXAM
For inquiries visit www.topnotchboardprep.com.ph or email us at topnotchmedicalboardprep@gmail.com
Item QUESTION EXPLANATION AUTHOR TOPNOTCH
# EXAM
54 What is the ideal dose to give a 10 kg child with The ideal dose for iron supplemention is 3-6 GEORGE MIDTERM 2
iron deficiency anemia? mg/kg/day. For ths child, it should be 30-60 mg/day MICHAEL EXAM -
A. Elemenal iron, 3-6 mg/day of elemental iron. SOSUAN (TOP 5 - MARCH 2016
B. Elemenal iron, 30-60 mg/day AUG 2015 MED
C. Elemenal iron, 300-600 mg/day BOARDS;
D. Elemenal iron, 33-66 mg/day TOPNOTCH MD
E. Elemenal iron, 0.3-0.6 mg/day FROM UST)

55 Which pf the following Plasmodium species causes Plasmodium falciparum is the most severe form and GEORGE MIDTERM 2
the most severe form of malaria has the highest fatality rate. MICHAEL EXAM -
A. P. falciparum SOSUAN (TOP 5 - MARCH 2016
B. P. malariae AUG 2015 MED
C. P. knowlesi BOARDS;
D. P. ovale TOPNOTCH MD
E. P. vivax FROM UST)

56 The severe acute respiratory syndrome outbreak in The SARS coronavirus, sometimes shortened to SARS- GEORGE MIDTERM 2
the early 2000s was caused by what virus CoV, is the virus that causes severe acute respiratory MICHAEL EXAM -
A. Rhinovirus syndrome (SARS). SOSUAN (TOP 5 - MARCH 2016
B. Influenza virus AUG 2015 MED
C. Coronavirus BOARDS;
D. Parainfluenza virus TOPNOTCH MD
E. Respiratory syncitial virus FROM UST)

57 A 5 y/o boy come in the ER with generalized The most common cause of nephrotic syndrome GEORGE MIDTERM 2
edema, BP 90/60, HR 80 bpm, BUN 250 mg/dL, Cr amongst children is MCD, responsive to steroids most MICHAEL EXAM -
0.5 mg/dL, urine protein is 3600 mg/day. The most of the time. SOSUAN (TOP 5 - MARCH 2016
common histologic finding in this patient is AUG 2015 MED
A. Membranous nephropathy BOARDS;
B. Focal segmental glomerulosclerosis TOPNOTCH MD
C. Minimal change disease FROM UST)
D. Membranoproliferative nephropathy
E. Mesangioproliferative nephropathy
58 The most common etiologic agent of the most The most common cause of spontaneous bacterial GEORGE MIDTERM 2
common major complication of the case above is peritonitis among patient with MCD is Streptococcus MICHAEL EXAM -
A. Klebsiella oxytoca pneumoniae. SOSUAN (TOP 5 - MARCH 2016
B. Streptococcus pneumoniae AUG 2015 MED
C. Pseudomonas aeruginosa BOARDS;
D. Staphylococcus aureus TOPNOTCH MD
E. Enterococcus faecalis FROM UST)

59 Anaphylaxis results in what type of shock GEORGE MIDTERM 2


A. Hypovolemic MICHAEL EXAM -
B. Cardiogenic SOSUAN (TOP 5 - MARCH 2016
C. Distributive AUG 2015 MED
D. Septic BOARDS;
E. Adrenal crisis TOPNOTCH MD
FROM UST)

60 Koplik spots are often seen in this stage of Measles GEORGE MIDTERM 2
infection MICHAEL EXAM -
A. Incubation period SOSUAN (TOP 5 - MARCH 2016
B. Prodromal phase AUG 2015 MED
C. Full blown stage BOARDS;
D. Convalescent stage TOPNOTCH MD
E. AOTA FROM UST)

61 An 11 month old baby presents for evaluation of Roseola infantum/exanthem subitum is characterized JAN CHRISTIAN MIDTERM 3
fever and rash. His mother noted a fever of 40˚C by the abrupt onset of high fever that usualy resovles FELICIANO (TOP EXAM -
two days ago. He appeared well and was eating and after 72 hours coincident with the appearance of a 2 - AUG 2015 MARCH 2016
playing normally, so his mother was not alarmed. faint pink-rose coloured morbiliform rash on the MED BOARDS;
After the fever resolved, he developed a red rash on trunk. TOPNOTCH MD
his trunk that progressed rapidly over the past 24 FROM UST)
hours? What is the most lilkely diagnosis?
A. Exanthem subitum
B. Rubella
C. Rubeola
D. Erythema infectiosum
E. Menigococcemia
62 In relation to the previous question, what do you IN asian countries, ulcers at the uvulopalatoglossal JAN CHRISTIAN MIDTERM 3
call the ulcers at the uvulopalatoglossal junction junction called Nagayama's spots are commonly FELICIANO (TOP EXAM -
characteristic of this disease? reported in infants with roseola 2 - AUG 2015 MARCH 2016
A. Koplik's spots MED BOARDS;
B. Herpangina TOPNOTCH MD
C. Dewdrop on a rose petal FROM UST)
D. Forscheimer's spots
E. Nagayama's spots

63 An 8 year old male was brought to the pediatrician The initial stage of erythema infectiosum is an JAN CHRISTIAN MIDTERM 3
by his mother because he developed low grade erythematouus facial flushing often described as a FELICIANO (TOP EXAM -
fevers several days ago, and now has red cheeks slapped cheek appearance. The causative agent is 2 - AUG 2015 MARCH 2016
and a new rash on his body? What is the most Parvovirus B19 MED BOARDS;
lilkely diagnosis? TOPNOTCH MD
A. Exanthem subitum FROM UST)
B. Rubella
C. Rubeola
D. Erythema infectiosum
E. Infectious mononucleosis
TOPNOTCH MEDICAL BOARD PREP PEDIATRICS SUPEREXAM Page 11 of 97
For inquiries visit www.topnotchboardprep.com.ph or email us at topnotchmedicalboardprep@gmail.com
TOPNOTCH MEDICAL BOARD PREP PEDIATRICS SUPEREXAM
For inquiries visit www.topnotchboardprep.com.ph or email us at topnotchmedicalboardprep@gmail.com
Item QUESTION EXPLANATION AUTHOR TOPNOTCH
# EXAM
64 In relation to the previous question, what is trhe The primary target of B19 infection is the erythroid JAN CHRISTIAN MIDTERM 3
primary target of the causative organism? cell line, specifcially erythroid precursors near the FELICIANO (TOP EXAM -
A. Lymphoid cells pronormoblast stage. There is progressive depletion 2 - AUG 2015 MARCH 2016
B. Myeloid cells of erythroid precursos and a transient arrest of MED BOARDS;
C. Erythroid cells erythropoesis. TOPNOTCH MD
D. Peyer's patches FROM UST)
E Skin langerhans cells

65 A mother is concerned because her 6 year old son Coxsackie virus A 16 is the most common cause of JAN CHRISTIAN MIDTERM 3
developed low grade fevers, painful ulcers in the hand, foot and mouth disease which is characteristic FELICIANO (TOP EXAM -
mouth, and rashes on his hands and feet. What of the case. 2 - AUG 2015 MARCH 2016
could be the likely causative agent? MED BOARDS;
A. Adenovirus TOPNOTCH MD
B. Coxsackievirus A16 FROM UST)
C. Cytomegalovirus
D. Echovirus 22
E. Epstein Barr virus
66 A previously healthy girl presents with a 1 week Rubeola will classically present with the the prodrome JAN CHRISTIAN MIDTERM 3
history of cough, runny nose, fever, sore throat and of fever, malaise, conjunctivitis, cough, coryza and an FELICIANO (TOP EXAM -
red eyes. exanthem of erythematous macules and papules 2 - AUG 2015 MARCH 2016
She went to her pediatrician 2 days ago and was beginning on the face and spreading cephalocaudally MED BOARDS;
prescribed Augmentin (amoxicillin and and centrifugally (by the 3rd day, the whole body is TOPNOTCH MD
clavulanate) for presumed pharyngitis. Yesterday, involved). FROM UST)
Ana developed a red rash which started on her face
and has spread to her trunk. Her mother insists that
the rash is from her new medication. What is the
most likely diagnosis?
A. Exanthem subitum
B. Rubella
C. Rubeola
D. Erythema infectiosum
E. Fixed drug eruption
67 Pertaining to the previosu question all of the ff are In individuals with passively aquired antibody such JAN CHRISTIAN MIDTERM 3
true regardign this condition EXCEPT? as infants and recipeinets of blood products, a FELICIANO (TOP EXAM -
A. In measles with no bacterial infection, the ESR subclinical form of measles may occur callled 2 - AUG 2015 MARCH 2016
and CRP are normal inapparent measles. Atypical measles is a more severe MED BOARDS;
B. The portal of entry is both the respirartory tract form of measles due to the use of the original formalin TOPNOTCH MD
and the conjunctivae containing vaccine. All other statemetns are true. FROM UST)
C. Measles infection causes necrosis of the
respiraroty tract epithelium with lymphocytic
infiltrates
D. Patients are infctious from 3 days before up to 4-
6 days after the onet of rashes.
E. In individuals with passively aquired antibody
such as infants and recipeinets of blood products, a
subclinical form of measles may occur called
atypical measles
68 What is the most common cause of death of Pneumonia is the most common cause of death in JAN CHRISTIAN MIDTERM 3
pediatric patients afflicted with measles? measles. It may manifest as giant cell pneumonia FELICIANO (TOP EXAM -
A. Otitis media caused directly by the viral infection or as 2 - AUG 2015 MARCH 2016
B. Meningitis superimposed bacterial infection. MED BOARDS;
C. Encephalitis TOPNOTCH MD
D. Dehydration due to diarrhea and vomiting FROM UST)
E. Pneumonia

69 Which of the ff screening tests results is condiered An immature/total neutrophil ratio of more than 0.2, JAN CHRISTIAN MIDTERM 3
to be positive for sepsis? WBC of less tha 5000, ANC of less than 1,500 and FELICIANO (TOP EXAM -
A. Total WBC 3000/mm3; immature/total platelet of less than 100,000 is indicative of sepsis in 2 - AUG 2015 MARCH 2016
neutrophil ratio of 0.1 and gastric aspirate with the newborn MED BOARDS;
neutrophils 3/hpf TOPNOTCH MD
B. Micro-ESR 10 mm in 1 hr, total WBC of FROM UST)
4500/mm3 and gastric aspirate with neutrophil of
2/hpf
C. CRP 3 mg/dl; immature/total neutrophil ratio of
0.5 and gastric aspirate with neutrophil 6/hpf
D. Total WBC 6300/mm3; CRP 0.5 mg/dl
immature/total neutrophil ratio of 0.2
E. All of the above
70 Which of the ff statements describes accurately Routine intubation to aspirate the lungs of vigorous JAN CHRISTIAN MIDTERM 3
meconium aspiration syndrome? infnts born through meconium stained fluid is not FELICIANO (TOP EXAM -
A. 10-15% of births occur in premature babies effective in reducing MAS. 10-15% occurs in term or 2 - AUG 2015 MARCH 2016
B. Respiratory distress usually manifests within 24 postterm babies. Respiratry distress occurs within the MED BOARDS;
hours after birth first hour after birth. A typcial CXR is char by patchy TOPNOTCH MD
C. Chest xray is usually normal infiltrates and coarse streaking of both lugn fields. FROM UST)
D. Routine intubation is not indicated for vigorous
babies
E. None of the above
71 A newly born baby girl os observed to have tremors Heroin is associated with tremors and hyperirrtiablity JAN CHRISTIAN MIDTERM 3
and hyperirritability. Moter finally admitted taking of the infant FELICIANO (TOP EXAM -
an illicit drug. Which of the ff substances/drugs is 2 - AUG 2015 MARCH 2016
commonly associated with the symptoms? MED BOARDS;
A. Tobacoo TOPNOTCH MD
B. Alcohol FROM UST)
C. Cocaine
D. Heroin
E. All of the above

TOPNOTCH MEDICAL BOARD PREP PEDIATRICS SUPEREXAM Page 12 of 97


For inquiries visit www.topnotchboardprep.com.ph or email us at topnotchmedicalboardprep@gmail.com
TOPNOTCH MEDICAL BOARD PREP PEDIATRICS SUPEREXAM
For inquiries visit www.topnotchboardprep.com.ph or email us at topnotchmedicalboardprep@gmail.com
Item QUESTION EXPLANATION AUTHOR TOPNOTCH
# EXAM
72 You have been following an 8 year old child in your BA<CA - GH deficiency, constitutional growth delay, JAN CHRISTIAN MIDTERM 3
iffce for the past several years and have noted that hypothroidism, hypercortisolism
FELICIANO (TOP EXAM -
durign the past year, his height has remained below BA=CA - genetic short stature, skeletal dysplasias, 2 - AUG 2015 MARCH 2016
the third percentile. You are conerned about his IUGR, Turner's syndrome MED BOARDS;
short stature and decide to begin a workup. On TOPNOTCH MD
bone age determination it was discoviered that the FROM UST)
px's bone age is 4 years younger than his
chornological age. Which ofthe ff diagnosis should
be considered?
A. Genetic short stature
B. Skeletal dysplasias
C. Intrauterine grwoth retardation
D. Turner's syndrome
E. Growth hormone deficiency
73 An 8 year old girl is brought to the ER with a fever The CSF analysis is most consistent with aseptic JAN CHRISTIAN MIDTERM 3
of 38.5C, stiff neck, photophobia and headache. mengitis, specifically viral menigitis. Enterovirus are FELICIANO (TOP EXAM -
Lumbar pucnture was perfromed revealign the ff the most common cause of viral menigitis. There is 2 - AUG 2015 MARCH 2016
results: WBC 38- cells/mm3 with 40% PMN and lymphocytic predominance and normal protein and MED BOARDS;
60% lymphocyte; normal protein and glucose and sugar. TOPNOTCH MD
negative gram stain. Which of the ff is the most FROM UST)
likely cause of her mengitis?
A. Neisseria mengitidis
B. Streptococcus pneumoniae
C. Enterovirus
D. Cryptococcus neoformans
E. Mycobacterium tuberculosis
74 An 8 year old girl presents with sore throat, fever The patient's clinical presentation is consistent wit JAN CHRISTIAN MIDTERM 3
and a rough sandpaper like rash over her trunk and hscarlet fever, caused by the erythrogenic strain of FELICIANO (TOP EXAM -
extremities. A throat culture is positive for group A GAHBS. Althoigh there are multiple complication of 2 - AUG 2015 MARCH 2016
Beta hemolytic streptococcus. Treatment of her GABHS as mentioned in the question, only rheumatic MED BOARDS;
infection with antibiotics will prevent which of the fever will be prevented by treatment of antibiotics. TOPNOTCH MD
ff complications? FROM UST)
A. Reactive arthritis
B. Rheumatic fever
C. Post strep glomerlonephritis
D. Guillan barre syndrome
E. All of the above
75 A newborn male had cyanosis on birth. On Tricuspid atresia is the only casue of cyanosis in the JAN CHRISTIAN MIDTERM 3
auscultation, you hear a single S2 but no murmur. newborn period that manifests with left axis deviation FELICIANO (TOP EXAM -
O2 sat is 72% in room air. An ECG reveals left axis and left ventricular hypertrophy on ECG. Patients with 2 - AUG 2015 MARCH 2016
deviation and elft ventricular hypertrophy. What is TA withot a ventricular septal defect have a single S2 MED BOARDS;
the most likely diagnosis? as a result of the sual coexistence of pulmonary atresia TOPNOTCH MD
A. Tetralogy of Fallot and do not have a murmur. FROM UST)
B. Transposition of the great arteries
C. Truncus arteriosus
D. Total anomalous pulmonary venous connection
E. Tricuspid atresia with intact ventricular septum
76 An 8 year old has a chest radiograph that shows The classic CXR appearance of TAPVR is the snowman JAN CHRISTIAN MIDTERM 3
cadiomegaly with a snowman appearance. What is appearance FELICIANO (TOP EXAM -
the most likely diagnosis? 2 - AUG 2015 MARCH 2016
A. Tetraloogy of Fallot MED BOARDS;
B. Transposition of the great arteries TOPNOTCH MD
C. Truncus arteriosus FROM UST)
D. Total anomalous pulmonary venous connection
E. Tricuspid atresia with intact ventricular septum

77 A 3 year old boy presents with high grade fever, All of the symptoms and PE findings of the patient JAN CHRISTIAN MIDTERM 3
anorexia and drooliong. He has been previously points to a possible epiglottitis. One must avoid FELICIANO (TOP EXAM -
well. His immunixation records are up to date. On erxcessive stimulation, including examination of the 2 - AUG 2015 MARCH 2016
examination he appears very ill and prefers to lean pharynx with a tongue depressor bec this will induce MED BOARDS;
forward on his hands and his neck hyperextended. respiratory distress. Evaluation of the airway and TOPNOTCH MD
His voice is muffled. Which of the ff is correct intubation is necessary. FROM UST)
regarding his diagnosis?
A The patient likely has bacterial tracheitis and
should be started on antistaphylococcal antibiotics.
B. Racemic epinephrine should be immediately
administered.
C. The patient should have his airway visualized
and intubated in a controlled environment
D. A neck radiograph will show a steeple sign
E. The throat should be examined to rule out a
retropharngeal abscess
78 A 5 month old infant in a daycare facility develops The patient's clinical features are conistent with JAN CHRISTIAN MIDTERM 3
low grade fever, rhonorrhea and cough. A few days bronchiolitis, a lower resp. tract infection. Still, FELICIANO (TOP EXAM -
later, she is brought to the ER with tachypnea, chest supportive care is the most effective management and 2 - AUG 2015 MARCH 2016
retractions, diffuse expiratory whezing and fine inhaled bornchodilators have some benefit as well. MED BOARDS;
inspiratory rales bilateraly. Which of the ff is Chest xray will dmeosntrate hyperinflation. Evidence TOPNOTCH MD
correct regarding his diagnosis? of ribavarin in the treatment if bronchiolitis is lacking FROM UST)
A. Chest xray will demonstrate decreased lung and should only be given in severely ill infants.
volumes with bilateral lobar consolidation
B. Chlamydia trachomatis should be condered as a
possible etiologic agent
C. Supportive care is the most important
management
D. IV antibiotics should be started
E. Ribavarin should be administered.
TOPNOTCH MEDICAL BOARD PREP PEDIATRICS SUPEREXAM Page 13 of 97
For inquiries visit www.topnotchboardprep.com.ph or email us at topnotchmedicalboardprep@gmail.com
TOPNOTCH MEDICAL BOARD PREP PEDIATRICS SUPEREXAM
For inquiries visit www.topnotchboardprep.com.ph or email us at topnotchmedicalboardprep@gmail.com
Item QUESTION EXPLANATION AUTHOR TOPNOTCH
# EXAM
79 A 9 month old girl is brought to the ER with a 1 day The clinical picture is consistent with intussception. JAN CHRISTIAN MIDTERM 3
history of intermittent, inconsolable crying. She The most impt diagnostic test at this time is the FELICIANO (TOP EXAM -
seems comfrotable in between the attacks. She has contrast enema which demonstrates the involved 2 - AUG 2015 MARCH 2016
vomited twice and has had one bowel movement segment. In addition, the pressure from the contrast MED BOARDS;
that the mother described as bloody. Based on the enema will usually reduce the intussception. TOPNOTCH MD
clinical presentation what is the most appropriate FROM UST)
diagnostic procedure?
A. Plain abdominal xray
B. Plain whole abdomen CT scan
C. Barium enema
D. Surgical exploration
E. Abdominal UTZ
80 A 14 year old boy has 3+ protein and 4+ blood with This patient's clinical presentation are consistnet with JAN CHRISTIAN MIDTERM 3
RBC casts on urinalysis. Further questioning IgA nephropathy. They usually present with recurrent FELICIANO (TOP EXAM -
revelaed 2 prior episodes of tea coloured urine bouts of gross hematuria assoc with respiratory 2 - AUG 2015 MARCH 2016
concurrent with upper respiratory tract infection infections. MED BOARDS;
during the last 3 years. Which of the ff is the most TOPNOTCH MD
likely diagnosis? FROM UST)
A. Membranous nephropathy
B. SLE nephritis
C. IgA nephropathy
D. Membranoproliferatifve GN
E. Post streptococcal GN
81 1. A 5 year old female came in for complaints of Nelson’s Textbook of Pediatrics 19th edition p.1477 ANDREW TIU FINAL EXAM -
cough and watery nasal discharges 1 week PTA. (TOP 1 - AUG MARCH 2016
Upon examination at the ER of CVG hospital, T = 2015 MED
38C RR = 42 with nasal flaring and subcostal BOARDS;
retractions. Crackles are heard on both lung fields. TOPNOTCH MD
Abdominal distention was also noted. CXR revealed FROM CIM)
hyperinflation with bilateral interstitial infiltrates
and peribronchial cuffing. Which of the following is
the most likely diagnosis?
a. bronchiolitis
b. pneumonia
c. acute bronchitis
d. asthma
e. NOTA
82 2. An 18 year old male came in for complaints of Nelson’s Textbook of Pediatrics 19th edition p.1457 ANDREW TIU FINAL EXAM -
cough and watery nasal discharges 1 week PTA. (TOP 1 - AUG MARCH 2016
Upon examination at the ER, T = 38.5C, RR = 65, 2015 MED
dyspnea and difficulty to console the child. After 1 BOARDS;
hour, pulse oximetry revealed 70% thus patient TOPNOTCH MD
was intubated. Auscultation of chest revealed fine FROM CIM)
crackles on both lung fields with occasional
wheezes. CBC taken was normal. CXR revealed
hyperinflation and air trapping. Which of the
following is the most likely diagnosis?
a. bronchiolitis
b. pneumonia
c. acute bronchitis
d. asthma
e. NOTA
83 3. A 2 year old female comes in for complaints of Nelson’s Textbook of Pediatrics 19th edition p.1460 ANDREW TIU FINAL EXAM -
low grade fever and watery nasal discharges 1 (TOP 1 - AUG MARCH 2016
week PTA. Two days PTA, dry hacking cough was 2015 MED
noted now becoming productive with yellowish BOARDS;
sputum. Auscultation of the chest revealed coarse TOPNOTCH MD
breath sounds. CXR and CBC were unremarkable. FROM CIM)
Which of the following is the most likely diagnosis?
a. bronchiolitis
b. pneumonia
c. acute bronchitis
d. asthma
e. NOTA
84 4. A five year old male presents to the emergency The acute onset of a focal neurologic deficit in a child ANDREW TIU FINAL EXAM -
room with sudden onset right sided hemiparesis. is stroke until proven otherwise. THe msot common (TOP 1 - AUG MARCH 2016
Two months ago, patient was diagnosed with iron focal presentation is hemiparesis but acute visual, 2015 MED
deficiency anemia and ascariasis. Which of the speech, sensory or balance deficits also occur. Three BOARDS;
following is the most likely diagnosis? main etiologies are arteriopathic, cardiac, and TOPNOTCH MD
a. Todd paralysis hematological (sickle cell anemia and IDA) FROM CIM)
b. migraine Nelson’s Textbook of Pediatrics 19th edition p.2080
c. meningitis (CNS infection)
d. ischemic stroke
e. NOTA
85 5. Which of the following laboratory tests in the acute ischemic stroke is a clinical and radiographic ANDREW TIU FINAL EXAM -
above situation would confirm your impression? diagnosis. CT imaging can demonstrate larger mature (TOP 1 - AUG MARCH 2016
a. blood cultures AIS and exclude hemorrhage, however, MRI identifies 2015 MED
b. MRI early and small infarcts and is therefore required to BOARDS;
c. EEG exclude ischemic stroke. TOPNOTCH MD
d. lumbar tap Nelson’s Textbook of Pediatrics 19th edition p.2080 FROM CIM)
e. CT scan

TOPNOTCH MEDICAL BOARD PREP PEDIATRICS SUPEREXAM Page 14 of 97


For inquiries visit www.topnotchboardprep.com.ph or email us at topnotchmedicalboardprep@gmail.com
TOPNOTCH MEDICAL BOARD PREP PEDIATRICS SUPEREXAM
For inquiries visit www.topnotchboardprep.com.ph or email us at topnotchmedicalboardprep@gmail.com
Item QUESTION EXPLANATION AUTHOR TOPNOTCH
# EXAM
86 6. Which of the following would be the most Children with these presentations require urgent ANDREW TIU FINAL EXAM -
appropriate initial management strategy for the neuroimaging and consultation with a child (TOP 1 - AUG MARCH 2016
patient above? neurologist as emergency interventions may be 2015 MED
a. call your friendly pediatric neurologist indicated. BOARDS;
b. neuroprotective strategies Nelson’s Textbook of Pediatrics 19th edition p.2080 TOPNOTCH MD
c. antithrombotic strategies FROM CIM)
d. correction of iron deficiency anemia
e. rehabilitation
87 7. What is the definitive treatment in the above emergency thrombolysis is not yet established for ANDREW TIU FINAL EXAM -
condition? children. Antithrombotic strategies depend on the (TOP 1 - AUG MARCH 2016
a. broad spectrum antibiotics suspected cause but include anticoagulation with 2015 MED
b. carbamazepine heparins or antiplatelet strategies such as aspirin. BOARDS;
c. careful control of cerebral perfusion pressure Nelson’s Textbook of Pediatrics 19th edition p.2080 TOPNOTCH MD
d. antithrombosis FROM CIM)
e. NOTA

88 8. A previously healthy male 3 year old child Nelson’s Textbook of Pediatrics 19th edition p.1002 ANDREW TIU FINAL EXAM -
followed up in your clinic for official reading of TST (TOP 1 - AUG MARCH 2016
which turned out to be 5mm. Which of the 2015 MED
following is true? BOARDS;
a. patient is immunosuppressed TOPNOTCH MD
b. results is equivocal FROM CIM)
c. negative skin test
d. positive skin test
e. confirmation of history of exposure
89 9. A 2 year old male, CF, presents to the clinic with Measles vaccine only effective in prevention or ANDREW TIU FINAL EXAM -
fever and rash which began on the forehead and on modification of measles if given within 72 hours of (TOP 1 - AUG MARCH 2016
upper neck spreading downward to extremities exposure. Immune globulin may be given up to 6 days 2015 MED
which subsequently faded leaving a fine after exposure. Vitamin A dosages is 50,000 IU for less BOARDS;
desquamation. CF also has a brother who is 13 than 6 months, 100,000 6mo-1 yr, and 200,000 >= 1 TOPNOTCH MD
months old who has been exposed 4 days ago when year old FROM CIM)
his grandmother brought him along to visit him. Nelson’s Textbook of Pediatrics 19th edition p.1073
Which of the following is the most appropriate
management?
a. give measles vaccine to CF’s brother
b. give measles immune globulin to CF’s brother
c. give both vaccine and immune globulin to CF’s
brother
d. Give vitamin A 50,000 IU to CF
e. give vitamin A 100,000 IU to CF’s brother
90 10. A 25 year old G3P2 (2002) mother, KEC, was Nelson’s Textbook of Pediatrics 19th edition p.1110 ANDREW TIU FINAL EXAM -
admitted to VCMC hospital due to hyperemesis (TOP 1 - AUG MARCH 2016
gravidarum. Her 2 year old child started to have 2015 MED
intensely pruritic erythematous macules that BOARDS;
evolve through papules then vesicles distributed TOPNOTCH MD
centripetally 2 days ago. KEC also has a 12 month FROM CIM)
old child at home and a 14 year old neighbor who
got exposed. Which of the following is the most
appropriate management?
a. give varicella vaccine to the 12month old child
b. give VariZIG to KEC
c. give VariZIG to the 14 year old neighbor
d. both A and B
e. both B and C
91 11. When should you suspect the 14 year old Nelson’s Textbook of Pediatrics 19th edition p.1105 ANDREW TIU FINAL EXAM -
neighbor to develop the same symptoms with the 2 (TOP 1 - AUG MARCH 2016
year old child of KEC? 2015 MED
a. 3 - 5 days BOARDS;
b. 10 - 21 days TOPNOTCH MD
c. 24 - 48 hours FROM CIM)
d. 7 - 10 days
e. 3 - 4 weeks
92 12. After 2 days, KEC was discharged. Nurse JoAnne Varicella vaccine is contraindicated during pregnancy ANDREW TIU FINAL EXAM -
who is also a neighbor of KEC came to visit her to Nelson’s Textbook of Pediatrics 19th edition p.1109 (TOP 1 - AUG MARCH 2016
check on her blood pressure at home but was also 2015 MED
exposed to KEC’s 2 year old child. Which of the BOARDS;
following is not true regarding prevention of VZV TOPNOTCH MD
transmission? FROM CIM)
a. person is contagious 24-48 hours before rash
appears
b. unvaccinated health care workers who are
exposed should be furloughed for 8 - 21 days
c. varicella vaccination can be given to KEC
d. evidence of immunity may be through
verification of history of herpes zoster disease by
health care provider
e. NOTA
93 13. A 3 year old female comes to you with Nelson’s Textbook of Pediatrics 19th edition ANDREW TIU FINAL EXAM -
complaints of a rapidly growing left abdominal (TOP 1 - AUG MARCH 2016
mass that crosses the midline associated with a 2015 MED
purplish rash and weight loss. Which of the BOARDS;
following is the most likely diagnosis? TOPNOTCH MD
a. rhabdomyosarcoma FROM CIM)
b. hepatoblastoma
c. wilm’s tumor
d. neuroblastoma

TOPNOTCH MEDICAL BOARD PREP PEDIATRICS SUPEREXAM Page 15 of 97


For inquiries visit www.topnotchboardprep.com.ph or email us at topnotchmedicalboardprep@gmail.com
TOPNOTCH MEDICAL BOARD PREP PEDIATRICS SUPEREXAM
For inquiries visit www.topnotchboardprep.com.ph or email us at topnotchmedicalboardprep@gmail.com
Item QUESTION EXPLANATION AUTHOR TOPNOTCH
# EXAM
e. nephroblastoma

94 14. RL, a 16 year old male, came in for recurrent in severe hemophilia, PTT value is usually 2 - 3 times ANDREW TIU FINAL EXAM -
tonsilitis 5x this year. Patient underwent bilateral the upper limit of normal. REsults of other screening (TOP 1 - AUG MARCH 2016
tonsillectomy and during the 1st postoperative day, tests (platelet count, bleeding time, PT, thrombin 2015 MED
recurrent bleeding at operative site was noted. time) are normal. BOARDS;
Laboratories requested were as follows: bleeding Nelson’s Textbook of Pediatrics 19th edition TOPNOTCH MD
time = 3 minutes (2 - 8); PTT= 78 seconds (21 - 35); FROM CIM)
INR = 1.0; mixing studies = correction of PTT; CBC
and thrombin time within normal limits. Which of
the following is the most likely diagnosis?
a. von willebrand’s disease
b. antiphospholipid antibody syndrome
c. hemophilia
d. DIC
e. parahemophilia
95 15. A 7 year old male came in for high grade fever 6 in the acute stage of Kawasaki disease, give IVIG and ANDREW TIU FINAL EXAM -
days PTC associated with a polymorphous rash, aspirin. Long term therapy for patients with coronary (TOP 1 - AUG MARCH 2016
diffuse injection of pharynx, cervical abnormalities include aspirin, clopidogrel, warfarin, 2015 MED
lymphadenopathy, and periungual peeling of toes. LMWH. BOARDS;
Which of the following is the most appropriate Nelson’s Textbook of Pediatrics 19th edition TOPNOTCH MD
management for this patient? FROM CIM)
a. ASA
b. IVIG
c. dexamethasone
d. both A and B
e. all of the above
96 16. A 12 year old female came in for knee pain and It is characterized by arthritis, fever, and prominent ANDREW TIU FINAL EXAM -
swelling 3 months PTA. Patient cannot participate visceral involvement including HSM, LAD, and (TOP 1 - AUG MARCH 2016
in physical education classes due to limited range of serositis (pericarditis). The evanescent salmon 2015 MED
motion. One month PTA, patient now noted ankle colored lesions classic for systemic onset disease are BOARDS;
and wrist pain and swelling associated with fever, linear or circular and most commonly distributed over TOPNOTCH MD
generalized lymphadenopathy, and migratory trunk and proximal extremities. FROM CIM)
salmon colored rash. Upon physical examination, Nelson’s Textbook of Pediatrics 19th edition
pericardial friction rub was heard. ANA was
negative while CBC showed anemia. Which of the
following is the most likely diagnosis?
a. systemic JIA
b. sarcoidosis
c. SLE
d. rheumatoid arthritis
e. acute rheumatic fever
97 17. An 8 year old presented to the OPD of CVGH M3 subtype acute promyeloblastic leukemia presents ANDREW TIU FINAL EXAM -
with multiple subcutaneous nodules. On physical with DIC due to granules present in the blast cells (TOP 1 - AUG MARCH 2016
examination, HR = 120, RR = 42, O2 sats = 89%, which trigger the coagulation cascade. 2015 MED
multiple palatal petechiae, pale palpebral Nelson’s Textbook of Pediatrics 19th edition BOARDS;
conjunctivae, coarse crackles on left lung field, TOPNOTCH MD
subcostal retractions, generalized FROM CIM)
lymphadenopathy. Patient was then transported
immediately to ER and the following labs were
taken: CBC = anemia with leukocytosis; PTT and PT
were elevated. Patient had difficulty breathing and
went into respiratory arrest. Upon intubation,
pinkish frothy sputum was noted and IV sites were
oozing with blood. Which of the following is not
true?
a. this is diagnosed through bone marrow biopsy
b. a t(15;17) has favorable prognosis
c. M2 subtype usually presents with DIC
d. remission occurs in 85 - 90% with chemotherapy
e. cell of origin is myeloblast
98 18. Which of the following congenital heart diseases also includes TGA, DORV without PS, TA, single ANDREW TIU FINAL EXAM -
presents with increased pulmonary blood flow? ventricle (TOP 1 - AUG MARCH 2016
a. total anomalous pulmonary venous return if decreased pulmonary blood flow: TOF, TA, Ebstein 2015 MED
b. hypoplastic left heart syndrome anomaly BOARDS;
c. double outlet right ventricle Nelson’s Textbook of Pediatrics 19th edition TOPNOTCH MD
d. both A and B FROM CIM)
e. both B and C

99 19. A neonate 36 weeks by BS was admitted to the if PaO2 rises above 150mmHg during 100% O2, an ANDREW TIU FINAL EXAM -
NICU for further evaluation due to an episode of intracardiac right to left shunt can usually be (TOP 1 - AUG MARCH 2016
apnea and cyanosis. Currently, patient is grunting excluded. If with pulmonary disease, PaO2 generally 2015 MED
with subcostal retractions. CXR revealed ground increases significantly with 100% oxygen as BOARDS;
glass infiltrates on both lung fields. O2 sats remain ventilation - perfusion inequalities are overcome. If TOPNOTCH MD
89%. Which of the following is not true? due to CNS disorder, PaO2 will normalize through FROM CIM)
a. hyperoxia test can distinguish cyanotic artifical ventilation.
congenital heart disease from pulmonary disease Nelson’s Textbook of Pediatrics 19th edition
b. neonates with Congenital heart disease can
increase PaO2 when given 100% oxygen
c. if due to a CNS disorder, PaO2 will not normalize
with 100% oxygen through artificial ventilation
TOPNOTCH MEDICAL BOARD PREP PEDIATRICS SUPEREXAM Page 16 of 97
For inquiries visit www.topnotchboardprep.com.ph or email us at topnotchmedicalboardprep@gmail.com
TOPNOTCH MEDICAL BOARD PREP PEDIATRICS SUPEREXAM
For inquiries visit www.topnotchboardprep.com.ph or email us at topnotchmedicalboardprep@gmail.com
Item QUESTION EXPLANATION AUTHOR TOPNOTCH
# EXAM
d. both A and B
e. both B and C

100 20. A 6 month old infant came in for complaints of vascular ring results from abnormal development of ANDREW TIU FINAL EXAM -
stridor. Upon workup, a barium esophagram shows aortic arch complex. THe double aortic arch is the (TOP 1 - AUG MARCH 2016
a posterior indentation of the esophagus. Which of most common complete vascular ring encircling both 2015 MED
the following is the most likely diagnosis? trachea and esophagus compressing both. BOARDS;
a. laryngomalacia Nelson’s Textbook of Pediatrics 19th edition TOPNOTCH MD
b. vascular ring FROM CIM)
c. posterior laryngeal cleft
d. congenital subglottic hemangioma
e. congenital subglottic stenosis
101 Males and females are equally affected except in:
Page 34 of Topnotch Handout. As the X chromosome is ANGELA DIAGNOSTIC
A. Autosomal dominant disorder one of the sex chromosomes (the other being the Y PAULINE P. EXAM - AUG
B. Autosomal recessive disorder chromosome), X-linked inheritance is determined by CALIMAG- 2015
C. X-linked disorder the sex of the parent carrying a specific gene and can LOYOLA (TOP 8 -
D. Y-linked disorder often seem complex. This is due to the fact that, FEB 2015 MED
E. Mitochondrial inheritance typically, females have two copies of the X- BOARDS;
chromosome, while males have only one copy. The TOPNOTCH MD
difference between dominant and recessive FROM UST)
inheritance patterns also plays a role in determining
the chances of a child inheriting an X-linked disorder
from their parentage.

Males are normally hemizygous for the X
chromosome, having only one copy. As a result, X-
linked dominant disorders usually show higher
expressivity in males than females
102 A 5 year old girl presenting with mental reatrdation Page 34 of Topnotch Handout. Phenylketonuria is an ANGELA DIAGNOSTIC
associated with fair complexion is most likely due autosomal recessive disorder. Infants with classic PKU PAULINE P. EXAM - AUG
to: A. Maple syrup urine appear normal until they are a few months old. CALIMAG- 2015
disease Without treatment, these children develop permanent LOYOLA (TOP 8 -
B. Phenylketonuria intellectual disability. Seizures, delayed development, FEB 2015 MED
C. Galactosemia behavioral problems, and psychiatric disorders are BOARDS;
D. Methylmalonic aciduria also common. Untreated individuals may have a musty TOPNOTCH MD
E. Tay-Sachs disease or mouse-like odor as a side effect of excess FROM UST)
phenylalanine in the body.
103 A 1 year old patient came in after 3 days of SIMILAR TO PREVIOUS BOARD EXAM ANGELA DIAGNOSTIC
vomiting and diarrhea. Last urine output was 12 CONCEPT/PRINCIPLE. Urine indices may be useful in PAULINE P. EXAM - AUG
hours ago. After giving fluid bolus and hydrating for differentiating prerenal from intrinsic ARF. Intrinsic CALIMAG- 2015
4 hours, urine output was 20 ml. Serum creatinine renal: Sp gr<1.010, low urine osmolality, high urine LOYOLA (TOP 8 -
was 1.2 mgs/dl and urine sp gr was 1.010. If sodium>40mEq/L and FeNa>2%. Pre renal: elevated FEB 2015 MED
intrinsic renal failure has set in, which finding will sp gr, elevated urine osmolality, low urine sodium <20 BOARDS;
be consistent?
mEq/L, low FeNa <1%. TOPNOTCH MD
A. Elevated urine osmolality FROM UST)
B. Urine Na >30
C. FENa <1
D. BUN/Crea ratio >20
E. None of the above
104 Urine culture finding that best indicate a diagnosis SIMILAR TO PREVIOUS BOARD EXAM ANGELA DIAGNOSTIC
of UTI:
CONCEPT/PRINCIPLE. Urine culture and sensitivity PAULINE P. EXAM - AUG
A. E.Coli 7,000 cfu/ml and Staph Epidermidis testing is the gold standard for the diagnosis of UTI. CALIMAG- 2015
10,000 cfu/ml by suprapubic aspiration Suprapubic count-any count of gram neg bacteria, LOYOLA (TOP 8 -
B. E. Coli 30,000 cfu/ml by midstream catch transurethral cath >50,000 cfu/ml of single organism, FEB 2015 MED
C. Proteus 2,000 cfu/ml urine collected by midstream sample >100,000 cfu/ml of single BOARDS;
cathetherization organissm, if count is 10,000 cfu/ml but symptomatic, TOPNOTCH MD
D. E.Coli 5,000 cfu/ml by suprapubic aspiration infection is likely. FROM UST)
E. E.Coli 70,000 cfu/ml by urine bag collection
105 A term male infant delivered via emergency CS at SIMILAR TO BOARD QUESTION. Page 5 of handout. ANGELA DIAGNOSTIC
39 weeks AOG presented with early onset jaundice, ABO incompatibility is the most common cause of PAULINE P. EXAM - AUG
the mother is O+ and the infant is B+, infant was hemolytic disease of the newborn. Mother is usually CALIMAG- 2015
breastfed, which is the most likely diagnosis:
type O and baby is either A or B.Most cases are mild LOYOLA (TOP 8 -
A. RH incompatibility jaundice and hepatosplenomegaly. FEB 2015 MED
B. Breastfeeding jaundice BOARDS;
C. Breastmilk jaundice TOPNOTCH MD
D. Sepsis FROM UST)
E. ABO incompatibility
106 A newly born female was noted to have jaundice, Page 5 of Topnotch Handout. CMV infection is the ANGELA DIAGNOSTIC
hepatomegaly, platelet count of 25,000/cu mm. most common congenital infection. Presents with PAULINE P. EXAM - AUG
Cranial ultrasound showed periventricular chorioretinitis, microcephaly, and periventricular CALIMAG- 2015
calcifications. What is the most likely diagnosis?
calcifications. Tx: is Ganciclovir. LOYOLA (TOP 8 -
A. Congenital Rubella FEB 2015 MED
B. Congenital Syphillis BOARDS;
C. Toxoplasmosis TOPNOTCH MD
D. VZV infection FROM UST)
E. CMV infection

TOPNOTCH MEDICAL BOARD PREP PEDIATRICS SUPEREXAM Page 17 of 97


For inquiries visit www.topnotchboardprep.com.ph or email us at topnotchmedicalboardprep@gmail.com
TOPNOTCH MEDICAL BOARD PREP PEDIATRICS SUPEREXAM
For inquiries visit www.topnotchboardprep.com.ph or email us at topnotchmedicalboardprep@gmail.com
Item QUESTION EXPLANATION AUTHOR TOPNOTCH
# EXAM
107 A 6 y/o boy developed anasarca. On PE, BP 100/70 SIMILAR TO PREVIOUS BOARD EXAM ANGELA DIAGNOSTIC
(p50). Laboratory findings showed Urine protein CONCEPT/PRINCIPLE. This nephrotic syndrome fits PAULINE P. EXAM - AUG
on dipstick 4+, Urine protein/urine creatinine ratio minimal change disease. Minimal change disease CALIMAG- 2015
was elevated. He was started on oral prednisone. usually responds well to initial treatment and over LOYOLA (TOP 8 -
After 2 weeks upon follow up, the edema resolved 90% of patients will respond to oral steroids within 6– FEB 2015 MED
and urinalysis was normal. If this is a nephrotic 8 weeks, with most of these having a complete BOARDS;
syndrome, it fits the course of:
remission. TOPNOTCH MD
A. Membranoproliferative GN FROM UST)
B. Focal segmental glomerulosclerosis
C. Mesangial proliferation
D. Minimal change disease
E. IgA nephropathy
108 A 15 month old male presents with high fever of 3 SIMILAR TO PREVIOUS BOARD EXAM ANGELA DIAGNOSTIC
days, coryza, dry cough, red eyes with purulent CONCEPT/PRINCIPLE. Page 57-58 of Topnotch PAULINE P. EXAM - AUG
discharge and rashes appearing around the hairline Handout. This is measles. The spots at the level of the CALIMAG- 2015
spreading downwards. Discrete red lesions with premolars are koplik spots. LOYOLA (TOP 8 -
white spots are noted in the buccal mucosa at the FEB 2015 MED
level of the premolars. The most likely diagnosis is:
BOARDS;
A. Roseola infantum TOPNOTCH MD
B. Kawasaki disease FROM UST)
C. Rubeola
D. Rubella
E. Exanthem subitum
109 A 5 y/o girl with a two month history of SIMILAR TO PREVIOUS BOARD EXAM ANGELA DIAGNOSTIC
progressive difficulty in ambulation particularly in CONCEPT/PRINCIPLE. Page 15 of Topnotch Handout. PAULINE P. EXAM - AUG
climbing up and down the stairs. After going on a The rash over the periorbital area is not malar rash, it CALIMAG- 2015
field trip, she came home with an erythematous is a heliotrope rash. Its onset is insidious. Rash over LOYOLA (TOP 8 -
rash in the malar area and violaceous discoloration sun exposed areas appear first followed by proximal FEB 2015 MED
with swelling over the periorbital areas. On the day muscle weakness after 2 months. BOARDS;
of consult, she was noted to have papules over the TOPNOTCH MD
fingerjoints and elbows. The most likely diagnosis FROM UST)
is:
A. Juvenile dermatomyositis
B. Juvenile rheumatoid arthritis
C. Systemic lupus erthematosus
D. Juvenile ankylosing spondylitis
E. Systemic scleroderma
110 An 8 year old girl was brought in for declining This is an absence seizure. The DOC is Ethosuximide. ANGELA DIAGNOSTIC
academic performance. Her teacher reported that PAULINE P. EXAM - AUG
she is frequently inattentive in class. The doctor CALIMAG- 2015
hyperventilates her in the office and she manifests LOYOLA (TOP 8 -
with blank stares and clonic movements of the eyes, FEB 2015 MED
there was no post-ictal drowsiness noted. The BOARDS;
whole event lasted a mere 10 seconds. What is the TOPNOTCH MD
treatment of choice for this type of seizures?
FROM UST)
A. Valproic acid
B. Phenobarbital
C. Ethosuximide
D. Diazepam
E. None of the above
111 In the management of benign febrile seizure, which SIMILAR TO PREVIOUS BOARD EXAM ANGELA DIAGNOSTIC
of the following is advisable?
CONCEPT/PRINCIPLE. In benign febrile seizure the PAULINE P. EXAM - AUG
A. Do a lumbar tap and CSF analysis to rule out CNS most important management is education of the CALIMAG- 2015
infection mother. Lumbar tap, anticonvulsants, EEG and LOYOLA (TOP 8 -
B. Give an anticonvulsant to prevent recurrence CT/MRI are not indicated. FEB 2015 MED
C. Education of the mother to manage the baby on BOARDS;
next attack TOPNOTCH MD
D. Do an EEG as this might ba a seizure disorder FROM UST)
E. Do a CT/MRI to rule out a brain mass
112 A 10 y/o girl presents to your clinic with a history SIMILAR TO PREVIOUS BOARD EXAM ANGELA DIAGNOSTIC
of fever for 4 days, headache, muscle aches and CONCEPT/PRINCIPLE. The most important initial test PAULINE P. EXAM - AUG
anorexia. The fever lyses but she still feels very to do first is a CBC. CALIMAG- 2015
weak and complains of some epigastric pain. VS are LOYOLA (TOP 8 -
within the normal range. She appears flushed. What FEB 2015 MED
is the initial diagnostic test that you will request?
BOARDS;
A. CBC with platelet count TOPNOTCH MD
B. Dengue NS1 FROM UST)
C. PT, PTT, BT
D. Typhidot
E. Dengue blot
113 The lab results of the patient in the above case SIMILAR TO PREVIOUS BOARD EXAM ANGELA DIAGNOSTIC
showed: Hgb 142, Hct 0.48, WBC 2,000/mm3, CONCEPT/PRINCIPLE. Patient is in the critical phase PAULINE P. EXAM - AUG
lymphocytes 0.78, segmenters 0.22, platelet of illness and its pathophysiological hallmark is CALIMAG- 2015
180,000. Prologed PT. By this time you are increased capillary permeability with extravasation of LOYOLA (TOP 8 -
considering a diagnosis of Dengue. What is the fluids. FEB 2015 MED
pathophysiological hallmark of this phase of illness BOARDS;
she is in? TOPNOTCH MD
A. Rapid increase in platelet count FROM UST)
B. Fluid redistribution from the ECF to ICF
C. Progressive drop in platelet count
D. Increased capillary permeability with
extravasation of fluids
E. None of the above

TOPNOTCH MEDICAL BOARD PREP PEDIATRICS SUPEREXAM Page 18 of 97


For inquiries visit www.topnotchboardprep.com.ph or email us at topnotchmedicalboardprep@gmail.com
TOPNOTCH MEDICAL BOARD PREP PEDIATRICS SUPEREXAM
For inquiries visit www.topnotchboardprep.com.ph or email us at topnotchmedicalboardprep@gmail.com
Item QUESTION EXPLANATION AUTHOR TOPNOTCH
# EXAM
114 A 13 y/o boy came in alert, afebrile with a swelling SIMILAR TO PREVIOUS BOARD EXAM ANGELA DIAGNOSTIC
over the neck area. On examination, he was noted CONCEPT/PRINCIPLE. This is a case of diptheric PAULINE P. EXAM - AUG
to have a grayish white membrane on the nostrils croup. Page 52 of Topnotch Handout. Tx is PenG IM or CALIMAG- 2015
and pharynx which bled upon removal. Which IV 100,000-150,000 U/Kg/day every 6 hours for 14 LOYOLA (TOP 8 -
statement is true of this disease? days. Immunization with DPT still should be given. FEB 2015 MED
A. Specific treatment is penicillin for 5 days. BOARDS;
B. The most serious late complication is TOPNOTCH MD
tachycardia out of proportion to fever suggesting FROM UST)
cardiomyopathy and autonomic dysfunction.
C. Immunization is not required after treatment.
D. The most serious immediate complication is
septicemia.
E. All of the above.
115 Which of the following is characteristic of complex SIMILAR TO PREVIOUS BOARD EXAM ANGELA DIAGNOSTIC
febrile seizures:
CONCEPT/PRINCIPLE. Page 48 of Topnotch Handout. PAULINE P. EXAM - AUG
A. 1 episode in a 24 hour febrile period Duration greater than 15 ins, repeated convulsions in CALIMAG- 2015
B. Duration <15 minutes 24 hours, focal seizure activity often manifested by a LOYOLA (TOP 8 -
C. Preferential gaze towards the left side left preferential gaze. FEB 2015 MED
D. Generalized tonic-clonic movements BOARDS;
E. None of the above TOPNOTCH MD
FROM UST)
116 A 10 y/o girl presents with fever, headache, SIMILAR TO PREVIOUS BOARD EXAM ANGELA DIAGNOSTIC
malaise, she also refuses to eat and drink. On PE CONCEPT/PRINCIPLE. Page 60 of Topnotch Handout. PAULINE P. EXAM - AUG
you find painful vesicular lesions on the cheeks and Remember the difference between herpangina caused CALIMAG- 2015
gingival area, halitosis, and submandibular by coxsackie from herpetic gingivostomatitis caused LOYOLA (TOP 8 -
lymphadenitis. What is the most probable etiologic by HSV1 FEB 2015 MED
agent?
BOARDS;
A. Coxsackie A16 TOPNOTCH MD
B. Varicella Zoster virus FROM UST)
C. HSV-1
D. HSV-2
E. EBV
117 A newly born infant has her right arm extended and SIMILAR TO PREVIOUS BOARD EXAM ANGELA DIAGNOSTIC
her left arm flexed. If she has Erb-Duchenne palsy, CONCEPT/PRINCIPLE. Page 3 of Topnotch Handout. PAULINE P. EXAM - AUG
which of the ff is true?
Erb-duchenne palsy (C5-C6, upper trunk) affected arm CALIMAG- 2015
A. The baby will have an absent Moro on the right is adducted, pronated and internally rotated. Hence LOYOLA (TOP 8 -
side the baby will have an absent on the affected arm. FEB 2015 MED
B. Grasp reflex will be absent on the right hand BOARDS;
C. Miosis and anhydrosis may be present TOPNOTCH MD
D. The left arm should be partially immobilized FROM UST)
and properly positioned
E. All of the above
118 An LGA infant was born to an insulin dependent SIMILAR TO PREVIOUS BOARD EXAM ANGELA DIAGNOSTIC
diabetic mother. He presented with jitteriness, CONCEPT/PRINCIPLE. These infants may have PAULINE P. EXAM - AUG
hypotonia, temperature instability, seizures and periods of low blood sugar (hypoglycemia) shortly CALIMAG- 2015
lethargy after 24 hours what is the most likely after birth because of increased insulin level in their LOYOLA (TOP 8 -
cause?
blood due to compensation for the high blood sugar FEB 2015 MED
A. Hypocalcemia levels of the mother. BOARDS;
B. Hyperkalemia TOPNOTCH MD
C. Hyponatremia FROM UST)
D. Hypoglycemia
E. Hyperglycemia
119 A baby whose antenatal ultrasound showed renal SIMILAR TO PREVIOUS BOARD EXAM ANGELA DIAGNOSTIC
agenesis is prone to this type of congenital CONCEPT/PRINCIPLE. Due to the renal agenesis there PAULINE P. EXAM - AUG
anomaly: A. Deformation would be oligohydramnios causing deformation. CALIMAG- 2015
B. Dysplasia LOYOLA (TOP 8 -
C. Disruption FEB 2015 MED
D. Malformation BOARDS;
E. Syndrome TOPNOTCH MD
FROM UST)
120 A 5 y/o child was brought for consult due to 9 days SIMILAR TO PREVIOUS BOARD EXAM ANGELA DIAGNOSTIC
fever (39 C), generalized body malaise, anorexia, CONCEPT/PRINCIPLE.Page 75 of Topnotch Handout. PAULINE P. EXAM - AUG
and hepatomegaly 3 cms below the costal margin. This is typical of kawasaki disease. CALIMAG- 2015
He was extremely irritable, had bilateral LOYOLA (TOP 8 -
conjunctival injection, with severe dryness and FEB 2015 MED
fissuring of the lip, markedly red erythematous BOARDS;
tongue, right cervical lymphadenopathy (2cm) with TOPNOTCH MD
erthema and swelling of the hands and feet. What is FROM UST)
the most likely diagnosis?
A. Typhoid fever
B. Infectious mononucleosis
C. Scarlet fever
D. Dengue Fever
E. Kawasaki disease
121 A pediatric patient arrives at your clinic for a well- A one year old child can walk with one hand held, LYNN DARYL MIDTERM 1
baby check up. You observed that he can walk with rises independenly, says a few words besides "mama" FELICIANO EXAM - AUG
one-held, and plays a ball. What is the most likely and "dada", plays simple ball game, and makes VILLAMATER, MD 2015
age of the child? postural adjustment to dressing. (Nelson's). SIMILAR (TOP 5 - FEB
A. 8 months TO PREVIOUS BOARD EXAM CONCEPT. 2015 MED
B. 10 months BOARDS;
C. 1 year TOPNOTCH MD
D. 2 years FROM EAC)
E. 3 years

TOPNOTCH MEDICAL BOARD PREP PEDIATRICS SUPEREXAM Page 19 of 97


For inquiries visit www.topnotchboardprep.com.ph or email us at topnotchmedicalboardprep@gmail.com
TOPNOTCH MEDICAL BOARD PREP PEDIATRICS SUPEREXAM
For inquiries visit www.topnotchboardprep.com.ph or email us at topnotchmedicalboardprep@gmail.com
Item QUESTION EXPLANATION AUTHOR TOPNOTCH
# EXAM
122 Which of the following will show an increase in The lesions that cause left-to-right shunting (ASD, LYNN DARYL MIDTERM 1
pulmonary circulation? VSD, PDA) resuls in shunting of fully oxygenated blood FELICIANO EXAM - AUG
A. Left-to-right shunt disorders back into the lungs. The resulting increase volume of VILLAMATER, MD 2015
B. Right-to-left shunt disorders blood in the lungs decreases pulmonary compliance (TOP 5 - FEB
C. Hypoplastic right ventricle and increases the work of breathing. The chronic 2015 MED
D. Coarctation of aorta exposure of the pulmonary circulation to high BOARDS;
E. All except B pressure and blood flow results in a gradual increase TOPNOTCH MD
in pulmonary vascular resistance (Eisenmenger FROM EAC)
syndrome) (Source: Nelson's) SIMILAR TO PREVIOUS
BOARD EXAM CONCEPT.
123 A 12-year old female presented with a easy A harsh, blowing holosystolic murmur heard at the left LYNN DARYL MIDTERM 1
fatigability. On physical examination, there is a sternal border is characteristic of VSD. FELICIANO EXAM - AUG
blowing holosystolic murmur heard at the left VILLAMATER, MD 2015
sternal border. The most likely diagnosis is: (TOP 5 - FEB
A. Tetralogy of Fallot 2015 MED
B. Mitral regurgitation BOARDS;
C. Ventricular septal defect TOPNOTCH MD
D. Aortic insufficiency FROM EAC)
E. Pulmonary stenosis
124 A 2-year old male presented with frequent Tracheomalacia is a congenital tracheal obstruction LYNN DARYL MIDTERM 1
coughing and wheezing. Radiologic finding showed characterized by complete absence of cartilaginous FELICIANO EXAM - AUG
a collapsed trachea. The findings are suggestive of: rings. They presented with wheezing, coughing which VILLAMATER, MD 2015
A. Bronchiolitis generally increases with activity and colds. They (TOP 5 - FEB
B. Bronchial asthma usually maintain a normal oxygenation and 2015 MED
C. Cystic fibrosis disposition and grow and and develop normally, BOARDS;
D. Tracheomalacia despite the chronic airway noises. SIMILAR TO TOPNOTCH MD
E. Bronchopulmonary dysplasia PREVIOUS BOARD EXAM CONCEPT FROM EAC)

125 A 6 year-old male presented with low grade fever, This is a case of Erythema Infectiosus caused by LYNN DARYL MIDTERM 1
colds, flushed cheeks, and diffuse macular rashes of Parvovirus B19, most prevalent in ages 5-15 years old. FELICIANO EXAM - AUG
the trunk and extremities, with notable sparing of The hallmark sign is erythamatous facial flushing VILLAMATER, MD 2015
the palms and soles. His condition is most likely (slapped cheek appearance) and rashes that spreads (TOP 5 - FEB
caused by: rapidly to the trunk and proximal extremities as a 2015 MED
A. Varicella zoster virus diffuse macular erythema. SIMILAR TO PREVIOUS BOARDS;
B. Rubeola BOARD EXAM CONCEPT TOPNOTCH MD
C. Rubella FROM EAC)
D. HHV-6
E. Parvovirus
126 An infant born to a mother with active varicella at Newborns whose mothers develop varicella 5 days LYNN DARYL MIDTERM 1
delivery should immediately receive: before to 2 days after delivery should receive 1 vial of FELICIANO EXAM - AUG
A. VZ Ig varicella-zoster immune globulin (VariZIG). Dosage: VILLAMATER, MD 2015
B. Acyclovir 125 units/10 kg IM given within 96 hrs after (TOP 5 - FEB
C. Active vaccine against varicella exposure. Although neonatal varicella may occur in 2015 MED
D. High dose of Vitamin A about half of these infants despite administration of BOARDS;
E. MMR VariZIG, it is usually mild. Perinatally acquired TOPNOTCH MD
varicella may be life threatening, the infant should be FROM EAC)
treated with acyclovir (10 mg/kg every 8 hr IV) when
lesions develop. (Nelson's). SIMILAR TO PREVIOUS
BOARD EXAM CONCEPT
127 A child was delivered preterm via emergency CS to APGAR score=5; 8-10 good cardiopulmonary LYNN DARYL MIDTERM 1
a 35 year-old primigravid because of fetal distress. adaptation, 4-7 need for rescucitation esp. ventilatory FELICIANO EXAM - AUG
At the first 5 minute of life, he was noted to have a support; 0-3 immediate rescucitation. The APGAR VILLAMATER, MD 2015
heart rate of 90, cyanotic on the extremities, score helps to rapidly assess th need to rescucitate (TOP 5 - FEB
irregular respiration, weak tone, and grimaces with neonates after birth. SIMILAR TO PREVIOUS BOARD 2015 MED
suction. This indicates/predicts EXAM CONCEPT. BOARDS;
A. Low umbilicard cord pH TOPNOTCH MD
B. A need for rescucitation FROM EAC)
C. Presence of cerebral palsy
D. A poor neurologic outcome
E. Neonatal death
128 A 7 year-old boy arrives at the emergency This is a case of PSGN. Confirmation of the diagnosis LYNN DARYL MIDTERM 1
department due to hematuria and scanty urine of Post-streptococcus glomerulonephritis requires FELICIANO EXAM - AUG
output for the past 24 hours. Ten days prior to clear evidence of invasive streptococcal infection. A VILLAMATER, MD 2015
consult, he had fever and throat pain but did not rising antibody titer to streptococcal antigen confirms (TOP 5 - FEB
seek consult. Which of the following will be the a recent streptococcal infection. Importantly, 2015 MED
most important in his diagnosis. antistreptolysin O titer is commonly elevated after a BOARDS;
A. ASO 180 pharyngeal infection. (Nelson's). ASO normal level (0- TOPNOTCH MD
B. elevated ESR 100 IU/mL). Although decreased C3 is also diagnostic FROM EAC)
C. Decreased C3 of PSGN, other conditions may also present with
D. (+) RBC in the urine depressed (MPGN, Lupus nephritis). SIMILAR TO
E. Diminished renal function PREVIOUS BOARD EXAM CONCEPT.
129 Which of the following correctly describes Upper limit/Normal protein: Urine protein/creatinine LYNN DARYL MIDTERM 1
proteinuria? ratio (Upr/Ucr) <0.5 in children younger than 2 yrs FELICIANO EXAM - AUG
A. Urine protein/crea ratio of +0.15 and <0.2 in children 2 yrs or older (A ratio >3 suggest VILLAMATER, MD 2015
B. Urine protein of 0.15 g/24 hrs nephrotic range proteinuria); 150 mg/24 hr or (TOP 5 - FEB
C. Urine protein of 120 mg/24 hrs 0.15g/24 hr; Normal protein excretion: <4 mg/m2/hr 2015 MED
D. Urine protein of 20 mg/m2/hr (ABNORMAL 4-40 mg/m2/hr and NEPHROTIC RANGE BOARDS;
E. None of the above. is defined as >40 mg/m2/hr) (SOURCE:NELSON'S) TOPNOTCH MD
SIMILAR TO PREVIOUS BOARD EXAM CONCEPT. FROM EAC)

TOPNOTCH MEDICAL BOARD PREP PEDIATRICS SUPEREXAM Page 20 of 97


For inquiries visit www.topnotchboardprep.com.ph or email us at topnotchmedicalboardprep@gmail.com
TOPNOTCH MEDICAL BOARD PREP PEDIATRICS SUPEREXAM
For inquiries visit www.topnotchboardprep.com.ph or email us at topnotchmedicalboardprep@gmail.com
Item QUESTION EXPLANATION AUTHOR TOPNOTCH
# EXAM
130 Which of the following causes of diarrhea in All other causative agents warrants only supportive LYNN DARYL MIDTERM 1
children warrants antibiotic treatment? treatment such as hydration. S. aureus and B. cereus FELICIANO EXAM - AUG
A. Rotavirus symptoms are due to preformed toxin. Shigella may VILLAMATER, MD 2015
B. Shigella be treated with Cefixime or Erythromycin. (TOP 5 - FEB
C. Staphylococcus aureus (Cotrimoxazole in IMCI). SIMILAR TO PREVIOUS 2015 MED
D. Bacillus cereus BOARD EXAM CONCEPT. BOARDS;
E. All of the above TOPNOTCH MD
FROM EAC)
131 A 10-year-old obese female presented with T1DM is characterized by low or absent level of LYNN DARYL MIDTERM 1
polyuria, polydypsia, and hyperpigmentation of the endogenously produced insulin. T2Dm is FELICIANO EXAM - AUG
skin of the beck arrives at your clinic for consult. chacaracterized by insulin resistance. Acanthosis VILLAMATER, MD 2015
Laboratory findings showed a high insulin level, nigrican or dark pigmentation of skin creases/flexural (TOP 5 - FEB
normal C peptide and FBS of 186 mg/dL. The most areas, a sign of insulin resistance, is present in the 2015 MED
likely diagnosis is: majority of patients with T2DM and is accompanied BOARDS;
A. Impaired glucose tolerance by a relative hyperinsulinemia at the time of TOPNOTCH MD
B. Type I DM diagnosis. Impaired glucose tolerance refers to a FROM EAC)
C. Type II DM metabolic stage that is intermediate between normal
D. Hypercortisolism glucose homeostasis and diabetes. (FBS 100-125
E. Cushing's Syndrome mg/dL). Source: Nelson's. SIMILAR TO PREVIOUS
BOARD EXAM CONCEPT.
132 A 12-year old female presented with short stature, Emotional deprivation is an important cause of LYNN DARYL MIDTERM 1
voracious appetite, and delayed sexual maturity. retardation of growth and mimics hypopituitarism. FELICIANO EXAM - AUG
You would suspect: The condition is known as psychosocial dwarfism, VILLAMATER, MD 2015
A. Hypopituitarism maternal deprivation dwarfiss, or hyperphagic short (TOP 5 - FEB
B. Hypothyroidism stature. Emotionally deprived children frquently have 2015 MED
C. Constitutional short stature perverted or voracious appetites, enuresis, encopresis, BOARDS;
D. Psychosocial dwarfism insomnia, crying spasms, and sudden tantrums. TOPNOTCH MD
E. Growth hormone insensitivity (Nelson's). Deprivation dwarfism is described as a FROM EAC)
triad of extreme short stature, voracious appetite, and
markedly delayed sexual maturation. SIMILAR TO
PREVIOUS BOARD EXAM CONCEPT.
133 A 7-year old female patient had recent history of Pneumonia due to S. aureus may be primary LYNN DARYL MIDTERM 1
cough, colds, and fever which lasted for 1 week. (hematogenous) or secondary after a viral infection FELICIANO EXAM - AUG
However, after a few days, patient presented with such as influenza. More common are high fever, VILLAMATER, MD 2015
recurrrence of cough, now with high fever, extreme abdominal pain, tachypnea, dyspnea, and localized or (TOP 5 - FEB
dyspnea and tachypnea. Physical findings revealed diffuse bronchopneumonia or lobar disease. S. aureus 2015 MED
inspiratory crackles with decreased breath sound often causes a necrotizing pneumonitis; empyema, BOARDS;
in the right lower lung field. Chest radiograph PNEUMATOCOELE pyopneumothorax, and TOPNOTCH MD
revealed pneumatocele. Her condition now is most bronchopleural fistulas develop frequently. FROM EAC)
likely caused by: Staphylococcal pneumonia occasionally produces a
A. Pseudomonas aeruginosa diffuse interstitial disease characterized by extreme
B. Streptococcus pneumoniae dyspnea, tachypnea, and cyanosis. (Nelson's).
C. Mycoplasma pneumoniae SIMILAR TO PREVIOUS BOARD EXAM CONCEPT.
D. Staphylococcus aureus
E. Haemophilus influenzae
134 The empiric treatment for neonatal sepsis consists SIMILAR TO PREVIOUS BOARD EXAM CONCEPT. LYNN DARYL MIDTERM 1
of? FELICIANO EXAM - AUG
A. Cloxacillin + Aminoglycoside VILLAMATER, MD 2015
B. Ampicillin + Sulfamethoxazole (TOP 5 - FEB
C. Aminoglycoside + Amplicillin 2015 MED
D. Erythromycin + Cloxacillin BOARDS;
E. Imipinem + Cilastatin TOPNOTCH MD
FROM EAC)
135 A 9 year old male patient was brought to your clinic TB disease : 3 or more of the following criteria: 1. LYNN DARYL MIDTERM 1
due to cough for more than a month accompanied Exposure to an adult/adolescent with active TB FELICIANO EXAM - AUG
by marked weight loss. Her mother also has disease; 2. (+) PPD; 3. s/sx suggestive of TB; 4. (+) VILLAMATER, MD 2015
chronic cough. If Mantoux test shows an induration Abnormal cxr findings; 5. Lab findings. TB infection : (TOP 5 - FEB
of 10 mm, the most likely diagnosis is: +/- exposure, (+) PPD, no s/sx, (-) cxr findings 2015 MED
A. TB disease BOARDS;
B. TB infection TOPNOTCH MD
C. TB exposure FROM EAC)
D. Latent TB
E. Miliary TB
136 A 17-year old female presented with itching and Herpes simplex virus-2 is the cause of genital herpes. LYNN DARYL MIDTERM 1
pain on the genital area. On examination, she had Most primary genital herpes simplex virus infections FELICIANO EXAM - AUG
vesicular lesions over the genital area. Based on the are asymptomatic. Lesions evolve from vesicles to VILLAMATER, MD 2015
presentation, this is most likely caused by: pustules to wet ulcers and heal by crusting. Lesions (TOP 5 - FEB
A. HSV-1 occur on the labia, mons pubis, vaginal mucosa, cervix 2015 MED
B. HSV-2 and on the shaft of the penis. Inguinal BOARDS;
C. VZV lymphadenopathy, dysuria, and vaginal discharge are TOPNOTCH MD
D. Neisseria gonorrhea frequent complaints. SIMILAR TO PREVIOUS BOARD FROM EAC)
E. Chlamydia trachomatis EXAM CONCEPT.
137 A 16 -year-old male presented with painful lesion in Lymphogranuloma is a sexually transmitted disease LYNN DARYL MIDTERM 1
the inguinal node with "groove sign". This is a case caused by Chlamydia trachomatis. Inguinal syndrome FELICIANO EXAM - AUG
of: occurs 2-6 weeks after exposure, and presents with VILLAMATER, MD 2015
A. Lymphogranuloma venereum large circular lymph nodes (buboes), which are (TOP 5 - FEB
B. Donovanosis painful, tender inguinal or femoral lymph adenopthy. 2015 MED
C. Chancroid SIMILAR TO PREVIOUS BOARD EXAM CONCEPT. BOARDS;
D. Herpes Genitalis TOPNOTCH MD
E. Early syphilis FROM EAC)

TOPNOTCH MEDICAL BOARD PREP PEDIATRICS SUPEREXAM Page 21 of 97


For inquiries visit www.topnotchboardprep.com.ph or email us at topnotchmedicalboardprep@gmail.com
TOPNOTCH MEDICAL BOARD PREP PEDIATRICS SUPEREXAM
For inquiries visit www.topnotchboardprep.com.ph or email us at topnotchmedicalboardprep@gmail.com
Item QUESTION EXPLANATION AUTHOR TOPNOTCH
# EXAM
138 Which of the following sign/s point to a primary 10 warning signs of primary immunodeficiency: 4 or LYNN DARYL MIDTERM 1
immunodeficiency problem in children? more new ear infections within 1 year, 2 or more FELICIANO EXAM - AUG
A. More than 2 pneumonias in a year serious sinus infections within 1 year; 2 or more VILLAMATER, MD 2015
B. More than 4 new ear infections in 1 year months on antibiotics with little effect; 2 or more (TOP 5 - FEB
C. Two or more serious sinus infections within 1 pneumonias within 1 year; Failure of an infant to gain 2015 MED
year. weight or grow normally; recurrent, deep skin or BOARDS;
D. Sepsis 3x a year organ abscess; persistent thrush in mouth or fungal TOPNOTCH MD
E. All of the above infection on skin; need for IV antibiotics to clear FROM EAC)
infections; 2 or more deep seated infections including
septicemia; a family history of primary
immunodeficiency. SIMILAR QUESTION IN THE
BOARDS.
139 A 10 year-old patient presented with retro-orbital Complete blood count will be helpful in determining LYNN DARYL MIDTERM 1
pain, nausea, vomiting, abdominal pain, and 4-day presence/absence of thrombocytopenia and FELICIANO EXAM - AUG
history of high grade fever. Which of the following hemoconcentration as these are part of clinical VILLAMATER, MD 2015
diagnostic test will you request first? diagnosis of Dengue fever. Note: 4-5 stem questions (TOP 5 - FEB
A. Typhidot were asked about Dengue in the Feb 2015 boards. 2015 MED
B. Dengue DNS-1 antigen BOARDS;
C. Dengue IgG TOPNOTCH MD
D. Complete blood count FROM EAC)
E. LeptoMAT
140 A 4 week old female was brought to your clinic with Biliary atresia is characterized by progressive fibrosis LYNN DARYL MIDTERM 1
parental reports of increasing jaundice over the last and obliteration of extra- and intra-hepatic bile ducts. FELICIANO EXAM - AUG
week. Her parents reported that 2 weeks ago, she Usually, patient is born at term with normal birth VILLAMATER, MD 2015
began to have yellowing of her eyes with weight. Jaundice can be present at birth or it be (TOP 5 - FEB
subsequent yellowing of her skin. Her stools have present as late as 3-5 weeks of life. Acholic stool is 2015 MED
been pale in color for the past 10 days. On history, another common complaints, highly suggestive of BOARDS;
she was born by spontaneous vaginal delivery to a cholestasis. On physical exam, patient will have an TOPNOTCH MD
G2P1 withno complicated noted at birth or at enlarged liver. Lab exams will show an elevated TB, FROM EAC)
discharge. The patient was not jaundiced at increased DB, normal or elevated aminotransferases,
discharge. What is your primary impression? and highly elevated Alk phos. Choledochal cyst may
A. Biliary atresia present with intermittent jaundice. Symptoms of
B. Choledochal cyst duodenal atresia may be present right after birth. BF
C. Breastfeeding jaundice jaundice and breastmilk jaundice are less likely to
D. Duodenal atresia cause progressive jaundice. Note: SIMILAR TO
E. Breastmilk jaundice PREVIOUS BOARD EXAM CONCEPT
141 A 2 year old child came at the ER due to convulsions Secure and protect the airway first before doing any EDWARD HARRY MIDTERM 2
and upon PE, you noted the child is in active other procedures by giving supplemental oxygen and VALLAJERA, MD EXAM - AUG
seizure, the mother told you that the child had fever placing the patient on his side to prevent aspiration (TOP 8 - FEB 2015
prior to the seizure episode, the first step you 2015 MED
should do as the ER ROD is: BOARDS;
A. Give diazepam 0.2mg/kg/dose TOPNOTCH MD
B. Place tongue guard FROM
C. Give anti pyretics PERPETUAL
D. Secure the airway BINAN)
E. None of the above
142 After the seizure episode, the mother asked about Benign febrile seizures are benign episodes of seizures EDWARD HARRY MIDTERM 2
his son's condition, you tell her that benign febrile related to elevated body temperature, therefore VALLAJERA, MD EXAM - AUG
seizures are educating the mother about the use of antipyretics (TOP 8 - FEB 2015
A. A serious illness that should be seen by a and cooling will keep her calm during such attacks. 2015 MED
neurologist for prescription of maintenance BOARDS;
medications TOPNOTCH MD
B. A benign episode and should tell her that she FROM
needs to give the child antipyretics and keep him PERPETUAL
cool during febrile episodes BINAN)
C. An isolated case that will never happen again
D. A sign of serious CNS infection
E. None of the above
143 SAO, a 4 year old boy was brought to the ER due to The case is ALL, the treatment is remission induction EDWARD HARRY MIDTERM 2
bleeding from the gums, CBC was done revealing with vincristine, prednisone and L-aspariginase. VALLAJERA, MD EXAM - AUG
WBC of 35,000, ANC was 4,200, BM aspiration (TOP 8 - FEB 2015
showed >25% of BM cells are lymphoblasts, what is 2015 MED
the treatment? BOARDS;
A. Chemotherapy TOPNOTCH MD
B. Radiation therapy FROM
C. All trans-retinoic acid PERPETUAL
D. Plasmapheresis BINAN)
E. None of the above
144 MO, a 7 year old male was brought by her mother Anti-DNAse B is the antibody titer test of choice for EDWARD HARRY MIDTERM 2
due to passage of cola like urine, upon careful documenting antibody titers of streptococcal skin VALLAJERA, MD EXAM - AUG
history, the patient had a skin infection 1 month infection, ASO is for throat infection. (TOP 8 - FEB 2015
prior. What is the diagnostic test of choice to 2015 MED
document the antibody titer of the skin infection. BOARDS;
A. Serum C3 TOPNOTCH MD
B. ASO titer FROM
C. DNAse B PERPETUAL
D. Serum IgA BINAN)
E. None of the above

TOPNOTCH MEDICAL BOARD PREP PEDIATRICS SUPEREXAM Page 22 of 97


For inquiries visit www.topnotchboardprep.com.ph or email us at topnotchmedicalboardprep@gmail.com
TOPNOTCH MEDICAL BOARD PREP PEDIATRICS SUPEREXAM
For inquiries visit www.topnotchboardprep.com.ph or email us at topnotchmedicalboardprep@gmail.com
Item QUESTION EXPLANATION AUTHOR TOPNOTCH
# EXAM
145 You are the medical intern assigned at the NICU, The one minute APGAR score signifies the need to EDWARD HARRY MIDTERM 2
you know that the importance of the 1 minute perform resuscitative measures to save the life of the VALLAJERA, MD EXAM - AUG
APGAR score is? newborn while the other options are the implications (TOP 8 - FEB 2015
A. The risk of development of neurologic sequelae of the 5 minute APGAR score 2015 MED
B. The adequacy of resuscitation BOARDS;
C. The need for resuscitation TOPNOTCH MD
D. Both A and B FROM
E. None of the above PERPETUAL
BINAN)
146 The vaccine BCG is made up of what BCG is a live attenuated form of Mycobacterium bovis EDWARD HARRY MIDTERM 2
A. Purified protein derivative of Mycobacterium that is grown on the laboratory and has similar VALLAJERA, MD EXAM - AUG
tuberculosis antigenic properties as Mycobacterium tuberculosis (TOP 8 - FEB 2015
B. Live attenuated form of Mycobacterium 2015 MED
tuberculosis BOARDS;
C. Killed Mycobacterium tuberculosis TOPNOTCH MD
D. Antigenic parts of Mycobacterium tuberculosis FROM
E. Attenuated live Mycobacterium bovis PERPETUAL
BINAN)
147 A child was brought to your clinic for a wellness At this age, the patient says dada/mama, the child's EDWARD HARRY MIDTERM 2
check up, you noted that the child walks held on first words, is assisted to walk but still can't walk VALLAJERA, MD EXAM - AUG
one hand by her mother, who knows mama and independently which is cruising. (TOP 8 - FEB 2015
dada. You know that the approximate age of this 2015 MED
child is? BOARDS;
A. 10 months TOPNOTCH MD
B. 12 months FROM
C. 24 months PERPETUAL
D. 48 months BINAN)
E. 36 months
148 KO, a 5 year old child was brought to your clinic This is a case of german measles and the causative EDWARD HARRY MIDTERM 2
because of fever, you noted the presence of fever at agent is a RNA virus from the Togaviridae family VALLAJERA, MD EXAM - AUG
37.8C and swollen posterior auricular nodes (TOP 8 - FEB 2015
without colds or photophobia, there were 2015 MED
maculopapular rashes all over the body, you know BOARDS;
that the most likely causative agent is: TOPNOTCH MD
A. Paramyxoviridae FROM
B. Togaviridae PERPETUAL
C. Human herpes virus 6 BINAN)
D. Herpes zoster
E. Coxsackie virus
149 A 3 year old child was brought to your clinic due to This is a case of PKU, the enzyme defective in this case EDWARD HARRY MIDTERM 2
strong body odor noted by the mother, no newborn is Phenylalanine hydroxylase causing buildup of VALLAJERA, MD EXAM - AUG
screening was done. Upon physical exam, you noted phenylalanine. This causes the formation of (TOP 8 - FEB 2015
that the child had strong musty odor. You know phenylacetate, phenylpyruvate and phenethylamine 2015 MED
that the defective enzyme in this patient is? which causes the characteristic musty body odor. BOARDS;
A. Phenylethanolamine N-methyltransferase TOPNOTCH MD
B. Dopamine B-hydroxylase FROM
C. Tyrosine hydroxylase PERPETUAL
D. Phenylalanine hydroxylase BINAN)
E. None of the above
150 SO, a 15 year old male came into your clinic due to This is a case of juvenile AS, supported by EDWARD HARRY MIDTERM 2
joint pains, you noted that the joint pains occur at oligoarthritis, (+) x-ray of sacroiliitis and (+) HLA B27 VALLAJERA, MD EXAM - AUG
the lower back, hips, knees and left ankle, x-ray of (TOP 8 - FEB 2015
the lumbar spine shows sacroiliitis, labs showed 2015 MED
slight elevation of ESR and (+) HLA B27. What is the BOARDS;
most likely diagnosis? TOPNOTCH MD
A. Juvenile Ankylosing Spondylitis (Juvenile FROM
Spondyloarthropathy) PERPETUAL
B. Juvenile Rheumatic Arthritis (Juvenile Idiopathic BINAN)
Arthritis)
C. Systemic Lupus Erythrematosus
D. Juvenile Dermatomyositis
E. None of the above
151 FS, a 5 year old female was brought by her mother This is a case of Henoch-Schonlein Purpura. The EDWARD HARRY MIDTERM 2
due to reddish purpuric lesions on her legs, upon characteristic of the rash as well as a prior bout of VALLAJERA, MD EXAM - AUG
PE, heart sounds were unremarkable, you noted URTI and HLA DRB1*07 (+) is suggestive of this case. (TOP 8 - FEB 2015
that the purpura is palpable and mostly located on 2015 MED
the legs and there is also knee pain, she also tested BOARDS;
positive for HLA-DRB1*07 you noted that prior to TOPNOTCH MD
the rashes' appearance, she had a bout of URTI, as FROM
her pediatrician, you know that she most likely has: PERPETUAL
A. Dengue Hemorrhagic Fever BINAN)
B. Kawasaki disease
C. Henoch-Schonlein purpura
D. Thrombotic Thrombocytopenic purpura
E. None of the above
152 A newborn was born to a mother known to have This is a case of neonatal lupus and the most dreaded EDWARD HARRY MIDTERM 2
SLE, PE findings include annular or macular rash complication from this is congenital heart block due to VALLAJERA, MD EXAM - AUG
typically affecting the face especially the periorbital Ro and La antigens that may be exposed on the (TOP 8 - FEB 2015
area. The most dreaded complication from this surface of cardiac cells in the proximity of the 2015 MED
disease is: atrioventricular node, thus making these antigens BOARDS;
A. Lupus nephritis accessible to maternal autoantibodies. Binding incites TOPNOTCH MD
B. Congenital heart block a local immune response, resulting in fibrosis within FROM
C. Serositis the conduction system. Lupus nephritis is seen in PERPETUAL
D. Arthritis older children and adults who manifest lupus later in BINAN)
E. None of the above life. (Nelson 19th ed., pp 845-846)

TOPNOTCH MEDICAL BOARD PREP PEDIATRICS SUPEREXAM Page 23 of 97


For inquiries visit www.topnotchboardprep.com.ph or email us at topnotchmedicalboardprep@gmail.com
TOPNOTCH MEDICAL BOARD PREP PEDIATRICS SUPEREXAM
For inquiries visit www.topnotchboardprep.com.ph or email us at topnotchmedicalboardprep@gmail.com
Item QUESTION EXPLANATION AUTHOR TOPNOTCH
# EXAM
153 An 18 month old child was brought to your clinic This is a case of MSUD in which there is a deficiency of EDWARD HARRY MIDTERM 2
because of developmental delay, you noted a sweet the enzyme branched-chain alpha-keto acid VALLAJERA, MD EXAM - AUG
smelling urine similar to a maple syrup, you know dehydrogenase complex causing accumulation of the (TOP 8 - FEB 2015
that the enzyme deficient in this case metabolizes above amino acids causing the characteristic sweet, 2015 MED
which amino acids maple odor urine. BOARDS;
A. Leucine, lysine, valine TOPNOTCH MD
B. Isoleucine, phenylalanine, tryptophan FROM
C. Leucine, isoleucine, aspartate PERPETUAL
D. Valine, leucine, isoleucine BINAN)
E. Phenylalanine, tryptophan, glycine
154 Which of the following is true regarding MHC All of the options are properties of MHC proteins EDWARD HARRY MIDTERM 2
proteins VALLAJERA, MD EXAM - AUG
A. Are involved in marking body cells as being “self” (TOP 8 - FEB 2015
B. Can be found on virtually all cells (Type I) 2015 MED
C. Can be found on antigen-presenting cells (Type BOARDS;
II) TOPNOTCH MD
D. Both A and B FROM
E. All of the above PERPETUAL
BINAN)
155 Proteins on the surface of mitral valve cells This is an example of molecular mimicry as the M EDWARD HARRY MIDTERM 2
resemble proteins on the surface of the bacterium protein of strep pyogenes causes rheumatic heart VALLAJERA, MD EXAM - AUG
streptococcus pyogenes. This: disease. (TOP 8 - FEB 2015
A. Is beneficial because it will help increase the 2015 MED
body’s anti-bacterial defenses. BOARDS;
B. Can result in the production of antibodies that TOPNOTCH MD
target mitral valve cells FROM
C. Is an example of cell-mediated immunity PERPETUAL
D. Is an example of passive artificial humoral BINAN)
immunity
E. None of the above
156 The only antibody that could attack the Rh The Rh blood group in the fetus would be attacked by EDWARD HARRY MIDTERM 2
complexes on fetal red blood cells is the: Rh IgG from the mother. VALLAJERA, MD EXAM - AUG
A. IgA (TOP 8 - FEB 2015
B. IgD 2015 MED
C. IgE BOARDS;
D. IgG TOPNOTCH MD
E. IgM FROM
PERPETUAL
BINAN)
157 Which of the following is a possible result of All of the following are possible consequences of EDWARD HARRY MIDTERM 2
activated complement activated complement VALLAJERA, MD EXAM - AUG
A. Formation of a membrane attack complex (TOP 8 - FEB 2015
B. Lysis of bacterial cells 2015 MED
C. Opsonization BOARDS;
D. Inflammation TOPNOTCH MD
E. All of the above FROM
PERPETUAL
BINAN)
158 The anterior pituitary gland is an embryologic The anterior pituitary gland is an invagination of the EDWARD HARRY MIDTERM 2
remnant of what germ layer? ectoderm of the mouth. VALLAJERA, MD EXAM - AUG
A. Endoderm (TOP 8 - FEB 2015
B. Mesoderm 2015 MED
C. Ectoderm BOARDS;
D. Yolk sac TOPNOTCH MD
E. None of the above FROM
PERPETUAL
BINAN)
159 The superior parathyroid glands are derived from The superior parathyroid and the C cells of the thyroid EDWARD HARRY MIDTERM 2
which pharyngeal pouch develop from the 4th pouch while the inferior VALLAJERA, MD EXAM - AUG
A. 1st parathyroid and the thymus develop from the 3rd (TOP 8 - FEB 2015
B. 2nd pouch 2015 MED
C. 3rd BOARDS;
D. 4th TOPNOTCH MD
E. 5th FROM
PERPETUAL
BINAN)
160 Which of the following is not true regarding All of the following statements about EDWARD HARRY MIDTERM 2
Immunoglobulins immunoglobulins are true. VALLAJERA, MD EXAM - AUG
A. IgA is a dimer often found in the same secretions (TOP 8 - FEB 2015
as lysozyme 2015 MED
B. IgG can be involved in hemolytic disease of the BOARDS;
newborn TOPNOTCH MD
C. IgM and IgG can activate complement FROM
D. IgD can be found acting as a receptor protein on PERPETUAL
the B cell plasma membrane BINAN)
E. None of the above

TOPNOTCH MEDICAL BOARD PREP PEDIATRICS SUPEREXAM Page 24 of 97


For inquiries visit www.topnotchboardprep.com.ph or email us at topnotchmedicalboardprep@gmail.com
TOPNOTCH MEDICAL BOARD PREP PEDIATRICS SUPEREXAM
For inquiries visit www.topnotchboardprep.com.ph or email us at topnotchmedicalboardprep@gmail.com
Item QUESTION EXPLANATION AUTHOR TOPNOTCH
# EXAM
161 Aspirin is usually contraindicated in children these 3 diseases are the only indication for giving ASA HAROLD JAY S. MIDTERM 3
because of the risk of developing Reye's syndrome. in children BAYTEC, MD EXAM - AUG
However, there are certain diseases in which the (TOP 10 - FEB 2015
benefit outweighs the risk. Which of among the 2015 MED
following diseases can you use ASA? BOARDS;
A. Kawasaki TOPNOTCH MD
B. Rheumatic Fever FROM FEU)
C. JRA
D. A and B
E. all of the above
162 In diphtheria, the most common cause of death of HAROLD JAY S. MIDTERM 3
patient is usually: BAYTEC, MD EXAM - AUG
A. pneumonia (TOP 10 - FEB 2015
B. myocarditis 2015 MED
C. sepsis BOARDS;
D. meningitis TOPNOTCH MD
E. Renal failure FROM FEU)

163 A premature newborn was just delivered vaginally. Color 1, HR 2, irritability 2, activity 0, respiration 1. HAROLD JAY S. MIDTERM 3
You noticed that his body is pink but the body pink but extremities are blue- 1; HR 115- 2; BAYTEC, MD EXAM - AUG
extremities are blue, HR of 115, limp, with good limp- 0; good cough- 2; slow and irregular respiration- (TOP 10 - FEB 2015
cough but slow and irregular respiration. What is 1... A good coughing reflex can also mean that the 2015 MED
the APGAR score of the baby? patient has good irritability BOARDS;
A. 4 TOPNOTCH MD
B. 5 FROM FEU)
C. 6
D. 7
E. 8
164 It is a major criteria in the diagnosis of Rheumatic SIMILAR TO PREVIOUS BOARD EXAM HAROLD JAY S. MIDTERM 3
Fever that can stand alone and does need ASO titer. CONCEPT/PRINCIPLE BAYTEC, MD EXAM - AUG
A. Migratory arthritis (TOP 10 - FEB 2015
B. Myocarditis 2015 MED
C. Subcutaneous nodules BOARDS;
D. Erythema marginatum TOPNOTCH MD
E. Sydenham's chorea FROM FEU)

165 Definitive diagnosis of infective endocarditis is give HAROLD JAY S. MIDTERM 3


if it fulfills: BAYTEC, MD EXAM - AUG
A. 2 major criteria (TOP 10 - FEB 2015
B. One major and 3 minor criteria 2015 MED
C. 5 minor criteria BOARDS;
D. A and C TOPNOTCH MD
E. All of the above FROM FEU)

166 A 14 year old patient came in at your clinic for for individual >13 years old and without immunity, HAROLD JAY S. MIDTERM 3
check up. He told you that he has not had chicken they should receive 2 doses of the vaccine with at least BAYTEC, MD EXAM - AUG
pox before and he wants to get vaccinated. How will 4 weeks apart. (TOP 10 - FEB 2015
you give the vaccine? 2015 MED
A. 1 dose only BOARDS;
B. 2 doses with at least 4 weeks apart TOPNOTCH MD
C. 2 doses with at least 6 months apart FROM FEU)
D. 3 doses with at least 4 weeks apart
E. 3 doses with at least 6 weeks apart
167 The last dose of the Rotavirus vaccination should HAROLD JAY S. MIDTERM 3
NOT be administered to infants older than how BAYTEC, MD EXAM - AUG
many weeks? (TOP 10 - FEB 2015
A. 24 2015 MED
B. 26 BOARDS;
C. 28 TOPNOTCH MD
D. 30 FROM FEU)
E. 32

168 Measles vaccine may be given as early as how many HAROLD JAY S. MIDTERM 3
months specially in cases of epidemics? BAYTEC, MD EXAM - AUG
A. 5 (TOP 10 - FEB 2015
B. 6 2015 MED
C. 7 BOARDS;
D. 8 TOPNOTCH MD
E. 9 FROM FEU)

169 in the 2014 guidelines of PPS, all of the following EPI includes BCG, DTwP-Hib-Hepatitis B, OPV, HAROLD JAY S. MIDTERM 3
are considered part of the EPI except: measles,MMR, rotavirus, PCV, and Td BAYTEC, MD EXAM - AUG
A. MMR (TOP 10 - FEB 2015
B. measles 2015 MED
C. Rotavirus vaccine BOARDS;
D. Infuenza TOPNOTCH MD
E. Td FROM FEU)

170 The WHO and DOH both recommend the use of the HAROLD JAY S. MIDTERM 3
following regimen in deworming children: BAYTEC, MD EXAM - AUG
A. Use of albendazole 200mg single dose every 6 (TOP 10 - FEB 2015
month for ages 12 to 24 months 2015 MED
B. Use of albendazole 400mg single dose every 6 BOARDS;
month for ages 24 months and above TOPNOTCH MD
C. use of mebendazole 500mg single dose every 6 FROM FEU)
months for ages 12 months and above
D. A and C
TOPNOTCH MEDICAL BOARD PREP PEDIATRICS SUPEREXAM Page 25 of 97
For inquiries visit www.topnotchboardprep.com.ph or email us at topnotchmedicalboardprep@gmail.com
TOPNOTCH MEDICAL BOARD PREP PEDIATRICS SUPEREXAM
For inquiries visit www.topnotchboardprep.com.ph or email us at topnotchmedicalboardprep@gmail.com
Item QUESTION EXPLANATION AUTHOR TOPNOTCH
# EXAM
E. all of the above

171 Vitamin A supplementation for infants 6-11 months 100,000 IU for infants 6-11 months and 200,000 IU for HAROLD JAY S. MIDTERM 3
is given at a single dose of _______IU as children 12-59 months. BAYTEC, MD EXAM - AUG
recommended by PPS and DOH. (TOP 10 - FEB 2015
A. 50,000 2015 MED
B. 100,000 BOARDS;
C. 150,000 TOPNOTCH MD
D. 200,000 FROM FEU)
E. 300,000

172 Deworming is contraindicated to children with the only high grade fever is contraindicated. HAROLD JAY S. MIDTERM 3
following conditions except: BAYTEC, MD EXAM - AUG
A. Severe malnutrition (TOP 10 - FEB 2015
B. Profuse diarrhea 2015 MED
C. Abdominal pain BOARDS;
D. Low grade fever TOPNOTCH MD
E. Serious illness FROM FEU)

173 The recommended Breastmilk storage period is: Based on the Philippine Pediatric Society Preventive HAROLD JAY S. MIDTERM 3
A. 1 hour at room temperature more than 25C Pediatric Health Care Handbook page 24, the ONLY BAYTEC, MD EXAM - AUG
B. 10 days in a refrigerator answer is A but, cross referencing it to pedia topnotch (TOP 10 - FEB 2015
C. 3 weeks in a freezer of a 1 door refrigerator handout, E can also be considered. If CDC guidelines 2015 MED
D. 4 months in a freezer of a 2 door refrigerator will be used, A D and E are the answers. BOARDS;
E. 7 months in a deep freezer with constant TOPNOTCH MD
temperature of -20C FROM FEU)

174 The following factors indicate worse prognosis in a t(12:21) usually indicates better prognosis. the HAROLD JAY S. MIDTERM 3
patient with Acute Lymphoblastic Leukemia presence of t(9:22) in ALL means that it has also a BAYTEC, MD EXAM - AUG
EXCEPT: CML property which makes the case worse. Robbins (TOP 10 - FEB 2015
A. Age under 2 8th ed page 603. Factors associated with worse 2015 MED
B. Presentation in adolescence prognosis are (1) age under 2; (2) presentation in BOARDS;
C. Peripheral blood blast greater than 100,000 adolescence or adulthood; (3) peripheral blood blast TOPNOTCH MD
D. Presence of Philadelphia chromosome or counts greater than 100000; (4) the presence of FROM FEU)
t(9;22) Philadelphia chromosome. Favorable prognostic
E. translocation of chromosome 12 and 21 or markers include (1) an age of 2 to 10; (2) a low white
t(12;21) cell count; (3) hyperploidy; (4) trisomy of
chromosomes 4, 7, and 10, and; (5) the presence of a
t(12:21)
175 a 15 year old male came in to your clinic SIMILAR TO PREVIOUS BOARD EXAM HAROLD JAY S. MIDTERM 3
complaining of deep knee pain which often cause CONCEPT/PRINCIPLE BAYTEC, MD EXAM - AUG
night time awakening. X ray was done and revealed (TOP 10 - FEB 2015
a sunburst pattern. The most likely diagnosis of the 2015 MED
patient is: BOARDS;
A. Ewing sarcoma TOPNOTCH MD
B. osteosarcoma FROM FEU)
C. Rhabdomyosarcoma
D. osteochondroma
E. chondroblastoma
176 A child was rushed by bystanders to the ER after a age in years x 2+8 HAROLD JAY S. MIDTERM 3
vehicular accident. Her preschool ID indicates that BAYTEC, MD EXAM - AUG
she is 4 years old. What is the approximate weight (TOP 10 - FEB 2015
of the patient in kilograms? 2015 MED
A. 12 BOARDS;
B. 14 TOPNOTCH MD
C. 16 FROM FEU)
D. 18
E. 20
177 to diagnose Systemic Lupus Erythematosus in a only four criteria are needed HAROLD JAY S. MIDTERM 3
patient, you need to satisfy how many of the BAYTEC, MD EXAM - AUG
criteria? (TOP 10 - FEB 2015
A. 3 2015 MED
B. 4 BOARDS;
C. 6 TOPNOTCH MD
D. 7 FROM FEU)
E. 8
178 Bird's beak sign, inverted U sign and coffee bean HAROLD JAY S. MIDTERM 3
sign are seen in abdominal Xray with barium in BAYTEC, MD EXAM - AUG
what disease? (TOP 10 - FEB 2015
A. Pyloric stenosis 2015 MED
B. Duodenal atresia BOARDS;
C. intussusception TOPNOTCH MD
D. Volvulus FROM FEU)
E. Hirschprung's disease

TOPNOTCH MEDICAL BOARD PREP PEDIATRICS SUPEREXAM Page 26 of 97


For inquiries visit www.topnotchboardprep.com.ph or email us at topnotchmedicalboardprep@gmail.com
TOPNOTCH MEDICAL BOARD PREP PEDIATRICS SUPEREXAM
For inquiries visit www.topnotchboardprep.com.ph or email us at topnotchmedicalboardprep@gmail.com
Item QUESTION EXPLANATION AUTHOR TOPNOTCH
# EXAM
179 In treatment Plan B for diarrhea using ORS, ___ HAROLD JAY S. MIDTERM 3
cc/kg is given to a patient for 4 hours. BAYTEC, MD EXAM - AUG
A. 50 (TOP 10 - FEB 2015
B. 75 2015 MED
C. 100 BOARDS;
D. 125 TOPNOTCH MD
E. 150 FROM FEU)

180 The murmur heard in Atrial septal defect originates due to increased blood flow passing through this HAROLD JAY S. MIDTERM 3
from: normal sized valve producing a relative stenosis of BAYTEC, MD EXAM - AUG
A. Pulmonary valve pulmonary valve----> systolic ejection murmur at the (TOP 10 - FEB 2015
B. Tricuspid valve 2nd left ICS. Blood flow across the ASD does not cause 2015 MED
C. ASD a murmur at the site of the shunt because no BOARDS;
D. Aortic valve substantial pressure gradient exists between the atria. TOPNOTCH MD
E. Mitral valve However, ASD with moderate-to-large left-to-right FROM FEU)
shunts result in increased right ventricular stroke
volume across the pulmonary outflow tract creating a
crescendo-decrescendo systolic ejection murmur. This
murmur is heard in the second intercostal space at the
upper left sternal border.
181 The 1st teeth to erupt at 6 months are the SIMILAR TO PREVIOUS BOARD EXAM JEAN PAOLO M. FINAL EXAM -
A. 2nd upper molars CONCEPT/PRINCIPLE.. 1st teeth to erupt at 6 to 6 DELFINO, MD AUG 2015
B. Upper cental incisors 1/2 months are the lower central incisors (TOP 10 - FEB
C. Lower central incisors 2015 MED
D. Upper canines BOARDS;
E. Lower canines TOPNOTCH MD
FROM FATIMA)

182 Steroids are used in the management of Rheumatic SIMILAR TO PREVIOUS BOARD EXAM JEAN PAOLO M. FINAL EXAM -
Fever if which of the following develops? CONCEPT/PRINCIPLE.. Patients with carditis and DELFINO, MD AUG 2015
A. carditis cardiomegaly or congestive heart failure should (TOP 10 - FEB
B. chorea receive corticosteroids. 2015 MED
C. arthritis BOARDS;
D. erythema marginatum TOPNOTCH MD
E. subcutaneous nodules FROM FATIMA)

183 A patient with evidence of previous Streptococcal SIMILAR TO PREVIOUS BOARD EXAM JEAN PAOLO M. FINAL EXAM -
throat infection is diagnosed with Rheumatic fever CONCEPT/PRINCIPLE.. The diagnosis of acute DELFINO, MD AUG 2015
if there is which among the following? rheumatic fever can be established by the Jones (TOP 10 - FEB
A. 0 major, 3 minor Jones criteria criteria when a patient fulfills 2 major criteria or 1 2015 MED
B. 1 major, 1 minor Jones criteria major and 2 minor criteria and meets the absolute BOARDS;
C. chorea requirement. There are 3 circumstances in which the TOPNOTCH MD
D. 0 major, 4 minor criteria diagnosis of acute rheumatic fever can be made FROM FATIMA)
E. None of the above without strict adherence to the Jones criteria. Chorea
may occur as the only manifestation of acute
rheumatic fever. Similarly, indolent carditis may be
the only manifestation in patients who 1st come to
medical attention months after the onset of acute
rheumatic fever. Finally, although most patients with
recurrences of acute rheumatic fever fulfill the Jones
criteria, some may not.
184 2 year old patient was brought to your clinic due to EDTA should be given as a chelation therapy for JEAN PAOLO M. FINAL EXAM -
neurocognitive delay. Mother recalled that for the chronic lead poisoning DELFINO, MD AUG 2015
past year, the walls of their house have peeling and (TOP 10 - FEB
cracking paint. What should be given to the child? 2015 MED
A. EDTA BOARDS;
B. Penicillamine TOPNOTCH MD
C. succimer FROM FATIMA)
D. activated charcoal
E. deferoxamine
185 5 year old patient has arthritis of the knees, rashes Henoch-schonlein purpura cause palpable purpuric JEAN PAOLO M. FINAL EXAM -
on the legs and intermittent abdominal pain for the rash, arthritis and intermittent abdominal pain. It is DELFINO, MD AUG 2015
past 3 days. History revealed previous upper the most common cause of non-thrombocytopenic (TOP 10 - FEB
respiratory tract infection. If the patient's kidney is purpura in children. The cause is unknown but it 2015 MED
involved, IgA mesangial deposition.is expected to typically follows an URTI. BOARDS;
be seen in kidney biopsy. What is the most probable TOPNOTCH MD
diagnosis? FROM FATIMA)
A. Post-strep glomerulonephritis
B. Kawasaki disease
C. Wilms tumor
D. Henoch-schonlein purpura
E. IgA nephropathy
186 What is the most common esophageal disorder in GERD is the most common esophageal disorder in JEAN PAOLO M. FINAL EXAM -
children of all ages? children of all ages DELFINO, MD AUG 2015
A. Achalasia (TOP 10 - FEB
B. Tracheoesophageal fistula 2015 MED
C. GERD BOARDS;
D. Foreign body obstruction TOPNOTCH MD
E. None of the above FROM FATIMA)

TOPNOTCH MEDICAL BOARD PREP PEDIATRICS SUPEREXAM Page 27 of 97


For inquiries visit www.topnotchboardprep.com.ph or email us at topnotchmedicalboardprep@gmail.com
TOPNOTCH MEDICAL BOARD PREP PEDIATRICS SUPEREXAM
For inquiries visit www.topnotchboardprep.com.ph or email us at topnotchmedicalboardprep@gmail.com
Item QUESTION EXPLANATION AUTHOR TOPNOTCH
# EXAM
187 11 months old infant was brought in the emergency Routine management of a normal infant with simple JEAN PAOLO M. FINAL EXAM -
room due to fever of 2 days duration. Upon arrival brief febrile convulsions includes a careful search for DELFINO, MD AUG 2015
at the ER, patient started to have a seizure episode, the cause of the fever and reassurance and education (TOP 10 - FEB
generalized tonic-clonic, which lasted for 2 minutes of the parents. Although antipyretics have not been 2015 MED
until diazepam was given. Temperature was 39.4 shown to prevent seizure recurrences, active BOARDS;
deg C. What is the next important step in the measures to control the fever, including the use of TOPNOTCH MD
management? antipyretics, may reduce discomfort and are FROM FATIMA)
A. do an EEG reassuring.
B. support ventilation
C. give antipyretics
D. search for the cause of fever
E. give phenobarbital as anticonvulsant
prophylaxis
188 10 year old patient developed periorbital edema SIMILAR TO PREVIOUS BOARD EXAM JEAN PAOLO M. FINAL EXAM -
and oliguria 2 weeks after treatment for a throat CONCEPT/PRINCIPLE.. low C3 is the only compatible DELFINO, MD AUG 2015
infection. Urine findings showed hematuria and finding for post-strep gn in this. ASO titer should be (TOP 10 - FEB
cylindriuria. Which among the following findings is gretaer than 200 in adults and 400 in children. Anti- 2015 MED
diagnostic of this case? DNAse is used in PSGN if it is preceded by BOARDS;
A. low C3 streptococcal skin infection. TOPNOTCH MD
B. leukopenia FROM FATIMA)
C. ASO titer 180
D. A and C
E. All of the above
189 Unilateral Moro reflex was noted in a newborn after JEAN PAOLO M. FINAL EXAM -
a difficult labor and delivery. The affected arm was DELFINO, MD AUG 2015
adducted, extended and medially rotated while the (TOP 10 - FEB
forearm is pronated and the carpal flexors flex the 2015 MED
hand at the wrist. What is the diagnosis? BOARDS;
A. Winged scapula TOPNOTCH MD
B. Klumpke's paralysis FROM FATIMA)
C. Erb-Duchenne syndrome
D. Clavicular fracture
E. Ulnar nerve injury
190 8 year old male was brought to the clinic due to 3 The diagnosis is chickenpox/ varicella. It is caused by JEAN PAOLO M. FINAL EXAM -
day history of fever and development of rashes.The varicella-zoster virus. The infection becomes latent in DELFINO, MD AUG 2015
rashes were characterized with an appearance dorsal root ganglion and reactivation is possible (TOP 10 - FEB
described as "dewdrops on a rose petal". Which of causing herpes zoster. 2015 MED
the the following is true about the diagnosis? BOARDS;
A. posterior auricular lymphadenopathy is TOPNOTCH MD
positive FROM FATIMA)
B. cough, coryza and conjunctivitis are also
present
C. the described skin lesion is known as the
Herman's rash
D. there may be reactivation of the infection
E the drug of choice is ganciclovir
191 2 years old patient developed stridor. Vaccination The diagnosis is epiglottitis caused by H. influenza. JEAN PAOLO M. FINAL EXAM -
history is unrecalled. Upon examination, you noted The drug of choice for this condition is Ceftriaxone. DELFINO, MD AUG 2015
the patient is sitting up on hands, with tongue out (TOP 10 - FEB
and the head forward. There is also drooling noted 2015 MED
and the voice is muffled. Imaging test revealed BOARDS;
"thumb sign". What is the drug of choice? TOPNOTCH MD
A. Penicillin FROM FATIMA)
B. Racemic epinephrine
C. Ceftriaxone
D. Azithromycin
E. Tetracycline
192 All of the following are live vaccines except? All vaccines for viral infections are live except for "RIP JEAN PAOLO M. FINAL EXAM -
A. Rotavirus vaccine Always". Rabies, Influenza, Papilloma and hep A DELFINO, MD AUG 2015
B. MMR vaccine vaccine. BCG is the only live vaccine against a bacterial (TOP 10 - FEB
C. BCG vaccine infection. 2015 MED
D. HPV vaccine BOARDS;
E. OPV vaccine TOPNOTCH MD
FROM FATIMA)

193 A 4 years old male tripped while playing in the Contaminated wounds in patients with >3doses of JEAN PAOLO M. FINAL EXAM -
streets. He sustained abrasions in the knees and anti-tetanus vaccine and last dose <5 years do not DELFINO, MD AUG 2015
forearms and his right hand was punctured by a require tetanus prophylaxis. (TOP 10 - FEB
fallen branch of a tree. He was brought to your 2015 MED
clinic for treatment. The child is completely BOARDS;
immunized for age. After cleaning and disinfecting TOPNOTCH MD
the wound, what should be given for tetanus FROM FATIMA)
prophylaxis?
A. Give Tetanus Toxoid and TIG
B. Give TT only
C. Give TIG only
D. Give DPT
E. None of the above
194 Decreased C3 levels that does not normalize in 6-8 MPGN JEAN PAOLO M. FINAL EXAM -
weeks is suggestive of DELFINO, MD AUG 2015
A. PSGN (TOP 10 - FEB
B. MPGN 2015 MED
C. MCD BOARDS;
D. MGN TOPNOTCH MD
E. FSGS FROM FATIMA)

TOPNOTCH MEDICAL BOARD PREP PEDIATRICS SUPEREXAM Page 28 of 97


For inquiries visit www.topnotchboardprep.com.ph or email us at topnotchmedicalboardprep@gmail.com
TOPNOTCH MEDICAL BOARD PREP PEDIATRICS SUPEREXAM
For inquiries visit www.topnotchboardprep.com.ph or email us at topnotchmedicalboardprep@gmail.com
Item QUESTION EXPLANATION AUTHOR TOPNOTCH
# EXAM
195 3 years old patient had 8 episodes of passing loose Patient has severe dehydration. Management is to JEAN PAOLO M. FINAL EXAM -
watery stool. Patient was lethargic, not able to administer IVF PLR 420ml for the first 30 minutes DELFINO, MD AUG 2015
drink, with sunken eyes and skin pinch goes back then 980ml for the next 2 1/2 hours (TOP 10 - FEB
slowly. How will you manage the patient? 2015 MED
A. Give 200ml fluid after each loose stool BOARDS;
B. Give 1050 ml ORS for 4 hours then reassess TOPNOTCH MD
C. Administer IVF PLR 420ml for the first 30 FROM FATIMA)
minutes then 980ml for the next 2 1/2 hours
D. Administer IVF PLR 420ml for the first hour
then 980ml for the next 5 hours
E. Administer IVF PLR 300ml for the first hour
then 700 ml for the next 5 hours
196 4 years old male had history of fever. Physical Diagnosis for this case is Kawasaki Disease. JEAN PAOLO M. FINAL EXAM -
examination revealed strawberry tongue, redness Management is IVIG and aspirin. DELFINO, MD AUG 2015
of hands and feet, rash on trunk and cervical (TOP 10 - FEB
lymphadenopathy. What is the most appropriate 2015 MED
management? BOARDS;
A. Penicillin G 25 000 units/kg/day TOPNOTCH MD
B. Ceftriaxone 50mg/kg FROM FATIMA)
C. Prednisone 1 mg/kg/day
D. IVIG and aspirin
E. Metronidazole 15mg/kg/day
197 In what condition will you see absence of CD18 In Leukocyte adhesion defect, there is absence of JEAN PAOLO M. FINAL EXAM -
which manifests with leukocytosis and inability to CD18. This manifests as leukocytosis and inability to DELFINO, MD AUG 2015
form pus? form pus. Another presentation would be a delay in (TOP 10 - FEB
A. Leukocyte adhesion defect umbilical cord sloughing 2015 MED
B. Chronic granulomatous disease BOARDS;
C. Chediak-Higashi Syndrome TOPNOTCH MD
D. Job's syndrome FROM FATIMA)
E. Wiskott-Aldrich Disease

198 A 12 month old infant, born premature, developed Diagnosis is bronchiolitis which is most commonly JEAN PAOLO M. FINAL EXAM -
low grade fever, rhinorrhea and cough. On PE, there caused by Respiratory Syncitial Virus. Risk factors DELFINO, MD AUG 2015
was wheezing and hyperresonance to percussion. include prematurity, immunodeficiency, heart or lung (TOP 10 - FEB
CXR showed hyperinflation and interstitial disease. Ribavirin is the drug of choice. 2015 MED
infiltrates. What drug can be given to the patient for BOARDS;
this condition? TOPNOTCH MD
A. Isoniazid FROM FATIMA)
B. Amoxicillin
C. Ribavirin
D. Azithromycin
E. Oxacillin
199 Breastfeeding is contraindicated in which of the Contraindications to breastfeeding are the following: JEAN PAOLO M. FINAL EXAM -
following? (Mnemonics- Girl, Bawal Ako Sa SuSo Mo). DELFINO, MD AUG 2015
A. Malaria Galactosemia, Breast Ca, Active TB, Septicemia, (TOP 10 - FEB
B. Galactosemia Substance abuse, Severe Psychosis, Malaria 2015 MED
C. Active TB BOARDS;
D. All of the above TOPNOTCH MD
E. B and C FROM FATIMA)

200 A 2 years old male had chronic constipation since Diagnosis is Hirschprung Disease. Gold Standard for JEAN PAOLO M. FINAL EXAM -
infanthood. PE revealed globular abdomen and the diagnosis is rectal suction biopsy. DELFINO, MD AUG 2015
rectal vault is empty, no feces on examining finger. (TOP 10 - FEB
What test should be done to confirm the diagnosis? 2015 MED
A. Abdominal x-ray BOARDS;
B. CT scan TOPNOTCH MD
C. Ultrasound FROM FATIMA)
D. Colonoscopy
E. Suction biopsy
201 A newborn infant presents with generalized APGAR score is not designed to predict neurologic GRACE ARVIOLA, DIAGNOSTIC
cyanosis, irregular respirations, and some flexion of outcome. Immaturity, adminstration of analgesics and MD (TOP 3 - AUG EXAM - FEB
the extremities. Heart rate is 80 bpm. There is no sedatives to the mother, precipitous delivery are some 2014 MED 2015
response when a catheter was placed on the nostril. of the causes of a falsely low Apgar score. BOARDS;
Which of the following statements is NOT true TOPNOTCH MD)
about this infant?
A. The APGAR score is 3.
B. This infant requires immediate resuscitation.
C. This infant is at high risk of developing cerebral
palsy.
D. A low Apgar score predicts neonatal death.
E. A low Apgar score may be present in the
absence of fetal acidosis or hypoxia.
202 In a newborn infant, the presence of a third This is also seen among preterm infants. GRACE ARVIOLA, DIAGNOSTIC
fontanel suggests: MD (TOP 3 - AUG EXAM - FEB
A. Patau syndrome 2014 MED 2015
B. Edward syndrome BOARDS;
C. Down syndrome TOPNOTCH MD)
D. Johanson-Blizzard syndrome
E. Turner syndrome

TOPNOTCH MEDICAL BOARD PREP PEDIATRICS SUPEREXAM Page 29 of 97


For inquiries visit www.topnotchboardprep.com.ph or email us at topnotchmedicalboardprep@gmail.com
TOPNOTCH MEDICAL BOARD PREP PEDIATRICS SUPEREXAM
For inquiries visit www.topnotchboardprep.com.ph or email us at topnotchmedicalboardprep@gmail.com
Item QUESTION EXPLANATION AUTHOR TOPNOTCH
# EXAM
203 Which among the following patients with measles is Measles at an early age favors the development of GRACE ARVIOLA, DIAGNOSTIC
most likely to develop SSPE? SSPE. 50% of SSPE patients had primary measles MD (TOP 3 - AUG EXAM - FEB
A. A 2 year-old male patient before 2 years of age and 75% before 4 years of age. 2014 MED 2015
B. A 6 year-old patient who was not given vitamin Males are affected twice as often as females. BOARDS;
A TOPNOTCH MD)
C. A 10 year-old patient who has developed
pneumonia
D. A 14 year-old patient who is
immunocompromised
E. A 16 year-old with subclinical measles
204 A 2 week-old infant has developed the bronze baby The terms bronze baby syndrome refers to a GRACE ARVIOLA, DIAGNOSTIC
syndrome. This infant most likely has: sometimes-noted dark, grayish-brown skin MD (TOP 3 - AUG EXAM - FEB
A. Hyaline membrane disease discoloration in infants undergoing phototherapy. 2014 MED 2015
B. Congenital diaphragmatic hernia Almost all infants observed with this syndrome have BOARDS;
C. Truncus arteriosus had significant elevation of direct-reacting bilirubin TOPNOTCH MD)
D. Choledochal cyst and other evidence of obstructive liver disease.
E. Hemachromatosis

205 A 1 year-old female was brought in by her mother This is severe dehydration. GRACE ARVIOLA, DIAGNOSTIC
because of 3 days of loose bowel movement. The MD (TOP 3 - AUG EXAM - FEB
patient is lethargic and does not tolerate oral fluids. 2014 MED 2015
Her mother noted her last urine output to be BOARDS;
scanty. She is tachycardic with very slow capillary TOPNOTCH MD)
refill. Pulses are thready. What should be your
immediate management?
A. Trial of ORS solution, 50-100 mL/kg body
weight over 4 hours
B. Administer antibiotics
C. Intubate patient
D. Send stool exam for fecalysis
E. Administer PNSS 20mL/kg IV bolus
206 A 1 week-old SGA infant born at 35 weeks AOG A: CMV is the most common. B: indirect transmission GRACE ARVIOLA, DIAGNOSTIC
presents with jaundice, hepatomegaly, a blueberry- is possible. C: Ganciclovir, not acyclovir. E: treatment MD (TOP 3 - AUG EXAM - FEB
muffin like rash, bleeding and microcephaly. Which is not indicated for immunocompetent persons. 2014 MED 2015
statement is correct? BOARDS;
A. The disease is the second most common cause TOPNOTCH MD)
of congenital infection.
B. Transmission occurs by direct person-to-
person contact; indirect transmission is not
possible.
C. Acyclovir and foscarnet are treatment options.
D. Primary infections in pregnant women cause
the most severe symptomatic congenital infections.
E. Treatment is indicated for all infected persons.
207 In the diagnosis of febrile sezires, a lumbar A LP should be strongly considered in children less GRACE ARVIOLA, DIAGNOSTIC
puncture is strongly considered among children of than 12 months of age and considered in those 12-18 MD (TOP 3 - AUG EXAM - FEB
what age group? months old. 2014 MED 2015
A. <6 months BOARDS;
B. <12 months TOPNOTCH MD)
C. <18 months
D. <24 months
E. <36 months

208 Which antibody mediates the vasculitis present in There is deposition of IgA and C3 in the small vessels GRACE ARVIOLA, DIAGNOSTIC
Henoch-Schonlein purpura? of the skin and the renal glomeruli. MD (TOP 3 - AUG EXAM - FEB
A. IgA 2014 MED 2015
B. IgG BOARDS;
C. IgM TOPNOTCH MD)
D. IgD
E. IgE

209 Which is not consistent with a diagnosis of Prematuriy is uncommon with this condition. GRACE ARVIOLA, DIAGNOSTIC
Hirschsprung disease? MD (TOP 3 - AUG EXAM - FEB
A. Delayed passage of meconium 2014 MED 2015
B. A premature infant BOARDS;
C. Rectum is empty of feces TOPNOTCH MD)
D. Increased acetylcholinesterase staining on
rectal biopsy
E. Absent Auerbach's plexus
210 A 4 year-old male with Kawasaki disease has a A repeat study should be performed 6-8 weeks after GRACE ARVIOLA, DIAGNOSTIC
normal echocardiography at the time of diagnosis. the onset of illness if both test were negative. MD (TOP 3 - AUG EXAM - FEB
Repeat test after 2 weeks revealed no coronary 2014 MED 2015
abnormalities. When should you schedule the next BOARDS;
2D-echo? TOPNOTCH MD)
A. 1 months after the onset of illness
B. 2 months after the onset of illness
C. 3 months after the onset of illness
D. 4 months after the onset of illness
E. 6 months after the onset of illness
211 What is the daily maintenance fluid requirement of For 11-20 kg: 1,000 mL + 50 ml for each kg above 10 GRACE ARVIOLA, DIAGNOSTIC
a 19 kg-patient? kg MD (TOP 3 - AUG EXAM - FEB
A. 1,900 mL 2014 MED 2015
B. 1,450 mL BOARDS;
C. 1,180 mL TOPNOTCH MD)
D. 1,950 mL
E. 1,680 mL

TOPNOTCH MEDICAL BOARD PREP PEDIATRICS SUPEREXAM Page 30 of 97


For inquiries visit www.topnotchboardprep.com.ph or email us at topnotchmedicalboardprep@gmail.com
TOPNOTCH MEDICAL BOARD PREP PEDIATRICS SUPEREXAM
For inquiries visit www.topnotchboardprep.com.ph or email us at topnotchmedicalboardprep@gmail.com
Item QUESTION EXPLANATION AUTHOR TOPNOTCH
# EXAM
212 Which among the following is INCORRECT about Age of onset is inversely related to gestational age. GRACE ARVIOLA, DIAGNOSTIC
necrotizing enterocolitis? MD (TOP 3 - AUG EXAM - FEB
A. Preamturity is the greatest risk factor. 2014 MED 2015
B. In most cases, no pathogen is identified. BOARDS;
C. Premature infants exhibit symptoms earlier TOPNOTCH MD)
than term infants.
D. The finding of pneumatosis intestinalis is
diagnostic.
E. Exclusively breast-fed infants have a reduced
risk.
213 Which finding in a patient with post-streptococcal The acute phase generally resolves within 6-8 weeks. GRACE ARVIOLA, DIAGNOSTIC
acute glomerulonephritis is the last to resolve? Although urinary protein excretion and hypertension MD (TOP 3 - AUG EXAM - FEB
A. Edema usually normalize by 4-6 weeks after onset, persistent 2014 MED 2015
B. Proteinuria microscopic hematuria may persist for 1-2 years after BOARDS;
C. Elevated ESR the initial presentation. TOPNOTCH MD)
D. Hypertension
E. Hematuria

214 Folic acid supplementation should be given at what 0.4 mg daily of folic acid is recommended for women GRACE ARVIOLA, DIAGNOSTIC
dose to a woman with who has given birth to a child of childbearing age and is capable of being pregnant. MD (TOP 3 - AUG EXAM - FEB
with meningocele? This case, however, is a high risk woman. 2014 MED 2015
A. 0.3 mg daily BOARDS;
B. 0.4 mg daily TOPNOTCH MD)
C. 3 mg daily
D. 4 mg daily
E. 5 mg daily

215 The ECG of a patient with Tetralogy of Fallow A dominant R wave appears in the right precordial GRACE ARVIOLA, DIAGNOSTIC
showed right axis deviation. Which component of leads or an RSR' pattern. MD (TOP 3 - AUG EXAM - FEB
the tetralogy is directly responsible for this finding? 2014 MED 2015
A. Pulmonary stenosis BOARDS;
B. Overriding of the aorta TOPNOTCH MD)
C. Ventricular septal defect
D. Right ventricular hypertrophy
E. Coarctation of the aorta

216 Mantoux testing of 7 year-old patient in remission Induration of >5 mm is considered positive among GRACE ARVIOLA, DIAGNOSTIC
from acute leukemia after receiving patients receiving immunosuppressive therapy or MD (TOP 3 - AUG EXAM - FEB
immunosuppressive doses of corticosteroids with immunosuppressive conditions including HIV 2014 MED 2015
showed 7 mm of induration. Interpret this finding. infection. BOARDS;
A. Positive TOPNOTCH MD)
B. Negative
C. Indeterminate
D. Latent
E. Active
217 To prevent cardiac damage, the cumulative dose of Congestive cardiac failure may occur after treatment GRACE ARVIOLA, DIAGNOSTIC
doxorubicin should be limited to what value in with doxorubicin particularly if administered in MD (TOP 3 - AUG EXAM - FEB
patients with Wilms tumor? combination with radiation therapy to the lungs. 2014 MED 2015
A. 50-100 mg/m2 BOARDS;
B. 100-200 mg/m2 TOPNOTCH MD)
C. 200-300 mg/m2
D. 300-400 mg/m2
E. 400-500 mg/m2

218 A ball was placed in front of a young child Object permanence seen around 9 months of age. GRACE ARVIOLA, DIAGNOSTIC
afterwhich a piece of cloth was placed over the ball, MD (TOP 3 - AUG EXAM - FEB
hiding it in sight. The child lifted the cloth and 2014 MED 2015
found the ball beneath it. The youngest age that this BOARDS;
child could possibly be is: TOPNOTCH MD)
A. 1 month old
B. 4 months old
C. 9 months old
D. 15 months old
E. 18 months old
219 The Steeple sign of croup represents: This sign may be absent in patient with croup, present GRACE ARVIOLA, DIAGNOSTIC
A. Deviation of the trachea in patients without croup as a normal variant, and MD (TOP 3 - AUG EXAM - FEB
B. Subglottic narrowing may occasionally be seen in patient with epiglottitis. 2014 MED 2015
C. Edema of the glottis BOARDS;
D. Retropharyngeal abscess TOPNOTCH MD)
E. Hyperinflated lungs

220 During measles outbreaks, at what earliest age can GRACE ARVIOLA, DIAGNOSTIC
you vaccinate chilldren with the measles vaccine? MD (TOP 3 - AUG EXAM - FEB
A. At birth 2014 MED 2015
B. 2 months BOARDS;
C. 4 months TOPNOTCH MD)
D. 6 months
E. 8 months

TOPNOTCH MEDICAL BOARD PREP PEDIATRICS SUPEREXAM Page 31 of 97


For inquiries visit www.topnotchboardprep.com.ph or email us at topnotchmedicalboardprep@gmail.com
TOPNOTCH MEDICAL BOARD PREP PEDIATRICS SUPEREXAM
For inquiries visit www.topnotchboardprep.com.ph or email us at topnotchmedicalboardprep@gmail.com
Item QUESTION EXPLANATION AUTHOR TOPNOTCH
# EXAM
221 in what phase of kawasaki disease will have the kawasaki disease is divided into three clinical phases. LEAN ANGELO MIDTERM
greatest risk for coronary aneurysm formation? The acute febrile phase (1-2 wks) presents with SILVERIO, MD EXAM 1 - FEB
A. Acute febrile phase myocarditis, fever and other classic signs of acute (TOP 4 - AUG 2015
B. Subacute phase illness. Subacute phase - abatement of fever, 2014 MED
C. Convalescent phase associated with desquamationm thrombocytosis and BOARDS;
D. Early recovery phase development of coronary aneurysm. it is also in this TOPNOTCH MD),
E. Risk is not defined by the phase stage that there is highest risk of sudden cardiac MD
death. convalescent phase - when all clinical
symptoms disappear up to the time there is
normalization of ESR and CRP
222 which of the following is true about systemic type A- the evanescent salmon colored rash which only LEAN ANGELO MIDTERM
of juvenile rheumatoid arthritis ? exhibit during the height of the fever is non pruritic. C- SILVERIO, MD EXAM 1 - FEB
A. It presents with pruritic evanescent among the types of JRA, it is the polyarticular type and (TOP 4 - AUG 2015
maculopapular rash on proximal extremities and not the systemic type that highly resembles classic 2014 MED
trunk rheumatoid arthritis. D- there is prominent visceral BOARDS;
B. It exhibits koebner phenomenon involvement in systemic JRA. TOPNOTCH MD),
C. It highly resembles characteristic presentation MD
of rheumatoid arthritis and with similar HLA
profile
D. visceral involvement ( hepatosplenomegaly)
although part of the spectrum is rare in this type of
JRA
E. all of the above
223 A 2 y/o male presents with rapidly enlarging neuroblastoma is the 3rd most common pediatric LEAN ANGELO MIDTERM
abdominal mass. During PE, you noted that there cancer. Its classic presentation is an rapidly enlarging SILVERIO, MD EXAM 1 - FEB
was also bilateral proptosis and echymosis of both abdominal mass that crosses the midline. It also (TOP 4 - AUG 2015
periorbital region. Neurological examination presents with proptosis, ecchymosis, opsomyoclonus 2014 MED
showed, miosis of the right eye as well as difficulty and sometimes horner syndrome. BOARDS;
in performing finger to nose test. what is the most TOPNOTCH MD),
likely diagnosis? MD
A. Wilms tumor
B. ALL with brain metastasis
C. Neuroblastoma
D. pheochromocytoma
E. none of the above
224 a 4 y/o female was referred to your clinic similar to aSIMILAR TO PREVIOUS BOARD EXAM LEAN ANGELO MIDTERM
secondary to a mass occupying the nasopharyngeal CONCEPT/PRINCIPLE. Rhabdomyosarcoma (RS) is the SILVERIO, MD EXAM 1 - FEB
area. this is accompanied by congestion, recurrent most common pediatric soft tissue sarcoma. They are (TOP 4 - AUG 2015
epistaxis and multiple cranial nerve palsies. Patient usually found in the head and neck, genitourinary, and 2014 MED
is lethargic with complains of headache and retroperitoneal area. nasopharyngeal RS presents BOARDS;
recurrent projectile vomitting. what is the most with mass accompanied by epistaxis, congestion, TOPNOTCH MD),
likely diagnosis for this patient? dysphagia. regional extension leads to cranial nerve MD
A. rhabdomyosarcoma palsies, blindness and signs of increased intracranial
B. neuroblastoma pressure. medullablastoma is the most common
C. medulloblastoma malignant primary brain tumor in pediatrics. it is
D. chloroma intraaxial and will not present with mass in the
E. all of the above nasopharyngeal area. its most common site is the
cerebellar hemisphere. Neuroblastoma seldomly seen
in head and neck region. along with wilms, it usually
present with an abdominal mass. chloroma is a solid
collection of leukemic cells in acute myelogenous
leukemia. it was not stated in the case if the patient
presents with signs and symptoms of leukemia.
225 what is the most common cause of persistent SIMILAR TO PREVIOUS BOARD EXAM LEAN ANGELO MIDTERM
proteinuria in school aged children and CONCEPT/PRINCIPLE. A- fever induced, along with SILVERIO, MD EXAM 1 - FEB
adolescents? exercise, dehydration, cold exposure, CHF , seizure (TOP 4 - AUG 2015
A. Fever induced proteinuria and stress are all types of transient proteinuria.B and 2014 MED
B. PSGN C are most likely nephritic in nature. E- minimal BOARDS;
C. IgA nephropathy change disease is the most common glomerular type TOPNOTCH MD),
D. Orthostatic of nephrotic syndrome in pediatrics. however, MD
E. Minimal change disease orthostatic proteinuria still accounts for 60%
pediatric population with incidental finding of
proteinuria. Nelson 18 th ed 2189
226 which is a correct statement regarding Fever of Nelson 18th ed pp 1090-1091 LEAN ANGELO MIDTERM
unknown origin? SILVERIO, MD EXAM 1 - FEB
A. cause cannot be identified after 3 weeks OPD (TOP 4 - AUG 2015
evaluation or 1 week as an Inpatient 2014 MED
B. It is usually an atypical presentation of a BOARDS;
common disease TOPNOTCH MD),
C. The most frequently associated connective tissue MD
disorder in FUO is SLE and JRA
D. all of the above
E. none of the above
227 During a check up of a well child, what would you a 3 y/o child can do the following: rides tricycle, LEAN ANGELO MIDTERM
expect a 3 y/o child can perform? stands momentarily on one foot, makes tower of 10 SILVERIO, MD EXAM 1 - FEB
A. Knows age and sex cubes, copies circle, imitates cross, knows age and sex, (TOP 4 - AUG 2015
B. Plays with other children with beginning social play simple games, washes hands. All the other 2014 MED
interaction choices are general characteristic of a 4 y/o child BOARDS;
C. Hops on one foot TOPNOTCH MD),
D. Throws ball overhand MD
E. Identifies longer of two lines

TOPNOTCH MEDICAL BOARD PREP PEDIATRICS SUPEREXAM Page 32 of 97


For inquiries visit www.topnotchboardprep.com.ph or email us at topnotchmedicalboardprep@gmail.com
TOPNOTCH MEDICAL BOARD PREP PEDIATRICS SUPEREXAM
For inquiries visit www.topnotchboardprep.com.ph or email us at topnotchmedicalboardprep@gmail.com
Item QUESTION EXPLANATION AUTHOR TOPNOTCH
# EXAM
228 which of the following is not a cause of abnormal delayed eruption ( no teeth at 13 months of age) - LEAN ANGELO MIDTERM
early exfoliation of teeth? hypothyroid, hypoparathyroid, idiopathic. Early SILVERIO, MD EXAM 1 - FEB
A. Histiocytosis X exfoliation- histiocytosis x, cyclic neutropenia, (TOP 4 - AUG 2015
B. neutropenia leukemia, trauma. Nelson 18th ed p 73 2014 MED
C. leukemia BOARDS;
D. hypoparathyroidism TOPNOTCH MD),
E. None of the above MD

229 what vitamin/mineral deficiency is associated with Vitamin A - nyctalopsia, xeropthalmia, bitot spits, LEAN ANGELO MIDTERM
intention tremor, decreased proprioception, epiphyseal bone abnormalities. Vitamin B6 - SILVERIO, MD EXAM 1 - FEB
nystagmus, dysarthria, and retinopathies? irritability, convulsion, hypochromic anemia, oxaluria, (TOP 4 - AUG 2015
A. Vitamin A peripheral neuritis. Zinc - abdominal pain, diarrhea, 2014 MED
B. Vitamin B6 vomitting, acrodermatitis enterohepatica. Vitamin E BOARDS;
C. Vitamin E deficiency is associated with severe neurologic TOPNOTCH MD),
D. Zinc manifestation. it usually presents with limb and MD
E. Selenium truncal ataxia, ophthalmoplegia, positive rhomberg
test, dysarthria
230 An 8 y/o male child presented to the ER with fever, patient is most likely suffering from measles infection. LEAN ANGELO MIDTERM
cough, colds, and photophobia. There was also All of the choices are true. SILVERIO, MD EXAM 1 - FEB
presence of generalized maculopapular rash. Which (TOP 4 - AUG 2015
of the following is true about his condition ? 2014 MED
A. There may be an increaesed rate of pulmonary BOARDS;
tuberculosis activation TOPNOTCH MD),
B. low serum retinol is associated with increased MD
risk for pneumonia or encephalitis
C. pathologic hallmark is the presence of warthin
finkeldey giant cells in the respiratory tissue
D. all of the above.
E. none of the above.
231 which of the following is not a clinical criteria in the APGAR score of 0-5 in > 5 mins LEAN ANGELO MIDTERM
diagnosis of perinatal asphyxia? SILVERIO, MD EXAM 1 - FEB
A. Acidosis (pH 7.0) (TOP 4 - AUG 2015
B. Evidence of neurological injury 2014 MED
C. APGAR score of 0-5 in > 2 mins BOARDS;
D. Multisystem organ failure TOPNOTCH MD),
E. None of the above MD

232 Which of the following parameters is last to oliguria -2 weeks, hypertension - 3 weeks, gross LEAN ANGELO MIDTERM
normalized during the clinical course of APSGN? hematuria -3 weeks, low C3 - up to 8 weeks, SILVERIO, MD EXAM 1 - FEB
A. Microscopic hematuria proteinuria - 6 months,microscopic hematuria- >1 (TOP 4 - AUG 2015
B. Low C3 year. 2014 MED
C. Oliguria BOARDS;
D. proteinuria TOPNOTCH MD),
E. ASO titier MD

233 A 12 y/o male went for consult secondary to tea MPGN is the most common cause of chronic LEAN ANGELO MIDTERM
colored urine. Physical examination: BP 150/90, PR glomerulonephritis in children and adolescent. Its SILVERIO, MD EXAM 1 - FEB
102 RR 21 T 37.0C. Pale palpebral conjunctiva, presentation is very similar to APSGN however, the (TOP 4 - AUG 2015
anicteric sclerae, clear breath sounds, soft, only difference is a persistently low C3 even after 2 2014 MED
nontender abdomen. There was grade 1 bipedal months. BOARDS;
edema and with slight periorbital swelling. lab TOPNOTCH MD),
shows the following result (+) ASO, low C3, and MD
subneprotic range proteinuria. patient was
hospitalized for a week and was successfully
treated. he was lost to follow up, and just come
back after 3 months. laboratories still show
persistently low C3. what is your diagnosis ?
A. FSGS
B. MPGN
C. APSGN
D. MGN
E. RPGN
234 A 3 mo old infant has a history of failure to thrive this is a classic case of hirschprung disease. the gold LEAN ANGELO MIDTERM
accompanied by infrequent bowel movement. standard for the diagnosis is rectal biopsy. Showing SILVERIO, MD EXAM 1 - FEB
During rectal examination, patient subsequently absence of ganglion. Rectal motility and barium enema (TOP 4 - AUG 2015
passes large amount of stool. Rectal tone was will also highly yield a diagnostic clue for the presence 2014 MED
normal. Which of the following is not needed in of this disease. colonoscopy will provide normal result BOARDS;
establishing the diagnosis? as there are no visible mucosal defect in hirschprung. TOPNOTCH MD),
A. rectal motility test MD
B. rectal biopsy
C. barium enema
D. colonoscopy
E. none of the above
235 A 6 y/o female presents with a 2 day history of the highest yield to detect meckels diverticulum at this LEAN ANGELO MIDTERM
painless blood stained stool. During the consult, stage is the use of a meckels scan. Since it is located at SILVERIO, MD EXAM 1 - FEB
patient is comfortable and denies any feeling of the mid to late portion of ileum, colonoscopy will not (TOP 4 - AUG 2015
pain. abdominal examination is normal. However, yield any abnormal result. Xray and abdominal utz 2014 MED
she has pale palpebral conjunctiva and noted to be also will be normal. Angiography can be use if the BOARDS;
tachycardic for her age. what is the best initial test patient is in the state of massive bleeding. TOPNOTCH MD),
to request to establish your diagnosis? MD
A. Angiography
B. Meckel scan
C. air contrast enema
D. ultrasound of the abdomen
E. abdominal Xray

TOPNOTCH MEDICAL BOARD PREP PEDIATRICS SUPEREXAM Page 33 of 97


For inquiries visit www.topnotchboardprep.com.ph or email us at topnotchmedicalboardprep@gmail.com
TOPNOTCH MEDICAL BOARD PREP PEDIATRICS SUPEREXAM
For inquiries visit www.topnotchboardprep.com.ph or email us at topnotchmedicalboardprep@gmail.com
Item QUESTION EXPLANATION AUTHOR TOPNOTCH
# EXAM
236 A 2 day old infant was noted to be cyanotic during based on the ancillary results, this might be a case of LEAN ANGELO MIDTERM
its hospital stay. Cardiac examination showed 3-4/6 pulmonary atresia. Since this condition is ductal SILVERIO, MD EXAM 1 - FEB
systolic murmur with prominent second heart dependent. It is better to administer prostaglandin E1 (TOP 4 - AUG 2015
sound without any splitting. 12LECG - LVH, CXR: to prevent ductal closure. Administration of ibuprofen 2014 MED
decrease pulmonary markings with a normal will lead to obliteration of ductal patency and the BOARDS;
cardiac size without displacement. which of the patient will succumb to acidosis and later death. TOPNOTCH MD),
following medications is needed to administer ? MD
A. Prostaglandin E1
B. Digoxin
C. Ibuprofen
D. Morphine
E. Furosemide
237 A neonate was transferred to ICU d/t clinical delayed separation of umbilical stump is a primary LEAN ANGELO MIDTERM
findings of pneumonia, peritonitis and omphalitis. characteristic of leukocyte adhesion defect. SILVERIO, MD EXAM 1 - FEB
there was also a delayed separation of the umbilical Phagocytes are unable to migrate to extravascular (TOP 4 - AUG 2015
stump. Which type of immune disorder is most spaces d/t deficiency in integrin receptors. The 2014 MED
likely affected in this case? standard treatment for this condition is bone marrow BOARDS;
A. Phagocyte function transplantation TOPNOTCH MD),
B. B cell MD
C. T cell
D. Combined B and T cell
E. Phagocyte enzyme deficiency
238 A 8 y/o female presenting with hyperaldosteronism a normal CT findings does not exclude the presence of LEAN ANGELO MIDTERM
is unresponsive to dexamethasone. You requested an adrenal mass. This is particularly true especially if SILVERIO, MD EXAM 1 - FEB
for CT scan of the abdoment however results are the mass is subcentimeter in size. Local production of (TOP 4 - AUG 2015
equivocal. What is your next to establish your aldosterone can be detected using adrenal vein 2014 MED
diagnosis ? catheterization. BOARDS;
A. Explorative laparotomy TOPNOTCH MD),
B. adrenal venous blood sampling MD
C. metyrapone therapy
D. increase the dose of dexamethasone
E. abdominal MRI
239 which of the following is not a principal feature of osteogenesis imperfecta is an autosomal dominant LEAN ANGELO MIDTERM
osteogenesis imperfecta? disorder affecting type 1 collagen. Its principal SILVERIO, MD EXAM 1 - FEB
A. microcephaly features include macrocephaly, fractures, blue sclerae (TOP 4 - AUG 2015
B. Defect in type 1 collagen and early deafness. 2014 MED
C. Autosomal dominant trait BOARDS;
D. Early conduction deafness TOPNOTCH MD),
E. Vertebral compression fractures MD

240 the purpose of administering BCG vaccine is? BCG vaccination prevents the development of life LEAN ANGELO MIDTERM
A. Prevention of miliary TB threatening forms of TB disease ( meningeal, miliary, SILVERIO, MD EXAM 1 - FEB
B. Reduce the incidence of primary complex TB hepatic). It does not prevent one person from (TOP 4 - AUG 2015
C. Increase herd immunity against PTB acquiring pulmonary TB or primary complex TB. 2014 MED
D. All of the above BOARDS;
E. None of the above TOPNOTCH MD),
MD

241 A 14 year old male patient was admitted due to SIMILAR TO PREVIOUS BOARD EXAM KEVIN BRYAN LO, MIDTERM 2
dehydration, mother reported that the patient was CONCEPT/PRINCIPLE., unresponsive to DDAVP - MD (TOP 7 - AUG EXAM - FEB
urinating large volumes of urine and was said to be nephrogenic DI 2014 MED 2015
frequently thirsty and drinks a lot of water. Serum BOARDS;
sodium was 128, plasma osmolality was 310, urine TOPNOTCH MD)
osmolality was only at 200mosm/kg of water, after
administration of DDAVP, urine osmolality
increased to 230mosm/kg, which of the following
conditions does the patient most likely have?
A. renal tubular acidosis
B. nephrogenic diabetes insipidus
C. central diabetes insipidus
D. gittelman's syndrome
E. none of the above
242 12 year old female patient came with with altered SIMILAR TO PREVIOUS BOARD EXAM KEVIN BRYAN LO, MIDTERM 2
sensorium. Patient had 2 month history of chronic CONCEPT/PRINCIPLE., csf picture and pellicle MD (TOP 7 - AUG EXAM - FEB
cough accompanied by occasional fevers, weight formation = TB meningitis 2014 MED 2015
loss and body malaise. Over the course of 3 days, BOARDS;
patient had on and off fevers, increasing daytime TOPNOTCH MD)
sleepiness, irritability and noted to complain of
headaches. On physical examination, patient was
irritable, conscious but confused, (+) nuchal
rigidity, no motor or sensory deficits, no CN deficits.
Lumbar puncture was done which revealed glucose
of 30 serum glucose of 90, protein of 60mg, WBC of
5000 predominantly lymphocytes with pellicle
formation. Which of the following conditions does
the patient most likely have?
A. acute bacterial meningitis
B. subacute partially treated bacterial meningitis
C. TB meningitis
D. fungal meningitis
E. viral encephalomeningitis

TOPNOTCH MEDICAL BOARD PREP PEDIATRICS SUPEREXAM Page 34 of 97


For inquiries visit www.topnotchboardprep.com.ph or email us at topnotchmedicalboardprep@gmail.com
TOPNOTCH MEDICAL BOARD PREP PEDIATRICS SUPEREXAM
For inquiries visit www.topnotchboardprep.com.ph or email us at topnotchmedicalboardprep@gmail.com
Item QUESTION EXPLANATION AUTHOR TOPNOTCH
# EXAM
243 An 8 year old female patient was reported by her SIMILAR TO PREVIOUS BOARD EXAM KEVIN BRYAN LO, MIDTERM 2
teacher to be constantly daydreaming, lacked focus CONCEPT/PRINCIPLE, EEG for evaluation of MD (TOP 7 - AUG EXAM - FEB
and motivation, her academic performance had childhood non induced seizures about 3 questions 2014 MED 2015
started to deteriorate for the past 3 months, mother came out BOARDS;
also noted the patient to be occasionally day TOPNOTCH MD)
dreaming at home for regular short intervals and
that she would not remember the things that were
said or had happened a few moments before, which
is the appropriate next step in evaluation of this
patient?
A. plain cranial CT scan
B. plain cranial MRI
C. serum electrolytes
D. EEG
E. sleep study
244 A 7 year old female child had fever of 4 days SIMILAR TO PREVIOUS BOARD EXAM KEVIN BRYAN LO, MIDTERM 2
duration, body aches, and malaise. The concerned CONCEPT/PRINCIPLE, increased hematocrit may MD (TOP 7 - AUG EXAM - FEB
mother brought her to the clinic for evaluation indicate dengue due to increased capillary 2014 MED 2015
since according to her there were already a lot of permeability BOARDS;
dengue cases arising in the area where they lived, TOPNOTCH MD)
which of the following laboratory parameters
would indicate a possible dengue infection?
A. increased Hematocrit
B. increased WBC neutrophilia
C. (-) NS1 antigen test
D. platelet count of 180,0000
E. none of the above
245 An 11 year old male patient presented to a clinic SIMILAR TO PREVIOUS BOARD EXAM KEVIN BRYAN LO, MIDTERM 2
with 2 day history of fever accompanied by the CONCEPT/PRINCIPLE, a rubella case, only A is correct MD (TOP 7 - AUG EXAM - FEB
onset of a maculopapular rash beginning at the face it is 3 day measles, toga virus, no orchitis 2014 MED 2015
progressing downwards to the trunk and to the BOARDS;
extremities, there was anorexia, body malaise as TOPNOTCH MD)
well. Physical exam revealed posterior auricular
lymphadenopathy, which of the following is true
regarding this condition?
A. this condition is relatively self limiting
B. usually called 5 day measles
C. it is caused by a paramyxovirus
D. virtually no complications arise among pregnant
women
E. can cause orchitis and infertility among
adolescent males
246 A neonate was born via emergency cesarean SIMILAR TO PREVIOUS BOARD EXAM KEVIN BRYAN LO, MIDTERM 2
section from a mother who had seizurers secondary CONCEPT/PRINCIPLE, criteria for asphyxia MD (TOP 7 - AUG EXAM - FEB
to eclampsia, the infant was noted to be floppy, 2014 MED 2015
cyanotic, with weak cry and HR of 80, initial BOARDS;
resuscitation was instituted, the criteria for the TOPNOTCH MD)
diagnosis of perinatal asphyxia involves all
EXCEPT?
A. blood pH of <7.0
B. 5 minute apgar score of 0-3
C. hypoxic ischemic encephalopathy - altered tone
decreased level of consciousness
D. signs of multiorgan system dysfunction
E. all of the above are true
247 A 6 year old male patient was admitted due to flank SIMILAR TO PREVIOUS BOARD EXAM KEVIN BRYAN LO, MIDTERM 2
pain, peristently high fever, vomiting. Urinalysis CONCEPT/PRINCIPLE, usually repeated infections on MD (TOP 7 - AUG EXAM - FEB
revealed wbc 15-20, rbc 3-5, wbc casts and (-) males are a result of a congenital anomaly most 2014 MED 2015
crystals, (+) leukocyte esterase, patient was noted commonly posterior urethral valves work up would BOARDS;
to have had 2 previous episodes of UTI over the be VCUG TOPNOTCH MD)
past month, which of the following diagnostic
procedures should be done to work up the possible
etiology of the repeated infections?
A. DMSA renal scan
B. voiding cystourethrogram
C. retrograde pyelography
D. urine culture
E. all of the above
248 A neonate was born from a 32 year old G1P1 SIMILAR TO PREVIOUS BOARD EXAM KEVIN BRYAN LO, MIDTERM 2
mother, he was noted to be acrocyanotic, the nurse CONCEPT/PRINCIPLE, Cranial CT scan can diagnose MD (TOP 7 - AUG EXAM - FEB
noted difficulty in passing a suction catheter choanal atresia conveniently 2014 MED 2015
through the right nostril, unilateral choanal atresia BOARDS;
is suspected, a nasogastric tube was tried but was TOPNOTCH MD)
unable to insert also through the right nostril,
which is the next appropriate step of evaluation?
A. cranial CT scan
B. cranial MRI
C. anterior rhinoscopy
D. needle guided NGT insertion
E. none of the above

TOPNOTCH MEDICAL BOARD PREP PEDIATRICS SUPEREXAM Page 35 of 97


For inquiries visit www.topnotchboardprep.com.ph or email us at topnotchmedicalboardprep@gmail.com
TOPNOTCH MEDICAL BOARD PREP PEDIATRICS SUPEREXAM
For inquiries visit www.topnotchboardprep.com.ph or email us at topnotchmedicalboardprep@gmail.com
Item QUESTION EXPLANATION AUTHOR TOPNOTCH
# EXAM
249 A 6 month old infant was brought to the clinic due SIMILAR TO PREVIOUS BOARD EXAM KEVIN BRYAN LO, MIDTERM 2
to severe dyspnea and coughing, he was noted to CONCEPT/PRINCIPLE, all of the above are correct MD (TOP 7 - AUG EXAM - FEB
have subcostal retractions, bilateral crackles and possible management 2014 MED 2015
wheezes on both lung fields, RR of 70, slightly BOARDS;
febrile at 37.6 degrees celsius, which of the TOPNOTCH MD)
following is are appropriate management?
A. IV corticosteroids
B. salbutamol nebulization
C. ribavirin
D. intubation as necessary
E. all of the above
250 In the Philippines, the most common cause of SIMILAR TO PREVIOUS BOARD EXAM KEVIN BRYAN LO, MIDTERM 2
neonatal meningitis is ? CONCEPT/PRINCIPLE, in developing countries usually MD (TOP 7 - AUG EXAM - FEB
A. E. coli E coli is the most common cause of neonatal 2014 MED 2015
B. group B streptococci meningitis BOARDS;
C. Listeria monocytogenes TOPNOTCH MD)
D. streptococcus pneumonia
E. Haemophilus influenzae

251 A 7 day old neonate was rushed to the ER because SIMILAR TO PREVIOUS BOARD EXAM KEVIN BRYAN LO, MIDTERM 2
the mother noticed that the patient had difficulty CONCEPT/PRINCIPLE, a case of sepsis, in selecting the MD (TOP 7 - AUG EXAM - FEB
breathing and was unable to breastfeed. On closer best answer = IV antibiotics should be given 2014 MED 2015
examination, patient was jaundiced, tachypneic BOARDS;
with subcostal retractions, crackles all over TOPNOTCH MD)
bilateral lung bases, difficult to arouse, cold clammy
skin, mottled in appearance, what is the
appropriate course of action?
A. give oral amoxicillin by dropper and supportive
therapy
B. give IV ampicillin + gentamicin and supportive
therapy
C. nebulize with salbutamol immediately and
supportive therapy
D. hydrate the infant immediately
E. none of the above
252 A 3 year old child was brought by the mother for SIMILAR TO PREVIOUS BOARD EXAM KEVIN BRYAN LO, MIDTERM 2
consult, the mother noted decreased activity over CONCEPT/PRINCIPLE, TOF, pulmonic stenosis MD (TOP 7 - AUG EXAM - FEB
the past few months, noted episodes of interrupted determines cyanosis 2014 MED 2015
physical play, episodes of bluish tinge over the lips BOARDS;
and fingers, noted behavioral changes like TOPNOTCH MD)
squatting, the degree of cyanosis is proportional to
the severity of which anatomic defect?
A. VSD
B. overriding aorta
C. RVH
D. pulmonic stenosis
E. aortic stenosis
253 An 8 year old male child was brought for consult for SIMILAR TO PREVIOUS BOARD EXAM KEVIN BRYAN LO, MIDTERM 2
an 8 month history of gradually enlarging cheek CONCEPT/PRINCIPLE, rapidly expanding mass, local MD (TOP 7 - AUG EXAM - FEB
mass 5 by 5cm on the right cheek, there was noted mass effects causing signs and symptoms, not much 2014 MED 2015
some difficulty of breathing and nasal obstruction, systemic manifestations as heralded by other cancers BOARDS;
frequent complaints of headaches, and mild pain especially hematological TOPNOTCH MD)
over the mass.
A. acute lymphocytic leukemia
B. non hodgkin's lymphoma
C. rhabdomyosarcoma
D. ewing's sarcoma
E. salivary gland mixed tumor
254 A 12 year old male presented to the clinic jaundice SIMILAR TO PREVIOUS BOARD EXAM KEVIN BRYAN LO, MIDTERM 2
and edema. Patient noted decreased urine output CONCEPT/PRINCIPLE, history of previous infection MD (TOP 7 - AUG EXAM - FEB
accompanied by bipedal pitting edema. 5 days gastroenteritis, jaundice, renal failure, 2014 MED 2015
prior, patient had a bout of watery diarrhea with thrombocytopenia = HUS BOARDS;
fever which resolved spontaenously. Patient TOPNOTCH MD)
presented with some petechia over the lower
extremities. Evaluation revealed anemia with Hgb
of 10g/dL WBC of 12000, platelet count of 70,000
which of the following conditions might be present?
A. Thrombotic thrombocytopenic purpura
B. hemolytic uremic syndrome
C. Iga nephropathy
D. henoch schonlein purpura
E. none of the above
255 A 16 year old female patient came in due to bipedal patient most likely has IgA nephropathy, with signs of KEVIN BRYAN LO, MIDTERM 2
pitting edema. On further probing, patient had autoimmunity joint pains, abdominal pain, previous MD (TOP 7 - AUG EXAM - FEB
occasional abdominal pains, joint pains involving URTI, relatively normal laboratory parameters except 2014 MED 2015
the ankles and knees intermittently during the past hematuria in the urine, no other symptoms noted, BOARDS;
week after a bout of cough and colds after she came FSGS causes nephrotic syndrome TOPNOTCH MD)
down with a flu. Occasional mild fevers were also
noted with no other remarkable symptoms noted.
CBC revealed Hgb of 13, WBC of 6000, Platelet
count of 200,000, no other symptoms were noted.
Urinalysis revealed RBC 5-10/hpf, WBC 0-1, with
(+) RBC casts, protein +1, which of the following
conditions does the patient most likely have?
A. IgA nephropathy
B. Hemolytic uremic syndrome

TOPNOTCH MEDICAL BOARD PREP PEDIATRICS SUPEREXAM Page 36 of 97


For inquiries visit www.topnotchboardprep.com.ph or email us at topnotchmedicalboardprep@gmail.com
TOPNOTCH MEDICAL BOARD PREP PEDIATRICS SUPEREXAM
For inquiries visit www.topnotchboardprep.com.ph or email us at topnotchmedicalboardprep@gmail.com
Item QUESTION EXPLANATION AUTHOR TOPNOTCH
# EXAM
C. henoch schonlein purpura
D. focal segmental glomerulosclerosis
E. none of the above

256 16 year old male student in a dormitory presented SIMILAR TO PREVIOUS BOARD EXAM KEVIN BRYAN LO, MIDTERM 2
with increasing daytime sleepiness, irritability and CONCEPT/PRINCIPLE, most deaths occur when there MD (TOP 7 - AUG EXAM - FEB
headache, patient noted to have nuchal rigidity and is blood infection, meningococcemia without 2014 MED 2015
fever over the past 2 days. After a course of 8 hours, meningitis = high rate of death compared to BOARDS;
patient was noted to have purpuric rashes from the meningitis alone TOPNOTCH MD)
trunk extending to the lower extremities, which of
the following found in the patient infers a better
prognosis in terms of this disease?
A. meningococcal bacteremia
B. meningitis
C. waterhouse friederichsen syndrome
D. septicemia
E. septic shock and low WBC count
257 A 5 year old male patient was brought to the clinic SIMILAR TO PREVIOUS BOARD EXAM KEVIN BRYAN LO, MIDTERM 2
because his mother noticed a gradual enlarging CONCEPT/PRINCIPLE, abdominal mass does not cross MD (TOP 7 - AUG EXAM - FEB
abdominal mass since 6 months ago which she midline hematuria - wilms tumor, no other symptoms 2014 MED 2015
didn't mind at first until she noticed it growing in noted BOARDS;
size while she was bathing the child and there was TOPNOTCH MD)
onset of tea colored urine observed for the past 2
months. Physical exam was unremarkable except
for the 5x5cm painless abdominal mass which does
not cross the midline. There was also noted
hematuria on futher workup which of the following
conditions does the child probably have?
A. neuroblastoma
B. nephroblastoma
C. rhabdmyosarcoma
D. lymphoma
E. none of the above
258 A male neonate was born to a G1P1 mother, on recalled from the board exams, baby looks mature KEVIN BRYAN LO, MIDTERM 2
grading the newborn's maturity, all extremities based on the limited data seems term 36-38 weeks MD (TOP 7 - AUG EXAM - FEB
recall immediately to flexed position, square same with what was given during our boards 2014 MED 2015
window less than 30 degrees, heal to ear test heal BOARDS;
only up to the level of genitals, sparse lanugo, TOPNOTCH MD)
creases anterior 2/3, testes down with good rugae,
raised areola 3-4mm bud, what is the approximate
maturity in weeks of the baby?
A. 32-34 weeks
B. 36-38 weeks
C. 40-42 weeks
D. 28-30 weeks
E. none of the above
259 A 5 year old female infant was noted by the mother recalled from the board exams, unsteady gait, KEVIN BRYAN LO, MIDTERM 2
to be always clumsy and falls easily while walking heterogenous calcification over the mass on the MD (TOP 7 - AUG EXAM - FEB
and playing, she was also noted to walk with an cerebellar area 2014 MED 2015
unsteady gait, further workup upon consult showed BOARDS;
a mass located centrally on the cerebellum with TOPNOTCH MD)
heterogenous calcifications, which of the following
is the most likely condition?
A. glioblastoma
B. astrocytoma
C. medulloblastoma
D. meningioma
E. all of the above
260 The predominant pathophysiologic mechanism increased capillary permeability is responsible for 3rd KEVIN BRYAN LO, MIDTERM 2
present in dengue shock syndrome is due to? spacing and dengue shock syndrome MD (TOP 7 - AUG EXAM - FEB
A. increased capillary permeability 2014 MED 2015
B. septic shock from secondary bacterial infection BOARDS;
C. hemmorhage secondary to thrombocytopenia TOPNOTCH MD)
D. dehydration secondary to fever and decreased
intake
E. none of the above
261 A newborn patient born via NSVD was observed to Characteristic description of choanal atresia RAYMUND MIDTERM 3
be cyanotic in the nursery. There was noted MARTIN LI, MD EXAM - FEB
difficulty upon trying to insert a feeding catheter (TOP 1 - AUG 2015
through the nose. The neonate was observed to 2014 MED
turn pink upon crying. What is the diagnosis? BOARDS;
A. Tracheo-esophageal fistula TOPNOTCH MD)
B. Congenital Heart Disease
C. Choanal Atresia
D. Cleft palate
E. Respiratory distress syndrome
TOPNOTCH MEDICAL BOARD PREP PEDIATRICS SUPEREXAM Page 37 of 97
For inquiries visit www.topnotchboardprep.com.ph or email us at topnotchmedicalboardprep@gmail.com
TOPNOTCH MEDICAL BOARD PREP PEDIATRICS SUPEREXAM
For inquiries visit www.topnotchboardprep.com.ph or email us at topnotchmedicalboardprep@gmail.com
Item QUESTION EXPLANATION AUTHOR TOPNOTCH
# EXAM
262 You are assessing a child's heart sounds. Which of An innocent physiologic murmur is grade 2/6 or less, RAYMUND MIDTERM 3
the following will you most likely suspect as systolic, soft, musical, and usually heard better during MARTIN LI, MD EXAM - FEB
pathologic? fever or anxious states (TOP 1 - AUG 2015
A. Heard during fever, anxiety, or an infectious 2014 MED
illness BOARDS;
B. Vibrating and musical in quality TOPNOTCH MD)
C. Systolic murmur heard over the left lower
sternum with a grade of 2/6
D. A soft, faintly heard diastolic murmur with a
grade of 1/6
E. None of the above
263 What is the most common cause of acute renal Post-streptococcal glomerulonephritis and minimal RAYMUND MIDTERM 3
failure in children? change disease do not usually lead to renal failure. MARTIN LI, MD EXAM - FEB
A. Hemolytic uremic syndrome Most cases resolve (TOP 1 - AUG 2015
B. Post-streptococcal glomerulonephritis 2014 MED
C. Minimal change disease BOARDS;
D. Henoch Schonlein purpura TOPNOTCH MD)
E. SLE nephritis

264 You are treating a pediatric patient with anemia. An increase of reticulocyte count by the 3rd day RAYMUND MIDTERM 3
Your primary impression is iron-deficiency. What indicates response to iron therapy. Hgb levels don't MARTIN LI, MD EXAM - FEB
laboratory test will you request on the 3rd day after usually start to rise until after the 4th day (TOP 1 - AUG 2015
the start of treatment to determine response? 2014 MED
A. Hemoglobin level BOARDS;
B. Serum ferritin TOPNOTCH MD)
C. Serum iron
D. Reticulocyte count
E. Bone marrow aspiration
265 What is the most common type of congenital heart Endocardial cushion defect is the most common CHD RAYMUND MIDTERM 3
defect in patients with trisomy 21? in trisomy 21 MARTIN LI, MD EXAM - FEB
A. VSD (TOP 1 - AUG 2015
B. ASD 2014 MED
C. Tricuspid atresia BOARDS;
D. Patent ductus arteriosus TOPNOTCH MD)
E. Endocardial cushion defect

266 A term neonate born to a 32 year old mother via TTN usually occurs in term newborn, cesarean RAYMUND MIDTERM 3
cesarean delivery for CPD, becomes tachypneic. delivery, rapid 2nd stage of labor; usually resolves MARTIN LI, MD EXAM - FEB
Chest X-ray reveals air trapping, fluid in fissures with minimal oxygen requirement (TOP 1 - AUG 2015
and perihilar streaking? What is the most likely 2014 MED
diagnosis BOARDS;
A. Hyaline membrane disease TOPNOTCH MD)
B. Meconium aspiration syndrome
C. Transient tachypnea of the newborn
D. Bronchopulmonary dysplasia
E. None of the above
267 A 6 year old female patient is brought to you by her Hepatosplenomegaly is not consistent with ITP and RAYMUND MIDTERM 3
mother due to complaints of petechiae, purpura, indicates a different diagnosis. All others are MARTIN LI, MD EXAM - FEB
and bleeding after a dental tooth extraction. She characteristics of ITP (TOP 1 - AUG 2015
was previously healthy with no significant history. 2014 MED
Which of the following will NOT support a BOARDS;
diagnosis of Idiopathic thrombocytopenic purpura? TOPNOTCH MD)
A. Severe thrombocytopenia < 20,000
B. Normal to increased megakaryocytes in the bone
marrow
C. Presence of hepatosplenomegaly
D. Preceded by a history of viral infection 2-3 weeks
prior
E. None of the above
268 A newborn patient born to a G1P1 mother was Severe cyanosis at birth with associated decreased RAYMUND MIDTERM 3
noted to have severe cyanosis. Physical pulmonary circulation at X-ray and LVH with left axis MARTIN LI, MD EXAM - FEB
examination revealed a holosystolic murmur along deviation is charfacteristic of tricuspid atresia (TOP 1 - AUG 2015
the left sternal border and ECG shows left axis 2014 MED
deviation with left ventricular hypertrophy. What is BOARDS;
the most likely diagnosis? TOPNOTCH MD)
A. Tricuspid atresia
B. VSD
C. ASD
D. Transposition of the great arteries
E. Tetralogy of Fallot
269 A 5 year old patient is brought to your clinic. Her Gower sign, pseudohypertrophy of the calf muscles, RAYMUND MIDTERM 3
parents claim that the child started to show scoliosis, lordotic posturing - signs of Duchenne MARTIN LI, MD EXAM - FEB
difficulty in attempting to stand from a sitting muscular dystrophy, an X-linked recessive disorder (TOP 1 - AUG 2015
position and has lordotic posturing. They can also 2014 MED
recall that they noted poor head control when he BOARDS;
was an infant. Physical examination reveals TOPNOTCH MD)
hypertrophy of the calf muscles. What is the
diagnosis?
A. Amyotrophic lateral sclerosis
B. Duchenne muscular dystrophy
C. Charcot Marie Tooth Disease
D. Myotonic dystrophy
E. Gullain-Barre syndrome

TOPNOTCH MEDICAL BOARD PREP PEDIATRICS SUPEREXAM Page 38 of 97


For inquiries visit www.topnotchboardprep.com.ph or email us at topnotchmedicalboardprep@gmail.com
TOPNOTCH MEDICAL BOARD PREP PEDIATRICS SUPEREXAM
For inquiries visit www.topnotchboardprep.com.ph or email us at topnotchmedicalboardprep@gmail.com
Item QUESTION EXPLANATION AUTHOR TOPNOTCH
# EXAM
270 A 10 year old patient is diagnosed with Acute Prednisone is only given to patients with RAYMUND MIDTERM 3
Rheumatic fever. Which of the following is true carditis/CHF; Phenobarbital is the drug of choice if MARTIN LI, MD EXAM - FEB
regarding the treatment of this condition? chorea is an isolated finding; Penicillin prophylaxis is (TOP 1 - AUG 2015
A. Prednisone is usually given to patients with or may be given for life in patients with carditis but only 2014 MED
without carditis or CHF symptoms until the early twenties in those without out. IM Pen G BOARDS;
B. Penicillin prophylaxis therapy is given to patients prophylaxis is given once every 4 weeks TOPNOTCH MD)
with no carditis or cardiac complications only up to
their early twenties
C. Aspirin is the drug of choice for patient who
present with chorea alone
D. A daily dose of intramuscular benzathine
penicillin G is required for prophylaxis
E. None of the above
271 An 11 month old infant is seen in the ER presenting This is a case of viral croup which requires supportive RAYMUND MIDTERM 3
with dyspnea. His mother tells you that it all started management which may include nebulized MARTIN LI, MD EXAM - FEB
with coryza, cough, and low-grade fever and then epinephrine. No need to intubate unless patient is in (TOP 1 - AUG 2015
with recent onset of barky cough that is getting severe respiratory distress/hemodynamically 2014 MED
worse. Upon examining the infant, you hear seal- unstable BOARDS;
like coughing and there is hoarseness and TOPNOTCH MD)
inspiratory stridor. Other than tachypnea, vital
signs remain stable. What will be an appropriate
management?
A. IV antibiotic therapy
B. Intubate immediately
C. Nebulized epinephrine
D. Ribavirin therapy
E. None of the above
272 Which of the following pediatric diarrheal Patients with viral gastroenteritis, HUS, and RAYMUND MIDTERM 3
conditions will require antibiotic therapy? nontyphoidal salmonellosis will usually not require MARTIN LI, MD EXAM - FEB
A. A child presenting with watery diarrhea with a antibiotic therapy. (TOP 1 - AUG 2015
suspected viral cause 2014 MED
B. A child with diarrhea due to ingestion of BOARDS;
nontyphoidal salmonella-infected, improperly TOPNOTCH MD)
cooked eggs
C. A child who presents with bloody diarrhea that is
associated with anemia, thrombocytopenia, and
hypertension
D. A child with high fever and mucoid stool with
frank blood
E. B, C, and D
273 A neonate is born to a mother with known HIV All infants < 18 months with or without HIV will test RAYMUND MIDTERM 3
infection. The mother did not receive any positive for IgG antibodies because maternal IgG MARTIN LI, MD EXAM - FEB
antiretroviral treatment during pregancy. What test crosses the placenta so testing should done using HIV- (TOP 1 - AUG 2015
will you request to screen the newborn for HIV? DNA PCR 2014 MED
A. HIV-DNA PCR BOARDS;
B. IgG Ab by ELISA TOPNOTCH MD)
C. IgG Ab by Western blot
D. CD4 count
E. None of the above
274 A 8 year old patient who had a previous skin All of the choices are anti-streptococcal antibodies RAYMUND MIDTERM 3
infection 3 weeks prior now presents with which may be elevated after a GABHS throat infection. MARTIN LI, MD EXAM - FEB
hematuria, elevated BP, and bilateral lower However, in GABHS pyoderma or skin infection, anti- (TOP 1 - AUG 2015
extremity edema. What test will you use to Dnase B is the best test because the others may be 2014 MED
document previous GABHS infection? falsely low or negative BOARDS;
A. anti-ASO TOPNOTCH MD)
B. anti-Hyaluronidase
C. anti-Streptokinase
D. anti-NAD
E. anti-DNase B
275 What is the most common associated congenital RAYMUND MIDTERM 3
anomaly in a patient with esophgeal atresia and MARTIN LI, MD EXAM - FEB
tracheoesophageal fistula? (TOP 1 - AUG 2015
A. Vertebral anomalies 2014 MED
B. Imperforate anus BOARDS;
C. Cardiac anomalies TOPNOTCH MD)
D. Genitourinary anomalies
E. Limb defects

276 Which is of the following is true regarding brain Infratentorial is most common. Benign astrocytoma is RAYMUND MIDTERM 3
tumors in children? the most common brain tumor followed by MARTIN LI, MD EXAM - FEB
A. Most commonly supratentorial in location in medulloblastoma. ALL is the most common (TOP 1 - AUG 2015
children <10 malignancy in children but brain tumors is the most 2014 MED
B. Benign astrocytoma is the most common brain common solid malignancy. BOARDS;
tumor in children TOPNOTCH MD)
C. Brain is the most frequent solid malignancy in
children
D. A and B
E. B and C

TOPNOTCH MEDICAL BOARD PREP PEDIATRICS SUPEREXAM Page 39 of 97


For inquiries visit www.topnotchboardprep.com.ph or email us at topnotchmedicalboardprep@gmail.com
TOPNOTCH MEDICAL BOARD PREP PEDIATRICS SUPEREXAM
For inquiries visit www.topnotchboardprep.com.ph or email us at topnotchmedicalboardprep@gmail.com
Item QUESTION EXPLANATION AUTHOR TOPNOTCH
# EXAM
277 A 4 week-old infant is brought to the emergency Ultrasound is the best test to conform cases of RAYMUND MIDTERM 3
room by his mother. She claims that there is a congenital pyloric stenosis MARTIN LI, MD EXAM - FEB
recent onset of projectile vomiting after feeding and (TOP 1 - AUG 2015
that the infant usually appears hungry and wants to 2014 MED
feed more. Physical examination of the child reveals BOARDS;
mild dehydration and a firm, movable mass is TOPNOTCH MD)
palpated. What is the best test to order to confirm
this diagnosis?
A. Ultrasound
B. Abdominal X-ray
C. CT scan
D. MRI
E. None of the above
278 Which of the following immunizations may given to Live vaccines are usually contraindicated in children RAYMUND MIDTERM 3
a child with diagnosed HIV/AIDS who is not with HIV/AIDS but MMR and varicella may be given if MARTIN LI, MD EXAM - FEB
severely immunocompromised? the child is not severely immunocompromised (TOP 1 - AUG 2015
A. OPV 2014 MED
B. Vaccinia BOARDS;
C. BCG TOPNOTCH MD)
D. MMR
E. None of the above

279 An 11 month old infant presented at the ER with A is bacterial, B is partially treated bacterial, C is RAYMUND MIDTERM 3
fever, irritability, and nausea and vomiting. viral/aseptic MARTIN LI, MD EXAM - FEB
Currently, patient is now lethargic and still febrile. (TOP 1 - AUG 2015
P.E. fails to reveal meningeal irritation but you still 2014 MED
suspect meningitis so you perform a lumbar tap. BOARDS;
Which of the following CSF findings will indicate a TOPNOTCH MD)
mycobacterial etiology?
A. 3000 cells/mL, predominantly PMN, low glucose,
high protein, high pressure
B. 200 cells/mL, predominantly PMN, low glucose,
high protein, high pressure
C. 350 cells/mL, mixed PMN and lymphocytes,
normal glucose, slightly high protein, normal
pressure
D. 400 cells/mL, predominantly lymphocytes, low
glucose, high protein, high pressure
E. None of the above
280 All of the following cases of urinary tract infection Cases that will require additional assessment with RAYMUND MIDTERM 3
in pediatric patients will require additional VCUG include: All males, females <5yo, females >5yo MARTIN LI, MD EXAM - FEB
assessment with Voiding cystourethrogram (VCUG) with a second UTI, Febrile UTI (TOP 1 - AUG 2015
EXCEPT? 2014 MED
A. All Males with UTI BOARDS;
B. Females > 5 years old with a first episode of UTI TOPNOTCH MD)
C. Febrile UTI
D. All cases will require VCUG
281 What is the easiest way to confirm the diagnosis of Abdominal ultrasound is a helpful diagnostic tool in ERIC ROYD FINAL EXAM -
bilary atresia in a 1 month old infant presenting the evaluation of neonatal cholestasis because it will TALAVERA, MD FEB 2015
with cholestatic jaundice? identify choledocholithiasis, perforation of the bile (TOP 1 - AUG
A. Presence of a triangular cord sign in the duct, or other structural abnormalities of the biliary 2014 MED
ultrasound of the liver tree such as a choledochal cyst. In patients with biliary BOARDS;
B. Intact lobular structure on liver biopsy atresia, ultrasound may detect associated anomalies TOPNOTCH MD)
C. Poor hepatic uptake of radioactive material but such as abdominal polysplenia and vascular
with slow excretion into the gut malformations. Ultrasonographic triangular cord (TC)
D. Firm consistency of the liver on palpation sign, which represents a cone-shaped fibrotic mass
E. Elevated levels of anti-mitochondrial antibodies cranial to the bifurcation of the portal vein, may be
seen in patients with biliary atresia
282 A 13 year old girl sought consult due to easy A child with an ostium secundum ASD is most often ERIC ROYD FINAL EXAM -
fatigability and poor weight gain. PE was asymptomatic; the lesion may be discovered TALAVERA, MD FEB 2015
essentially normal except for a grade 3/6 systolic inadvertently during physical examination. Even an (TOP 1 - AUG
murmur at the 2nd left interspace and a fixed extremely large secundum ASD rarely produces 2014 MED
splitting of S2. What Is the diagnosis for this case? clinically evident heart failure in childhood. In most BOARDS;
A. Coarctation of the aorta patients, the 2nd heart sound is characteristically TOPNOTCH MD)
B. VSD widely split and fixed in its splitting in all phases of
C. ASD respiration. . A systolic ejection murmur is heard; it is
D. PDA medium pitched, without harsh qualities, seldom
E. TGA accompanied by a thrill, and best heard at the left
middle and upper sternal border
283 Based on the ARIA guidelines, a patient is classified Intermittent AR: Symptoms occur < 4 days a week or ERIC ROYD FINAL EXAM -
to have persistent allergic rhinitis if which of the <4 consecutive weeks. Mild AR: normal sleep, no TALAVERA, MD FEB 2015
following is present? impairment of ADL, school or work. Moderate to (TOP 1 - AUG
A. Symptoms occur < 4 days a week severe: troublesome sleep, impairment of ADL, work 2014 MED
B. Symptoms occur < 4 consecutive weeks and school BOARDS;
C. Impairment of activities of daily living TOPNOTCH MD)
D. Symptoms occur > 4 days a week OR > 4
consecutive weeks
E. Symptoms occur > 4 days a week AND > 4
consecutive weeks

TOPNOTCH MEDICAL BOARD PREP PEDIATRICS SUPEREXAM Page 40 of 97


For inquiries visit www.topnotchboardprep.com.ph or email us at topnotchmedicalboardprep@gmail.com
TOPNOTCH MEDICAL BOARD PREP PEDIATRICS SUPEREXAM
For inquiries visit www.topnotchboardprep.com.ph or email us at topnotchmedicalboardprep@gmail.com
Item QUESTION EXPLANATION AUTHOR TOPNOTCH
# EXAM
284 Which of the following if present confers a better Poor prognostic factors on presentation include ERIC ROYD FINAL EXAM -
prognosis for meningococcemia? hypothermia or extreme hyperpyrexia, hypotension TALAVERA, MD FEB 2015
A. Leukopenia or shock, purpura fulminans, seizures, leukopenia, (TOP 1 - AUG
B. Presence of petechiae <12 hours before thrombocytopenia (including DIC), acidosis, and high 2014 MED
admission circulating levels of endotoxin and TNF-α. The BOARDS;
C. Nuchal rigidity presence of petechiae for <12 hr before admission, TOPNOTCH MD)
D. Hypothermia absence of meningitis, and low or normal erythrocyte
E. Low ESR sedimentation rate indicate rapid, fulminant
progression and poorer prognosis.
285 An 8 year old girl was brought in for declining Simple (typical) absence (petit mal) seizures are ERIC ROYD FINAL EXAM -
academic performance. Her teachers have reported characterized by a sudden cessation of motor activity TALAVERA, MD FEB 2015
that she is frequently "daydreaming" in class and or speech with a blank facial expression and flickering (TOP 1 - AUG
inattentive. The doctor hyperventilates her in the of the eyelids. Patients do not lose body tone, but their 2014 MED
clinic and she manifests with blank stares and head may fall forward slightly. Immediately after the BOARDS;
clonic movements of the eyes, there was no post seizure, patients resume preseizure activity with no TOPNOTCH MD)
ictal drowsiness noted. If an EEG would be indication of postictal impairment. Automatic
performed on this patient, what would be the behavior frequently accompanies simple absence
expected findings? seizures. Hyperventilation for 3–4 min routinely
A. Normal tracing produces an absence seizure. The EEG shows a typical
B. Typical 3/sec spike and generalized wave 3/sec spike and generalized wave discharge
discharge
C. Fast spike wave complexes of ≥2.5 Hz and a
normal background rhythm
D. 4–6/sec irregular spike and wave pattern,
which is enhanced by photic stimulation
E. Chaotic pattern of high-voltage, bilaterally
asynchronous, slow-wave activity
286 A newborn infant was delivered via elective CS at Dx: Choanal atresia. Infants with bilateral choanal ERIC ROYD FINAL EXAM -
38 weeks AOG, with an apgar score of 8,9. On atresia who have difficulty with mouth breathing TALAVERA, MD FEB 2015
feeding, he was observed to become cyanotic and make vigorous attempts to inspire, often suck in their (TOP 1 - AUG
would often suck in his lips. The cyanosis would be lips, and develop cyanosis. Distressed children then 2014 MED
relieved once he begins to cry. Which of the cry (which relieves the cyanosis) and become calmer, BOARDS;
following will establish the diagnosis in this case? with normal skin color. Diagnosis is established by TOPNOTCH MD)
A. CT scan the inability to pass a firm catheter through each
B. Administering 100% oxygen nostril 3–4 cm into the nasopharynx. The atretic plate
C. Cranial MRI may be seen directly with fiberoptic rhinoscopy. The
D. Inserting an NGT anatomy is best evaluated by using high-resolution CT
E. Rhinoscopy
287 A couple sought consult for genetic counseling. On For X-linked recessive inheritance usually the male ERIC ROYD FINAL EXAM -
history, they relayed that both their children, one offspring would be affected, while the female offspring TALAVERA, MD FEB 2015
male and one female died at 1 week of life due to an would be a carrier. For X-linked and autosomal (TOP 1 - AUG
acute encephalopathy. Both parents are healthy, dominant, the parents would also manifest with 2014 MED
with no known co-morbids, asymptomatic and non- similar symptoms to their offsprings. Autosomal BOARDS;
consanguineous. No other relatives are affected or recessive pattern is more compatible since both TOPNOTCH MD)
have died under similar circumstances. What is the parents are normal but carry the genes (carriers) both
most probable pattern of inheritance in this case? children were affected since they inherited both sets
A. Autosomal recessive of genes from the mother and father
B. Autosomal dominant
C. X-linked recessive
D. X-linked dominant
E. All of the above
288 A 9 year old boy was brought to the clinic due to Dx: ALL. In general, the initial therapy is designed to ERIC ROYD FINAL EXAM -
pallor, easy fatigability and numerous petechiae. A eradicate the leukemic cells from the bone marrow; TALAVERA, MD FEB 2015
CBC was done which showed anemia, this is known as remission induction. During this (TOP 1 - AUG
thrombocytopenia and leukocytosis. A bone phase, therapy usually is given for 4 wk. Staging of 2014 MED
marrow aspiration was subsequently done which ALL is based partly on a cerebrospinal fluid (CSF) BOARDS;
showed numerous lymphoblasts. Which of the examination. If lymphoblasts are found and the CSF TOPNOTCH MD)
following statements regarding the patient's leukocyte count is elevated, overt CNS or meningeal
disease is true? leukemia is present. Hepatomegaly is an uncommon
A. Initial therapy is usually given for a period of 7 manifestation. Children >10 and with an inital WBC
days. count >50,000 are at higher risk
B. CSF examination is essential for staging the
disease
C. Children who are >10 yr of age or who have an
initial leukocyte count of >50,000/μL are
considered to be at a lower risk
D. Hepatomegaly is a common manifestation of
this disease
E. Treatment is the single most important
prognostic factor
289 A 5 year old girl presented with a 2 month history Juvenile dermatomyositis (JDM), the most common of ERIC ROYD FINAL EXAM -
of progressive difficulty in ambulation, particularly the pediatric inflammatory myopathies, is TALAVERA, MD FEB 2015
in climbing up and down the stairs. After going on a distinguished by a characteristic rash and proximal, (TOP 1 - AUG
field trip, she came home with an erythematous symmetric muscle weakness that is often responsive 2014 MED
rash in the malar area and violaceous discoloration to the immunosuppressive therapy. The rash often has BOARDS;
with swelling over the periorbital area. On the day onset in sun-exposed areas and develops as the first TOPNOTCH MD)
of consult, she was noted to have papules over the symptom in 50% of cases, and concomitantly with
elbows and interphalangeal joints. What is the most weakness in 25% of cases. The characteristic
likely diagnosis of this case? periorbital violaceous erythema (heliotrope) may
A. Systemic Scleroderma cross the bridge of the nose, in a masklike distribution,
B. Juvenile Dermatomyositis and involve the ears as well. The rash is often florid
C. Juvenile Rheumatoid Arthritis and is usually palpable over joints, especially the
D. Systemic Lupus Erythematosus metacarpal phalangeal, intercarpal phalangeal
E. Ankylosing Spondylitis (Gottron papules), knees, elbows, and medial malleoli
of the ankles

TOPNOTCH MEDICAL BOARD PREP PEDIATRICS SUPEREXAM Page 41 of 97


For inquiries visit www.topnotchboardprep.com.ph or email us at topnotchmedicalboardprep@gmail.com
TOPNOTCH MEDICAL BOARD PREP PEDIATRICS SUPEREXAM
For inquiries visit www.topnotchboardprep.com.ph or email us at topnotchmedicalboardprep@gmail.com
Item QUESTION EXPLANATION AUTHOR TOPNOTCH
# EXAM
290 A 3 year old girl was brough to the ER due to pallor, The hemolytic-uremic syndrome (HUS) is the most ERIC ROYD FINAL EXAM -
lethargy and decreased urine output. The said common cause of acute renal failure in young children. TALAVERA, MD FEB 2015
symptoms were preceded by a 5 day history of It is classically characterized by the triad of (TOP 1 - AUG
bloody diarrhea and fever. Blood tests revealed microangiopathic hemolytic anemia, 2014 MED
anemia, thrombocytopenia and an elevated thrombocytopenia, and uremia.acute enteritis with BOARDS;
creatinine level. What is the most common diarrhea caused by Shiga-like toxin–producing TOPNOTCH MD)
causative agent that initiated the events that led to Escherichia coli 0157: H7 precedes 80% or more of
the patient's currrent condition ? HUS cases
A. Giardia lamblia
B. Vibrio cholerae
C. Escherichia coli
D. Staphylococcus aureus
E. Cytomegalovirus
291 A 6 year old girl was admitted in the PICU because Cerebral salt wasting appears to be the result of ERIC ROYD FINAL EXAM -
of multiple organ injuries secondary to a vehicular hypersecretion of atrial natriuretic peptide and is seen TALAVERA, MD FEB 2015
accident. On the 2nd hosptial day, there was note of primarily with central nervous system disorders (TOP 1 - AUG
sudden increase in urine output. Blood and urine including brain tumors, head trauma, hydrocephalus, 2014 MED
tests were done which showed a serum sodium of neurosurgery, cerebral vascular accidents, and brain BOARDS;
115 mEq/L , urine sodium of 160 mEq/L and a low death. Hyponatremia is accompanied by elevated TOPNOTCH MD)
serum ADH. What is the most likely diagnosis for urinary sodium excretion (often >150 mEq/L),
this case? excessive urine output, hypovolemia, normal or high
A. SIADH uric acid, suppressed vasopressin, and elevated atrial
B. Cerebral Salt Wasting natriuretic peptide concentrations (>20 pmol/L).
C. Central Diabetes Insipidus Thus, it is distinguished from SIADH, in which normal
D. Nephrogenic Diabetes Insipidus or decreased urine output, euvolemia, low uric acid,
E. Pseudohyponatremia only modestly elevated urine sodium concentration,
and an elevated vasopressin level occur.
292 Which of the following statements best describes a Pathologic jaundice: (1) it appears in the 1st 24–36 hr ERIC ROYD FINAL EXAM -
"physiologic" jaundice in the newborn? of life, (2) serum bilirubin is rising at a rate faster than TALAVERA, MD FEB 2015
A. Direct bilirubin >10% of the total bilirubin 5 mg/dL/24 hr, (3) serum bilirubin is >12 mg/dL in (TOP 1 - AUG
B. Present in the first 24 hours of life full-term infants (especially in the absence of risk 2014 MED
C. Resolves on the 10th day of life factors) or 10–14 mg/dL in preterm infants, (4) BOARDS;
D. Rate of rise of bilirubin is 6 mg/dL/24hr jaundice persists after 10–14 days of life, or (5) direct- TOPNOTCH MD)
E. None of the above reacting bilirubin is >2 mg/dL at any time.

293 A 15 month old male presented with a high grade The presence of koplik spots is highly pathognomonic ERIC ROYD FINAL EXAM -
fever of 3 days, cough, coryza, conjunctivits and for measles TALAVERA, MD FEB 2015
multiple erythematous maculopapular rashes (TOP 1 - AUG
appearing around the hairline and spreading 2014 MED
downwards. The rashes are more confluent in the BOARDS;
upper extremities and exhibit branny TOPNOTCH MD)
desquamation. Discrete red lesions with white
spots are noted in the buccal mucosa at the level of
the premolars. What is the most likely diagnosis for
this case?
A. Infectious Mononucleosis
B. Varicella
C. Roseola infantum
D. Rubella
E. Rubeola
294 Which of the following statements does not Pneumonia, lymphadenitis, osteomyelitis, and skin ERIC ROYD FINAL EXAM -
describe Chronic Granulomatous Disease (CGD)? infections are the most common illnesses encountered TALAVERA, MD FEB 2015
A. It is characterized by the inability of in CGD. Bacteremia or fungemia occur but are much (TOP 1 - AUG
neutrophils and macrophage to kill catalase- less common than focal infections 2014 MED
positive microorganisms BOARDS;
B. The most common pathogen involved in TOPNOTCH MD)
recurrent infections is S. aureus
C. Bacteremia and fungemia are more common
than local infections
D. For screening of CGD, the nitroblue tetrazolium
(NBT) dye test is used
E. Patients with CGD should be given daily oral
trimethoprim-sulfamethoxazole for prophylaxis of
infections
295 Delayed dental eruption is usually considered when Delayed eruption is usually considered when there are ERIC ROYD FINAL EXAM -
there are no teeth noted at what age (in months)? no teeth by approximately 13 mo of age (mean + 3 TALAVERA, MD FEB 2015
A. 24 months standard deviations). Common causes include (TOP 1 - AUG
B. 8 months hypothyroid, hypoparathyroid, familial, and (the most 2014 MED
C. 16 months common) idiopathic. BOARDS;
D. 13 months TOPNOTCH MD)
E. 48 months

296 A 10 year old boy presented with a 4 day history of Peritonsillar cellulitis/abscess, which is relatively ERIC ROYD FINAL EXAM -
fever, sore throat, dysphagia and odynophagia. On common compared to the deep neck infections, is TALAVERA, MD FEB 2015
PE his VS are BP 100/60, PR 118, RR 21, T 38.6. caused by bacterial invasion through the capsule of (TOP 1 - AUG
There was a noted asymmetric tonsillar bulge with the tonsil, leading to cellulitis and/or abscess 2014 MED
displacement of the uvula. The rest of the formation in the surrounding tissues. Clinical BOARDS;
examination was normal. What is the most likely manifestations include sore throat, fever, trismus, and TOPNOTCH MD)
diagnosis for this case? dysphagia. Physical examination reveals an
A. Peritonsillar abscess asymmetric tonsillar bulge with displacement of the
B. Laryngeotracheobronchitis uvula. An asymmetric tonsillar bulge is diagnostic, but
C. Retropharyngeal abscess it may be poorly visualized because of trismus
D. Diphtheria
E. Epiglottitis

TOPNOTCH MEDICAL BOARD PREP PEDIATRICS SUPEREXAM Page 42 of 97


For inquiries visit www.topnotchboardprep.com.ph or email us at topnotchmedicalboardprep@gmail.com
TOPNOTCH MEDICAL BOARD PREP PEDIATRICS SUPEREXAM
For inquiries visit www.topnotchboardprep.com.ph or email us at topnotchmedicalboardprep@gmail.com
Item QUESTION EXPLANATION AUTHOR TOPNOTCH
# EXAM
297 Which of the following is not a parameter for Posture, square window, arm recoil, popliteal angle, ERIC ROYD FINAL EXAM -
physical maturity in the Ballard Maturational scarf sign and heel to ear are parameters for TALAVERA, MD FEB 2015
Assessment? neuromuscular maturity (TOP 1 - AUG
A. Lanugo 2014 MED
B. Posture BOARDS;
C. Breast bud TOPNOTCH MD)
D. Plantar surface
E. Ears
298 A 4 year old boy was brough in due to abdominal Intussusception occurs when a portion of the ERIC ROYD FINAL EXAM -
pain. The abdominal pain is paroxysmal in nature alimentary tract is telescoped into an adjacent TALAVERA, MD FEB 2015
with the boy's leg drawn up and is accompanied by segment. It is the most common cause of intestinal (TOP 1 - AUG
passage of bloody stools. On abdominal obstruction between 3 mo and 6 yr of age. In typical 2014 MED
examination a palpable sausage shaped mass at the cases, there is sudden onset, in a previously well child, BOARDS;
right upper quadrant was appreciated. Rectal of severe paroxysmal colicky pain that recurs at TOPNOTCH MD)
examination revealed the presence of bloody frequent intervals and is accompanied by straining
mucus in the tactating finger. What is the most efforts with legs and knees flexed and loud cries.
likely diagnosis for this case? Palpation of the abdomen usually reveals a slightly
A. Hypertrophic pyloric stenosis tender sausage-shaped mass, sometimes ill defined,
B. Inflammatory bowel disease which may increase in size and firmness during a
C. Acute gastroenteritis paroxysm of pain and is most often in the right upper
D. Irritable bowel syndrome abdomen
E. Intussusception
299 Ataxia, myoclonus ("dancing feet") and opsoclonus Neuroblastoma (NB) is an embryonal cancer of the ERIC ROYD FINAL EXAM -
("dancing eyes") are manifestations of a peripheral sympathetic nervous system with TALAVERA, MD FEB 2015
paraneoplastic syndrome of what form of heterogeneous clinical presentation and course. NB (TOP 1 - AUG
malignancy? can present as a paraneoplastic syndrome of 2014 MED
A. Wilm's Tumor autoimmune origin manifesting as ataxia or BOARDS;
B. Osteosarcoma opsomyoclonus (“dancing eyes and dancing feet”). In TOPNOTCH MD)
C. Rhabdomyosarcoma such cases, the primary tumor is in the chest or
D. Neuroblastoma abdomen, and the brain is negative for tumor.
E. Medulloblastoma
300 At what age does the apex beat of a child shift from the apex beat is normally located at the 4th ICS MCL ERIC ROYD FINAL EXAM -
the 4th left ICS MCL to the 5th left ICS MCL? until 7 y/o when it shifts to the 5th ICS MCL TALAVERA, MD FEB 2015
A. 7 years old (TOP 1 - AUG
B. 10 years old 2014 MED
C. 3 years old BOARDS;
D. 15 years old TOPNOTCH MD)
E. None of the above

301 A 4 y/o female was referred to your clinic SIMILAR TO PREVIOUS BOARD EXAM LEAN ANGELO BACK-UP
secondary to a mass occupying the nasopharyngeal CONCEPT/PRINCIPLE. Rhabdomyosarcoma (RS) is the SILVERIO, MD MIDTERM
area. this is accompanied by congestion, recurrent most common pediatric soft tissue sarcoma. They are (TOP 4 - AUG EXAM - FEB
epistaxis and multiple cranial nerve palsies. Patient usually found in the head and neck, genitourinary, and 2014 MED 2015
is lethargic with complains of headache and retroperitoneal area. nasopharyngeal RS presents BOARDS;
recurrent projectile vomitting. what is the most with mass accompanied by epistaxis, congestion, TOPNOTCH MD),
likely diagnosis for this patient? dysphagia. regional extension leads to cranial nerve MD
A. rhabdomyosarcoma palsies, blindness and signs of increased intracranial
B. neuroblastoma pressure. medullablastoma is the most common
C. medulloblastoma malignant primary brain tumor in pediatrics. it is
D. chloroma intraaxial and will not present with mass in the
E. all of the above ? nasopharyngeal area. its most common site is the
cerebellar hemisphere. Neuroblastoma seldomly seen
in head and neck region. along with wilms, it usually
present with an abdominal mass. chloroma is a solid
collection of leukemic cells in acute myelogenous
leukemia. it was not stated in the case if the patient
presents with signs and symptoms of leukemia.
302 what vitamin/mineral deficiency is associated with Vitamin A - nyctalopsia, xeropthalmia, bitot spits, LEAN ANGELO BACK-UP
intention tremor, decreased proprioception, epiphyseal bone abnormalities. Vitamin B6 - SILVERIO, MD MIDTERM
nystagmus, dysarthria, and retinopathies? irritability, convulsion, hypochromic anemia, oxaluria, (TOP 4 - AUG EXAM - FEB
A. Vitamin A peripheral neuritis. Zinc - abdominal pain, diarrhea, 2014 MED 2015
B. Vitamin B6 vomitting, acrodermatitis enterohepatica. Vitamin E BOARDS;
C. Vitamin E deficiency is associated with severe neurologic TOPNOTCH MD),
D. Zinc manifestation. it usually presents with limb and MD
E. Selenium? truncal ataxia, ophthalmoplegia, positive rhomberg
test, dysarthria
303 A 12 y/o male went for consult secondary to tea MPGN is the most common cause of chronic LEAN ANGELO BACK-UP
colored urine. Physical examination: BP 150/90, PR glomerulonephritis in children and adolescent. Its SILVERIO, MD MIDTERM
102 RR 21 T 37.0C. Pale palpebral conjunctiva, presentation is very similar to APSGN however, the (TOP 4 - AUG EXAM - FEB
anicteric sclerae, clear breath sounds, soft, only difference is a persistently low C3 even after 2 2014 MED 2015
nontender abdomen. There was grade 1 bipedal months. BOARDS;
edema and with slight periorbital swelling. lab TOPNOTCH MD),
shows the following result (+) ASO, low C3, and MD
subneprotic range proteinuria. patient was
hospitalized for a week and was successfully
treated. he was lost to follow up, and just come
back after 3 months. laboratories still show
persistently low C3. what is your diagnosis ?
A. FSGS
B. MPGN
C. APSGN
D. MGN
E. RPGN?

TOPNOTCH MEDICAL BOARD PREP PEDIATRICS SUPEREXAM Page 43 of 97


For inquiries visit www.topnotchboardprep.com.ph or email us at topnotchmedicalboardprep@gmail.com
TOPNOTCH MEDICAL BOARD PREP PEDIATRICS SUPEREXAM
For inquiries visit www.topnotchboardprep.com.ph or email us at topnotchmedicalboardprep@gmail.com
Item QUESTION EXPLANATION AUTHOR TOPNOTCH
# EXAM
304 A 4 months old infant develops protracted Food protein-induced enterocolitis syndrome LEAN ANGELO BACK-UP
projectile vomiting, diarrhea and lethargy 2 hours typically manifests in the first several months of life SILVERIO, MD MIDTERM
after ingesting a powdered milk formula. What is with irritability, protracted vomiting and diarrhea, not (TOP 4 - AUG EXAM - FEB
the most likely diagnosis? infrequently resulting in dehydration. Vomiting 2014 MED 2015
A. Food protein induced enterocolitis generally occurs 1-3 hr following feeding, and BOARDS;
B. Allergic eosinophilic esophagitis continued exposure may result in bloody diarrhea, TOPNOTCH MD),
C. allergic eosinophilic gastroenteritis anemia, abdominal distention, and failure to thrive. MD
D. anaphylaxis Symptoms are most commonly provoked by cow's
E. none of the above milk- or soy protein-based formulas but occasionally
result from food proteins passed in maternal breast
milk.
305 Which of the following is a false statement the absolute lymphocyte count is usually only LEAN ANGELO BACK-UP
regarding DiGeorge syndrome? moderately low SILVERIO, MD MIDTERM
A. Very low absolute lymphocyte count for age. Lymphocyte responses to mitogen stimulation (TOP 4 - AUG EXAM - FEB
B. Equal male and female predisposition are 2014 MED 2015
C. Normal levels of serum Immunoglobulins absent, reduced, or normal, depending on the degree BOARDS;
D. Variable hypoplasia of the thymus and of TOPNOTCH MD),
parathyroids is common thymic deficiency. SIMILAR TO PREVIOUS BOARD MD
E. none of the above EXAM CONCEPT/PRINCIPLE
306 Recurrent meningococcal infection is correlated Congenital deficiencies of the terminal components of LEAN ANGELO BACK-UP
with what type of immunologic disorder? complement (C5, C6, C7, or C8) have been associated SILVERIO, MD MIDTERM
A. T cell defect with repeated neisseria infection. SIMILAR TO (TOP 4 - AUG EXAM - FEB
B. B cell defect PREVIOUS BOARD EXAM CONCEPT/PRINCIPLE 2014 MED 2015
C. Combined T and B cell defect BOARDS;
D. Phagocyte function defect TOPNOTCH MD),
E. None of the above MD

307 what is the most common complication of acute Hearing deficits are most common complication LEAN ANGELO BACK-UP
bacterial meningitis ? followed by neuropsychiatric disturbances ( more SILVERIO, MD MIDTERM
A. Recurrent seizures predominant in H influenza infection). Nelson 17th ed (TOP 4 - AUG EXAM - FEB
B. Mental retardation page 594. 2014 MED 2015
C. Decreased hearing acuity BOARDS;
D. Behavioral changes TOPNOTCH MD),
E. Anterograde amnesia MD

308 the abnormal proliferation of metanephric nephroblastoma or wilms tumor is the 2nd most LEAN ANGELO BACK-UP
blastema is a characteristic finding of which common malignancy in the pediatric population. It is SILVERIO, MD MIDTERM
malignancy? characterized by proliferation of metanephric (TOP 4 - AUG EXAM - FEB
A. Neuroblastoma blastema. 2014 MED 2015
B. ARPKD BOARDS;
C. Wilms tumor TOPNOTCH MD),
D. Rhabdomyosarcoma MD
E. Medullary sponge kidney
309 which of the following is not associated with Nelson 18th ed page 1037. other features of kawasaki LEAN ANGELO BACK-UP
Kawasaki disease? disease are extreme irritability, aseptic meningitis, SILVERIO, MD MIDTERM
A. Gallbladder hydrops diarrhea, hepatitis, gallbladder hydrops, urethritis, (TOP 4 - AUG EXAM - FEB
B. Otitis media otitis media, and arthritis. ( SIMILAR TO PREVIOUS 2014 MED 2015
C. splenomegaly BOARD EXAM CONCEPT/PRINCIPLE) BOARDS;
D. Nondeforming arthritis TOPNOTCH MD),
E. None of the above MD

310 A 2-mo-old infant has a history of infrequent bowel the gold standard for the diagnosis of hirschprung LEAN ANGELO BACK-UP
movements and poor weight gain. Physical disease is rectal biopsy. Colonoscopy will not yield any SILVERIO, MD MIDTERM
examination reveals a thin, poorly nourished child additional information since the classic presentation ( (TOP 4 - AUG EXAM - FEB
with abdominal distention. Rectal examination delayed passage of meconium) along with biopsy or 2014 MED 2015
reveals normal tone, and the infant subsequently barium enema will suffice. BOARDS;
passes an explosive, foul-smelling stool.which of TOPNOTCH MD),
the following procedures is not useful in the MD
diagnosis of this condition?
A. rectal biopsy
B. barium enema
C. proctosigmoidoscopy
D. rectal motility studies
E. none of the above
311 the most common etiologic agent for bacterial Nelson 18th ed page 1766 LEAN ANGELO BACK-UP
tracheitis? SILVERIO, MD MIDTERM
A. Hemophilus Influenza (TOP 4 - AUG EXAM - FEB
B. Staphylococcus aureus 2014 MED 2015
C. Streptococcus pyogenes BOARDS;
D. Streptococcus pneumoniae TOPNOTCH MD),
E. Moraxella catarrhalis MD

312 a 3 month old child was brougt for consult due to stacatto cough is characteristic of chlamydia LEAN ANGELO BACK-UP
staccato cough with mild respiratory distress. She trachomatis infection. SILVERIO, MD MIDTERM
is afebrile. Lung auscultation showed occasional (TOP 4 - AUG EXAM - FEB
bibasilar wheezing and fair entry. What is the most 2014 MED 2015
likely etiologic agent? BOARDS;
A. Chlamydia trachomatis TOPNOTCH MD),
B. Mycoplasma pneumoniae MD
C. Influenza virus
D. streptococcus pneumoniae
E. Listeria monocytogenes

TOPNOTCH MEDICAL BOARD PREP PEDIATRICS SUPEREXAM Page 44 of 97


For inquiries visit www.topnotchboardprep.com.ph or email us at topnotchmedicalboardprep@gmail.com
TOPNOTCH MEDICAL BOARD PREP PEDIATRICS SUPEREXAM
For inquiries visit www.topnotchboardprep.com.ph or email us at topnotchmedicalboardprep@gmail.com
Item QUESTION EXPLANATION AUTHOR TOPNOTCH
# EXAM
313 handedness is usually established at an age of? Nelson 18th ed p54 LEAN ANGELO BACK-UP
A. 2 y/o SILVERIO, MD MIDTERM
B. 3y/o (TOP 4 - AUG EXAM - FEB
C. 4y/o 2014 MED 2015
D. 5y/o BOARDS;
E. None of the above TOPNOTCH MD),
MD

314 Jaundice is most likely physiologic in a term infant parameters of physiologic jaundice: visible on 2nd - LEAN ANGELO BACK-UP
in which of the following? 3rd day, peaks on 4th day at 5-6mg/dl, decreasees on SILVERIO, MD MIDTERM
A. serum bilirubin level <5mg/dL/24 hours in the 5th -7th day at <2mg/dl. (TOP 4 - AUG EXAM - FEB
first 2-4 days of life 2014 MED 2015
B. jaundice at 18 hours of life BOARDS;
C. B2 greater than 1.5mg/dL TOPNOTCH MD),
D. jaundice at 12 days of life MD
E. none of the above
315 The most common abnormality associated with UTI Nelson 18th ed p 2228 LEAN ANGELO BACK-UP
is: SILVERIO, MD MIDTERM
A. ureteropelvic junction obstruction (TOP 4 - AUG EXAM - FEB
B. urolithiasis 2014 MED 2015
C. vesicoureteral reflux BOARDS;
D. posterior urethral reflux TOPNOTCH MD),
E. None of the above MD

316 the success rate for establishing good bile flow after Nelson 18th ed p 1673 LEAN ANGELO BACK-UP
Kasai procedure is much higher if it is done before SILVERIO, MD MIDTERM
in what age? (TOP 4 - AUG EXAM - FEB
A. 1 year 2014 MED 2015
B. 8mo BOARDS;
C. 5mo TOPNOTCH MD),
D. 4mo MD
E. 2mo

317 A 4 weeks old male manifests cyanosis and Critical pulmonic stenosis often presents in the LEAN ANGELO BACK-UP
hepatomegaly. There is a grade 4/6 systolic neonate.Cyanosis is due to elevated right-sided SILVERIO, MD MIDTERM
ejection murmur without an audible ejection click. pressures and right-to-left shunting at (TOP 4 - AUG EXAM - FEB
The ECG tall P waves and right ventricular the patent foramen ovale. 2014 MED 2015
hypertrophy. What is the most likely diagnosis BOARDS;
A. Tetralogy of fallot TOPNOTCH MD),
B. pulmonic stenosis MD
C. Truncus arteriosus
D. tricuspid atresia
E. TAPVR
318 which of the following is not a characteristic of fabry disease is an x linked recessive disorder caused LEAN ANGELO BACK-UP
Fabry's disease? by deficiency in a-galactosidase A. the manifestation SILVERIO, MD MIDTERM
A. Mental retardation includes angiokeratoma, hypohidrosis, corneal and (TOP 4 - AUG EXAM - FEB
B. acroparesthesia lenticular opacities, acroparesthesias and vascular 2014 MED 2015
C. Lenticular opacities diseases of the kidney,heart and brain. mental BOARDS;
D. Hypohidrosis retardation is not a feature of this disease. Nelson TOPNOTCH MD),
E. None of the above 18th ed 598 ( SIMILAR TO PREVIOUS BOARD EXAM MD
CONCEPT/PRINCIPLE)
319 what is the single most predictive factor in acute Nelson 18th ed chapter 487 LEAN ANGELO BACK-UP
lymphoblastic leukemia ? SILVERIO, MD MIDTERM
A. Age at onset (TOP 4 - AUG EXAM - FEB
B. Treatment 2014 MED 2015
C. Initial leukocyte count BOARDS;
D. Chromosomal abnormalities TOPNOTCH MD),
E. None of the above MD

320 a 9 y/o male patient brought for consult secondary presence of ashleaf spots and adenoma sebaceum LEAN ANGELO BACK-UP
to recurrent seizures. Upon secondary survey, you seen in this patient on the background recurrent SILVERIO, MD MIDTERM
noted multiple melanotic macules around his trunk seizures suggest tuberous sclerosis as the primary (TOP 4 - AUG EXAM - FEB
and red minute nodules on his nose and malar area. diagnosis. 2014 MED 2015
what is the most likely diagnosis? BOARDS;
A. Von Hippel Lindau disease TOPNOTCH MD),
B. NF1 MD
C. NF2
D. Tuberous sclerosis
E. sturge weber syndrome
321 Which paranasal sinuses are pneumatized at birth? SIMILAR TO PREVIOUS BOARD EXAM DEBBIE ROSE BACK-UP
A. maxillary CONCEPT/PRINCIPLE. Nelson Textbook of Pediatrics TANENGSY, MD MIDTERM
B. frontal 18th edition p.1749 (TOP 5 - AUG EXAM - FEB
C. ethmoidal 2014 MED 2015
D. sphenoidal BOARDS;
TOPNOTCH MD)
322 Fetuses infected at that age of gestation may have Congenital varicella syndrome at 16-20 weeks: eye DEBBIE ROSE BACK-UP
maximal interruption with lumb development in and brain development are affected. Nelson Textbook TANENGSY, MD MIDTERM
congenital varicella syndrome? of Pediatrics 18th edition p.1368 (TOP 5 - AUG EXAM - FEB
A. 6-12 weeks 2014 MED 2015
B. 16-20 weeks BOARDS;
C. 20-24 weeks TOPNOTCH MD)
D. 24-28 weeks
E. 28-32 weeks

TOPNOTCH MEDICAL BOARD PREP PEDIATRICS SUPEREXAM Page 45 of 97


For inquiries visit www.topnotchboardprep.com.ph or email us at topnotchmedicalboardprep@gmail.com
TOPNOTCH MEDICAL BOARD PREP PEDIATRICS SUPEREXAM
For inquiries visit www.topnotchboardprep.com.ph or email us at topnotchmedicalboardprep@gmail.com
Item QUESTION EXPLANATION AUTHOR TOPNOTCH
# EXAM
323 The most frequently reported cardiac defect in Nelson Textbook of Pediatrics 18th edition p.1340 DEBBIE ROSE BACK-UP
Congenital Rubella Syndrome: TANENGSY, MD MIDTERM
A. Tetralogy of fallot (TOP 5 - AUG EXAM - FEB
B. right pulmonary artery stenosis 2014 MED 2015
C. left pulmonary artery stenosis BOARDS;
D. patent ductus arteriosus TOPNOTCH MD)
E. pulmonic valve stenosis

324 What is the most common cause of death in Hemorrhagic skin eruption is rare; it is seen in DEBBIE ROSE BACK-UP
measles? hemorrhagic/black measles. Myocarditis is also a rare TANENGSY, MD MIDTERM
A. encephalitis complication of measles. Acute otitis media is the most (TOP 5 - AUG EXAM - FEB
B. hemorrhagic skin eruption common complication of measles infection. Nelson 2014 MED 2015
C. myocarditis Textbook of Pediatrics 18th edition p.1334 BOARDS;
D. acute otitis media TOPNOTCH MD)
E. pneumonia

325 Ten years after a primary measles infection, a 13- SSPE stage I - subtle changes in school performance; DEBBIE ROSE BACK-UP
year-old boy was diagnosed to have subacute stage II - massive myoclonus; stage III - TANENGSY, MD MIDTERM
sclerosing panencephalitis. He now exhibits choreoathetosis, rigidity, dystonia with deteriorating (TOP 5 - AUG EXAM - FEB
repetitive myoclonic jerks and involuntary consciousness; stage IV - loss of cortical centers that 2014 MED 2015
movements, but consciousness is maintained. He is support breathing, HR, BP. There is no stage V in SSPE. BOARDS;
now in this stage of SSPE: Nelson Textbook of Pediatrics 18th edition p.1335 TOPNOTCH MD)
A. I
B.II
C.III
D. IV
E. V
326 Microorganism associated with bullous myringitis: Nelson Textbook of Pediatrics 18th edition p.1279 DEBBIE ROSE BACK-UP
A. S. aureus TANENGSY, MD MIDTERM
B. M. pneumoniae (TOP 5 - AUG EXAM - FEB
C. H. influenzae 2014 MED 2015
D. S. pneumoniae BOARDS;
E. N. meningitidis TOPNOTCH MD)

327 What is/are the easiest and most reliable indicators But rectal suction biopsy is the procedure of choice. DEBBIE ROSE BACK-UP
of Hirschsprung disease? Nelson Textbook of Pediatrics 18th edition p.1566 TANENGSY, MD MIDTERM
A. rectal manometry (TOP 5 - AUG EXAM - FEB
B. rectal suction biopsy 2014 MED 2015
C. barium enema BOARDS;
D. A & B TOPNOTCH MD)
E. all of the above

328 Which is a true statement? Stridor is heard in extrathoracic airway obstruction. DEBBIE ROSE BACK-UP
A. In asthma, the patient exhibitis a polyphonic Most wheezing in infants is caused by inflammation. TANENGSY, MD MIDTERM
wheezing. Infants with acute bronchiolitis usually do not exhibit (TOP 5 - AUG EXAM - FEB
B. Stridor is heard in intrathoracic airway diarrhea or vomiting. Bronchiolitis is more common in 2014 MED 2015
obstruction. males, babies not breastfed, and those living in BOARDS;
C. Most wheezing in infants is caused by anatomic crowded conditions. Nelson Textbook of Pediatrics TOPNOTCH MD)
abnormalities. 18th edition pp.1774-1775
D. Infants with acute bronchiolitis usually also
exhibits diarrhea and/or vomiting.
E. Bronchiolitis is more common in breastfed
females.
329 The following clinical manifestations of infective Nelson Textbook of Pediatrics 18th edition p.1954 DEBBIE ROSE BACK-UP
endocarditis are described accurately: TANENGSY, MD MIDTERM
A. Janeway lesions are painless, small, (TOP 5 - AUG EXAM - FEB
erythematous/hemorrhagic lesions on the palms 2014 MED 2015
and soles. BOARDS;
B. Osler nodes are tender, pea-sized intradermal TOPNOTCH MD)
nodules in the pads of fingers and toes.
C. Splinter hemorrhages are linear lesions beneath
nails.
D. A & B
E. all of the above
330 Why should milk consumption be limited in the SIMILAR TO PREVIOUS BOARD EXAM DEBBIE ROSE BACK-UP
treatment of a child with iron deficiency anemia? CONCEPT/PRINCIPLE. Nelson Textbook of Pediatrics TANENGSY, MD MIDTERM
A. to increase the amount of iron rich foods 18th edition p.2016 (TOP 5 - AUG EXAM - FEB
consumed 2014 MED 2015
B. to decrease blood loss from intolerance to cow's BOARDS;
milk proteins TOPNOTCH MD)
C. for optimal absorption of the oral iron
D. A & B
E. all of the above
331 The first laboratory evidence of response to iron 12-24 hours - subjective improvement DEBBIE ROSE BACK-UP
deficiency anemia treatment occurs in: 36-48 hours - initial bone marrow response TANENGSY, MD MIDTERM
A. 12-24 hours 48-72 hours - reticulocytosis (TOP 5 - AUG EXAM - FEB
B. 36-48 hours 4-30 days - increase in hemoglobin 2014 MED 2015
C. 48-72 hours 1-3 months - repletion of stores NelsonTextbook of BOARDS;
D. 4-30 days Pediatrics 18th edition p.2017 TOPNOTCH MD)
E. 1-3 months

TOPNOTCH MEDICAL BOARD PREP PEDIATRICS SUPEREXAM Page 46 of 97


For inquiries visit www.topnotchboardprep.com.ph or email us at topnotchmedicalboardprep@gmail.com
TOPNOTCH MEDICAL BOARD PREP PEDIATRICS SUPEREXAM
For inquiries visit www.topnotchboardprep.com.ph or email us at topnotchmedicalboardprep@gmail.com
Item QUESTION EXPLANATION AUTHOR TOPNOTCH
# EXAM
332 The most common hereditary bleeding disorder is: Nelson Textbook of Pediatrics 18th edition p.2071 DEBBIE ROSE BACK-UP
A. Glanzmann thrombasthenia TANENGSY, MD MIDTERM
B. von Willebrand factor deficiency (TOP 5 - AUG EXAM - FEB
C. hemophilia A 2014 MED 2015
D. hemophilia B BOARDS;
E. Bernard Soulier syndrome TOPNOTCH MD)

333 True of methimazole intake in Graves Disease, Transient granulocytopenia is not a harbinger of DEBBIE ROSE BACK-UP
except: agranuloctyosis in methimazole intake. E is also true TANENGSY, MD MIDTERM
A. Transient granulocytopenia should be taken of PTU. Nelson Textbook of Pediatrics 18th edition (TOP 5 - AUG EXAM - FEB
seriously as it is a harbinger of agranulocytosis. p.2335 2014 MED 2015
B. The most severe reactions are hypersensitive. BOARDS;
C. It is ten times more potent than PTU on weight TOPNOTCH MD)
basis.
D. It has a longer serum T1/2 than PTU.
E. It inhibits incorporation of trapped inorganic
iodide into organic compounds.
334 What is the preferred drug in the management of ACTH is the preferred drug in the management of DEBBIE ROSE BACK-UP
infantile spasms? infantile spasms. Prednison is equally effective. TANENGSY, MD MIDTERM
A. prednisone Vigabatrin is also effective for infantile spasms. (TOP 5 - AUG EXAM - FEB
B. ACTH Leviractam is an adjunct treatment for partial seizure; 2014 MED 2015
C. vigabatrin its mechanism of action is unknown. Zonisamide is BOARDS;
D. leviractam also an adjunct treatment for partial seizures and in TOPNOTCH MD)
E. zonisamide myoclonic syndromes with an unclear mechanism of
action. Nelson Textbook of Pediatrics 18th edition
p.2469
335 SMR stage that corresponds to the appearance of 1 - preadolescent DEBBIE ROSE BACK-UP
enlarged breasts and areola with no contour 2 - breast and papilla elevated as small mound TANENGSY, MD MIDTERM
separation in girls? 4 - areola and papilla form a secondary mound (TOP 5 - AUG EXAM - FEB
A. 1 5 - nipple projects, areola part of generalized breast 2014 MED 2015
B. 2 contour BOARDS;
C. 3 TOPNOTCH MD)
D. 4
E. 5

336 true of SIDS: Nelson Textbook of Pediatrics 18th edition p.1738 DEBBIE ROSE BACK-UP
A. Infants should be placed for sleep in supine or TANENGSY, MD MIDTERM
side sleeping position. (TOP 5 - AUG EXAM - FEB
B. Soft mattresses are preferred over firm ones. 2014 MED 2015
C. It is recommended that infants sleep in the same BOARDS;
room as parents. TOPNOTCH MD)
D. Sleeping prone has consistently been shown to
increase the risk for SIDS.
E. Bed sharing has not been shown to increase the
risk for SIDS.
337 What is the only accurate method for diagnosis in Nelson Textbook of Pediatrics 18th edition p.1750 DEBBIE ROSE BACK-UP
sinusitis? TANENGSY, MD MIDTERM
A. transillumination of sinus cavities (TOP 5 - AUG EXAM - FEB
B. CT scan of sinuses 2014 MED 2015
C. rigid nasal endoscopy BOARDS;
D. sinus aspirate culture TOPNOTCH MD)
E. clinical evaluation

338 Consistent with croup: The rest of the choices refer to acute epiglottitis. DEBBIE ROSE BACK-UP
A. It is a clinical diagnosis that doesn't require a Nelson Textbook of Pediatrics 18th edition p.1763 TANENGSY, MD MIDTERM
neck x-ray. (TOP 5 - AUG EXAM - FEB
B. Drooling is present and the neck is 2014 MED 2015
hyperextended. BOARDS;
C. It requires visualization of large cherry red TOPNOTCH MD)
swollen epiglottis on laryngoscopy.
D. Classic x-ray finding is the thumb sign.
E. The child may assume a tripod position.
339 The most common pediatric soft tissue sarcoma is: Nelson Textbook of Pediatrics 18th edition p.2145 DEBBIE ROSE BACK-UP
A. synovial sarcoma TANENGSY, MD MIDTERM
B. fibrosarcoma (TOP 5 - AUG EXAM - FEB
C. rhabdomyosarcoma 2014 MED 2015
D. malignant fibrous histiocytoma BOARDS;
TOPNOTCH MD)
340 The most common cause of suppurative arthritis in Nelson Textbook of Pediatrics 18th edition p.2845 DEBBIE ROSE BACK-UP
infants and children is: TANENGSY, MD MIDTERM
A. E. coli (TOP 5 - AUG EXAM - FEB
B. S. pneumoniae 2014 MED 2015
C. Candida BOARDS;
D. S. aureus TOPNOTCH MD)
341 A newborn whose mother is both HBsAg and JESSICA MAE BACK-UP
HBeAg (+) should be given Hepa B vaccine and SANCHEZ, MD MIDTERM
HBIG simultaneously at the latest by how many (TOP 4 - AUG EXAM - FEB
hours after birth? 2014 MED 2015
A. 36 BOARDS;
B. 24 TOPNOTCH MD)
C. 12
D. 6

TOPNOTCH MEDICAL BOARD PREP PEDIATRICS SUPEREXAM Page 47 of 97


For inquiries visit www.topnotchboardprep.com.ph or email us at topnotchmedicalboardprep@gmail.com
TOPNOTCH MEDICAL BOARD PREP PEDIATRICS SUPEREXAM
For inquiries visit www.topnotchboardprep.com.ph or email us at topnotchmedicalboardprep@gmail.com
Item QUESTION EXPLANATION AUTHOR TOPNOTCH
# EXAM
342 A 5 year old girl was brought to the clinic due to a 2 JESSICA MAE BACK-UP
week history of fever, joint pains, and loss of SANCHEZ, MD MIDTERM
appetite. On PE: enlarged cervical lymph nodes and (TOP 4 - AUG EXAM - FEB
hepatosplenomegaly. CBC showed Hgb 6 g/dL, Hct 2014 MED 2015
18%, WBC 30x10^9/L, segmenters 5%, BOARDS;
lymphocytes 95%, platelets 50x10^9/L, (+) blasts TOPNOTCH MD)
in the peripheral smear. The most likely diagnosis
is:
A. Neuroblastoma
B. Non-Hodgkin’s lymphoma
C. Acute leukaemia
D. Rhabdomyosarcoma
343 A 4 year old male had 1 month on and off cough, Sputum AFB is not usually a part of initial work-up for JESSICA MAE BACK-UP
weight loss, and afternoon rise of temperature. His TB in young children who cannot yet follow SANCHEZ, MD MIDTERM
grandfather has apical infiltrates which are most instructions and give adequate samples for testing. (TOP 4 - AUG EXAM - FEB
likely tuberculous in origin. What is the initial 2014 MED 2015
work-up of this child? BOARDS;
A. Mantoux test and chest X-ray TOPNOTCH MD)
B. Chest CT scan
C. Bactec TB culture
D. Sputum AFB smear
344 The most frequent cause of food protein induced JESSICA MAE BACK-UP
enteropathy in young infants is sensitivity to: SANCHEZ, MD MIDTERM
A. Soy (TOP 4 - AUG EXAM - FEB
B. Egg 2014 MED 2015
C. Chicken BOARDS;
D. Cow milk TOPNOTCH MD)
345 A 7 year old male with Dengue fever developed Improvement of urine output with fluid replacement JESSICA MAE BACK-UP
upper GI bleeding. He was then noted to have points to a pre-renal cause of renal failure. SANCHEZ, MD MIDTERM
decreased urine output. On PE: BP 80/50 PR (TOP 4 - AUG EXAM - FEB
120/min with cold, clammy skin and weak pulses. 2014 MED 2015
He was given several boluses of plasma expanders. BOARDS;
This resulted in improved urine volume. His renal TOPNOTCH MD)
failure is classified as:
A. Pre-renal
B. Intrinsic renal
C. Post-renal
D. Initially pre-renal then post-renal
346 A 15 year old female developed low grade fever, JESSICA MAE BACK-UP
malaise, sore throat, and anorexia. Irregular pink SANCHEZ, MD MIDTERM
rashes were noted on the face, spreading to the (TOP 4 - AUG EXAM - FEB
trunk and extremities, with multiple large lymph 2014 MED 2015
nodes on the suboccipital, posterior auricular and BOARDS;
anterior cervical areas. The most likely diagnosis is: TOPNOTCH MD)
A. Varicella
B. Rubeola
C. Rubella
D. Dengue fever
347 A 3 year old girl arrives at the ER with a one day The fulminant course of this disease, as well as the JESSICA MAE BACK-UP
history of fever and macular rash that evolved into presence of multiple purpura, make meningococcemia SANCHEZ, MD MIDTERM
petechiae and purpura. She was hypotensive with the most likely diagnosis among the given choices. (TOP 4 - AUG EXAM - FEB
cold clammy extremities. What is the most likely 2014 MED 2015
diagnosis? BOARDS;
A. ITP TOPNOTCH MD)
B. Meningococcemia
C. Kawasaki disease
D. Dengue hemorrhagic fever
348 A 5 year old girl presents with a 2 month history of Difficulty in ambulating, particularly in movements JESSICA MAE BACK-UP
progressive difficulty in ambulation, particularly in which need proximal muscle strength, plus the SANCHEZ, MD MIDTERM
climbing up the stairs. After going on a field trip, presence of violaceous discolouration around the eyes (TOP 4 - AUG EXAM - FEB
she came home with an erythematous rash in the (Heliotrope rash), and papules over the finger and 2014 MED 2015
malar area and violaceous discolouration with elbow joints (Gottron rash) are characteristic of BOARDS;
swelling over the periorbital areas. On the day of Dermatomyositis. TOPNOTCH MD)
consult, she was noted to have papules over the
finger and elbow joints. What is the most likely
diagnosis?
A. SLE
B. Juvenile idiopathic arthritis
C. Juvenile dermatomyositis
D. Systemic scleroderma
349 A 10 year old boy is noted to have hematuria and Diagnosis is IgA nephropathy JESSICA MAE BACK-UP
proteinuria on routine PE. He is asymptomatic, and SANCHEZ, MD MIDTERM
examination findings are normal. Blood chemistry (TOP 4 - AUG EXAM - FEB
results were normal, but 24 hr urine specimen 2014 MED 2015
reveals 2g of protein. A renal biopsy is performed BOARDS;
which reveals a mesangial proliferative TOPNOTCH MD)
glomerulonephritis with bright IgA deposition on
immunofluorescence. Which of the following
statements is true regarding this child’s form of
glomerulonephritis?
A. This disease is more common in females
B. The primary treatment is blood pressure control
C. Progressive kidney disease occurs in a majority
of children
D. C3 value is usually low

TOPNOTCH MEDICAL BOARD PREP PEDIATRICS SUPEREXAM Page 48 of 97


For inquiries visit www.topnotchboardprep.com.ph or email us at topnotchmedicalboardprep@gmail.com
TOPNOTCH MEDICAL BOARD PREP PEDIATRICS SUPEREXAM
For inquiries visit www.topnotchboardprep.com.ph or email us at topnotchmedicalboardprep@gmail.com
Item QUESTION EXPLANATION AUTHOR TOPNOTCH
# EXAM
350 A 5 year old girl presents with tea coloured urine, C3 levels usually return to normal after 8 weeks in JESSICA MAE BACK-UP
oliguria and body edema 2 weeks after being patients with post-streptococcal glomerulonephritis. SANCHEZ, MD MIDTERM
treated for streptococcal pharyngitis. Her C3 level is (TOP 4 - AUG EXAM - FEB
noted to be very low. When should this patient’s C3 2014 MED 2015
level be repeated in order to confirm your BOARDS;
suspected diagnosis? TOPNOTCH MD)
A. In 1 week
B. In 2 weeks
C. In 4 weeks
D. In 8 weeks
351 A 3 year old boy presents at the ER with a 3 day Findings which are characteristic of HSP are purpuric JESSICA MAE BACK-UP
history of abdominal pain and difficulty walking. rash, especially over the lower extremities; abdominal SANCHEZ, MD MIDTERM
Abnormal findings include BP of 120/80, diffuse pain and other GI symptoms; joint pains and swelling; (TOP 4 - AUG EXAM - FEB
abdominal tenderness, purpuric rash of the legs and renal involvement. 2014 MED 2015
and ankles, and swelling and tenderness over the BOARDS;
ankles. The most likely diagnosis is: TOPNOTCH MD)
A. SLE
B. Kawasaki disease
C. Juvenile idiopathic arthritis
D. Henoch Schonlein purpura
352 All of the following statements about hemolytic- JESSICA MAE BACK-UP
uremic syndrome are true except: SANCHEZ, MD MIDTERM
A. HUS is the most common cause of acute renal (TOP 4 - AUG EXAM - FEB
failure in young children 2014 MED 2015
B. Verotoxin elaborated by E. coli O157:H7 initiated BOARDS;
endothelial injury in HUS TOPNOTCH MD)
C. HUS always presents after an enteritis with
diarrhoea
D. The diagnosis of HUS requires microangiopathic
haemolytic anemia, thrombocytopenia, and acute
renal failure
353 A 7 month old girl presents to the ER with gradual Based on the CSF findings, this is most likely a JESSICA MAE BACK-UP
onset of fever, lethargy, and irritability. Her bacterial infection (predominant segmenters, elevated SANCHEZ, MD MIDTERM
immunisations are up to date. Examination reveals protein, decreased sugar). Group B strep is unlikely (TOP 4 - AUG EXAM - FEB
a febrile infant who is crying inconsolably. A since the patient is already 7 months old. Hib is also 2014 MED 2015
lumbar tap is performed with the following result: unlikely because it was noted that her immunisation BOARDS;
WBC 1500/mm3, 84% segmenters; glucose 12 history is complete. TOPNOTCH MD)
mg/dL; protein 70 mg/dL. Gram stain is negative.
The most likely etiologic agent for this infection is:
A. An enterovirus
B. Haemophilus influenzae type b
C. Neisseria meningitides
D. Group B streptococci
354 A 3 year old girl has a 2 week history of fever Based on the patient’s history, she most likely has JESSICA MAE BACK-UP
associated with headache, lethargy and vomiting. Tetralogy of Fallot, which predisposes her to having a SANCHEZ, MD MIDTERM
She has a history of perioral cyanosis and dyspnea brain abscess. Other findings supporting this aetiology (TOP 4 - AUG EXAM - FEB
with exertion ever since she was an infant. She is the presence of increased ICP (papilledema) and a 2014 MED 2015
suddenly has a 10 minute tonic-clonic seizure. The focal neurologic deficit, which makes meningitis less BOARDS;
child is obtunded afterwards, with a temperature of likely. TOPNOTCH MD)
38.2C, PR 118/min, BP of 100/70. Perioral cyanosis
is noted at rest. A harsh pan systolic murmur is
heard along the left sternal border. Fundoscopy
reveals papilledema. She is also noted to have right
sided weakness. The most likely cause of her
hemiparesis is:
A. A brain tumor
B. Neurocysticercosis
C. Meningitis
D. A brain abscess
355 Major features of atopic dermatitis in children JESSICA MAE BACK-UP
include all of the following except: SANCHEZ, MD MIDTERM
A. Pruritus (TOP 4 - AUG EXAM - FEB
B. Facial and extensor eczema 2014 MED 2015
C. Angioedema BOARDS;
D. Chronic or relapsing course TOPNOTCH MD)
356 A 6 month old infant develops protracted projectile JESSICA MAE BACK-UP
vomiting and lethargy about 2 hours after ingesting SANCHEZ, MD MIDTERM
a milk formula. The most likely diagnosis is: (TOP 4 - AUG EXAM - FEB
A. Generalized anaphylaxis 2014 MED 2015
B. Milk induced enterocolitis syndrome BOARDS;
C. Allergic eosinophilic esophagitis TOPNOTCH MD)
D. Allergic eosinophilic gastroenteritis
357 All of the following statements concerning allergic It is mediated by IgE. JESSICA MAE BACK-UP
reaction to foods are true, except: SANCHEZ, MD MIDTERM
A. Cow’s milk sensitivity is the most common cause (TOP 4 - AUG EXAM - FEB
of protein-induced enteropathy 2014 MED 2015
B. Gastrointestinal anaphylaxis is mediated by IgA BOARDS;
C. Majority of children with positive results on skin TOPNOTCH MD)
prick tests to a food will not react when the food is
ingested
D. Elimination diets are the only means to establish
the diagnosis of food allergies

TOPNOTCH MEDICAL BOARD PREP PEDIATRICS SUPEREXAM Page 49 of 97


For inquiries visit www.topnotchboardprep.com.ph or email us at topnotchmedicalboardprep@gmail.com
TOPNOTCH MEDICAL BOARD PREP PEDIATRICS SUPEREXAM
For inquiries visit www.topnotchboardprep.com.ph or email us at topnotchmedicalboardprep@gmail.com
Item QUESTION EXPLANATION AUTHOR TOPNOTCH
# EXAM
358 A 6 week old infant has gained no weight since JESSICA MAE BACK-UP
birth. Her skin appears mottled, extremities are SANCHEZ, MD MIDTERM
cold, and her temperature is 35C. The most likely (TOP 4 - AUG EXAM - FEB
diagnosis is: 2014 MED 2015
A. Sepsis BOARDS;
B. Galactosemia TOPNOTCH MD)
C. Hypothermia
D. Hypothyroidism
359 All of the following are prominent features of It’s an X-linked recessive syndrome. JESSICA MAE BACK-UP
Wiskott-Aldrich syndrome except: SANCHEZ, MD MIDTERM
A. Atopic dermatitis (TOP 4 - AUG EXAM - FEB
B. Thrombocytopenia 2014 MED 2015
C. Autosomal dominant inheritance BOARDS;
D. Recurrent infections with encapsulated bacteria TOPNOTCH MD)
360 The most important extra medullary site of relapse The CNS and the testes are the two most important JESSICA MAE BACK-UP
in childhood ALL is: sites of extra medullary relapse of ALL. SANCHEZ, MD MIDTERM
A. Kidney (TOP 4 - AUG EXAM - FEB
B. Lung 2014 MED 2015
C. Heart BOARDS;
D. CNS TOPNOTCH MD)
361 Patients with cystic fibrosis are susceptible to Nelsons 19th pg 1361 MAIRRE JAMES BACK-UP
which of the following? GADDI, MD (TOP MIDTERM
A. Pancreatic insufficiency 4 - AUG 2013 EXAM - FEB
B. Recurrent pneumonia MED BOARDS; 2015
C. Rectal mucosal prolapse TOPNOTCH MD)
D. A and B
E. All of the above

362 TRUE of simple febrile seizures EXCEPT not recurrent within a 24-hour period Nelsons 19th MAIRRE JAMES BACK-UP
A. generalized tonic-clonic pg 2017 GADDI, MD (TOP MIDTERM
B. lasts no more than 15 minutes 4 - AUG 2013 EXAM - FEB
C. more than 1 episode in 24 hours MED BOARDS; 2015
D. B and C TOPNOTCH MD)
E. All of the above

363 Most common cause of hemolytic disease of the ABO incompatibility is the most common cause of MAIRRE JAMES BACK-UP
newborn hemolytic disease of the newborn. Nelsons 19th pg GADDI, MD (TOP MIDTERM
A. ABO incompatibility 619 4 - AUG 2013 EXAM - FEB
B. Rh incompatibility MED BOARDS; 2015
C. Autoimmune TOPNOTCH MD)
D. Kell incompatibility
E. None of the above

364 A 1 day old neonate was brought to the hospital for Choices arranged from most common to least common MAIRRE JAMES BACK-UP
frothing and bubbling at the mouth & nose. A few with the following frequencies: 85%, 8%, 4%, 2%, 1% GADDI, MD (TOP MIDTERM
hours prior, the mother attempted to feed the baby 4 - AUG 2013 EXAM - FEB
which resulted to coughing and cyanosis. Which is MED BOARDS; 2015
the most common form of this disorder? TOPNOTCH MD)
A. EA with distal fistula
B. EA with no fistula
C. H type fistula with no EA
D. EA with proximal and distal fistula
E. EA with proximal fistula
365 Failure to thrive is defined as: All are correct. Topnotch MAIRRE JAMES BACK-UP
A. Growth at less than 5th percentile on the growth GADDI, MD (TOP MIDTERM
chart 4 - AUG 2013 EXAM - FEB
B. Weight or heigt less than the 5th percentile MED BOARDS; 2015
C. Growth patterns cross two major percentiles TOPNOTCH MD)
within the past 6 months
D. All of the above
366 The period of communicability for measles is: A - rubella; C - mumps (1-2 days before until 5 days MAIRRE JAMES BACK-UP
A. 7 days before until 7 days after the onset of the after the onset of the swelling; D - varicella; E - roseola GADDI, MD (TOP MIDTERM
rash Topnotch 4 - AUG 2013 EXAM - FEB
B. 4 days before until 4 days after the onset of the MED BOARDS; 2015
rash TOPNOTCH MD)
C. 1-2 days before until 5 days after the onset of the
rash
D. 1-2 days before until 7 days after the onset of the
rash
E. Unknown
367 What is the strongest stimulus for the closure of the Increase in O2 saturation of the systemic circulation is MAIRRE JAMES BACK-UP
ductus arteriosus? the strongest stimulus for the constriction of the GADDI, MD (TOP MIDTERM
A. Increase in O2 saturation of the systemic ductal smooth muscles which leads to the closure of 4 - AUG 2013 EXAM - FEB
circulation the ductus. Topnotch MED BOARDS; 2015
B. Increase in the pressure of the systemic TOPNOTCH MD)
circulation
C. Decrease in the O2 saturation of the pulmonary
circulation
D. Decrease in the pressure of the pulmonary
circulation
E. A and D

TOPNOTCH MEDICAL BOARD PREP PEDIATRICS SUPEREXAM Page 50 of 97


For inquiries visit www.topnotchboardprep.com.ph or email us at topnotchmedicalboardprep@gmail.com
TOPNOTCH MEDICAL BOARD PREP PEDIATRICS SUPEREXAM
For inquiries visit www.topnotchboardprep.com.ph or email us at topnotchmedicalboardprep@gmail.com
Item QUESTION EXPLANATION AUTHOR TOPNOTCH
# EXAM
368 The pentalogy of cantrell consists of the following Pentalogy of Cantrell consists of ectopia cordis, MAIRRE JAMES BACK-UP
EXCEPT midline supraumbilical abdominal defect, deficiency GADDI, MD (TOP MIDTERM
A. Omphalocele of the anterior diaphragm, defect of the lower 4 - AUG 2013 EXAM - FEB
B. Sternal cleft sternum, and an intra- cardiac defect (either a MED BOARDS; 2015
C. Ectopia cordis ventricular septal defect, tetralogy of Fallot or TOPNOTCH MD)
D. Posterior diaphragmatic hernia diverticulum of the left ventricle). Nelsons 19th pg
E. VSD 1599

369 A 2 hour old neonate was brought to the ER for The resulting small airway obstruction may produce MAIRRE JAMES BACK-UP
cyanosis, tachypnea, grunting and retractions. On respiratory distress within the 1st hours, with GADDI, MD (TOP MIDTERM
further history, the neonate was delivered by a hilot tachypnea, retractions, grunting, and cyanosis 4 - AUG 2013 EXAM - FEB
after a prolonged labor of more than a day. CXR was observed in severely affected infants. The typical chest MED BOARDS; 2015
done which showed patchy infiltrate and coarse roentgenogram is characterized by patchy infiltrates, TOPNOTCH MD)
streaking of both lung fields. What is your coarse streaking of both lung fields, increased
diagnosis? anteroposterior diameter, and flattening of the
A. Hyaline membrane disease diaphragm.A normal chest roentgenogram in an infant
B. Neonatal pneumonia with severe hypoxemia and no cardiac malformation
C. Meconium aspiration syndrome suggests the diagnosis of pulmonary hypertension
D. Transient tachypnea of the newborn Nelsons 19th pg 591
E. Persistent pulmonary hypertension of the
newborn
370 The most common symptom of rheumatic fever: Choices arranged from most common to least common MAIRRE JAMES BACK-UP
A. Migratory polyarthritis with the following frequencies: 75%, 50-60%, 10- GADDI, MD (TOP MIDTERM
B. Carditis 15%, <3%, <1% Nelsons 19th pg 921-922 4 - AUG 2013 EXAM - FEB
C. Sydenham chorea MED BOARDS; 2015
D. Erythema marginatum TOPNOTCH MD)
E. Subcutaneous nodules

371 An 14/M presented to you with headaches, visual Children with craniopharyngioma often present with MAIRRE JAMES BACK-UP
disturbances, growth failure and delayed sexual endocrinologic abnormalities such as growth failure GADDI, MD (TOP MIDTERM
maturation. A CT scan was done which revealed a and delayed sexual maturation. Visual changes can 4 - AUG 2013 EXAM - FEB
large and heterogenous mass, with solid and cystic occur and may include decrease acuity or visual field MED BOARDS; 2015
components, in the suprasellar region. What is your deficits. These tumors are often quite large and TOPNOTCH MD)
diagnosis? heterogenous, displaying both solid and cystic
A. Pituitary adenoma components, and occur within the suprasellar region.
B. Craniopharyngioma They are minimally invasive, adhere to adjacent brain
C. Germ cell tumor parenchyma, and engulf normal brain structures.
D. Ependymal tumor Nelsons 19th pg 1752
E. Astrocytoma
372 The caudal end of the neural tube closes during: Normally, the rostral end of the neural tube closes on MAIRRE JAMES BACK-UP
A. 23rd day the 23rd day and the caudal neuropore closes by a GADDI, MD (TOP MIDTERM
B. 27th day process of secondary neurulation by the 27th day of 4 - AUG 2013 EXAM - FEB
C. 4th month development Nelsons 19th pg 1998 MED BOARDS; 2015
D. 8th month TOPNOTCH MD)
E. None of the above

373 Perinatal asphyxia severe enough to result in acute > 5 minute Apgar of 0-3 MAIRRE JAMES BACK-UP
neurologic injury if all of the following variables are GADDI, MD (TOP MIDTERM
present EXCEPT? 4 - AUG 2013 EXAM - FEB
A. Umbilical artery blood pH <7 MED BOARDS; 2015
B. 3 minute Apgar of 0-3 TOPNOTCH MD)
C. Seizures, coma or hypotonia
D. Multiple organ involvement
E. No exception

374 An 16/M presents with recurrent muscle cramps Patients with Gitelman syndrome often have a history MAIRRE JAMES BACK-UP
and spasms. Serum chemistries were done of recurrent muscle cramps and spasms, presumably GADDI, MD (TOP MIDTERM
revealing hypokalemia, hypomagnesemia and caused by low serum magnesium levels. Biochemical 4 - AUG 2013 EXAM - FEB
metabolic alkalosis. Urinary electrolyte analysis abnormalities include hypokalemia, metabolic MED BOARDS; 2015
showed low urinary calcium levels and elevated alkalosis, and hypomagnesemia. The urinary calcium TOPNOTCH MD)
urinary magnesium levels. What is your diagnosis? level is usually very low (in contrast to the elevated
A. Gitelman syndrome urinary calcium level often seen in Bartter syndrome),
B. Bartter syndrome and the urinary magnesium level is elevated. Nelsons
C. RTA type IV 19th pg 1814
D. RTA type I
E. None of the above
375 A 9/M presented with a 3 day history of symmetric The diagnosis of HSP is a clinical one and is often MAIRRE JAMES BACK-UP
multiple non-blanching purple discolorations in the straightforward when the typical rash is present. GADDI, MD (TOP MIDTERM
lower extremities and buttocks. This was Classification criteria for HSP: 4 - AUG 2013 EXAM - FEB
associated with postprandial abdominal pain, AMERICAN COLLEGE OF RHEUMATOLOGY MED BOARDS; 2015
arthralgia and hematuria. What is your diagnosis? CLASSIFICATION CRITERIA Two of the following TOPNOTCH MD)
A. HSP criteria must be present:
B. Wegener granulomatosis • Palpable purpura
C. Churg-Strauss syndrome • Age at onset ≤20 yr
D. Microscopic polyangiitis • Bowel angina (postprandial abdominal pain, bloody
E. None of the above diarrhea)
• Biopsy demonstrating intramural granulocytes in
small arterioles and/or
venules
EUROPEAN LEAGUE AGAINST
RHEUMATISM/PEDIATRIC RHEUMATOLOGY
EUROPEAN SOCIETY CRITERIA Palpable
purpura (in absence of coagulopathy or
thrombocytopenia) and one or more of the following
criteria must be present:
TOPNOTCH MEDICAL BOARD PREP PEDIATRICS SUPEREXAM Page 51 of 97
For inquiries visit www.topnotchboardprep.com.ph or email us at topnotchmedicalboardprep@gmail.com
TOPNOTCH MEDICAL BOARD PREP PEDIATRICS SUPEREXAM
For inquiries visit www.topnotchboardprep.com.ph or email us at topnotchmedicalboardprep@gmail.com
Item QUESTION EXPLANATION AUTHOR TOPNOTCH
# EXAM
• Diffuse abdominal pain
• Arthritis or arthralgia
• Biopsy of affected tissue demonstrating
predominant immunoglobulin A
deposition Nelsons 19th pg 870

376 A 9 month old was brought to you for recurrent Children with LAD-1 present in infancy with MAIRRE JAMES BACK-UP
pneumonia and recurrent bacterial infections of the recurrent, indolent bacterial infections of the skin, GADDI, MD (TOP MIDTERM
skin. On futher history, it was noted that the infant mouth, respiratory tract, lower intestinal tract, and 4 - AUG 2013 EXAM - FEB
developed an infection of the cord stump due to its genital mucosa. They may have a history of delayed MED BOARDS; 2015
delayed separation? What is your diagnosis? separation of the umbilical cord, usually with TOPNOTCH MD)
A. SCID associated infection (omphalitis) of the cord stump.
B. Chronic granulomatous disease Nelsons 19th pg 741
C. Chediak-Higashi syndrome
D. Leukocyte adhesion deficiency I
E. Complement deficiency
377 A 1/M presents with a 4 day history of low-grade This is a case of bronchiolitis which is an acute MAIRRE JAMES BACK-UP
fever, cough and colds. On PE you noted inflammation of the small airways in children < 2y GADDI, MD (TOP MIDTERM
hyperresonance on percussion and wheezing on Topnotch 4 - AUG 2013 EXAM - FEB
auscultation. The etiologic agent of the patient's MED BOARDS; 2015
illness is? TOPNOTCH MD)
A. Parainfluenza virus
B. Respiratory syncitial virus
C. Hemophilus influezae type B
D. Streptococcus pneumoniae
E. Mycoplasma sp.
378 A 16/M presents with acute onset scrotal pain and Prehn sign: relief when scrotum is lifted. Prehn sign MAIRRE JAMES BACK-UP
swelling. Upon further history, the patient also had positive in epididymitis and negative in torsion GADDI, MD (TOP MIDTERM
nausea, vomiting and fever. On PE, the scrotum did Topnotch 4 - AUG 2013 EXAM - FEB
not transilluminate, Prehn sign was positive and MED BOARDS; 2015
you were not able to elicit the cremasteric reflex. TOPNOTCH MD)
What is your diagnosis?
A. Testicular torsion
B. Epididymitis
C. Varicocele
D. Hydrocele
E. None of the above
379 The following is/are TRUE regarding breastmilk Breastmilk has higher digestibility and better MAIRRE JAMES BACK-UP
and milk formula? absorption; Milk formula has a little higher iron than GADDI, MD (TOP MIDTERM
A. Milk formula has higher digestibility and better breastmilk Topnotch 4 - AUG 2013 EXAM - FEB
absorption MED BOARDS; 2015
B. Breastmilk contains higher amounts of iron TOPNOTCH MD)
compared to milk formula
C. Breastmilk is more allergenic than milk formula
D. B and C
E. All of the above
380 A 15/F presents with pallor and easy fatiguability. 12-24 hours subjective improvement; 26-48 hours MAIRRE JAMES BACK-UP
CBC was done revealing a low Hgb. You decide to initial BM response; 72 hours retic count increases; 4- GADDI, MD (TOP MIDTERM
start the patient on iron supplementation. When 6 weeks Hgb increases; 1-3 months repletion of iron 4 - AUG 2013 EXAM - FEB
will you repeat the CBC in order to check if the Hgb stores Topnotch MED BOARDS; 2015
is already rising? TOPNOTCH MD)
A. 12-24 hours after
B. after 3 days
C. after 1 month
D. after 3 months
E. after 6 months
381 A child is considered enuretic if he/she has not A child is expected to be toilet trained by day at 2 SCOTT RILEY BACK-UP
attained full bladder control by what age? years old and dry by night at 3 years old. He/she is ONG, MD (TOP 5 - MIDTERM
A. 2 years considered enuretic if he/she remains unable to AUG 2014 MED EXAM - FEB
B. 3 years exhibit bladder control by 5 years old. BOARDS; 2015
C. 4 years TOPNOTCH MD)
D. 5 years
E. 6 years

382 A child is expected to have a complete set of The first permanent molars erupt at 6 years of age. SCOTT RILEY BACK-UP
primary teeth by what age? ONG, MD (TOP 5 - MIDTERM
A. 2 years AUG 2014 MED EXAM - FEB
B. 2.5 years BOARDS; 2015
C. 3 years TOPNOTCH MD)
D. 4 years
E. 6 years

TOPNOTCH MEDICAL BOARD PREP PEDIATRICS SUPEREXAM Page 52 of 97


For inquiries visit www.topnotchboardprep.com.ph or email us at topnotchmedicalboardprep@gmail.com
TOPNOTCH MEDICAL BOARD PREP PEDIATRICS SUPEREXAM
For inquiries visit www.topnotchboardprep.com.ph or email us at topnotchmedicalboardprep@gmail.com
Item QUESTION EXPLANATION AUTHOR TOPNOTCH
# EXAM
383 An infant is expected to double his/her birth weight Birth weight triples by 1 year old. SCOTT RILEY BACK-UP
by what age? ONG, MD (TOP 5 - MIDTERM
A. 2-3 months AUG 2014 MED EXAM - FEB
B. 4-5 months BOARDS; 2015
C. 8-9 months TOPNOTCH MD)
D. 12 months
E. 15 months

384 A 2-day old full-term infant was brought to you due Diagnosis: Erythema toxicum SCOTT RILEY BACK-UP
to multiple 1-2 mm papules, each with a distinct ONG, MD (TOP 5 - MIDTERM
erythematous base, scattered over his trunk. There AUG 2014 MED EXAM - FEB
are no other associated symptoms. Biopsy of these BOARDS; 2015
lesions will reveal predominance of which of the TOPNOTCH MD)
following cells?
A. Lymphocytes
B. Macrophages
C. Neutrophils
D. Eosinophils
E. No cells will be seen.
385 Which of the following is a known teratogenic effect Lithium: Ebstein anomaly. Thalidomide: phocomelia. SCOTT RILEY BACK-UP
that results from cocaine abuse during pregnancy? Rubella: PDA. ONG, MD (TOP 5 - MIDTERM
A. Ebstein anomaly AUG 2014 MED EXAM - FEB
B. Cystic cortical lesion BOARDS; 2015
C. Phocomelia TOPNOTCH MD)
D. Patent ductus arteriosus
E. Spina bifida

386 Which of the following findings would make you The rest of the choices are consistent with physiologic SCOTT RILEY BACK-UP
consider a pathologic cause for neonatal jaundice? jaundice. ONG, MD (TOP 5 - MIDTERM
A. Bilirubin level peaks at 2nd to 4th day of life AUG 2014 MED EXAM - FEB
B. Jaundice resolves within 14 days BOARDS; 2015
C. Total bilirubin of 10 mg/dL TOPNOTCH MD)
D. Direct bilirubin is 7% of total bilirubin
E. Total bilirubin increases by 1.0 mL/dL/hr

387 Which of the following is not consistent with Choice A refers to breastmilk jaundice. SCOTT RILEY BACK-UP
breastfeeding jaundice? ONG, MD (TOP 5 - MIDTERM
A. It results from the presence of glucuronidase in AUG 2014 MED EXAM - FEB
breastmilk. BOARDS; 2015
B. It typically occurs within the 1st week of life TOPNOTCH MD)
C. It is corrected with continued breastfeeding
>10x/day
D. It presents with laboratory finding of indirect
hyperbilirubinemia.
E. It typically lasts for only a few days.
388 Based on Philippine guidelines, deworming every 6 SCOTT RILEY BACK-UP
months should be performed on children between ONG, MD (TOP 5 - MIDTERM
what ages? AUG 2014 MED EXAM - FEB
A. 12 months to 5 years BOARDS; 2015
B. 12 months to 14 years TOPNOTCH MD)
C. 5 years to 14 years
D. 5 years to 18 years
E. 6 months to 5 years

389 Which of the following is the mainstay of therapy in Plasma exchange is the mainstay of management in SCOTT RILEY BACK-UP
idiopathic thrombocytopenic purpura? thrombotic thrombocytopenic purpura. Platelet ONG, MD (TOP 5 - MIDTERM
A. Corticosteroid therapy transfusion is contraindicated in both ITP and TTP. AUG 2014 MED EXAM - FEB
B. Plasma exchange BOARDS; 2015
C. Platelet transfusion TOPNOTCH MD)
D. IVIg therapy
E. None of the above

390 Which of the following features is not consistent Absence seizures are rare in children below 5 years SCOTT RILEY BACK-UP
with absence seizures? old. ONG, MD (TOP 5 - MIDTERM
A. 3-Hz spike waves on EEG AUG 2014 MED EXAM - FEB
B. No postictal state BOARDS; 2015
C. Impaired consciousness during attacks TOPNOTCH MD)
D. Typically lasts for less than 30 seconds
E. Most common among children less than 5 years
old
391 Which of the following disorders is a classic SCOTT RILEY BACK-UP
example of pure T-cell deficiency? ONG, MD (TOP 5 - MIDTERM
A. Chediak-Higashi syndrome AUG 2014 MED EXAM - FEB
B. Severe combined immunideficiency BOARDS; 2015
C. Wiskott-Aldrich syndrome TOPNOTCH MD)
D. Di George syndrome
E. Chronic granulomatous disease

392 Which of the following best describes proteinuria in SIMILAR TO PREVIOUS BOARD EXAM SCOTT RILEY BACK-UP
the pediatric population? CONCEPT/PRINCIPLE. Definitions of proteinuria in ONG, MD (TOP 5 - MIDTERM
A. > 4 mg/m2/hr the pediatric population: > 4 mg/m2/hr OR > 150 AUG 2014 MED EXAM - FEB
B. > 300 mg / day mg/day OR protein:crea ratio > 0.2-0.3. Values of > 40 BOARDS; 2015
C. > 40 mg/m2/hr mg/m2/hr describes nephrotic range proteinuria. TOPNOTCH MD)
D. protein:creatinine ratio > 1.7
E. > 150 mg/m2/hr

TOPNOTCH MEDICAL BOARD PREP PEDIATRICS SUPEREXAM Page 53 of 97


For inquiries visit www.topnotchboardprep.com.ph or email us at topnotchmedicalboardprep@gmail.com
TOPNOTCH MEDICAL BOARD PREP PEDIATRICS SUPEREXAM
For inquiries visit www.topnotchboardprep.com.ph or email us at topnotchmedicalboardprep@gmail.com
Item QUESTION EXPLANATION AUTHOR TOPNOTCH
# EXAM
393 A 5-year old female presented to your clinic due to Diagnosis: Herpangina SCOTT RILEY BACK-UP
sudden onset of fever with sore throat and ONG, MD (TOP 5 - MIDTERM
vomiting. On physical examination, you noted AUG 2014 MED EXAM - FEB
tender vesicles and ulcers on her tonsils and BOARDS; 2015
oropharynx. Which of the following is the most TOPNOTCH MD)
likely causative agent for her disease?
A. Epstein-Barr virus
B. Coxsackievirus A
C. Parvovirus B19
D. Corynebacterium diphtheriae
E. Poliovirus
394 Which of the following findings would you see SCOTT RILEY BACK-UP
during the subacute phase of Kawasaki disease? ONG, MD (TOP 5 - MIDTERM
A. Periungual desquamation of the fingers and toes AUG 2014 MED EXAM - FEB
B. Elevated ESR BOARDS; 2015
C. Thrombocytosis TOPNOTCH MD)
D. Development of coronary artery aneurysm
E. All of the above

395 A 7-year old male presented to your clinic with SIMILAR TO PREVIOUS BOARD EXAM SCOTT RILEY BACK-UP
symptoms of multiple raised erythematous lesion CONCEPT/PRINCIPLE. ONG, MD (TOP 5 - MIDTERM
on both lower legs, abdominal pain and mild AUG 2014 MED EXAM - FEB
hematuria. On further history, you noted an episode BOARDS; 2015
of upper respiratory tract infection 1 week prior to TOPNOTCH MD)
the development of the said problems. What is your
most likely diagnosis?
A. Lupus nephritis
B. Hemolytic uremic syndrome
C. Henoch-Schonlein purpura
D. Dengue hemorrhagic fever
E. Immune thrombocytopenic purpura
396 A 3-year old boy was brought to the emergency SCOTT RILEY BACK-UP
department due to moderate cramping abdominal ONG, MD (TOP 5 - MIDTERM
pain, accompanied by nausea and vomiting. On AUG 2014 MED EXAM - FEB
further history, you noted that he has had previous BOARDS; 2015
similar episodes in the past and has been TOPNOTCH MD)
asymptomatic in between episodes. There has been
no febrile episodes. Which of the following
conditions would be your primary diagnosis at this
point?
A. Acute appendicitis
B. Intussusception
C. Colonic volvulus
D. Irritable bowel syndrome
E. Celiac disease
397 A 6-year female was brought to your clinic due to SIMILAR TO PREVIOUS BOARD EXAM SCOTT RILEY BACK-UP
symptoms of excessive urination with increased CONCEPT/PRINCIPLE. ONG, MD (TOP 5 - MIDTERM
thirst, interfering with her appetite, weight gain AUG 2014 MED EXAM - FEB
and growth. Urination persisted despite fluid BOARDS; 2015
deprivation. Following administration of TOPNOTCH MD)
desmopressin, urine remained diluted with low
specific gravity. What is the most likely diagnosis?
A. Primary polydipsia
B. Central diabetes insipidus
C. Nephrogenic diabetes insipidus
D. Addison disease
E. Conn syndrome
398 The MOPP regimen (mustine, oncovin, SIMILAR TO PREVIOUS BOARD EXAM SCOTT RILEY BACK-UP
procarbazine, prednisone) is used in the treatment CONCEPT/PRINCIPLE. ONG, MD (TOP 5 - MIDTERM
of which of the following malignant diseases? AUG 2014 MED EXAM - FEB
A. Lymphoma BOARDS; 2015
B. Wilms tumor TOPNOTCH MD)
C. Neuroblastoma
D. Seminoma
E. Acute lymphoblastic leukemia

399 Six days after a major bout of bloody diarrhea, a 5- SIMILAR TO PREVIOUS BOARD EXAM SCOTT RILEY BACK-UP
year old boy developed crampy abdominal pain CONCEPT/PRINCIPLE. ONG, MD (TOP 5 - MIDTERM
accompanied by irritability, pallor and multiple AUG 2014 MED EXAM - FEB
petecchial rashes throughout his trunk and BOARDS; 2015
extremities. There was a decrease in his urine TOPNOTCH MD)
output with note of hematuria. What is your most
likely diagnosis?
A. Lupus nephritis
B. Hemolytic uremic syndrome
C. Henoch-Schonlein purpura
D. Thrombotic thrombocytopenic purpura
E. Dengue hemorrhagic fever
400 Which of the following features is least consistent Uveitis is associated with the oligoarthritic type of SCOTT RILEY BACK-UP
with juvenile rheumatoid arthritis? JRA. ONG, MD (TOP 5 - MIDTERM
A. Onset before 16 years old AUG 2014 MED EXAM - FEB
B. Duration of symptoms for more than 6 weeks BOARDS; 2015
C. Most commonly oligoarthritic, affecting the knees TOPNOTCH MD)
and ankles
D. Polyarthritic form is associated with uveitis in
20%

TOPNOTCH MEDICAL BOARD PREP PEDIATRICS SUPEREXAM Page 54 of 97


For inquiries visit www.topnotchboardprep.com.ph or email us at topnotchmedicalboardprep@gmail.com
TOPNOTCH MEDICAL BOARD PREP PEDIATRICS SUPEREXAM
For inquiries visit www.topnotchboardprep.com.ph or email us at topnotchmedicalboardprep@gmail.com
Item QUESTION EXPLANATION AUTHOR TOPNOTCH
# EXAM
E. Systemic type is associated with characteristic
quotidian fever

401 Klienfelter's Syndrome: JOSE CARLO DIAGNOSTIC


A. 47 XXY MASANGKAY III, EXAM - AUG
B. 45 XO MD (TOP 8 - FEB 2014
C. 47 XXY/48 XXXY 2014 MED
D. 47 XYY/47 XXY BOARDS;
E. 47 XYY TOPNOTCH MD)

402 Among which of the following statements regarding Milk formulas have higher levels of Vitamin D,K and JOSE CARLO DIAGNOSTIC
breastmilk is/are true? Iron than breastmilk. MASANGKAY III, EXAM - AUG
A. Milk formula has lower Vitamin D levels than MD (TOP 8 - FEB 2014
breastmilk 2014 MED
B. Milk formula has lower Vitamin K levels than BOARDS;
breastmilk TOPNOTCH MD)
C. Milk formula has higher Iron levels than
breastmilk
D. All of the above
E. None of the above
403 A 3-year old patient presented with a palpable rash Regardless of the severity of symptoms, patients JOSE CARLO DIAGNOSTIC
and arthritis on the knees after an apparent URTI require urinalysis every 3 months for 1 year. MASANGKAY III, EXAM - AUG
few weeks prior to consult. What would be the next MD (TOP 8 - FEB 2014
step for follow-up knowing your consideration? 2014 MED
A. Platelet count after 1 month BOARDS;
B. Arthrocentesis after 6 months TOPNOTCH MD)
C. Chest Xray after 6 months
D. Urinalysis after 3 months
E. C3 determination after 2 months
404 A 3-day old infant born to an indigent family, SIMILAR TO PREVIOUS BOARD EXAM JOSE CARLO DIAGNOSTIC
developed jaundice. The mother was unable to CONCEPT/PRINCIPLE MASANGKAY III, EXAM - AUG
provide breast milk due to mastitis and MD (TOP 8 - FEB 2014
occasionally gives rice-water with sugar as an 2014 MED
alternative. Which of the following will be your next BOARDS;
course of action? TOPNOTCH MD)
A. Sepsis neonatorum
B. Physiologic jaundice
C. Breastfeeding jaundice
D. Breast milk jaundice
E. None of the above
405 A 2-year old suddenly develops maculopapular Roseola, caused by Human herpesvirus type 6 JOSE CARLO DIAGNOSTIC
rashes suddenly after the disappearance of a 2-day MASANGKAY III, EXAM - AUG
fever. What is the probable cause of this disease? MD (TOP 8 - FEB 2014
A. Rubella Virus 2014 MED
B. Rubeola Virus BOARDS;
C. Parvovirus B19 TOPNOTCH MD)
D. HHV-6
E. EBV

406 A febrile 10-year old patient was brought to you by Measles. Period of communicability is 4 days before JOSE CARLO DIAGNOSTIC
his mother due to the appearance of maculopapular and 4 days after the onset of rash. MASANGKAY III, EXAM - AUG
rashes, which occurred the night before consult MD (TOP 8 - FEB 2014
where the highest fever grade was noted at 39.2 2014 MED
deg C. She then asks you how early can her child go BOARDS;
back to school? TOPNOTCH MD)
A. 4 days after the onset of rash
B. 6 days after the onset of rash
C. 4 days after the onset of fever
D. 6 days after the onset of fever
E. 8 days after the onset of fever
407 A 9-year old patient was brought to you with A patient with german measles does not develop JOSE CARLO DIAGNOSTIC
maculopapular rashes for 3 days. Among which of photophobia. MASANGKAY III, EXAM - AUG
the following manifestations would you NOT MD (TOP 8 - FEB 2014
suspect a Rubella virus infection? 2014 MED
A. Posterior auricular lymphadenopathy BOARDS;
B. Photophobia TOPNOTCH MD)
C. Forscheimer spots
D. Low grade fever
E. Polyarthritis
408 An 18-year old patient presented with generalized The case is Infectious Mononucleosis. Hodkin's JOSE CARLO DIAGNOSTIC
lymphadenopathy, and hepatosplenomegaly, on instead of Non-Hodkins MASANGKAY III, EXAM - AUG
diagnostic examinations atypical lymphocytes were MD (TOP 8 - FEB 2014
seen with positive Paul-Bunnell antibodies. Which 2014 MED
of the following Neoplastic diseases will the patient BOARDS;
not be at risk of having? TOPNOTCH MD)
A. Nasopharyngeal CA
B. Burkitt's Lymphoma
C. Non-Hodkin's Lymphoma
D. Lymphoproliferative Disease
E. Leiomyosarcoma

TOPNOTCH MEDICAL BOARD PREP PEDIATRICS SUPEREXAM Page 55 of 97


For inquiries visit www.topnotchboardprep.com.ph or email us at topnotchmedicalboardprep@gmail.com
TOPNOTCH MEDICAL BOARD PREP PEDIATRICS SUPEREXAM
For inquiries visit www.topnotchboardprep.com.ph or email us at topnotchmedicalboardprep@gmail.com
Item QUESTION EXPLANATION AUTHOR TOPNOTCH
# EXAM
409 A 15-year old was brought to the ED due to TBSA involvement of >40%, TEN may already be JOSE CARLO DIAGNOSTIC
dyspnea, multiple blisters, severe denudation of considered instead of SJS MASANGKAY III, EXAM - AUG
skin (45% TBSA) with intensive mucosal MD (TOP 8 - FEB 2014
involvement. It appearad after she was prescribed 2014 MED
Co-trimoxazole for an apparent UTI. What would be BOARDS;
your primary consideration? TOPNOTCH MD)
A. Stevens Johnson Syndrome
B. Toxic Epidermal Necrolysis
C. Erythema Multiforme
D. A and B
E. All of the above
410 After giving birth the ductus arteriosus closes due postnasal increase in oxygen saturation of the JOSE CARLO DIAGNOSTIC
to which of the following mechanisms? systemic circulation is the strongest stimulus for MASANGKAY III, EXAM - AUG
A. Increase in systemic vascular resistance due to constriction of the ductal smooth musclesurrounding MD (TOP 8 - FEB 2014
removal of the low resistance placenta the ductus arteriosus. 2014 MED
B. Lack of blood return from the placenta BOARDS;
C. Increased pressure in the Left Atrium TOPNOTCH MD)
D. Increased arterial Oxygen saturation
E. None of the above
411 In a patient with ASD, which of the following would An ASD usually does not result in LA enlargement due JOSE CARLO DIAGNOSTIC
you least expect? to the increased pulmonary venous return. MASANGKAY III, EXAM - AUG
A. Enlarged Left Atrium MD (TOP 8 - FEB 2014
B. Systolic ejection murmur 2014 MED
C. Widely split S2 BOARDS;
D. Increased pulmonary venous return TOPNOTCH MD)
E. Right bundle branch block

412 Among the major criteria for Rheumatic fever, is Arthritis-most common, carditis-most severe, JOSE CARLO DIAGNOSTIC
the most specific? Subcutaneous nodules-least common, Sydenham's MASANGKAY III, EXAM - AUG
A. Arthritis chorea-most specific MD (TOP 8 - FEB 2014
B. Carditis 2014 MED
C. Erythema marginatum BOARDS;
D. Subcutaneous nodules TOPNOTCH MD)
E. Sydenham's chorea

413 A 6 year old Male patient presents with a 4-day The following characteristics does not match the JOSE CARLO DIAGNOSTIC
fever, strawberry tongue, Erythema of the hand and criteria for Kawasaki, Scarlet fever is a more MASANGKAY III, EXAM - AUG
Bilateral cervical lymphadenopathy.Among which appropriate option, hence antibiotics should be MD (TOP 8 - FEB 2014
of the following would be the appropriate administered. 2014 MED
treatment for the patient? BOARDS;
A. IV Ig 2g/kg over 10-12 hours TOPNOTCH MD)
B. Aspirin 3-5 mg/kg orally once daily
C. Observe and treat conservatively
D. Give steroids
E. Antibiotics
414 A 12 month-old born term, presented to the ED JOSE CARLO DIAGNOSTIC
because of an abnormally enlarging head, Macewen MASANGKAY III, EXAM - AUG
sign, and a setting-sun sign. Among which of the MD (TOP 8 - FEB 2014
following statements is the reason for the Occular 2014 MED
manifestation of the child? BOARDS;
A. Impingement of the cranial nerves TOPNOTCH MD)
B. Impingement of the tectum
C. Retraction of the orbit
D. Enlargement of the cranium creates the illusion
of a setting sun
E. Impingement of the Occipital lobe
415 A 2- year old patient was diagnosed with Acute Dexamethasone is given to reduce auditory nerve JOSE CARLO DIAGNOSTIC
Bacterial Meningitis. CSF culture was performed damage, given 1-2 hours before antibiotics are started. MASANGKAY III, EXAM - AUG
revealing a H. influenza infection. Prior to antibiotic MD (TOP 8 - FEB 2014
treatment Dexamethasone was given, what is the 2014 MED
rationale for this action? BOARDS;
A. To reduce CSF production TOPNOTCH MD)
B. To reduce brain inflammation
C. To reduce brain edema
D. To reduce auditory nerve damage
E. To reduce optic nerve damage
416 A newborn with tachypnea underwent a Babygram JOSE CARLO DIAGNOSTIC
revealing Coarse streaking granular pattern of both MASANGKAY III, EXAM - AUG
lung fields, what would be your primary MD (TOP 8 - FEB 2014
consideration? 2014 MED
A. Respiratory Distress Syndrome BOARDS;
B. Transient Tachypnea of the Newborn TOPNOTCH MD)
C. Bronchopulmonary Dysplasia
D. Meconium Aspiration Syndrome
E. Neonatal Pneumonia
417 Signs of Cephalhematoma except: Cephalhematoma may cause jaundice. JOSE CARLO DIAGNOSTIC
A. Subperiosteal Hemorrhage MASANGKAY III, EXAM - AUG
B. Does not cross the suture lines MD (TOP 8 - FEB 2014
C. No discoloration of the scalp 2014 MED
D. Does not cause jaundice BOARDS;
E. No Exception TOPNOTCH MD)

TOPNOTCH MEDICAL BOARD PREP PEDIATRICS SUPEREXAM Page 56 of 97


For inquiries visit www.topnotchboardprep.com.ph or email us at topnotchmedicalboardprep@gmail.com
TOPNOTCH MEDICAL BOARD PREP PEDIATRICS SUPEREXAM
For inquiries visit www.topnotchboardprep.com.ph or email us at topnotchmedicalboardprep@gmail.com
Item QUESTION EXPLANATION AUTHOR TOPNOTCH
# EXAM
418 A 7 year old child with severe cough and petechial Classic case of Pertussis. Erythromycin is the drug of JOSE CARLO DIAGNOSTIC
hemorrhages was brought tou you by his mother, choice MASANGKAY III, EXAM - AUG
apparently no fever was noted, no malaise, no sore MD (TOP 8 - FEB 2014
throat was noted, However on CBC, The WBC count 2014 MED
was 40,000/L, Neutrophils: 20%, Lymphocytes BOARDS;
70%, Monocytes 8%, Eosinophiles 2%. What would TOPNOTCH MD)
be the drug of choice for your patient?
A. Penicillin G
B. Racemic Epinephrine
C. Oseltamivir
D. Erythromycin
E. None, conservative treatment only
419 A neurologic disease with Albuminocytologic Albuminocytologic Dissociation: increased protein JOSE CARLO DIAGNOSTIC
dissociation on Lumbar tap is: with very low or normal Cell count on CSF analysis. MASANGKAY III, EXAM - AUG
A. Myasthenia Gravis MD (TOP 8 - FEB 2014
B. Meningitis 2014 MED
C. Neuroblastoma BOARDS;
D. Tuberous Sclerosis TOPNOTCH MD)
E. Guillane-Barre Syndrome

420 A 6 month old patient presents with low-grade JOSE CARLO DIAGNOSTIC
fever, rhinorrhea and cough. On PE wheezing was MASANGKAY III, EXAM - AUG
noted all over with hyperresonance on percussion. MD (TOP 8 - FEB 2014
Hyperinflated lungs are noted on CXR, among 2014 MED
which of the following is your initial diagnosis: BOARDS;
A. Bronchiolitis TOPNOTCH MD)
B. Bronchial Asthma
C. Pneumonia
D. Croup
E. Epiglotitis
421 A 4-year old male was brought to the ER after *Favorite question. Remember that PSGN can follow WEBSTER MIDTERM 1
passing tea-colored urine. Patient was also noted to either a throat (type 12) or cutaneous (type 49) ALINDOG, MD EXAM - AUG
have periorbaital edema and with BP of 130/90. infection by the nephrritogenic strain of S. pyogenes. (TOP 3 - FEB 2014
Urinalysis revealed 2-3 RBC casts. Mother insists Such test is the best single antibody that will 2014 MED
that the child has no history of recent pharyngeal or document skin infection-related PSGN. BOARDS;
oral infections. To support your hint that this is a TOPNOTCH MD)
case of cutaneous PSGN you would order for:
A. C3 levels
B. Streptozyme test
C. ASO titer
D. Anti-DNase B titer

422 A child was admitted to your hospital due to cough To differentiate diphtheria from strep throat, the WEBSTER MIDTERM 1
for 8 days with associated restlessness, headache, former relatively presents afebrile and with the well- ALINDOG, MD EXAM - AUG
anorexia, sore throat, hoarseness and eye redness. distinguished pseudomembrane in the pharynx. (TOP 3 - FEB 2014
On physical exam, he was noted to have a swollen Diphtheria is notorious for its grave complications of 2014 MED
neck with an adherent, dense, grey toxic cardiomyopathy and toxic neuropathy (previous BOARDS;
pseudomembrane covering the posterior aspect of question as well!). Penicillin is the treatment of choice. TOPNOTCH MD)
pharynx. If you are considering to eradicate
possible nasopharyngeal carriage of the caregivers
you will give them which drug?
A. Erythromycin
B. Ciprofloxacin
C. Cotrimoxazole
D. Ampicillin-Sulbactam
423 How is rubella different from measles? The most characteristic sign of rubella is the presence WEBSTER MIDTERM 1
A. Presence of enanthem of retroauricular lymphadenopathy (*buzz word). It ALINDOG, MD EXAM - AUG
B. Low grade fever for up to 3 days can be differentiated from measles by having low (TOP 3 - FEB 2014
C. Photophobia grade fever for 1-3 days, polyarthritis, and absence of 2014 MED
D. Development of dystonia and altered photophobia. BOARDS;
sensorium TOPNOTCH MD)
E. All of the above

424 A 2-year old male infant with down syndrome is This is a case of patent ductus arteriosus. Anatomic WEBSTER MIDTERM 1
brought to your clinic for routine check-up. On closure is normally completed at 2-3 weeks of age ALINDOG, MD EXAM - AUG
auscultation you appreciate a continuous whereas functional closure is already achieved as (TOP 3 - FEB 2014
machinery-like murmur suggesting the presence of early as 10-15 hours. PDA and other congenital heart 2014 MED
a shunt occuring throught the cycle. You are diseases are common among Down syndrome BOARDS;
immediately convinced that this is abnormal patients. TOPNOTCH MD)
because you know that the anatomic closure of the
fetal shunt involved is at:
A. 10 hours after birth
B. 2 weeks after birth
C. 10 months after birth
D. 2 years after birth
425 These are benign cysts appearing on the hard A ranula is a mucocoele found on the floor of the WEBSTER MIDTERM 1
palate of the newborn and are composed of mouth, a connective tissue swelling after rupture of a ALINDOG, MD EXAM - AUG
accumulated epithelial cells: salivary gland duct as a result of local trauma. Epulis, (TOP 3 - FEB 2014
A. Ranula on the other hand, simply refers to any benign mass 2014 MED
B. Epulis situated on the gingiva. Both differ from epstein pearls BOARDS;
C. Mucous retention cysts by not being present in normal neonates and also by TOPNOTCH MD)
D. Epstein pearls anatomic location.

TOPNOTCH MEDICAL BOARD PREP PEDIATRICS SUPEREXAM Page 57 of 97


For inquiries visit www.topnotchboardprep.com.ph or email us at topnotchmedicalboardprep@gmail.com
TOPNOTCH MEDICAL BOARD PREP PEDIATRICS SUPEREXAM
For inquiries visit www.topnotchboardprep.com.ph or email us at topnotchmedicalboardprep@gmail.com
Item QUESTION EXPLANATION AUTHOR TOPNOTCH
# EXAM
426 A 5-year old male patient was seen at the ER due to The patient is suffering from Kawasaki disease. The WEBSTER MIDTERM 1
one week history of fever with associated eye diagnostic criteria for such: fever lasting for at least 5 ALINDOG, MD EXAM - AUG
redness, swollen erythematous hands and soles, days; bilateral bulbar conjunctival injection, non- (TOP 3 - FEB 2014
and nonvesicular truncal rash. On examination, purulent; Mucosal changes in the oropharynx 2014 MED
cervical lymphadenopathy and strawberry tongue (including strawberry tongue); peripheral extremity BOARDS;
were observed. The single most important changes (including edema and erythema); rash; and TOPNOTCH MD)
diagnostic test you must order for this patient is: cervical lymphadenopathy (>1.5 cm, usuallu
A. ESR unilateral); and illness should not be explained by
B. Acute phase reactants other identifiable disease. There is no specific
C. CT/MRI diagnostic test, however, 2D echo is essential to
D. 2D echocardiography monitor the possible development of a coronary
disease, the most serious complication of Kawasaki.
427 An infant was noted to have vomiting after every GI obstructive conditions and other anomalies WEBSTER MIDTERM 1
feeding. No other symptoms reported. Physical (volvulus, duodenal atresia, Meckel, gastroschisis vs ALINDOG, MD EXAM - AUG
exam shows a restless tachycardic infant with a omphalocele) are high yield topics. (TOP 3 - FEB 2014
palpable firm, movable olive-shaped mass in the 2014 MED
abdomen. Your initial impression of pyloric BOARDS;
stenosis will be confirmed by: TOPNOTCH MD)
A. An abdominal UTZ showing pyloric length
>14mm and thickness of >4 mm
B. A barium study showing double tract sign
corresponding to a narrowed channel
C. An Xray exhibiting a double bubble sign
D. An endoscopic biopsy revealing hypertonic and
hypertrophic pyloric muscle
428 A 12-month old infant underwent an endorectal The recctosigmoid is affected in majority of the cases. WEBSTER MIDTERM 1
pull-through procedure to correct his Hirschprung. There is arrest in neuroblast migration that leads to ALINDOG, MD EXAM - AUG
If you are to examine the biopsied segment you absence of Meissner and Auerbach plexuses. Such (TOP 3 - FEB 2014
would say that there is: condition results in decreased motility in the affected 2014 MED
A. Excessive hypertrophy of the muscular and segment; lack of propagation of the peristaltic waves BOARDS;
submucosal layers of the upper and lower rectum going to the aganglionic part; and abnormal TOPNOTCH MD)
B. Lymphoid proliferation with notable relaxation. All these explain its other common name
enlargement of the submucosal layer congenital aganglionic megacolon.
C. Absence of ganglion cells in the wall
D. Foci of muscular hypertrophy and sporadic
villous masses with necrotic core
429 This immunoglobulin does not fix complement. It is IgA, particularly the secretory type ( a dimer Ig). It is WEBSTER MIDTERM 1
synthesized in the GI walls to bind with possible not only found in GI but also in the respiratory tract ALINDOG, MD EXAM - AUG
harmful elements in the lumen: and in excretory glands. IgM is best in fixing the (TOP 3 - FEB 2014
A. IgM complement because it is pentameric, and hence the 2014 MED
B. IgG largest of all Igs. IgG is able to cross the placenta and is BOARDS;
C. IgD important in natural passive immunity. IgD on the TOPNOTCH MD)
D. IgA other hand is said to play role in isotypic switching of
Igs. IgE plays role both in hypersensitivity and
parasitism.
430 What is the most important risk factor that WEBSTER MIDTERM 1
predisposes a newborn to sepsis? ALINDOG, MD EXAM - AUG
A. An HIV infected mother (TOP 3 - FEB 2014
B. Chorioamnionitis 2014 MED
C. Prematurity BOARDS;
D. Severe combined immunodeficiency (SCID) TOPNOTCH MD)

431 A 15-year old male sought consult in a local clinic Migraine with aura aka Alice in wonderland syndrome WEBSTER MIDTERM 1
complaining of recurrent headache. He described it is characterized by photopsia, blurring of vision and ALINDOG, MD EXAM - AUG
as a band-like tightness sometimes throbbing fortification spectra. Hemiplegic aura on the hand, has (TOP 3 - FEB 2014
occuring in the frontal region that radiates to his unilateral sensory or motor signs that may persist for 2014 MED
left parietal area. There was no nausea, vomiting, days. Basilar-type migraine is due to vasoconstriction BOARDS;
dizziness, anorexia, photophobia, change in of basilar and posterior cerebral arteries that results TOPNOTCH MD)
alertness or intellect. He also reported that the in vertigo, tinnitus and diploma, with altered
condition seems to worsen during classes. At this consciousness and seizures.
point, the best possible impression is:
A. Migraine without aura
B. Hemiplegic aura
C. Basilar-type migraine
D. Tension headache
432 Laboratory findings in congenital adrenal 21-hydroxylase deficiency also accounts for 90% of WEBSTER MIDTERM 1
hyperplasia (21-hydroxylase deficiency) will show patients affected with CAH. Please review the adrenal ALINDOG, MD EXAM - AUG
LOW serum Na, Cl, and cortisol; and HIGH serum K, steroidogenesis pathway, high yield. (TOP 3 - FEB 2014
renin, and: 2014 MED
A. Deoxycorticosterone BOARDS;
B. 11-deoxycortisol TOPNOTCH MD)
C. 17-hydroxyprogesterone
D. Pregnenelone

433 Karyotyping result of a patient with Klinefelter SIMILAR TO PREVIOUS BOARD EXAM WEBSTER MIDTERM 1
syndrome will show: CONCEPT/PRINCIPLE ALINDOG, MD EXAM - AUG
A. 45 XO (TOP 3 - FEB 2014
B. 47 XXY 2014 MED
C. 46 XY BOARDS;
D. None of the above TOPNOTCH MD)

TOPNOTCH MEDICAL BOARD PREP PEDIATRICS SUPEREXAM Page 58 of 97


For inquiries visit www.topnotchboardprep.com.ph or email us at topnotchmedicalboardprep@gmail.com
TOPNOTCH MEDICAL BOARD PREP PEDIATRICS SUPEREXAM
For inquiries visit www.topnotchboardprep.com.ph or email us at topnotchmedicalboardprep@gmail.com
Item QUESTION EXPLANATION AUTHOR TOPNOTCH
# EXAM
434 A 2-year old female patient was diagnosed with Ferrous sulfate is given at a dose 6 mg/kg/day WEBSTER MIDTERM 1
iron deficiency anemia. Iron supplement therapy (elemental iron). Subjective improvement is expected ALINDOG, MD EXAM - AUG
was immediately instituted. At what time after after 12-24 hours; preliminary BM response after 48 (TOP 3 - FEB 2014
beginning of treatment do you expect hemoglobin hours; increase in reticulocyte count after 72 hours; 2014 MED
to increase? hemoglobin increase after 4-6 weeks; and repletion of BOARDS;
A. 1 day iron stores after 3 months. TOPNOTCH MD)
B. 3 days
C. 6 weeks
D. 3 months
435 What is the most common cause of Differentiate ITP from TTP. ITP is an autoimmune WEBSTER MIDTERM 1
thrombocytopenic purpura in children? destruction of platelets, and is usually precipitated by ALINDOG, MD EXAM - AUG
A. Idiopathic thrombocytopenic purpura a viral infection. Bruising and petechial rash are the (TOP 3 - FEB 2014
B. Thrombotic thrombocytopenic purpura prominent signs. TTP, on the other hand, is more 2014 MED
C. Microangiopathic hemolytic anemia toxic-looking - similar to HUS. It has five cardinal BOARDS;
D. Acute lymphoblastic leukemia symptoms: fever, hemolytic anemia, TOPNOTCH MD)
thrombocytopenia, renal dysfunction, and nervous
sytem changes (mnemonic is FAT RN).
436 Which of the following is not true about brain WEBSTER MIDTERM 1
tumors in children? ALINDOG, MD EXAM - AUG
A. Highest incidence in ages 1-10 years (TOP 3 - FEB 2014
B. Commonly located in the infratentorial area 2014 MED
C. Considered to be the most common solid tumor BOARDS;
in pediatrics TOPNOTCH MD)
D. All of the above
E. None of the above
437 A 6-year old boy was brought to pedia ER because This is a case of acute epiglottitis, caused by H. WEBSTER MIDTERM 1
of acute onset of high fever and for appearing to be influenzae. IV antibiotics 3rd generation ALINDOG, MD EXAM - AUG
dyspneic. Mother reports that patient also has cephalosporin or ampicillin-sulbactam are the (TOP 3 - FEB 2014
difficulty swallowing, drooling and muffled voice. primary drugs of choice. 2014 MED
Xray shows a thumbprint sign. Later on, the patient BOARDS;
was intubated due to worsening symptoms. Being TOPNOTCH MD)
able to identify the most likely organism involved,
the patient should be started with:
A. Penicillin
B. Trimethoprim-Sulfamethoxazole
C. Ceftriaxone
D. Racemic epinephrine
438 A 3-year old malnourished male patient is brought Fever, cough, tachypnea - triad of pneumonia. WEBSTER MIDTERM 1
to your health center due to difficulty of breathing. Dullness, faint or decreased breath sounds suggest ALINDOG, MD EXAM - AUG
Initial assessment reveals T 39.0, RR 51 with consolidation. (TOP 3 - FEB 2014
episodes of severe coughing, and stable 2014 MED
hemodynamics. There is focal dullness and faint BOARDS;
vesicular breath sounds on left lower posterior TOPNOTCH MD)
chest. Given these quick initial findings, your
strongest consideration is:
A. Lobar pneumonia
B. Bronchopneumonia
C. Bronchiolitis
D. Primary pulmonary tuberculosis
439 What is the main purpose of giving BCG vaccine? WEBSTER MIDTERM 1
A. To promote latency of TB infection ALINDOG, MD EXAM - AUG
B. To prevent extra-pulmonary TB (TOP 3 - FEB 2014
C. To prevent primary TB infection 2014 MED
D. All of the above BOARDS;
E. None of the above TOPNOTCH MD)

440 Which serologic marker is found only in SLE? ANA serves as an excellent screening parameter but it WEBSTER MIDTERM 1
A. ANA is not required in the diagnosis. It is nonspecific ALINDOG, MD EXAM - AUG
B. Anti-Smith because it can also be significant in other rheumatic (TOP 3 - FEB 2014
C. Anti-ds DNA conditions. Anti-ds DNA is more specific to SLE than 2014 MED
D. ESR ANA and is used to assess the degree of disease BOARDS;
activity. ESR is also non-specific and is a marker of TOPNOTCH MD)
acute inflammation.

441 A woman was noted to have a large volume of Duodenal atresia is due to failure to recanalize the JULIET KRISTINE MIDTERM 2
amniotic fluid at the time of her delivery of her lumen of the duodenum. Obstruction is usually distal EVANGELISTA, EXAM - AUG
child. At about 6-8 hours of age, the neonate begins to the ampulla of Vater. Hallmarks iclude bilious MD (TOP 9 - FEB 2014
regurgitating small amount of mucus and bile- vomiting without abdominal distention, 2014 MED
stained fluid. Physical examination was normal. X- polyhydramnios in 50% due to failure of absorption of BOARDS;
ray was requested and done revealing a double- amniotic fluid in the distal intestine. Double-bubble TOPNOTCH MD)
bubble sign. The most commonly affected part of sign is a radiologic finding due to distended and gas-
the gastrointestinal tract associated with the filled stomach and proximal duodenum.
diagnosis is:
A. Esophagus
B. Stomach
C. Pylorus
D. Duodenum
E. Large colon

TOPNOTCH MEDICAL BOARD PREP PEDIATRICS SUPEREXAM Page 59 of 97


For inquiries visit www.topnotchboardprep.com.ph or email us at topnotchmedicalboardprep@gmail.com
TOPNOTCH MEDICAL BOARD PREP PEDIATRICS SUPEREXAM
For inquiries visit www.topnotchboardprep.com.ph or email us at topnotchmedicalboardprep@gmail.com
Item QUESTION EXPLANATION AUTHOR TOPNOTCH
# EXAM
442 A 5 year-old boy was brought by his mother at the Erythema infectiosum is caused by Parvovirus B19 JULIET KRISTINE MIDTERM 2
out-patient department. He presented with a mild characterized by low grade fever, headache, upper EVANGELISTA, EXAM - AUG
respiratory prodrome. The initial stage was respiratory tract symptoms and a characteristic MD (TOP 9 - FEB 2014
characterized with erythematous facial flushing erythematous facial flushing or the slapped-cheek 2014 MED
likened to a slapped cheek. The disease is most appearance which spreads rapidly to the trunk and BOARDS;
likely caused by: proximal extremities as a diffuse macular erythema. TOPNOTCH MD)
A. Rubella
B. Rubeola
C. HHV-6
D. Parvovirus B19
E. Varicella
443 A 9 year-old boy has crampy abdominal pain and Henoch-schonlein purpura is a mucocutaneous JULIET KRISTINE MIDTERM 2
purpuric rashes on the back of his legs and buttocks syndrome characterized by a rash which is palpable EVANGELISTA, EXAM - AUG
as well as the extensor surfaces of his forearm. The petechia or purpura that evolve from red to brown MD (TOP 9 - FEB 2014
most probable diagnosis is: that last from 3-10 days. It is associated with arthritis, 2014 MED
A. Postrep GN intermittent abdominal pain with renal involvement. BOARDS;
B. Henoch-Schonlein Purpura TOPNOTCH MD)
C. IgA Nephropathy
D. Kawasaki Disease
E. Hemolytic Uremic Syndrome
444 An infant was noted of intractable neonatal Beckwith-Wiedemann syndrome is an overgrowth JULIET KRISTINE MIDTERM 2
hypoglycemia associated with multiple congenital disorder usually present at birth characterized by an EVANGELISTA, EXAM - AUG
defects including macroglossia, visceromegaly, mild increased risk of childhood cancer and certain MD (TOP 9 - FEB 2014
microcephaly, omphalocephaly, facial nevus congenital features. Five common features used to 2014 MED
flammeus and characteristic earlobe creases. The define BWS are: macroglossia, macrosomia, midline BOARDS;
above findings pertains to the syndrome: abdominal wall defects (omphalocele/exomphalos, TOPNOTCH MD)
A. Turner syndrome umbilical hernia, diastasis recti), ear creases or ear
B. Klinefelter syndrome pits, and neonatal hypoglycemia. Other findings
C. Beckwith-Wiedemann syndrome include nevus flammeus, prominent occiput, midface
D. Down syndrome hypoplasia, hemihypertrophy, genitourinary
E. Edward's syndrome anomalies, cardiac anomalies, musculoskeletal
abnormalities, and hearing loss.
445 A 15 month-old male suddenly draws up his leg and Intussusception occurs when a portion of the JULIET KRISTINE MIDTERM 2
scream in pain. It was repeated periodically alimentary tract is telescoped into an adjacent EVANGELISTA, EXAM - AUG
throughout the night interspersed with periods of segment. Clinical manifestations include severe MD (TOP 9 - FEB 2014
quiet sleep. He was brought to the Emergency paroxysmal colicky pain that recurs at frequent 2014 MED
Room after 12 hours and he looked pale. He intervals with straining efforts, legs and knees are BOARDS;
vomited and passed out blood-streaked stools. PE flexed with loud crying, about 60% of infants and TOPNOTCH MD)
revealed a palpable mass around the umbilicus. The children pass currant jelly stool. PE findings include
most likely diagnosis is: palpation of slightly tender sausage-shaped mass in
A. Acute appendicitis the RUQ which may increase in size and firmness
B. Volvulus during paroxysm of pain.
C. Meckel's diverticulum
D. Intussusception
E. Mesenteric adenitis
446 An 8 year-old girl has been having on-and-off fever On the acute phase of Kawasaki disease, IV JULIET KRISTINE MIDTERM 2
for the past 10 days. On examination, there were immunoglobulin results in rapid defervescence and EVANGELISTA, EXAM - AUG
conjunctivitis, strawberry tongue and periungual resolution of clinical signs if illness. Aspirin is given MD (TOP 9 - FEB 2014
desquamation. The appropriate drug to be during the convalescent stage, for prevention of acute 2014 MED
prescribed to the patient is: coronary thrombosis. BOARDS;
A. Steroids TOPNOTCH MD)
B. Aspirin
C. Antibiotics
D. Vitamin A
E. Zinc sulfate
447 A 7 year-old child developed high-grade fever and Staphylococcus aureus is one of the causes of JULIET KRISTINE MIDTERM 2
cough. Chest x-ray was done and revealed overwhelming hospital-acquired pneumonia. In EVANGELISTA, EXAM - AUG
pneumatoceles. What is the most common children, Xray findings in children are rapidly MD (TOP 9 - FEB 2014
pathogenic organism responsible for the above developing lobar/multilobar consolidation, pleural 2014 MED
findings: effusion and pneumatocoele. BOARDS;
A. Haemophilus influenzae TOPNOTCH MD)
B. Streptococcus pneumoniae
C. Staphylococcus aureus
D. Parainfluenza virus
E. Respiratory syncitial virus
448 The World Health Organization recommends that The World Health Organization recommends that JULIET KRISTINE MIDTERM 2
breastfeeding can be the infant's sole source of food breastfeeding can be the infant's sole source of food EVANGELISTA, EXAM - AUG
up to about: up to about 6 months of age. MD (TOP 9 - FEB 2014
A. 3 months of age 2014 MED
B. 6 months of age BOARDS;
C. 9 months of age TOPNOTCH MD)
D. 12 months of age
E. 2 years of age

449 Very low birth weight infants weigh less than ___ Low birth weight infants weigh <2500 grams, very JULIET KRISTINE MIDTERM 2
grams: low birth weight infants <1500 grams, extremely low EVANGELISTA, EXAM - AUG
A. 1000 grams birth weight infants <1000grams MD (TOP 9 - FEB 2014
B. 1500 grams 2014 MED
C. 2000 grams BOARDS;
D. 2500 grams TOPNOTCH MD)
E. 3000 grams

TOPNOTCH MEDICAL BOARD PREP PEDIATRICS SUPEREXAM Page 60 of 97


For inquiries visit www.topnotchboardprep.com.ph or email us at topnotchmedicalboardprep@gmail.com
TOPNOTCH MEDICAL BOARD PREP PEDIATRICS SUPEREXAM
For inquiries visit www.topnotchboardprep.com.ph or email us at topnotchmedicalboardprep@gmail.com
Item QUESTION EXPLANATION AUTHOR TOPNOTCH
# EXAM
450 A 6 month-old boy was brought at the OPD for Paraphimosis is a condition that affects uncircumcised JULIET KRISTINE MIDTERM 2
check-up. On physical examination, you noted that males and occurs when the retracted foreskin can no EVANGELISTA, EXAM - AUG
the prepuce can be retacted but cannot be returned longer be pulled forward over the tip of the penis. MD (TOP 9 - FEB 2014
back. He has: 2014 MED
A. Hydrocele BOARDS;
B. Hypospadia TOPNOTCH MD)
C. Chordee
D. Paraphimosis
E. Epispadia
451 A neonate was born with a heart rate over Heart rate over 100/minute=2, slow irregular JULIET KRISTINE MIDTERM 2
100/minute, slow irregular breathing, some flexion breathing=1, some flexion of the extremities=1, EVANGELISTA, EXAM - AUG
of the extremities, grimacing, pink body with blue grimacing=1, pink body with blue extremities=1. MD (TOP 9 - FEB 2014
extremities. APGAR score at 1 minute is: 2+1+1+1+1=6 2014 MED
A. 5 BOARDS;
B. 6 TOPNOTCH MD)
C. 7
D. 8
E. 9
452 A neonate developed jaundice in the first 24 hours Neonatal jaundice is physiologic if it presents after the JULIET KRISTINE MIDTERM 2
of life. These are differential diagnoses except: 24-48th hour of life. Pathologic jaundice presents in EVANGELISTA, EXAM - AUG
A. Physiologic jaundice the 1st 24 hours of life. Among the causes include: MD (TOP 9 - FEB 2014
B. ABO incompatibility sepsis, TORCHS, Rh and ABO incompatibility, 2014 MED
C. Sepsis hemorrhage BOARDS;
D. Hemorrhage TOPNOTCH MD)
E. None of the above

453 Precocious puberty is defined as onset of puberty Precocious puberty is the onset of secondary sexual JULIET KRISTINE MIDTERM 2
before age ___ years in girls and age 9 years in boys: characteristics before 8 years old in girls and 9 years EVANGELISTA, EXAM - AUG
A. 7 old in boys. MD (TOP 9 - FEB 2014
B. 8 2014 MED
C. 9 BOARDS;
D. 10 TOPNOTCH MD)
E. 11

454 A 15 year-old consulted for leg swelling and pain. Ewing's sarcoma is a rare bone cancer that primarily JULIET KRISTINE MIDTERM 2
Xray showed lytic lesion with periosteal reaction of affects children and adolescents. On conventional EVANGELISTA, EXAM - AUG
the tibia. The most likely diagnosis is: radiographs, the most common osseous presentation MD (TOP 9 - FEB 2014
A. Osteosarcoma is a lytic lesion with periosteal reaction. The classic 2014 MED
B. Ewing's sarcoma description of lamellated or "onion skin" type BOARDS;
C. Rhabdomyosarcoma periosteal reaction is often associated with this lesion. TOPNOTCH MD)
D. Chondroblastoma
E. Enchondroma

455 A child complains of anal pruritus causing sleep Enterobiasis is a pinworm infection, a human parasitic JULIET KRISTINE MIDTERM 2
disruption. You do a scotch tape test to check for disease and one of the most common parasitic worm EVANGELISTA, EXAM - AUG
the parasite. Upon identification, what is the drug of infections in the developed world. It is caused by MD (TOP 9 - FEB 2014
choice that you will prescribed for the patient? infestation of the parasitic roundworm Enterobius 2014 MED
A. Ivermectin vermicularis. The drug of choice for pinworm is BOARDS;
B. Mebendazole Pyrantel pamoate which causes depolarization- TOPNOTCH MD)
C. Pyrantel pamoate induced paralysis in woms. Ivermectin is the drug of
D. Diethylcarbamazine choice for strongyloides, mebendazole is for trichuris,
E. Thiabendazole diethylcarbamazine is for wuchereria and brugia and
thiabendazole is for trichinella.
456 Radiographic examination to determine skeletal A bone age study helps doctors estimate the maturity JULIET KRISTINE MIDTERM 2
maturity or bone age is best evaluated by x-rays of of a child's skeletal system. It's usually done by taking EVANGELISTA, EXAM - AUG
the: a single X-ray of the wrist. A child's bones, such as MD (TOP 9 - FEB 2014
A. Wrist those in the fingers and wrist, contain "growing 2014 MED
B. Elbow zones" at both ends called growth plates. BOARDS;
C. Hip TOPNOTCH MD)
D. Knee joint
E. Ankle
457 A raised patch of epithelial debris on the Bitot's spots are the buildup of keratin debris located JULIET KRISTINE MIDTERM 2
conjunctiva of a child due to vitamin A deficiency is: superficially in the conjunctiva, which are oval, EVANGELISTA, EXAM - AUG
A. Corneal scar triangular or irregular in shape. These spots are a sign MD (TOP 9 - FEB 2014
B. Bitot's spots of vitamin A deficiency and are associated with 2014 MED
C. Kayser-fleischer rings conjunctival xerosis. Kayser–Fleischer rings are dark BOARDS;
D. Conjunctival xerosis rings that appear to encircle the iris of the eye due to TOPNOTCH MD)
E. Pterygium copper deposition in part of the cornea as a result of
particular liver diseases.
458 A 3 year-old boy had fever and sore throat which Acute postrep glomerulonephritis follows infection of JULIET KRISTINE MIDTERM 2
uneventfully recovered spontaneously. A week the throat by nephritogenic strains of group A Beta- EVANGELISTA, EXAM - AUG
later, he awakens with eyelid puffiness and scanty hemolytic streptococci. Clinical manifestations include MD (TOP 9 - FEB 2014
tea-colored urine. This is most likely: gross hematuria, periorbital edema, hypertension and 2014 MED
A. Nephrotic syndrome oliguria. BOARDS;
B. Acute pyelonephritis TOPNOTCH MD)
C. Acute glomerulonephritis
D. Nephrolithiasis
E. Acute cystitis
459 A typical skin finding of infective endocarditis Osler's nodules are tender, pea-sized intradermal JULIET KRISTINE MIDTERM 2
characterized by tender, pea-sized intradermal nodules in the pads of the fingers and toes found in EVANGELISTA, EXAM - AUG
nodules in the pads of the fingers and toes is known Infective endocarditis. Janeway lesions are painless MD (TOP 9 - FEB 2014
as: erythematous of hemorrhagic lesions on the palms 2014 MED
A. Janeway lesions and soles. Splinter hemorrhages are linear lesions BOARDS;
B. Subcutaneous nodules beneath the nails. Roth's spots are retinal TOPNOTCH MD)
C. Osler's nodules hemorrhages found in subacute bacterial endocarditis.
D. Roth's spots
E. Splinter hemorrhages
TOPNOTCH MEDICAL BOARD PREP PEDIATRICS SUPEREXAM Page 61 of 97
For inquiries visit www.topnotchboardprep.com.ph or email us at topnotchmedicalboardprep@gmail.com
TOPNOTCH MEDICAL BOARD PREP PEDIATRICS SUPEREXAM
For inquiries visit www.topnotchboardprep.com.ph or email us at topnotchmedicalboardprep@gmail.com
Item QUESTION EXPLANATION AUTHOR TOPNOTCH
# EXAM
460 Five years after a cutaneous viral infection, a 12 Subacute sclerosing panencephalitis is a chronic JULIET KRISTINE MIDTERM 2
year-old boy had lethargy, altered level of complication of measles with a delayed onset due to EVANGELISTA, EXAM - AUG
consciousness, headache, nausea, vomiting and persistent infection with an altered virus that is MD (TOP 9 - FEB 2014
fever. This most likely post-infection is caused by a harbored intracellularly in the CNS. It is caused by 2014 MED
virus: Rubeola from the family of Paramyxoviruses. BOARDS;
A. Rubella TOPNOTCH MD)
B. Rubeola
C. Herpes simplex
D. Epstein Barr
E. Cytomegalovirus
461 A 5 year old child presents with high grade fever, this is rubeola, which presents with fever, coryza and LUISA BACK-UP
rhinorrhea, and conjunctivitis. The mother also conjunctivitis in which rashes appear at the height of SARANILLO, MD MIDTERM
noted rashes at the height of fever, which appears the fever. In roseola, rashes appear when fever abates. (TOP 6 - FEB EXAM AUG
first on the face then spreads to the trunk and In Rubella, the most characteristic is the presence of 2014 MED 2014 - FOR
extremities. What is the diagnosis? retroauricular, posterior cervical, and postoccipital BOARDS; INCLUSION IN
A. Rubeola lymphadenopathy. TOPNOTCH MD) THE SAMPLEX
B. Rubella
C. Roseola
D. Varicella
E. Fifth disease
462 What is the normal blood volume of a child? The normal blood volume of a child is 80 ml/kg, while LUISA BACK-UP
A. 100 ml/kg for infant is 100 ml/kg. SARANILLO, MD MIDTERM
B. 90 ml/kg (TOP 6 - FEB EXAM AUG
C. 80 ml/kg 2014 MED 2014 - FOR
D. 125 ml/kg BOARDS; INCLUSION IN
E. 130ml/kg TOPNOTCH MD) THE SAMPLEX

463 A G3P2 mother delivered a live term baby Boy with The APGAR score is 9 as computed: good cry=2, good LUISA BACK-UP
good cry, good activity, peripheral cyanosis, activity = 2, peripheral cyanosis = 1, RR of 60cpm = 2, SARANILLO, MD MIDTERM
respiratory rate of 60cpm, heart rate of 110bpm. HR = 110bpm. (TOP 6 - FEB EXAM AUG
What is the APGAR score? 2014 MED 2014 - FOR
A. 10 BOARDS; INCLUSION IN
B. 9 TOPNOTCH MD) THE SAMPLEX
C. 8
D. 7
E. 6
464 A 4 year old child who just recently had impetigo, LUISA BACK-UP
presents with gross hematuria and periorbital SARANILLO, MD MIDTERM
edema. The Blood Pressure is 100/70mmHg, (TOP 6 - FEB EXAM AUG
Respiratory rate of 25cpm, heart rate of 80bpm, 2014 MED 2014 - FOR
and temperature of 37.3deg C. There are RBC cast BOARDS; INCLUSION IN
in urinalysis, CBC shows mild normochromic TOPNOTCH MD) THE SAMPLEX
anemia, and levels of C3 is low. In this case, usually
how long will hematuria persists?
A. 6-12 months
B. 6-12 weeks
C. 6-12 days
D. 2-4 years
E. 3-5 months
465 A 5 year old child presents with fever of 6 days This is a case of Kawasaki disease in which there is no LUISA BACK-UP
duration associated with nonpurulent specific diagnostic test. The diagnosis of Kawasaki is SARANILLO, MD MIDTERM
conjunctivitis, dry and fissured lips, nonvesicular primarily clinical. (TOP 6 - FEB EXAM AUG
rashes on the trunk, and cervical lymphadenopathy. 2014 MED 2014 - FOR
What is the most specific diagnostic test? BOARDS; INCLUSION IN
A. antibody determination TOPNOTCH MD) THE SAMPLEX
B. acute phase protein level determination
C. complete blood count
D. urinalysis
E. none of the choices
466 A 3 week old neonate presents with nonbilious Pyloric stenosis is the most common cause of LUISA BACK-UP
vomiting. On examination of the abdomen, there is nonbilious vomiting with a firm, movable, olive- SARANILLO, MD MIDTERM
olive-shaped mass. What is the diagnosis? shaped mass in the abdomen. (TOP 6 - FEB EXAM AUG
A. Pyloric stenosis 2014 MED 2014 - FOR
B. Duodenal atresia BOARDS; INCLUSION IN
C. Volvulus TOPNOTCH MD) THE SAMPLEX
D. Meckel diverticulum
E. GERD

467 A 30 year old father with a homozygous Vitamin D When an affected male transmits the trait to all his LUISA BACK-UP
resistant rickets transmitted the trait to all of his daughters but to none of his sons, it is X-linked SARANILLO, MD MIDTERM
daughters but to none of his sons. Therefore it is: dominant. Vitamin D resistant rickets is an example. (TOP 6 - FEB EXAM AUG
A. Autosomal dominant 2014 MED 2014 - FOR
B. Autosomal recessive BOARDS; INCLUSION IN
C. X-linked recessive TOPNOTCH MD) THE SAMPLEX
D. X-linked dominant
E. multifactorial inheritance

468 One of the following does not describe the colostrum has low fat and carbohydrates LUISA BACK-UP
colostrum. SARANILLO, MD MIDTERM
A. Produced in the first 0-7 days (TOP 6 - FEB EXAM AUG
B. More than 80% water 2014 MED 2014 - FOR
C. High levels of fat and carbohydrates BOARDS; INCLUSION IN
D. Rich in protein TOPNOTCH MD) THE SAMPLEX
E. None of the choices

TOPNOTCH MEDICAL BOARD PREP PEDIATRICS SUPEREXAM Page 62 of 97


For inquiries visit www.topnotchboardprep.com.ph or email us at topnotchmedicalboardprep@gmail.com
TOPNOTCH MEDICAL BOARD PREP PEDIATRICS SUPEREXAM
For inquiries visit www.topnotchboardprep.com.ph or email us at topnotchmedicalboardprep@gmail.com
Item QUESTION EXPLANATION AUTHOR TOPNOTCH
# EXAM
469 A 3 old child was brought for consultation due to this is a classic roseola infection, rashes appear when LUISA BACK-UP
rashes. Upon review of history, the mother said that fever abates. The etiologic agent is human herpes SARANILLO, MD MIDTERM
the child had fever for 4 days, and that rashes virus 6. (TOP 6 - FEB EXAM AUG
appeared when fever abated. What is the etiologic 2014 MED 2014 - FOR
agent? BOARDS; INCLUSION IN
A. RNA virus from paramyxoviridae TOPNOTCH MD) THE SAMPLEX
B. RNA virus from Togaviridae
C. Human herpes virus 6
D. Human herpes virus 1
E. Coxsackie virus A
470 A 1 year old child was brought to the emergency this is a case of status epilepticus and the drug of LUISA BACK-UP
department due to seizure at the height of the fever. choice is phenytoin. SARANILLO, MD MIDTERM
The seizure lasted for 2 minutes and then the (TOP 6 - FEB EXAM AUG
patient had another 3 seizure episodes within the 2014 MED 2014 - FOR
30 minute period without regaining consciousness. BOARDS; INCLUSION IN
What is the drug of choice to give? TOPNOTCH MD) THE SAMPLEX
A. valproic acid
B. diazepam
C. phenytoin
D. phenobarbital
E. carbamazepine
471 In congenital syphilis, one of its manifestations is mulberry molar - abnormal first lower molar; LUISA BACK-UP
mulberry molar. What tooth is affected? hutchinson teeth - barrel shaped upper central SARANILLO, MD MIDTERM
A. First lower molar incisors (TOP 6 - FEB EXAM AUG
B. Second lower molar 2014 MED 2014 - FOR
C. First upper molar BOARDS; INCLUSION IN
D. Second upper molar TOPNOTCH MD) THE SAMPLEX
E. Upper central incisors

472 A 17 year old sex worker came in for consultation, This is a case of infectious mononucleosis caused by LUISA BACK-UP
complaining of fever for 3 days associated with EBV. The most feared complication is splenic rupture SARANILLO, MD MIDTERM
fatigue and generalized lymphadenopathy. On caused due to trauma in the 2nd week of illness (TOP 6 - FEB EXAM AUG
examination, there is splenomegaly and severe 2014 MED 2014 - FOR
pharyngitis with marked tonsillar enlargement, BOARDS; INCLUSION IN
with petechiae at the junction of the hard and soft TOPNOTCH MD) THE SAMPLEX
palate. What is the most feared complication?
A. carcinoma
B. splenic rupture
C. heart disease
D. secondary bacterial infection
E. hepatic rupture
473 A 3 year old child presents with 2 day fever and LUISA BACK-UP
white discrete papules with central umbilication in SARANILLO, MD MIDTERM
the extremities, head and neck. What is the (TOP 6 - FEB EXAM AUG
diagnosis? 2014 MED 2014 - FOR
A. Yellow fever BOARDS; INCLUSION IN
B. Molluscum contagiosum TOPNOTCH MD) THE SAMPLEX
C. Allergic reaction
D. Chicken pox
E. Erythema multiforme
474 What is the most common congenital heart disease? LUISA BACK-UP
A. Ventricular septal defect SARANILLO, MD MIDTERM
B. Atrial septal defect (TOP 6 - FEB EXAM AUG
C. Tetralogy of fallot 2014 MED 2014 - FOR
D. Patent ductus arteriosus BOARDS; INCLUSION IN
E. Tricuspid atresia TOPNOTCH MD) THE SAMPLEX

475 A 1.8 kg newborn presents with chorioretinitis, intracerebral calcification is seen in toxoplasmosis, LUISA BACK-UP
microcephaly and hepatomegaly. On imaging, there while periventricular calcificartion is seen in CMV. SARANILLO, MD MIDTERM
is intracerebral calcifications seen. What is the Other choices do not have cerebral calcification. (TOP 6 - FEB EXAM AUG
diagnsosis? 2014 MED 2014 - FOR
A. Herpes simplex virus infection BOARDS; INCLUSION IN
B. Congenital rubella TOPNOTCH MD) THE SAMPLEX
C. Congenital syphilis
D. Cytomegalovirus infection
E. Toxoplasmosis
476 What type of rhabdomyosarcoma will present with LUISA BACK-UP
vaginal grape-like mass? SARANILLO, MD MIDTERM
A. Sarcoma botryoides (TOP 6 - FEB EXAM AUG
B. Alveolar 2014 MED 2014 - FOR
C. Embryonal BOARDS; INCLUSION IN
D. Pleomorphic TOPNOTCH MD) THE SAMPLEX
E. Undifferentiated

477 A 10 month old infant will have how many teeth? the formula is age in month-6, so 10-6=4. LUISA BACK-UP
A. 4 SARANILLO, MD MIDTERM
B. 2 (TOP 6 - FEB EXAM AUG
C. 1 2014 MED 2014 - FOR
D. 3 BOARDS; INCLUSION IN
E. 5 TOPNOTCH MD) THE SAMPLEX

TOPNOTCH MEDICAL BOARD PREP PEDIATRICS SUPEREXAM Page 63 of 97


For inquiries visit www.topnotchboardprep.com.ph or email us at topnotchmedicalboardprep@gmail.com
TOPNOTCH MEDICAL BOARD PREP PEDIATRICS SUPEREXAM
For inquiries visit www.topnotchboardprep.com.ph or email us at topnotchmedicalboardprep@gmail.com
Item QUESTION EXPLANATION AUTHOR TOPNOTCH
# EXAM
478 Periorbital violaceous or heliotropic rash followed LUISA BACK-UP
by proximal muscle weakness is seen in: SARANILLO, MD MIDTERM
A. Scleroderma (TOP 6 - FEB EXAM AUG
B. Systemic lupus erythematosus 2014 MED 2014 - FOR
C. Dermatomyositis BOARDS; INCLUSION IN
D. Erysipelas TOPNOTCH MD) THE SAMPLEX
E. Systemic sclerosis

479 A 2 year old child presents with non-tender In neuroblastoma, abdominal mass crosses the LUISA BACK-UP
abdominal mass crossing the midline. On midline; while in wilms tumor, it does not cross the SARANILLO, MD MIDTERM
abdominal CT scan there is calcification and midline. (TOP 6 - FEB EXAM AUG
hemorrhage seen. What is the diagnosis? 2014 MED 2014 - FOR
A. Wilms tumor BOARDS; INCLUSION IN
B. Neuroblastoma TOPNOTCH MD) THE SAMPLEX
C. Metastatic tumor
D. Intestinal obstruction
E. medulloblastoma
480 The following are the blood pictures of herediatry In hereditary spherocytosis, MCHC is increased LUISA BACK-UP
spherocytosis except: SARANILLO, MD MIDTERM
A. Loss of biconcave shape (TOP 6 - FEB EXAM AUG
B. Decreased MCHC 2014 MED 2014 - FOR
C. Normal MCV BOARDS; INCLUSION IN
D. High reticulocyte count TOPNOTCH MD) THE SAMPLEX
E. None of the choices

481 Honey, a 6-year old girl, presents with This is a case of atopic dermatitis which presents as an ANGELIS FINAL EXAM -
erythematous, weeping, papules and vesicles, some eczema in a population with a strong personal or ANDREA COCOS, AUG 2014
coalescing to form scaly plaques distributed over family history of atopy. The management is maximal MD (TOP 1 - FEB
the flexural areas. The mother said that the hydration with emollients, moisturizers and topical 2014 MED
symptoms began in infancy and that her 8-month- steroids. BOARDS;
old sibling, Dindin, also has similar symptoms. TOPNOTCH MD)
Which is the most appropriate treatment?
A. Permethrin
B. Antifungal cream
C. Topical antibiotics
D. Topical moisturizers and steroids
482 Nes is a mentally retarded 15-year-old boy who has The physical features of fragile X syndrome, long face, ANGELIS FINAL EXAM -
a long face, micropenis, large testes and large ears. large ears, prominent jaw, large testes, hypotonia, ANDREA COCOS, AUG 2014
Which of the following conditions does he most repetitive speech, hand flapping etcetera, becomes MD (TOP 1 - FEB
likely have? more prominent after puberty. 2014 MED
A. Fragile X syndrome BOARDS;
B. Trisomy 21 TOPNOTCH MD)
C. Trisomy 13
D. Trisomy 18
483 The mother of Madeen, a 6-day old infant, has SIMILAR TO PREVIOUS BOARD EXAM ANGELIS FINAL EXAM -
developed multiple vesicular lesions throughout CONCEPT/PRINCIPLE This is a case of varicella in a ANDREA COCOS, AUG 2014
her body. Which of the following is the correct new postpartum mother. VZIG is only administered to MD (TOP 1 - FEB
management? the infant immediately if the mother had the onset of 2014 MED
A. Isolate the infant from the mother. varicella within 5 days prior to delivery and 2 days BOARDS;
B. Administer acyclovir to the infant. after delivery. This is already a 6-day old infant so TOPNOTCH MD)
C. Administer varicella-zoster immunoglobulin to VZIG and isolation are not necessary. Acyclovir is
the infant. given only to treat infants with lesions.
D. Continue regular well-baby care for the infant.
484 Loi, a newborn, has microcephaly, intracranial SIMILAR TO PREVIOUS BOARD EXAM ANGELIS FINAL EXAM -
calcifications, hepatosplenomegaly, and marked CONCEPT/PRINCIPLE. Microcephaly is for CMV, while ANDREA COCOS, AUG 2014
hyperbilirubinemia and thrombocytopenia. This hydrocephalus is for toxoplasmosis. Both of them MD (TOP 1 - FEB
baby probably has: presents with chorioretinitis and intracranial 2014 MED
A. Congenital rubella calcifications (although some references would label BOARDS;
B. Congenital CMV the calcifications of CMV as periventricular). TOPNOTCH MD)
C. Congenital syphilis
D. Congenital toxoplasmosis
485 During a routine school physical examination, Jk, a MVP presents with an apical murmur noted late in ANGELIS FINAL EXAM -
17 year-old girl, was noted to have a late apical systole, which can be preceded by a click. It occurs ANDREA COCOS, AUG 2014
systolic murmur, which is preceded by a click. The with the billowing of the mitral valve leaflets into the MD (TOP 1 - FEB
rest of the cardiac examination is normal. What atria at the end of systole. ASD presents with fixed 2014 MED
kind of heart defect does she most likely have? split S2. SIMILAR TO PREVIOUS BOARD EXAM BOARDS;
A. None CONCEPT/PRINCIPLE TOPNOTCH MD)
B. ASD
C. VSD
D. MVP
486 Andy, a 3-day old infant was noted to have TGA is the most common cause of early cyanosis in the ANGELIS FINAL EXAM -
progressively deepening cyanosis since birth. newborns (unlike TOF which presents with cyanosis ANDREA COCOS, AUG 2014
Which of the following congenital cardiac after a fews days of life). MD (TOP 1 - FEB
malformations is most likely responsible for her 2014 MED
cyanosis? BOARDS;
A. Transposition of great arteries TOPNOTCH MD)
B. Tetralogy of Fallot
C. Tricuspid atresia
D. Total anomalous pulmonary venous return

TOPNOTCH MEDICAL BOARD PREP PEDIATRICS SUPEREXAM Page 64 of 97


For inquiries visit www.topnotchboardprep.com.ph or email us at topnotchmedicalboardprep@gmail.com
TOPNOTCH MEDICAL BOARD PREP PEDIATRICS SUPEREXAM
For inquiries visit www.topnotchboardprep.com.ph or email us at topnotchmedicalboardprep@gmail.com
Item QUESTION EXPLANATION AUTHOR TOPNOTCH
# EXAM
487 Sweet delivered to a term infant which was noted Congenital cystic adenomatoid malformation (CCAM) ANGELIS FINAL EXAM -
to be in respiratory distress shortly after birth. The arises from an embryonic disruption before 9 wks that ANDREA COCOS, AUG 2014
infant has diminshed breath sounds on the left. A causes improper development of bronchioles. Large MD (TOP 1 - FEB
CXR was requested revealing a multiloculated mass lesions may compress the affected lung and cause 2014 MED
in the left hemithorax. The tip of the NGT placed pulmonary hypoplasia. BOARDS;
reveals that the stomach is below the diaphragm. TOPNOTCH MD)
Which is the most likely diagnosis?
A. Congenital diaphragmatic hernia
B. Congenital lobar emphysema
C. Congenital cystic adenomatoid malformation
D. Congenital bronchogenic cysts
488 Joy, brought her 3-year old son in the middle of the Laryngotracheobronchitis or viral croup presents with ANGELIS FINAL EXAM -
night, who has developed noisy breathing on inspiratory stridor (extrathoracic symptom) as in this ANDREA COCOS, AUG 2014
inspiration, chest retractions, and a barking cough. case. Agents causing croup are usually parainfluenza MD (TOP 1 - FEB
He has a mild upper respiratory tract infection for 2 types 1 and 3. Racemic epinephrine can be used to 2014 MED
days. Which would be the most helpful alleviate respiratory symptoms. Epiglottitis presents BOARDS;
intervention? with a more toxic-looking child and more acute TOPNOTCH MD)
A. inhaled beta-agonist presentation. The causative agent is H. influenzae,
B. ceftriaxone managed usually with ceftriaxone.
C. racemic epinephrine
D. amoxicillin
489 One-month-old Argie is admitted to the hospital for SIMILAR TO PREVIOUS BOARD EXAM ANGELIS FINAL EXAM -
jaundice. Her total bilirubin is demonstrated to be CONCEPT/PRINCIPLE This is a case of obstructive ANDREA COCOS, AUG 2014
12 mg/dL with direct bilirubin of 4.5 mg/dl. Which jaundice (direct bilirubin >20% of total bilirubin). The MD (TOP 1 - FEB
among the following is a likely diagnosis? rest of the other causes of jaundice listed typically 2014 MED
A. Gilbert syndrome lead to indirect hyperbilirubinemia. BOARDS;
B. ABO incompatibility TOPNOTCH MD)
C. Rh incompatibility
D. Choledochal cyst
490 5-month-old Maricel presented at the emergency SIMILAR TO PREVIOUS BOARD EXAM ANGELIS FINAL EXAM -
room with irritability, repeated bouts of crying, and CONCEPT/PRINCIPLE. The classic description would ANDREA COCOS, AUG 2014
intermittent colicky abdominal pain. On physical usually include currant-jelly stool (not mentioned MD (TOP 1 - FEB
examination, there is a sausage-shaped mass in the during our exam) and a palpable sausage mass on the 2014 MED
right upper quadrant. There is empty rectal vault RUQ or RLQ. BOARDS;
on DRE, with blood per examining finger. What is TOPNOTCH MD)
the most likely diagnosis?
A. Volvulus
B. Intusussception
C. Imperforate anus
D. Malrotation
491 Young Brian had numerous infected wounds 4 Gross hematuria usually resolves within 3-6 months ANGELIS FINAL EXAM -
weeks ago. He presented at your clinic with gross but microscopic hematuria may persist as long as 12- ANDREA COCOS, AUG 2014
hematuria but he is generally asymptomatic. You 18 months. C3 normalizes two months after the onset MD (TOP 1 - FEB
also find him to have hypertension. If this is post- of illness. 2014 MED
streptococcal glomerulonephritis, when do you BOARDS;
expect the microscopic hematuria to resolve? TOPNOTCH MD)
A. after 1 week
B. after 6 months
C. after a year
D. never
492 Jarvie, a 6 year-old child, is noted to have 3+ Drug-induced nephrotic syndrome has been described ANGELIS FINAL EXAM -
protein on urinalysis. A 24-hr urine collection with the use of trimethadione, captopril, probenecid, ANDREA COCOS, AUG 2014
revealed a protein excretion of 3.7 g/24 hr. lithium, procainamide, phenytoin, penicillamine and MD (TOP 1 - FEB
Elevated levels of cholesterol and triglycerides some NSAIDs and heavy metals (e.g gold). 2014 MED
were also found. On your PE he has had unexpected BOARDS;
weight gain and has scrotal edema. Which of the TOPNOTCH MD)
following drugs might have caused such
presentation?
A. penicillamine
B. streptomycin
C. paracetamol
D. tetracycline
493 Examination of the CSF of Zyra, a 10-year-old child, There are only 3 stages of TB meningitis. Stage 1: non- ANGELIS FINAL EXAM -
shows a classic picture of TB meningitis. She came specific signs, conscious, no hydrocephalus. Stage 2: ANDREA COCOS, AUG 2014
in stuporous. What stage of TB meningitis is she in? confusion and or focal neurologic signs. Stage 3: MD (TOP 1 - FEB
A. Stage II stupor, delirium, hemiplegia. 2014 MED
B. Stage III BOARDS;
C. Stage IV TOPNOTCH MD)
D. Stage V

494 Rhea, a 2-year-old child, presents with infantile SIMILAR TO PREVIOUS BOARD EXAM ANGELIS FINAL EXAM -
spasms, hypopigmented lesions throughout her CONCEPT/PRINCIPLE. This case has the classic ANDREA COCOS, AUG 2014
back (ash-leaf spots) and a single raised lesion over findings for tuberous sclerosis. Sturge-Weber presents MD (TOP 1 - FEB
the sacrum (shagreen patch). Which is the most with a portwine unilateral facial lesion, and blindness. 2014 MED
likely diagnosis? NF presents classically with café au lait spots, Lisch BOARDS;
A. Sturge-Weber syndrome nodules, and neurofibromas. Mc-Cune Albright TOPNOTCH MD)
B. Neurofibromatosis presents with unilateral cafe au lait spots, fibrous
C. McCune Albright Syndrome dysplasia of bones and precocious puberty.
D. Tuberous sclerosis

TOPNOTCH MEDICAL BOARD PREP PEDIATRICS SUPEREXAM Page 65 of 97


For inquiries visit www.topnotchboardprep.com.ph or email us at topnotchmedicalboardprep@gmail.com
TOPNOTCH MEDICAL BOARD PREP PEDIATRICS SUPEREXAM
For inquiries visit www.topnotchboardprep.com.ph or email us at topnotchmedicalboardprep@gmail.com
Item QUESTION EXPLANATION AUTHOR TOPNOTCH
# EXAM
495 Ela presents with a 6-hour history of fever up to 39 Rocky mountain spotted fever is often a more indolent ANGELIS FINAL EXAM -
C. Two hours prior to arrival at the ER, mother infection with rash developing days after the onset of ANDREA COCOS, AUG 2014
states that she noted a few purple spots scattered fever. Henoch-schonlein purpura is actually a MD (TOP 1 - FEB
about the body on the patient especially on the vasculitis and its progression is not as rapid nor toxic 2014 MED
buttocks and legs. During your PE the purple areas as described in the case. Measles present with coryza, BOARDS;
has spread rapidly and coalesced, and the patient is conjunctivitis, and cough. Its rash is maculopapular. TOPNOTCH MD)
now obtunded. Which is the most likely diagnosis?
A. Measles
B. Rocky Mountain spotted fever
C. Henoch-Schonlein Purpura
D. Meningococcemia
496 Thirteen-year-old Mayda awakens with mild This is case is a classic description of Rubella. Rubeola ANGELIS FINAL EXAM -
sorethroat, low-grade fever and a diffuse is aka measles. Roseola is aka 6th disease or exanthem ANDREA COCOS, AUG 2014
maculopapular rash. She developed arthralgia and subitum. MD (TOP 1 - FEB
marked swelling of the posterior cervical 2014 MED
lymphnodes. Which is the most likely diagnosis? BOARDS;
A. Rubella TOPNOTCH MD)
B. Rubeola
C. Roseola
D. Erythema multiforme
497 Ivy, an 8-year-old child, was brought to your clinic "Growing pains" are commonly seen in the school-age ANGELIS FINAL EXAM -
for deep, aching pain over the lower extremities, child. They do not cause joint swelling, redness, heat, ANDREA COCOS, AUG 2014
most commonly noted in the evening and gone by or systemic signs or symptoms. SIMILAR TO MD (TOP 1 - FEB
the morning. The patient is otherwise PREVIOUS BOARD EXAM CONCEPT/PRINCIPLE 2014 MED
asymptomatic and your PE is essentially normal. BOARDS;
What is the best management? TOPNOTCH MD)
A. Reassure child and parents
B. Get an xray
C. Refer for bone marrow biopsy
D. Give amoxicillin
498 While bathing her 2 year old son, Marg feels a mass Usual presentation of Wilms (in decreasing ANGELIS FINAL EXAM -
in the abdomen. A thourough medical evaluation frequency): abdominal mass, abdominal pain, ANDREA COCOS, AUG 2014
reveals aniridia, hypospadias, horseshoe kidney hypertension, hematuria. The features of aniridia and MD (TOP 1 - FEB
and hemihypertrophy. Which is the most likely renal anomaly makes the rest of the choices less likely. 2014 MED
diagnosis? BOARDS;
A. neuroblastoma TOPNOTCH MD)
B. wilms tumor
C. hepatoblastoma
D. rhabdomyosarcoma
499 Jerry is a mentally retarded 4-year-old who was The lack of part of paternal chromosome 15 causes ANGELIS FINAL EXAM -
noted to be hypotonic at birth and is noted to have Prader-Willi syndrome (described in the case) and the ANDREA COCOS, AUG 2014
tremendous appetite, obesity, hypogonadism and lack of part of maternal chromosome 15 causes MD (TOP 1 - FEB
small hands and feet. The genetic mechanism Angelman syndrome. 2014 MED
responsible for this syndrome is due to: BOARDS;
A. mitochondrial inheritance TOPNOTCH MD)
B. genomic imprinting
C. mosaicism
D. nondisjunction
500 What is the most likely serum concentration of Na+ In 21-hydroxylase deficiency, the synthesis of both ANGELIS FINAL EXAM -
and K+ in a 4-month-old boy with 21-hydroxylase mineralocorticoids and cortisol is impaired. ANDREA COCOS, AUG 2014
deficiency? Aldosterone deficiency results to hyponatremia and MD (TOP 1 - FEB
A. Na+ 115, K+ 7.5 hyperkalemia. Choice B is a finding in central and 2014 MED
B. Na+ 155, K+ 5.5 nephrogenic diabetes insipidus. Choice C is for BOARDS;
C. Na+ 144, K+ 3.0 patients with hyperaldosteronism. TOPNOTCH MD)
D. Na+ 110, K+ 3.0

501 A newborn infant is noted to have pale blue Appearance=pale, blue extremities=1; Pulse of 150=2; JAN CHARMAINE BACK-UP
extremities, HR of 150, grimaces to stimulation and Grimace=1, Active=2; Respiration-good cry=2 PALOMAR, MD MIDTERM
is active with good cry. What is her APGAR score? (TOP 9 - FEB EXAM AUG
A. 4 2014 MED 2014
B. 6 BOARDS;
C. 7 TOPNOTCH MD)
D. 8
E. 5

502 APGAR score are routinely assessed at 1 and 5 1-minute score gives an idea of what was going on JAN CHARMAINE BACK-UP
minutes and every 5 minutes thereafter as long as during labor and delivery PALOMAR, MD MIDTERM
resuscitation is continuing. The 5-minute APGAR (TOP 9 - FEB EXAM AUG
score gives an idea of? 2014 MED 2014
A. what was going on during labor and delivery BOARDS;
B. response to therapy and resuscitation TOPNOTCH MD)
C. what was going on during the last 9 months of
pregnancy
D. All of the above
E. None of the above
503 A newborn infant shows an extraordinary division Nelson 19th ed., 533 JAN CHARMAINE BACK-UP
of the body from the forehead to the pubis into red PALOMAR, MD MIDTERM
and pale halves which is transient and harmless. (TOP 9 - FEB EXAM AUG
What is the most likely diagnosis? 2014 MED 2014
A. Mongolian spots BOARDS;
B. Acrocyanosis TOPNOTCH MD)
C. Ichthyosis congenita
D. Mottling
E. Harlequin color change

TOPNOTCH MEDICAL BOARD PREP PEDIATRICS SUPEREXAM Page 66 of 97


For inquiries visit www.topnotchboardprep.com.ph or email us at topnotchmedicalboardprep@gmail.com
TOPNOTCH MEDICAL BOARD PREP PEDIATRICS SUPEREXAM
For inquiries visit www.topnotchboardprep.com.ph or email us at topnotchmedicalboardprep@gmail.com
Item QUESTION EXPLANATION AUTHOR TOPNOTCH
# EXAM
504 What is the pathognomonic sign on plain pneumatosis intestinalis. Nelson 19th ed., 602 JAN CHARMAINE BACK-UP
abdominal radiograph of Neonatal Necrotizing PALOMAR, MD MIDTERM
Enterocolitis? (TOP 9 - FEB EXAM AUG
A. Gas accummulation in the submucosa of the 2014 MED 2014
bowel wall BOARDS;
B. Gas accumulation in the mucosa of the bowel TOPNOTCH MD)
wall
C. Coiled spring sign
D. Omega and Bird's beak sign
E. Gas accumulation in the mucosa of the small
intestines
505 Which of the following is true of breast-milk Significant elevation in unconjugated bilirubin (breast JAN CHARMAINE BACK-UP
jaundice? milk jaundice) develops after the 7th day of life; due to PALOMAR, MD MIDTERM
A. It means that the baby is not nursing well. glucoronidase in milk; lasts 3w-3months, (TOP 9 - FEB EXAM AUG
B. Manangement includes continuation of the management: stop breastfeeding for 2 days.p607 2014 MED 2014
breastfeeding BOARDS;
C. Occurs due to glucoronidase present in some TOPNOTCH MD)
breastmilk
D. May be due to decreased milk intake
E. All of the above
506 What is the most common cardiac anomaly PDA-78%, Right pulmonary artery stenosis-70%, L JAN CHARMAINE BACK-UP
associated with congenital rubella syndrome? pulmonary artery stenosis-56%, Valvular pulmonic PALOMAR, MD MIDTERM
A. VSD stenosis-40% p.1077 (TOP 9 - FEB EXAM AUG
B. ASD 2014 MED 2014
C. Coarctation of the aorta BOARDS;
D. PDA TOPNOTCH MD)
E. TOF

507 Which of the following late manifestations describe B. Anterior bowing of the midportion of the tibia- JAN CHARMAINE BACK-UP
Higouménaki sign of congenital syphillis? saber shins C. Peg-shaped upper central incisors PALOMAR, MD MIDTERM
A. Unilateral or bilateral thickening of the with notch along the biting surface - Hutchinson's (TOP 9 - FEB EXAM AUG
sternoclavicular third of the clavicle teeth D. Linear scars 2014 MED 2014
B. Anterior bowing of the midportion of the tibia that extend in a spoke-like pattern from previous BOARDS;
C. Peg-shaped upper central incisors with notch mucocutaneous fissures of the mouth, anus, and TOPNOTCH MD)
along the biting surface genitalia - Rhagades p. 1017 nelson's pediatric 19th ed
D. Linear scars that extend in a spoke-like pattern
from previous mucocutaneous fissures of the
mouth, anus, and genitalia
E. none of the above
508 A premature newborn presented with intrauterine Nelson 17th ed., 1116 Toxoplasmosis triad- JAN CHARMAINE BACK-UP
growth restriction, hepatosplenomegaly, jaundice, chorioretinitis, hydrocephalus & cerebral PALOMAR, MD MIDTERM
blueberry muffin-like rash, thrombocytopenia and calcifications (TOP 9 - FEB EXAM AUG
purpura,and microcephaly and intracranial 2014 MED 2014
calcifications. What is the condition of the patient? BOARDS;
A. Disseminated neuroblastoma TOPNOTCH MD)
B. Toxoplasmosis
C. Congenital CMV infection
D. Congenital rubella syndrome
E. None of the above
509 Deficiency of what vitamin leads to dermatitis this is a case of pellagra (3 D's= Dermatitis, diarrhea JAN CHARMAINE BACK-UP
presenting initially as erythema which then and delirium) caused by Vitamin b3 or niacin PALOMAR, MD MIDTERM
becomes dry, rough, thickened, cracked, and deficiency. (TOP 9 - FEB EXAM AUG
hyperpigmented. There is also diarrhea, 2014 MED 2014
disorientation and delirium. BOARDS;
A. Riboflavin TOPNOTCH MD)
B. B1
C. Niacin
D. B9
E. Pantothenic acid
510 Nocturnal enuresis and encopresis refers to the Nocturnal enuresis- refers to the occurrence of JAN CHARMAINE BACK-UP
occurrence of involuntary voiding at night and involuntary voiding at night after age 5 yr, the age PALOMAR, MD MIDTERM
passage of feces into inappropriate places when volitional control of micturition is expected. p. (TOP 9 - FEB EXAM AUG
respectively, after what ages? 1851 Encopresis- 2014 MED 2014
A. 4 years and 5 years voluntary and involuntary passage of feces into BOARDS;
B. 5 years and 4 years inappropriate places at least once a month for 3 TOPNOTCH MD)
C. 5 years and 6 years consecutive months once a chronologic or
D. 6 years and 5 years developmental age of 4 yr has been reached.p73
E. None of the above
511 The major features of atopic dermatitis include the also: Chronic or relapsing dermatitis
JAN CHARMAINE BACK-UP
following, except: Associated Features: Xerosis, cutaneous infections, PALOMAR, MD MIDTERM
A. Pruritus nonspecific dermatitis of the hands and feet, (TOP 9 - FEB EXAM AUG
B. Facial and extensor eczema in infants and Ichthyosis, palmar hyperlinearity, keratosis pilaris, 2014 MED 2014
children nipple eczema, white dermatographism and delayed BOARDS;
C. Flexural eczema in adolescents blanch response, anterior subscapular catarcts, TOPNOTCH MD)
D. Personal or family history of atopic disease keratoconus, elevated IgE, positive results of
E. All of the above immediate-type allergy skin tests, early age at onset,
Dennie lines, facial erythema or pallor, course
influenced by environmental and/or emotional factors
Nelson's 19th ed, p803

TOPNOTCH MEDICAL BOARD PREP PEDIATRICS SUPEREXAM Page 67 of 97


For inquiries visit www.topnotchboardprep.com.ph or email us at topnotchmedicalboardprep@gmail.com
TOPNOTCH MEDICAL BOARD PREP PEDIATRICS SUPEREXAM
For inquiries visit www.topnotchboardprep.com.ph or email us at topnotchmedicalboardprep@gmail.com
Item QUESTION EXPLANATION AUTHOR TOPNOTCH
# EXAM
512 The classic triad of infectious mononucleosis Nelson's 19th ed, p1111 JAN CHARMAINE BACK-UP
include the following, except: PALOMAR, MD MIDTERM
A. fever (TOP 9 - FEB EXAM AUG
B. fatigue 2014 MED 2014
C. pharyngitis BOARDS;
D. generalized lymphadenopathy TOPNOTCH MD)
E. None of the above

513 The following is true of Alport syndrome, except: mutation in type IV collagen -Nelson's 19th ed., 1782 JAN CHARMAINE BACK-UP
A. It is a genetically heterogenous disease caused PALOMAR, MD MIDTERM
by mutations in the genes coding for type III (TOP 9 - FEB EXAM AUG
collagen, a major component of the basement 2014 MED 2014
membranes. BOARDS;
B. All patients with Alport syndrome have TOPNOTCH MD)
asymptomatic microscopic hematuria, which may
be intermittent in girls and younger boys.
C. The bilateral sensorineural hearing loss that
occurs in most of the patient with the disease is
never congenital.
D. The presence of anterior lenticonus or the
extrusion of the central portion of the lens into the
anterior chamber is pathognomonic.
E. None of the above
514 A 1 year old baby girl was rushed to the ER because Nelson's 19th ed, p1090 JAN CHARMAINE BACK-UP
of poor feeding for 3 days now. Condition started 3 PALOMAR, MD MIDTERM
days ago when patient developed fever, sore throat (TOP 9 - FEB EXAM AUG
and lesions in the posterior pharynx. Physical exam 2014 MED 2014
revealed discrete 1-2mm vesicles and ulcers that BOARDS;
enlarge and are surrounded by erythematous rings TOPNOTCH MD)
on the anterior tonsillar pillars, soft palate, uvula,
tonsils and posterior pharyngeal wall. What is your
primary consideration?
A. German measles
B. Measles
C. Herpangina
D. Chickenpox
E. Streptococcal pharyngitis
515 A 4 year old boy is seen with pink, maculopapular IgA nephropathy is the most common chronic JAN CHARMAINE BACK-UP
rash on the buttocks and legs with vague, glomerular disease worldwide. It most commonly PALOMAR, MD MIDTERM
intermittent abdominal pain. His mother reported presents with gross hematuria within 1-2 days of (TOP 9 - FEB EXAM AUG
that patient has recently recovered from cough and onset of an upper respiratory or gastrointestinal 2014 MED 2014
colds a week ago. Urinalysis revealed microscopic infection. Nelson's 19th ed., 1781-82 BOARDS;
hematuria. the rest of the labs were unremarkable. TOPNOTCH MD)
What is your primary consideration?
A. Henoch-Schonlein Purpura
B. Kawasaki Disease
C. IgA Nephropathy
D. Post-Streptococcus Glomerulonephritis
E. None of the above
516 The classical clinical criteria of Kawasaki Disease Fever persisting at least 5 days, subacute changes in JAN CHARMAINE BACK-UP
include the following except: the extremities manifesting asperiungal peeling of PALOMAR, MD MIDTERM
A. Fever persisting at least 7 days fingers and toes, polymorphous exanthem, cervical (TOP 9 - FEB EXAM AUG
B. Acute changes in extremities manifesting as lymphadenopathy (>1.5 cm diameter) Nelson's 19th 2014 MED 2014
erythema of palms, soles; edema of hands and feet ed., 863 BOARDS;
C. Bilateral bulbar conjunctival injection without TOPNOTCH MD)
exudate
E. lip cracking, strawberry tongue
517 What is the pathognomonic microscopic finding in Forchheimer spots- petechial hemorrhages in the soft JAN CHARMAINE BACK-UP
the rash and exanthem of measles? palate in patients with German mealsesNelson-1076; PALOMAR, MD MIDTERM
A. Koplik spots Koplik spots represent the enanthem in measles and is (TOP 9 - FEB EXAM AUG
B. Warthin-Finkeldey giant cells the pathognomonic sign of measles. Nelson-1070 2014 MED 2014
C. Forchheimer spots BOARDS;
D. All of the above TOPNOTCH MD)
E. None of the above

518 A 6 year old girl was brought to the ER because of patient has >2 signs and symptoms belonging to the JAN CHARMAINE BACK-UP
episodes of loose bowel of 4 episodes per day severe dehydration category PALOMAR, MD MIDTERM
amounting to 2 cups per bout which started 2 days (TOP 9 - FEB EXAM AUG
ago. Physical exam reveals child to be irritable, 2014 MED 2014
eager to drink, tachycardic, weak pulse, with BOARDS;
parched lips and decreased urine output. What is TOPNOTCH MD)
your admitting impression?
A. Shigellosis
B. Acute Gastroenteritis with moderate
dehydration
C. Acute Gastroenteritis with severe dehydration
D. Ascariasis
E. None of the above
519 Hematologic work up of a 15 year old female Acute bleeding-Normocytic normochromic RBCs with JAN CHARMAINE BACK-UP
revealed microcytic hypochromic red blood cells low retic count PALOMAR, MD MIDTERM
with low reticulocyte count. Which of the following (TOP 9 - FEB EXAM AUG
is excluded form the differential diagnosis? 2014 MED 2014
A. Iron deficiency anemia BOARDS;
B. Thalassemia trait TOPNOTCH MD)
C. Lead poisoning
D. Acute bleeding
E. Sideroblastic anemia
TOPNOTCH MEDICAL BOARD PREP PEDIATRICS SUPEREXAM Page 68 of 97
For inquiries visit www.topnotchboardprep.com.ph or email us at topnotchmedicalboardprep@gmail.com
TOPNOTCH MEDICAL BOARD PREP PEDIATRICS SUPEREXAM
For inquiries visit www.topnotchboardprep.com.ph or email us at topnotchmedicalboardprep@gmail.com
Item QUESTION EXPLANATION AUTHOR TOPNOTCH
# EXAM
520 What is the most consistent clinical manifestation Tachypnea is the most consistent clinical JAN CHARMAINE BACK-UP
of pneumonia? manifestation of pneumonia. Nelson's 19th ed., 1476 PALOMAR, MD MIDTERM
A. Fever (TOP 9 - FEB EXAM AUG
B. Cough 2014 MED 2014
C. Crackles BOARDS;
D. Increased respiratory rate TOPNOTCH MD)
E. Tachycardia

521 A previously healthy 10 year old boy is brought to MIGUEL RAFAEL MIDTERM 1
the physician because of a rash over his arms and RAMOS, MD (TOP EXAM - FEB
legs for 7 days. His blood pressure is elevated. On 3 - FEB 2012 MED 2013
physical examination, a yellow, crusted, excoriated BOARDS;
rash over the upper and lower extremities are seen. TOPNOTCH MD)
Urinalysis reveals 100 RBC/hpf and 5-10 WBC/hpf.
Which of the following is the most likely diagnosis?
A) Acute glomerulonephritis
B) Henoch-Schonlein purpura
C) Lupus nephritis
D) Nephrolithiasis
522 A 13 year old boy is brought to the physician MIGUEL RAFAEL MIDTERM 1
because of a 2 week history of left hip pain with an RAMOS, MD (TOP EXAM - FEB
associated limp. He has had no redness or swelling 3 - FEB 2012 MED 2013
of the left leg or hip. There is no history of trauma BOARDS;
or musculoskeletal problems. Vital signs are stable. TOPNOTCH MD)
He holds his left lower extremity in slight external
rotation and hip flexion at rest. Internal rotation
and abduction of the left hip are decreased. He
walks with a limp and is unable to bear his full
weight on the left. Which of the following is the
most likely underlying cause of this patient’s
condition?
A) Acute inflammatory reaction of the synovial
lining of the hip joint
B) Avascular necrosis of the proximal femur
C) Bacterial infection of the hip joint
D) Disruption of the femoral head epiphyseal plate
523 A 10 month old infant is brought to the emergency MIGUEL RAFAEL MIDTERM 1
department because of labored breathing for 1 RAMOS, MD (TOP EXAM - FEB
hour. She has had cough, coryza, and fever for 18 3 - FEB 2012 MED 2013
hours. Her temperature is 39 C, pulse is 120 bpm, BOARDS;
respirations are 54 cpm. Pulse oximetry shows an TOPNOTCH MD)
oxygen saturation of 92%. Bilateral wheezes and
basilar crackles are heard. What is the most likely
diagnosis?
A) Community acquired viral disease
B) Contiguously spread bacterial infection
C) Toxin mediated capillary leak
D) Osmotically generated fluid shift
524 A 15 year old boy is brought to the physician MIGUEL RAFAEL MIDTERM 1
because of headaches for 3 months. The headaches RAMOS, MD (TOP EXAM - FEB
occur most often in the morning and are associated 3 - FEB 2012 MED 2013
with vomiting. Over the past month, he also has had BOARDS;
increasing visual difficulty. During this period, he TOPNOTCH MD)
has had a 4.5 kg weight gain, and his school
performance has declined. He is also concerned
that his pubertal development has been slower that
his friends. Fundoscopic examination shows mild
papilledema. Which of the following is the most
likely diagnosis?
A) Adrenal insufficiency
B) Craniopharyngioma
C) Medulloblastoma
D) Migraines
525 A 13 year old boy is brought to the physician MIGUEL RAFAEL MIDTERM 1
because of a 3 month history of left knee pain that RAMOS, MD (TOP EXAM - FEB
is exacerbated by vigorous exercise. He also has 3 - FEB 2012 MED 2013
had occasional pain in his right knee. There is no BOARDS;
history of trauma. He is at the 50th percentile for TOPNOTCH MD)
height and weight. His temperature is 37 C.
Examination shows mild swelling and tenderness of
the left tibial tubercle; range of motion of the knees
is full. Which of the following is the most
appropriate next step in management?
A) MRI of the affected knee
B) Antibiotic therapy
C) NSAIDs
D) Knee immobilization

TOPNOTCH MEDICAL BOARD PREP PEDIATRICS SUPEREXAM Page 69 of 97


For inquiries visit www.topnotchboardprep.com.ph or email us at topnotchmedicalboardprep@gmail.com
TOPNOTCH MEDICAL BOARD PREP PEDIATRICS SUPEREXAM
For inquiries visit www.topnotchboardprep.com.ph or email us at topnotchmedicalboardprep@gmail.com
Item QUESTION EXPLANATION AUTHOR TOPNOTCH
# EXAM
526 A previously healthy 10 month old boy is brought MIGUEL RAFAEL MIDTERM 1
to the ER by his parents because he has vomited RAMOS, MD (TOP EXAM - FEB
twice during the past 2 hours. During this period, 3 - FEB 2012 MED 2013
he has had episodes of inconsolable crying and has BOARDS;
passed one bloody stool. On arrival, he appears TOPNOTCH MD)
lethargic and is difficult to arouse. His temperature
is 38C, pulse is 110 bpm, respirations are 24 cpm,
and blood pressure is 90/65 mmHg. The abdomen
is soft with no masses or organomegaly. Rectal
examination shows bloody mucus in the rectal
vault. Which o the following is the most appropriate
next step in diagnosis?
A) Contrast enema
B) Upper GI series with small bowel follow-through
C) CT scan of the head
D) Colonoscopy
527 A 17 year old girl comes to the physician because of MIGUEL RAFAEL MIDTERM 1
fatigue, increased thirst, and increased urination RAMOS, MD (TOP EXAM - FEB
over the past 2 weeks; she has had a 4.5 kg weight 3 - FEB 2012 MED 2013
loss during this period despite an increased BOARDS;
appetite. At the onset of her symptoms, she had a TOPNOTCH MD)
mild upper respiratory tract infection that resolved
without treatment. Vital signs are within normal.
Cranial nerves are intact. Muscle strength is 5/5, ad
deep tendon reflexes are symmetric. Serum
chemistries reveal Na 132 Cl 96 K 3.7 HCO3 26 and
Crea 1.2. Which of the following is the most likely
location of the primary disease process?
A) Adrenal gland
B) Central nervous system
C) Pancreas
D) Kidney
528 A 3 month old infant has had tachypnea and MIGUEL RAFAEL MIDTERM 1
tachycardia for 10 days; during this period he has RAMOS, MD (TOP EXAM - FEB
been feeding poorly. A grade 3/6 holosystolic 3 - FEB 2012 MED 2013
murmur and a grade 2/6, apical mid-diastolic BOARDS;
murmur are heard. An x-ray of the chest shows TOPNOTCH MD)
cardiomegaly with increased pulmonary vascular
markings. Echocardiography shows a large
ventricular septal defect. Which of the following is
the most likely cause of these symptoms?
A) Excessive pulmonary blood flow
B) Mitral valve obstruction
C) Reduced left ventricular contractility
D) Reduced right ventricular preload
529 A 2 month old boy is brought to the physician for a MIGUEL RAFAEL MIDTERM 1
well-child examination. He was born at term RAMOS, MD (TOP EXAM - FEB
following an uncomplicated pregnancy, labor, and 3 - FEB 2012 MED 2013
delivery. He has been breast-feeding six to eight BOARDS;
times daily since birth. He smile and lifts his head. TOPNOTCH MD)
He is at the 50th percentile for length, 75th
percentile for weight, and 25th percentile for head
circumference. Examination shows no
abnormalities. His mother asks for nutritional
recommendations. Which of the following is the
most appropriate recommendation for the infant?
A) Begin vitamin K supplementation
B) Add rice cereal
C) Add cow milk-based formula
D) Begin vitamin D supplementation
530 A 7 year old boy is brought to the emergency MIGUEL RAFAEL MIDTERM 1
department because of facial edema and RAMOS, MD (TOP EXAM - FEB
respiratory distress since eating dinner 2 hours 3 - FEB 2012 MED 2013
ago. He has had coryza and cough during the past 2 BOARDS;
days. His temperature is 37.5 C, pulse is 100 bpm, TOPNOTCH MD)
respirations are 40 cpm, and blood pressure is
100/70 mmHg. Breath sounds are unequal with
decreased aeration and a prolonged expiratory
phase. Which of the following is the most likely
diagnosis?
A) Allergen induced bronchospasm
B) Community acquired viral disease
C) Toxin-mediated capillary leak
D) Chemical irritant pneumonitis

TOPNOTCH MEDICAL BOARD PREP PEDIATRICS SUPEREXAM Page 70 of 97


For inquiries visit www.topnotchboardprep.com.ph or email us at topnotchmedicalboardprep@gmail.com
TOPNOTCH MEDICAL BOARD PREP PEDIATRICS SUPEREXAM
For inquiries visit www.topnotchboardprep.com.ph or email us at topnotchmedicalboardprep@gmail.com
Item QUESTION EXPLANATION AUTHOR TOPNOTCH
# EXAM
531 A 5-year-old girl is brought to the physician MIGUEL RAFAEL MIDTERM 1
because of temperatures to 40 C (104 F), RAMOS, MD (TOP EXAM - FEB
tachypnea, and a nonproductive cough for 12 3 - FEB 2012 MED 2013
hours. Four days ago she was treated with an oral BOARDS;
antibiotic for suspected pneumococcal pneumonia. TOPNOTCH MD)
Examination shows diminished breath sounds over
the lower right lung fields and dullness to
percussion at the right costophrenic angle. Which of
the following is the most likely diagnosis?
A) Bronchopleural fistula
B) Empyema
C) Lung abscess
D) Pleurodynia
532 A 5-month-old boy is brought for a follow-up MIGUEL RAFAEL MIDTERM 1
examination. He was born at 37 weeks' gestation RAMOS, MD (TOP EXAM - FEB
and has had persistent wheezing since shortly after 3 - FEB 2012 MED 2013
birth despite treatment with nebulized and oral BOARDS;
bronchodilators and oral corticosteroids. His diet TOPNOTCH MD)
consists of 32 ounces of iron-fortified cow's milk-
based formula daily. He appears well nourished and
happy. On examination, there is moderate relief of
wheezing with extension of the neck. Which of the
following is the most likely mechanism of this
infant's wheezing?
A ) Allergic reaction to cow's milk
B ) Aspiration of a foreign body
C ) Compression of the airway by a vascular ring
D ) Concurrent upper respiratory tract infection
533 A 16-year-old girl is brought to the physician MIGUEL RAFAEL MIDTERM 1
because of episodes of palpitations over the past 6 RAMOS, MD (TOP EXAM - FEB
months. The episodes occur when she runs or plays 3 - FEB 2012 MED 2013
basketball. She is otherwise asymptomatic. Her BOARDS;
blood pressure is 124/46 mm Hg, pulse is 78/min, TOPNOTCH MD)
and respirations are 18/min. She weighs 55 kg
(121 lb) and is 180 cm (71 in) tall. Her arm span is
188 cm (74 in), and the upper segment to lower
segment ratio is 0.85. Her fingers appear long and
are hyperextensible. A grade 4/6, early diastolic
murmur is heard along the upper and middle left
sternal border with radiation to the apex.
Peripheral pulses are bounding. Which of the
following is the most likely cause of these findings?
A) Aortic incompetence
B) Aortic stenosis
C) Mitral incompetence
D) Mitral stenosis
534 A 5-week-old boy is brought to the physician MIGUEL RAFAEL MIDTERM 1
because of vomiting for 3 days. Switching from a RAMOS, MD (TOP EXAM - FEB
cow's milk-based formula to a soy-based formula 3 - FEB 2012 MED 2013
and one bottle of an electrolyte solution has not BOARDS;
decreased his vomiting. His mother says that there TOPNOTCH MD)
is no yellow color to the vomitus, but it is forceful
and occurs immediately after he has had 1 to 2
ounces of liquid. He appears to vomit more liquid
than he drank. He has one mustard-colored seedy
stool daily. Examination shows no abnormalities.
Which of the following is the most likely
explanation for his vomiting?
A) Duodenal atresia
B) Gastroesophageal reflux
C) Hypertrophic pyloric stenosis
D) Lactose intolerance

TOPNOTCH MEDICAL BOARD PREP PEDIATRICS SUPEREXAM Page 71 of 97


For inquiries visit www.topnotchboardprep.com.ph or email us at topnotchmedicalboardprep@gmail.com
TOPNOTCH MEDICAL BOARD PREP PEDIATRICS SUPEREXAM
For inquiries visit www.topnotchboardprep.com.ph or email us at topnotchmedicalboardprep@gmail.com
Item QUESTION EXPLANATION AUTHOR TOPNOTCH
# EXAM
535 A 3-year-old boy who is HIV positive is brought for MIGUEL RAFAEL MIDTERM 1
a routine examination. His diet is appropriate for RAMOS, MD (TOP EXAM - FEB
age. His medications include three antiretroviral 3 - FEB 2012 MED 2013
drugs and trimethoprim-sulfamethoxazole for BOARDS;
Pneumocystis carinii prophylaxis. Laboratory TOPNOTCH MD)
studies show:

Hemoglobin 8.6 g/dL
Mean corpuscular hemoglobin 38 pg/cell
Mean corpuscular hemoglobin concentration 30%
Hb/cell
Mean corpuscular volume 101 μm3
Leukocyte count 5600/mm3
Segmented neutrophils 60% (many
hypersegmented)
Bands 3%
Lymphocytes 37%
Red cell distribution width 21% (N=10–16)

Which of the following is most likely to have
prevented this patient's anemia?
A) Folic acid supplementation
B) Iron supplementation
C) Vitamin B12 (cyanocobalamin) supplementation
D) Monthly intravenous immune globulin therapy
536 A 10-year-old girl is brought to the emergency MIGUEL RAFAEL MIDTERM 1
department because of diffuse, aching abdominal RAMOS, MD (TOP EXAM - FEB
pain, nausea, and recurrent vomiting over the past 3 - FEB 2012 MED 2013
5 hours. She has an 8-year history of type 1 BOARDS;
diabetes mellitus treated with 20 U of NPH and 6 U TOPNOTCH MD)
of regular insulin in the morning and 14 U of NPH
and 5 U of regular insulin in the evening. She
appears lethargic but is easily arousable. There is
an obvious odor of ketones on her breath. Her
blood pressure is 100/70 mm Hg, pulse is 95/min,
and respirations are 20/min and deep. Serum
studies show:

Na+ 142 mEq/L
K+ 5.3 mEq/L
HCO3– 6 mEq/L
Glucose 710 mg/dL

Which of the following laboratory findings is most
likely to be increased?
A ) Arterial pH
B ) Serum C-peptide level
C ) Serum magnesium level
D ) Serum osmolality
537 A 5-year-old girl with ventricular septal defect is MIGUEL RAFAEL MIDTERM 1
scheduled for tonsillectomy in 2 weeks. She has had RAMOS, MD (TOP EXAM - FEB
an allergic reaction to penicillin in the past. Her 3 - FEB 2012 MED 2013
temperature is 37 C (98.6 F). Examination shows no BOARDS;
abnormalities. Which of the following is the most TOPNOTCH MD)
appropriate prophylaxis prior to tonsillectomy?
A) Amoxicillin
B) Ciprofloxacin
C) Erythromycin
D) No prophylaxis indicated
538 A previously healthy 16-year-old boy is brought to MIGUEL RAFAEL MIDTERM 1
the emergency department 20 minutes after an RAMOS, MD (TOP EXAM - FEB
episode of left arm shaking that lasted 3 - FEB 2012 MED 2013
approximately 3 minutes. Over the past 2 days, he BOARDS;
has had fever and emotional lability. On arrival, his TOPNOTCH MD)
temperature is 38.9 C (102 F). He is somnolent and
disoriented to person, place, and time. He responds
poorly to pain. Neurologic examination shows no
other abnormalities.Analysis of cerebrospinal fluid
shows:

Leukocyte count 120/mm3
Segmented neutrophils 20%
Lymphocytes 80%
Erythrocyte count 300/mm3
Glucose 60 mg/dL
Protein 400 mg/dL

Which of the following is the most likely cause of
this patient's neurologic findings?
A) Bacterial infection
B) Viral infection
C) Fungal infection
D) Parasitic infection

TOPNOTCH MEDICAL BOARD PREP PEDIATRICS SUPEREXAM Page 72 of 97


For inquiries visit www.topnotchboardprep.com.ph or email us at topnotchmedicalboardprep@gmail.com
TOPNOTCH MEDICAL BOARD PREP PEDIATRICS SUPEREXAM
For inquiries visit www.topnotchboardprep.com.ph or email us at topnotchmedicalboardprep@gmail.com
Item QUESTION EXPLANATION AUTHOR TOPNOTCH
# EXAM
539 A newborn is in severe respiratory distress MIGUEL RAFAEL MIDTERM 1
immediately following delivery. She was born at 35 RAMOS, MD (TOP EXAM - FEB
weeks' gestation to a 35-year-old woman, gravida 3 - FEB 2012 MED 2013
2, para 1, aborta 1, who did not receive prenatal BOARDS;
care. The newborn's pulse is 60/min, and TOPNOTCH MD)
respirations are irregular and labored. Examination
shows pallor with perioral cyanosis, anasarca,
hepatosplenomegaly, and scattered petechiae. Cord
blood hemoglobin is 4 g/dL, and reticulocyte count
is 18%. A direct antiglobulin (Coombs') test is
positive. Which of the following sets of blood
groups is most likely in the mother and her
newborn? (Mother, Newborn)
A) A, Rh-positive O, Rh-positive
B) A, Rh-positive O, Rh-negative
C) O, Rh-negative O, Rh-positive
D) O, Rh-positive O, Rh-negative
540 A 16-year-old boy with neurofibromatosis is MIGUEL RAFAEL MIDTERM 1
brought for a follow-up examination. His uncle also RAMOS, MD (TOP EXAM - FEB
has neurofibromatosis. He has a 1-year history of 3 - FEB 2012 MED 2013
headaches during which his parents say he appears BOARDS;
pale. Six months ago, he underwent operative TOPNOTCH MD)
treatment for an optic nerve glioma. His blood
pressure is 164/105 mm Hg, pulse is 102/min, and
respirations are 14/min. The thyroid glands are not
enlarged. No murmurs are heard, and radial pulses
are equal. Abdominal examination shows no
abnormalities. Which of the following is the most
likely cause of this patient's high blood pressure?
A) Catecholamine-producing tumor
B) Carcinoma of the thyroid gland
C) Essential hypertension
D) Overproduction of aldosterone from an adrenal
adenoma
541 A 12/F is brought to the clinic due to fever and SIMILAR TO PREVIOUS BOARD EXAM ABDELSIMAR FINAL EXAM -
painful joints. Five days PTC, the parents noted that CONCEPT/PRINCIPLE. Note that the patient already OMAR II, MD FEB 2014
patient was febrile and has a sore and swollen left satisfies 2 major criteria: fever + migratory (TOP 2 - AUG
knee. A few days later, her left knee improved; but polyarthritis. This plus a history of sore throat (likely 2013 MED
her left elbow started to become similarly sore and streptococcal) raises suspicion for rheumatic fever. BOARDS;
swollen. You note that the patient had a sore throat TOPNOTCH MD -
four weeks prior. What is the MOST likely 200 QUESTIONS)
diagnosis? AND MARC
A. Juvenile rheumatoid arthritis DENVER
B. Septic arthritis TIONGSON, MD
C. Reactive arthropathy (40 QUESTIONS)
D. Viral arthropathy
E. Rheumatic fever
542 A child is already able to run well and walk up and ABDELSIMAR FINAL EXAM -
down stairs, one step at a time. He is also able to OMAR II, MD FEB 2014
put three words together in sentences. These (TOP 2 - AUG
milestones are expected for a child aged ___: 2013 MED
A. 12 months BOARDS;
B. 18 months TOPNOTCH MD -
C. 24 months 200 QUESTIONS)
D. 36 months AND MARC
E. 48 months DENVER
TIONGSON, MD
(40 QUESTIONS)
543 At birth, an infant is noted to have a cardiac VSDs are the MOST COMMON of the CHDs. Small VSDs ABDELSIMAR FINAL EXAM -
murmur. A loud, harsh blowing holosystolic with trivial L->R shunts would present with a loud, OMAR II, MD FEB 2014
murmur is heard best over the lower left sternal harsh, blowing holosystolic murmur heard best over (TOP 2 - AUG
border. There is no clinical evidence of congestive the L sternal border. ASD would present with a 2013 MED
heart failure. What is the most likely diagnosis? systolic ejection murmur produced by increased flow BOARDS;
A. Atrial septal defect across the right ventricular outflow tract into the TOPNOTCH MD -
B. Ventricular septal defect pulmonary artery. PDA presents with a continuous 200 QUESTIONS)
C. Patent ductus arteriosus machinery like murmur. AND MARC
D. Coarctation of the aorta DENVER
E. Tetralogy of Fallot TIONGSON, MD
(40 QUESTIONS)
544 You are part of a team sent to assess the nutritional Height for age - a measure of skeletal growth - reflects ABDELSIMAR FINAL EXAM -
status of school children in a far-flung locale. Which cumulative impact of events affecting nutritional OMAR II, MD FEB 2014
of the following anthropometric measurements status. Stunting is a reflection of chronic malnutrition. (TOP 2 - AUG
would best screen for chronic malnutrition in the 2- Weight for height on the other hand screens for 2013 MED
10 year old age group? wasting which is a measure of acute malnutrition. BOARDS;
A. Height for age Weight for age has less clinical significance because it TOPNOTCH MD -
B. Weight for height combines stature with current health conditions. 200 QUESTIONS)
C. Weight for age MUAC is used in emergencies and in the field in lieu of AND MARC
D. Mid upper arm circumference weight for height. DENVER
E. BMI TIONGSON, MD
(40 QUESTIONS)

TOPNOTCH MEDICAL BOARD PREP PEDIATRICS SUPEREXAM Page 73 of 97


For inquiries visit www.topnotchboardprep.com.ph or email us at topnotchmedicalboardprep@gmail.com
TOPNOTCH MEDICAL BOARD PREP PEDIATRICS SUPEREXAM
For inquiries visit www.topnotchboardprep.com.ph or email us at topnotchmedicalboardprep@gmail.com
Item QUESTION EXPLANATION AUTHOR TOPNOTCH
# EXAM
545 A 3/F was rushed to the emergency room due to Streptococcus pneumoniae is the most common ABDELSIMAR FINAL EXAM -
difficulty breathing. At the emergency room, patient pathogen in children 3 weeks to 4 years of age. OMAR II, MD FEB 2014
was noted to have the following vital signs: RR 48, (TOP 2 - AUG
HR 104, and T 39.9C. On PE, there was note of chest 2013 MED
indrawing. What is the most likely causative agent BOARDS;
in this infection? TOPNOTCH MD -
A. Haemophilus influenzae 200 QUESTIONS)
B. Streptococcus pneumoniae AND MARC
C. Mycoplasma pneumoniae DENVER
D. Chlamydia pneumoniae TIONGSON, MD
E. Chlamydia trachomatis (40 QUESTIONS)
546 A 6 week old infant is brought to you because of Pneumonia in an infant with eye discharge is ABDELSIMAR FINAL EXAM -
difficulty breathing of 3 days duration. The patient consistent with Chlamydia. Treat with oral OMAR II, MD FEB 2014
was born vaginally, at term, and with no erythromycin x 2 weeks for either pneumonia / (TOP 2 - AUG
fetomaternal complications. On PE, you note that conjunctivitis. 2013 MED
the patient has clear rhinorrhea, bilaterally BOARDS;
erythematous conjunctivae with discharge; and TOPNOTCH MD -
scattered crackles without wheezing. What is the 200 QUESTIONS)
most likely causative agent in this infection? AND MARC
A. Rhinovirus DENVER
B. Streptococcus pneumoniae TIONGSON, MD
C. Respiratory syncytial virus (40 QUESTIONS)
D. Chlamydia pneumoniae
E. Chlamydia trachomatis
547 A 45/F who has not had any prenatal check-ups Approximately 50% of Down's Syndrome infants have ABDELSIMAR FINAL EXAM -
delivers a 3,800 g live baby girl. The infant has cardiac defects - most commonly an endocardial OMAR II, MD FEB 2014
decreased tone, upslanting palpebral fissures, cushion defect (60%), VSD (32%), and TOF (6%). (TOP 2 - AUG
epicanthal fold, redundant nuchal skin, fifth finger 2013 MED
clinodactyly and brachydactyly and a single BOARDS;
transverse palmar crease. A 2D Echo was TOPNOTCH MD -
performed by the attending physician to screen the 200 QUESTIONS)
child of congenital heart diseases; the most AND MARC
common of which in this population is: DENVER
A. Atrial septal defect TIONGSON, MD
B. Ventricular septal defect (40 QUESTIONS)
C. Tetralogy of Fallot
D. Coarctation of the aorta
E. Endocardial cushion defect
548 A history of bloody stools, draining ears and atopic Wiskott-Aldrich syndrome, an X-linked recessive ABDELSIMAR FINAL EXAM -
eczema in a 5-month old should prompt one to syndrome, is characterized by atopic dermatitis, OMAR II, MD FEB 2014
suspect: thrombocytopenic purpura and an undue (TOP 2 - AUG
A. Ataxia-telangiectasia syndrome susceptibility to infection. 2013 MED
B. DiGeorge Syndrome BOARDS;
C. Wiskott-Aldrich syndrome TOPNOTCH MD -
D. Severe Combined Immunodeficiency 200 QUESTIONS)
E. Combined Variable Immunodeficiency AND MARC
DENVER
TIONGSON, MD
(40 QUESTIONS)
549 A day old newborn who was under observation in Patient has DiGeorge Syndrome. Remember the ABDELSIMAR FINAL EXAM -
the NICU for temperture instability had a suspected mnemonic: CATCH-22. It is characterized by: Cardiac OMAR II, MD FEB 2014
seizure. On examination, you note that the patient abnormalities, Abnormal facies, Thymic aplasia, Cleft (TOP 2 - AUG
had wide set eyes, a small mandible and a cleft palate, Hypocalcemia and deletion in 22q11. 2013 MED
palate. You hear a murmur on auscultation. Chest BOARDS;
radiograph reveals no apparent thymus. On seeing TOPNOTCH MD -
the results of serum chemistries, you determine 200 QUESTIONS)
that the seizure was probably due to severe AND MARC
hypocalcemia. You now suspect a syndrome caused DENVER
by mutant genes in this chromosome: TIONGSON, MD
A. Chromosome 22 (40 QUESTIONS)
B. Chromosome 13
C. Chromosome 21
D. Chromosome 19
E. X chromosome
550 A 17 year old female was brought to you by her Turner syndrome is a condition characterized by ABDELSIMAR FINAL EXAM -
mother because she has not yet had her menses. On complete or partial monosomy of the X chromosome OMAR II, MD FEB 2014
PE, you note that the patient is below the 5th and defined by a combination of phenotypic features (TOP 2 - AUG
percentile for height. You also not ethat she has including redundant nuchal skin (in utero cytic 2013 MED
webbed neck and redundant nuchal skin. You hygroma), short stature and ovarian dysgenesis BOARDS;
suspect a chromosomal disroder and order (causing primary amenorrhea). TOPNOTCH MD -
karyotyping which would most likely reveal this 200 QUESTIONS)
karyotype: AND MARC
A. 47,XXY DENVER
B. 45,XO TIONGSON, MD
C. 47, XXX (40 QUESTIONS)
D. 47,XYY
E. 46,XY
551 A newborn infant presents with generalized Jaundice at birth or within the first 24 hours of life is a ABDELSIMAR FINAL EXAM -
jaundice within the first 24 hours of life. Which of cause for concern. Causes include: erythroblastosis OMAR II, MD FEB 2014
the following considerations is LEAST likely? fetalis, concealed hemorrhage, sepsis, congenital (TOP 2 - AUG
A. Erythroblastosis fetalis infections and hemolysis. 2013 MED
B. Neonatal sepsis BOARDS;
C. Congenital infections TOPNOTCH MD -
D. Physiologic jaundice 200 QUESTIONS)
E. Hemolysis AND MARC
DENVER
TOPNOTCH MEDICAL BOARD PREP PEDIATRICS SUPEREXAM Page 74 of 97
For inquiries visit www.topnotchboardprep.com.ph or email us at topnotchmedicalboardprep@gmail.com
TOPNOTCH MEDICAL BOARD PREP PEDIATRICS SUPEREXAM
For inquiries visit www.topnotchboardprep.com.ph or email us at topnotchmedicalboardprep@gmail.com
Item QUESTION EXPLANATION AUTHOR TOPNOTCH
# EXAM
TIONGSON, MD
(40 QUESTIONS)

552 A 10/M came in to the community clinic due to ear External otitis is caused most commonly by P. ABDELSIMAR FINAL EXAM -
pain accentuated by jaw motion. Prior to the pain, aeruginosa. Otitis media is most commonly caused by OMAR II, MD FEB 2014
patient reported itching in his ear. PE is remarkable S. pneumoniae. (TOP 2 - AUG
for severe tragal tenderness and ear pain with 2013 MED
manipulaton of the pinna. You had to defer BOARDS;
complete otoscopic examination due to severe TOPNOTCH MD -
canal tenderness. The most likely common etiologic 200 QUESTIONS)
agent in this condition is: AND MARC
A. Staphylococcus aureus DENVER
B. Streptococcus pneumoniae TIONGSON, MD
C. Klebsiella pneumonier (40 QUESTIONS)
D. Enterobacter aerogenes
E. Pseudomonas aeruginosa
553 For the patient above, you then advise: Topical otic preparations containing neomycin (active ABDELSIMAR FINAL EXAM -
A. Hospitalization for administration of IV against gram-positive organisms and some gram- OMAR II, MD FEB 2014
vancomycin negative organisms, notably Proteus spp.) with either (TOP 2 - AUG
B. Outpatient therapy with Amoxicillin 500mg BID x colistin or polymyxin (active against gram-negative 2013 MED
7 days bacilli, notably Pseudomonas spp.) and corticosteroids BOARDS;
C. Ciprofloxacin eardrops applied to wick inserted are highly effective in treating most forms of acute TOPNOTCH MD -
into ear 3x/day for 2 days external otitis. Newer preparations of eardrops (e.g., 200 QUESTIONS)
D. Hospitalization for administration of IV ofloxacin, ciprofloxacin) are preferable and do not AND MARC
levofloxacin contain potentially ototoxic antibiotics. DENVER
E. Daily cleansing and close follow up TIONGSON, MD
(40 QUESTIONS)
554 A patient who suffered kernicterus as a neonate A similar question was asked in the Aug 2013 boards. ABDELSIMAR FINAL EXAM -
would be at risk of developing which form of Nelsons 19e p.2062 OMAR II, MD FEB 2014
cerebral palsy? (TOP 2 - AUG
A. Spastic diplegic 2013 MED
B. Spastic quadriplegic BOARDS;
C. Hemiplegic TOPNOTCH MD -
D. Extrapyramidal 200 QUESTIONS)
AND MARC
DENVER
TIONGSON, MD
(40 QUESTIONS)
555 A 17/M comes in after having an episode of Patient has juvenile myoclonic epilepsy or Janz ABDELSIMAR FINAL EXAM -
generalized tonic clonic seizure upon waking up. On syndrome, the MOST COMMON generalized epilepsy OMAR II, MD FEB 2014
probing, you learn that the patient has been having in young adults. Patients usually present with (TOP 2 - AUG
myoclonic jerks in the morning, often causing the myoclonic jerks in the morning, often causing patients 2013 MED
patient to drop things. EEG in this patient to drop things. It is best treated with VALPROATE or BOARDS;
demonstrates 4-5 cycle per second generalized LAMOTRIGINE TOPNOTCH MD -
spike and waves. Which anticonvulsant is MOST 200 QUESTIONS)
appropriate for him? AND MARC
A. Ethosuximide DENVER
B. Carbamazepine TIONGSON, MD
C. Valproic acid (40 QUESTIONS)
D. Topiramate
E. Vigabatrin
556 A term infant, delivered via cesarean section, is Patient has transient tachypnea of the newborn. ABDELSIMAR FINAL EXAM -
referred to you for tachypnea and expiratory Premature birth, precipitous birth, and operative birth OMAR II, MD FEB 2014
grunting at the first hour of life. Otherwise, physical without labor have all been associated with an (TOP 2 - AUG
examination is unremarkable. You order a stat CXR increased risk of TTN. 2013 MED
which revealed prominent vascular markings and BOARDS;
fluid in intralobar fissures. Which of the following is TOPNOTCH MD -
the MOST IMPORTANT predisposing factor in this 200 QUESTIONS)
condition? AND MARC
A. Birth by cesarean delivery DENVER
B. Low birthweight TIONGSON, MD
C. Prolonged labor (40 QUESTIONS)
D. Administration of large amounts of IV fluids to
the mother
E. Birth to a diabetic mother
557 A 2 month old boy was brought to the emergency The epidemiologic evi- dence that positioning the ABDELSIMAR FINAL EXAM -
room because after the mother discovered the boy babies to sleep on their backs reduces the rate of SIDS OMAR II, MD FEB 2014
no longer breathing in his bed. You start CPR at the deaths by more than 50% suggests that position, and (TOP 2 - AUG
ER; but the patient was pronounced dead after 30 not prematurity, has been the primary cause of SIDS. 2013 MED
minutes of resuscitation. After thorough review of Avoid- ance of cigarette smoking exposure and of BOARDS;
the birth and medical history, you fail to identify a overheating the infant are also important in the TOPNOTCH MD -
clear cause of death. Which of the following could prevention of SIDS. 200 QUESTIONS)
have prevented these kinds of deaths? AND MARC
A. Use of a cardiac monitor DENVER
B. Positioning the infant to sleep on his/her back TIONGSON, MD
C. Home monitoring with cameras and baby (40 QUESTIONS)
monitors
D. Breastfeeding and administration of vitamins
E. All of the above

TOPNOTCH MEDICAL BOARD PREP PEDIATRICS SUPEREXAM Page 75 of 97


For inquiries visit www.topnotchboardprep.com.ph or email us at topnotchmedicalboardprep@gmail.com
TOPNOTCH MEDICAL BOARD PREP PEDIATRICS SUPEREXAM
For inquiries visit www.topnotchboardprep.com.ph or email us at topnotchmedicalboardprep@gmail.com
Item QUESTION EXPLANATION AUTHOR TOPNOTCH
# EXAM
558 A 17/F consulted due to recurrent episodes of IgA nephropathy or Berger nephropathy is the MOST ABDELSIMAR FINAL EXAM -
having "iced tea" colored urine. He has been having COMMON chronic glomerular disease; commonly OMAR II, MD FEB 2014
these episodes for several years; notably occuring a presenting as gross hematuria often occuring 1-2 days (TOP 2 - AUG
few days after onset of upper respiratory tract after onset of URTI or GI infection 2013 MED
infections. Patient has no other symptoms or BOARDS;
remarkable physical findings. What is the most TOPNOTCH MD -
likely diagnosis? 200 QUESTIONS)
A. Post-streptococcal glomerulonephritis AND MARC
B. Nephrolithiasis DENVER
C. Hemolytic-uremic syndrome TIONGSON, MD
D. Henoch Schonlein Purpura nephritis (40 QUESTIONS)
E. IgA nephropathy
559 A 24/F medical student comes in due to a 10-day Patient has infectious mononucleosis. ABDELSIMAR FINAL EXAM -
history of malaise, low-grade fever, headache and OMAR II, MD FEB 2014
nausea. On physical examination, enlarged (TOP 2 - AUG
posterior cervical lymph nodes are palpated, 2013 MED
hypertrophied tonsils with exudates were seen, and BOARDS;
the Traube's space was obliterated. CBC done TOPNOTCH MD -
revealed leukocytosis with lymphocytic 200 QUESTIONS)
predominance. Which of the following organisms is AND MARC
the most likely etiologic agent? DENVER
A. Group A b-hemolytic Streptococcus TIONGSON, MD
B. Coxsackie virus (40 QUESTIONS)
C. Adenovirus
D. Epstein-Barr virus
E. Haemophilus influenzae
560 A 7/F comes in to your clinic due to a rash. On Erythema infectiosum is a childhood illness caused by ABDELSIMAR FINAL EXAM -
examination, you note that the patient has bright infection with parvovirus B19. A "slapped cheek" OMAR II, MD FEB 2014
red macules on his cheeks and a lacy, reticulated appearance with a lacy eruption on the torso and (TOP 2 - AUG
erythema over the extremities. Patient has no other extremities is the most common presentation. 2013 MED
symptoms or remarkable examination findings. BOARDS;
What is the most likely diagnosis? TOPNOTCH MD -
A. Scarlet fever 200 QUESTIONS)
B. Rubella AND MARC
C. Roseola infantum DENVER
D. Erythema infectiosum TIONGSON, MD
E. Measles (40 QUESTIONS)
561 A 2 year old male was seen in the ER due to fever Urine CS is the gold standard in the microbiologic BLAKE WARREN MIDTERM 2
of 5 days duration. This wasassociated with diagnosis ANG, MD (TOP 1 - EXAM - FEB
vomiting and anorexia. There were no cough, colds AUG 2013 MED 2014
associated. CBC revealed leukocytosis with BOARDS;
predominance of Segmenters. Urinalysis revealed TOPNOTCH MD)
TNTC pus cells and 5-10/hpf RBC. Impression : UTI.
The gold standard for the diagnosis of UTI is:
A. Urine Culture and sensitivity
B. Leukocyte esterase test
C. Urinalysis
D. Nitrite test
562 The best single antibody titer to document Anti-DNAse is the marker for PSGN due to BLAKE WARREN MIDTERM 2
cutaneous streptococcal infection in PSGN is: streptococcal skin ANG, MD (TOP 1 - EXAM - FEB
A. Deoxyribonuclease B antigen (DNase) AUG 2013 MED 2014
B. ASO titer BOARDS;
C. Streptozyme test TOPNOTCH MD)
D. Phadebact test

563 A 2 year old afebrile male child was brought to you 80mL/kg; This patient presents with AGE with some BLAKE WARREN MIDTERM 2
for consult due to 2 days of persistent diarrhea x 8 a dehydration because of increased thirst and ANG, MD (TOP 1 - EXAM - FEB
day with vomiting episodes. Upon evaluation irritability. (Consider the major clinical parameters: AUG 2013 MED 2014
patient has no sunken eyeballs, awake and Sensorium, Thirst, and Skin pinch) BOARDS;
frequently cries with minimal handling. He drinks Some Dehydration equates to 50-100 mL per kg of TOPNOTCH MD)
more than enough water but ends up vomiting. deficit (hence, we resuscitate using 75ml per kg to
What is the estimated fluid deficit of this patient? meet half way between the two ranges).
A. 25 mL per kg
B. 45 mL per kg
C. 80 mL per kg
D. 110 mL per kg
564 A 3 year old boy who just had his birthday presents Although Tetracycline is generally not favorable for BLAKE WARREN MIDTERM 2
to you with profuse watery diarrhea that was later use among children, it may be so in cases of cholera by ANG, MD (TOP 1 - EXAM - FEB
diagnosed to be due to cholera. What is the virtue of its short duration of therapy (3 days). AUG 2013 MED 2014
antibiotic of choice? BOARDS;
A. No antibiotic is needed TOPNOTCH MD)
B. Tetracycline even if the patient is only < 8years
old
C. Ceftriaxone IV drip
D. Metronidazole PO dose
565 A 6 month old baby girl is expected to be able to do the patient is not expected to be able to stand by 6 BLAKE WARREN MIDTERM 2
the following, except: months of age. ANG, MD (TOP 1 - EXAM - FEB
A. Stand while holding the crib AUG 2013 MED 2014
B. Sit with some support BOARDS;
C. Grab large objects with both hands TOPNOTCH MD)
D. Social smile

TOPNOTCH MEDICAL BOARD PREP PEDIATRICS SUPEREXAM Page 76 of 97


For inquiries visit www.topnotchboardprep.com.ph or email us at topnotchmedicalboardprep@gmail.com
TOPNOTCH MEDICAL BOARD PREP PEDIATRICS SUPEREXAM
For inquiries visit www.topnotchboardprep.com.ph or email us at topnotchmedicalboardprep@gmail.com
Item QUESTION EXPLANATION AUTHOR TOPNOTCH
# EXAM
566 Most common cause of arf in children: HUS is the most common cause of ARF in children BLAKE WARREN MIDTERM 2
A. PSGN ANG, MD (TOP 1 - EXAM - FEB
B. IgA Nephropathy AUG 2013 MED 2014
C. Hemolytic Uremic Syndrome BOARDS;
D. FSGS TOPNOTCH MD)

567 A known case of congenital heart disease patient BLAKE WARREN MIDTERM 2
presents to you with an egg shaped chest Xray most ANG, MD (TOP 1 - EXAM - FEB
likely has: AUG 2013 MED 2014
A. TGA BOARDS;
B. ASD TOPNOTCH MD)
C. Truncus Arteriosus
D. Pericardial effusion

568 A box-shaped heart on Chest Xray is seen in: BLAKE WARREN MIDTERM 2
A. TGA ANG, MD (TOP 1 - EXAM - FEB
B. Ebstein Anomaly AUG 2013 MED 2014
C. Pericardial effusion BOARDS;
D. TAPVR TOPNOTCH MD)

569 the trisomy most commonly associated with a heart Most common chromosomal anonaly assoc with BLAKE WARREN MIDTERM 2
defect in more than 90% of the cases is due to what cardiac defect: trisomy 18 ANG, MD (TOP 1 - EXAM - FEB
chromosomal abnormality? Heart shapes: AUG 2013 MED 2014
A. 13 box shape: ebstein BOARDS;
B. 18 boot: tof TOPNOTCH MD)
C. 21 egg: tga
D. XO snowman: tapvr
scimitar: papvr
water bottle: pericardial tamponade

570 An infant brought to you with red eye due to mild most common cause of overgrowth in children: BLAKE WARREN MIDTERM 2
corneal erosions from exophthalmos was also macroglossia exophthalmos omphalocele is Beckwith ANG, MD (TOP 1 - EXAM - FEB
noted to have omphalocele as well as macroglossia. Weidenmann Syndrome AUG 2013 MED 2014
A probable cause of this is entertained: BOARDS;
A. beckwith weidenmann syndrome TOPNOTCH MD)
B. WAGR
C. Denys Drash syndrome
D. Pituitary hyperfunction

571 digeorge syndrome: thymic hypoplasia in chromosome 22, cardiac, anomaly of face, thymic BLAKE WARREN MIDTERM 2
connection with CATCH22 hypoplasia, cleft palate, hypocalcemia constitute ANG, MD (TOP 1 - EXAM - FEB
has the following clinical features except: CATCH22 AUG 2013 MED 2014
A. Cardiac anomaly BOARDS;
B. Thymic hyperplasia TOPNOTCH MD)
C. Cleft palate
D. Hypocalcemia

572 This is the most common cause of hematuria PSGN is the second most common cause of hematuria BLAKE WARREN MIDTERM 2
second only to Berger’s disease: in children following igA Nephropathy ANG, MD (TOP 1 - EXAM - FEB
A. PSGN AUG 2013 MED 2014
B. FSGS BOARDS;
C. Cresentic Glomerulonephritis TOPNOTCH MD)
D. Lupus nephritis

573 A 3 week old infant came in for non-bilous vomiting BLAKE WARREN MIDTERM 2
occurring immediately after feeding. This was ANG, MD (TOP 1 - EXAM - FEB
followed by progressive loss of fluids and AUG 2013 MED 2014
electrolytes and dehydration. Feeding was alright. BOARDS;
Most likely you are dealing with TOPNOTCH MD)
A. Duodenal atresia
B. Esophageal atresia
C. Hypertrophic pyloric stenosis
D. Duodenal stenosis

574 A major criteria in the diagnosis of atopic BLAKE WARREN MIDTERM 2


dermatitis is ANG, MD (TOP 1 - EXAM - FEB
A. chronic relapsing course AUG 2013 MED 2014
B. elevated serum IgE BOARDS;
C. early age of onset TOPNOTCH MD)
D. white dermatographism

TOPNOTCH MEDICAL BOARD PREP PEDIATRICS SUPEREXAM Page 77 of 97


For inquiries visit www.topnotchboardprep.com.ph or email us at topnotchmedicalboardprep@gmail.com
TOPNOTCH MEDICAL BOARD PREP PEDIATRICS SUPEREXAM
For inquiries visit www.topnotchboardprep.com.ph or email us at topnotchmedicalboardprep@gmail.com
Item QUESTION EXPLANATION AUTHOR TOPNOTCH
# EXAM
575 The use of short acting beta 2 agonists as sole Mild Intermittent Asthma is treated with SABA PRN, BLAKE WARREN MIDTERM 2
treatment is limited to: whereas mild persistent asthma warrants a low-dose ANG, MD (TOP 1 - EXAM - FEB
A. Mild Interval Asthma ICS. AUG 2013 MED 2014
B. Mild Persistent Asthma BOARDS;
C. Mild Recurring Asthma TOPNOTCH MD)
D. Mild Intermittent Asthma

576 The systolic blood pressure of a 4 year old boy with BP= Age x 2 + 90 (upper SBP range) BLAKE WARREN MIDTERM 2
no known morbidities will be considered low if it BP= Age x 2 + 70 (lower SBP range) ANG, MD (TOP 1 - EXAM - FEB
falls below: Therefore, 4 x 2 + 70 and 4 x 2+90 = 78 – 98 mmHg is AUG 2013 MED 2014
A: 59 mmHg the normal SBP range of a 4 year old child. Below BOARDS;
B. 67 mmHg 78mmHg is already considered low. TOPNOTCH MD)
C. 78 mmHg
D. 90 mmHg

577 Administration of this vaccine component in older the pertussis (cellular) component is not encouraged BLAKE WARREN MIDTERM 2
children causes a neuroparalytic side effect: among older children and adults because of its ANG, MD (TOP 1 - EXAM - FEB
A. DPT possible neuroparalytic side effect. Therefore an AUG 2013 MED 2014
B. TT acellular pertussis component is substituted (DTaP) BOARDS;
C. Rota Virus or is disregarded (Td TOPNOTCH MD)
D. MMR-V

578 The earliest possible age for an infant to receive his At least 1 ½ months after birth, the infant may be BLAKE WARREN MIDTERM 2
first dose DPT is: eligible for a first DPT dose together with OPV ANG, MD (TOP 1 - EXAM - FEB
A. At birth following the EPI recommendation. AUG 2013 MED 2014
B. 4 weeks of life BOARDS;
C. 6 weeks of life TOPNOTCH MD)
D. 8 weeks of life

579 The ratio of chest compressions and respiration in a A 15:2 ratio of compression and respiration is advised BLAKE WARREN MIDTERM 2
2 health care rescuer setting is: in a 2 rescuer setting. A 30:2 ratio is advised in a lone ANG, MD (TOP 1 - EXAM - FEB
A. 30: 1 rescuer situation. AUG 2013 MED 2014
B. 30: 2 BOARDS;
C. 15: 1 TOPNOTCH MD)
D. 15:2

580 The most common cause of cardiac arrest in the In contrast to adults in which cardiac causes account BLAKE WARREN MIDTERM 2
pediatric population is due to a primary failure of for majority of Sudden cardiac arrests, the pediatric ANG, MD (TOP 1 - EXAM - FEB
what organ system? population most commonly suffer arrests due to AUG 2013 MED 2014
A. Cardiovascular Respiratory causes. BOARDS;
B. Respiratory TOPNOTCH MD)
C. Metabolic
D. Neurologic

581 A 3-week old infant initially normal at birth was Deficiency of the enzyme phenylalanine hydroxylase TIMOTHY TANG MIDTERM 1
noted to have episodes of vomiting accompanied by or of its cofactor tetrahydrobiopterin causes LEE SAY, MD EXAM - FEB
purposeless movements, rhythmic rocking, and accumulation of phenylalanine in body fluids and the (TOP 4 - AUG 2014
athetosis. On physical examination, vital signs were central nervous system (CNS). The severity of 2013 MED
normal and patient was hypertonic with hyperphenylalaninemia depends on the degree of BOARDS;
hyperactive deep tendon reflexes. Newborn testing enzyme deficiency and may vary from very high TOPNOTCH MD)
revealed elevated plasma phenylalanine levels >20 plasma concentrations (>20 mg/dL or >1,200
mg/dL. What is the initial management μmole/L, classic phenylketonuria [PKU]) to mildly
recommended for the patient? elevated levels (2–6 mg/dL or 120–360 μmole/L).The
A. Restrict phenylalanine in the diet affected infant is normal at birth. Mental retardation
B. Oral administration of the cofactor may develop gradually and may not be evident for the
tetrahydrobiopterin 1st few months. Vomiting, sometimes severe enough
C. Sequential phenylalanine determination until to be misdiagnosed as pyloric stenosis, may be an
levels decrease early symptom. Older untreated children become
D. Repeat newborn screening hyperactive, with purposeless movements, rhythmic
E. Limit protein intake to amino acids rocking, and athetosis. The goal of therapy is to reduce
phenylalanine in the body; formulas low in or free of
this amino acid are available commercially. The diet
should be started as soon as diagnosis is established.
It is generally accepted that infants with persistent
plasma levels of phenylalanine >6 mg/dL (360
μmole/L) should be treated with a phenylalanine-
restricted diet similar to that for classic PKU.
582 An infant is born full term to a woman who had an A diaphragmatic hernia is defined as a communication TIMOTHY TANG MIDTERM 1
uncomplicated pregnancy. Immediately after between the abdominal and thoracic cavities with or LEE SAY, MD EXAM - FEB
delivery, the infant was noted to be cyanotic, without abdominal contents in the thorax. Early (TOP 4 - AUG 2014
tachypneic, and with intercostals retractions. There respiratory distress within 6 hr of life is thought to be 2013 MED
was absent breath sounds on both lungfields. Bowel a poor prognostic sign. The clinical signs of BOARDS;
sounds were heard on the left hemithorax. The respiratory distress are characterized by tachypnea, TOPNOTCH MD)
abdomen is scaphoid. The CXR shows a multicystic grunting, use of accessory muscles, and cyanosis.
mass in the left chest with a shift of the Children with CDH will also have a scaphoid abdomen
mediastinum to the right. What is the most likely and increased chest wall diameter. Bowel sounds may
diagnosis? also be heard in the chest with decreased breath
A. Congenital cystic adenomatoid malformation sounds bilaterally. The point of maximal cardiac
B. Diaphragmatic hernia impulse may be displaced away from the side of the
C. Respiratory distress syndrome hernia if mediastinal shift has occurred. A chest x-ray
D. Transient tachypnea of the newborn and nasal gastric tube is all that is usually required to

TOPNOTCH MEDICAL BOARD PREP PEDIATRICS SUPEREXAM Page 78 of 97


For inquiries visit www.topnotchboardprep.com.ph or email us at topnotchmedicalboardprep@gmail.com
TOPNOTCH MEDICAL BOARD PREP PEDIATRICS SUPEREXAM
For inquiries visit www.topnotchboardprep.com.ph or email us at topnotchmedicalboardprep@gmail.com
Item QUESTION EXPLANATION AUTHOR TOPNOTCH
# EXAM
E. Pulmonary hypoplasia confirm the diagnosis.

583 Which of the following is the recommended To prevent perinatal transmission through improved TIMOTHY TANG MIDTERM 1
vaccination schedule for neonates born mothers maternal screening and immunoprophylaxis of infants LEE SAY, MD EXAM - FEB
whose HBsAg status was positive? born to HBsAg -positive mothers. Infants born to (TOP 4 - AUG 2014
A. Hepatitis B immunoglobulin within the 12 HBsAg -positive women should receive vaccine at 2013 MED
hours after birth, Hepatitis B vaccine at 1 month, 3 birth, 1–2 mo, and 6 mo of age The 1st dose should be BOARDS;
months and 6 months old accompanied by administration of 0.5 mL of HBIG as TOPNOTCH MD)
B. Hepatitis B Immunoglobulin at birth and soon after delivery as possible (within 12 hr) because
Hepatitis B vaccine at birth, 1 month and 6 months the effectiveness decreases rapidly with increased
old time after birth. Postvaccination testing for HBsAg and
C. Hepatitis B immunoglobulin within the 24 anti-HBs should be done at 9–18 mo.
hours after birth, Hepatitis B vaccine at birth, one
and six months old
D. Hepatitis B vaccine at birth, one and six months
old
E. Hepatitis B immunoglobulin within the 24
hours after birth and after 3 days, Hepatitis B
vaccine at birth, one and six months old
584 While at school a 7 year old boy is noted to have Absence seizures are characterized by a sudden TIMOTHY TANG MIDTERM 1
failing grades at school. His teacher noted him to be cessation of motor activity or speech with a blank LEE SAY, MD EXAM - FEB
staring blankly for 15-20 seconds multiple times a facial expression and flickering of the eyelids. These (TOP 4 - AUG 2014
day during class. There was no noted Consult done seizures, which are uncommon before age 5 yr, are 2013 MED
and EEG shows 3 per second generalized spike and more prevalent in girls, are never associated with an BOARDS;
wave discharges. What is the MOST likely diagnosis aura, rarely persist longer than 30 sec, and are not TOPNOTCH MD)
of this boy? associated with a postictal state. Children with
A. Generalized tonic-clonicseizure Absence seizures may experience countless seizures
B. Attention deficit disorder daily, whereas complex partial seizures are usually
C. Complex partial seizures less frequent. The EEG shows a typical 3/sec spike and
D. Absence seizures generalized wave discharge.
E. Mental retardation
585 A 2 year old male was noted to have a right inguinal Most inguinal hernias in infants and children are TIMOTHY TANG MIDTERM 1
bulge. On physical examination, the mass is smooth, congenital and result from a persistent patency of the LEE SAY, MD EXAM - FEB
firm, and more prominent on crying. Both testes processus vaginalis. The pertinent developmental (TOP 4 - AUG 2014
were palpated in the scrotum. Transillumination anatomy of congenital indirect inguinal hernia relates 2013 MED
was negative. This condition is due to? to development of the gonads and descent of the testis BOARDS;
A. Weakness in the transverses abdominis muscle through the internal ring and into the scrotum late in TOPNOTCH MD)
B. Failure of the testes to descend gestation.
C. Patency of the procesus vaginalis
D. Accumulation of serous fluid in the scrotum
E. Enlarged and elongated pampiniform plexus
586 A 15-month old boy was brought for consult due to Treatment of the tetralogy of Fallot depends on the TIMOTHY TANG MIDTERM 1
cyanosis. Patient was noted to have bluish severity of the right ventricular outflow tract LEE SAY, MD EXAM - FEB
discoloration of the lips after vigorous crying. On obstruction. Infants with severe tetralogy require (TOP 4 - AUG 2014
physical examination, a systolic thrill is felt along medical treatment and surgical intervention in the 2013 MED
the left sternal border in the 3rd and 4th neonatal period. Therapy is aimed at providing an BOARDS;
parasternal spaces, there is a systolic murmur most immediate increase in pulmonary blood flow to TOPNOTCH MD)
intense at the left sternal border, and the 2nd heart prevent the sequelae of severe hypoxia. Neonates with
sound is single. On chest x-ray, the heart is boot- marked right ventricular outflow tract obstruction
shaped and hilar areas and lung fields are relatively may deteriorate rapidly because, as the ductus
clear. The management of this case will depend on? arteriosus begins to close, pulmonary blood flow is
A. The patency of the ductus arteriosus further compromised. The intravenous administration
B. The size of the ventricular septal defect of prostaglandin E1. Infants with less severe right
C. The severity of the right ventricular outflow ventricular outflow tract obstruction who are stable
tract obstruction and awaiting surgical intervention require careful
D. The presence of other congenital anomalies observation.
E. The degree of ventricular hypertrophy
587 A 14-year old female was brought to the clinic due In iron deficiency anemia, the therapeutic dose should TIMOTHY TANG MIDTERM 1
to pallor. This was associated with easy fatigability be calculated in terms of elemental iron; ferrous LEE SAY, MD EXAM - FEB
and frequent consumption of ice cubes. She had sulfate is 20% elemental iron by weight. A daily total (TOP 4 - AUG 2014
regular menses for 1 year. On physical examination, dose of 4–6 mg/kg of elemental iron in 3 divided 2013 MED
BP 100/60, PR 120, RR 22, temp 36.8, weight=40 doses provides an optimal amount of iron for the BOARDS;
kg, height=160 cm. There was pale palpebral stimulated bone marrow to use. TOPNOTCH MD)
conjunctivae and palmar pallor. CBC showed Hgb
90, Hct .28, WBC 5, platelet 250, 000. Serum ferritin
and serum iron were low and total iron binding
capacity was low. If you plan to start the patient on
oral iron therapy, what is the recommended dose?
A. 500 mg of elemental iron once a day
B. 2-3 mg/kg of elemental iron in 3 divided doses
daily
C. 250 mg of elemental iron twice daily
D. 4–6 mg/kg of elemental iron in 3 divided doses
daily
E. 1 g of elemental iron once a week
TOPNOTCH MEDICAL BOARD PREP PEDIATRICS SUPEREXAM Page 79 of 97
For inquiries visit www.topnotchboardprep.com.ph or email us at topnotchmedicalboardprep@gmail.com
TOPNOTCH MEDICAL BOARD PREP PEDIATRICS SUPEREXAM
For inquiries visit www.topnotchboardprep.com.ph or email us at topnotchmedicalboardprep@gmail.com
Item QUESTION EXPLANATION AUTHOR TOPNOTCH
# EXAM
588 A 3 year old male was brought to your clinic due to The laboratory screening test that is affected by a TIMOTHY TANG MIDTERM 1
right elbow swelling. Few hours prior to consult, reduced level of factor VIII or factor IX is PTT. Results LEE SAY, MD EXAM - FEB
patient fell and hit his right elbow. There was noted of the other screening tests of the hemostatic (TOP 4 - AUG 2014
rapid swelling of the affected joint and violaceous mechanism (platelet count, bleeding time, 2013 MED
discoloration of the overlying skin. On further prothrombin time, and thrombin time) are normal. BOARDS;
probing, the mother claimed that the patient was The specific assay for factors VIII and IX will confirm TOPNOTCH MD)
noted to have easy bruisability. On physical the diagnosis of hemophilia.
examination, vital signs were normal. There was
noted tenderness and swelling on the right elbow
with limitation in the range of motion. If you are
suspecting Hemophilia, what initial screening test
will you request for the patient?
A. Prothrombin time
B. Activated partial thromboplastin time
C. Bleeding time
D. Thrombin time
E. Clotting time
589 A 5 year old female was brought for consult due to The initial presentation of ALL usually is nonspecific TIMOTHY TANG MIDTERM 1
a 2 month history of anorexia, weight loss, low and relatively brief. Anorexia, fatigue, and irritability LEE SAY, MD EXAM - FEB
grade fever and easy bruisabilty. On physical often are present, as is an intermittent, low-grade (TOP 4 - AUG 2014
examination, patient had pale palpebral fever. On physical examination, findings of pallor, 2013 MED
conjunctivae, generalized lymphadenopathy and listlessness, purpuric and petechial skin lesions, or BOARDS;
splenomegaly. CBC showed anemia, mucous membrane hemorrhage may reflect bone TOPNOTCH MD)
thrombocytopenia, and leukocytosis with atypical marrow failure. The proliferative nature of the disease
lymphocytes. Bone marrow aspiration showed may be manifested as lymphadenopathy,
>25% of the bone marrow cells as a homogeneous splenomegaly, or, less commonly, hepatomegaly. The
population of lymphoblasts. What is the most likely diagnosis of ALL is strongly suggested by peripheral
diagnosis? blood findings indicative of bone marrow failure.
A. Acute myelogenous leukemia Anemia and thrombocytopenia are seen in most
B. Acute lymphoblastic leukemia patients. ALL is diagnosed by a bone marrow
C. Multiple myeloma evaluation that demonstrates >25% of the bone
D. Myelofibrosis marrow cells as a homogeneous population of
E. Plasmacytoma lymphoblasts.
590 A 12 year-old female was brought to your clinic Craniopharyngioma (WHO grade I) is a common TIMOTHY TANG MIDTERM 1
room due to blurring of vision. Patient had a one tumor of childhood, accounting for 7–10% of all LEE SAY, MD EXAM - FEB
month history of blurring of vision of both visual childhood tumors. These tumors are solid with cystic (TOP 4 - AUG 2014
fields described as being cloudy accompanied by components and occur within the suprasellar region. 2013 MED
headaches. Patient was given paracetamol which They are minimally invasive, adhere to adjacent brain BOARDS;
afforded relief of the headache. On fundoscopy, parenchyma, and engulf normal brain structures. MRI TOPNOTCH MD)
there was noted papilledema. MRI was requested demonstrates the solid tumor with cystic structures
which showed a solid tumor with cystic structures containing fluid of intermediate density. CT may show
containing fluid of intermediate density in the calcifications associated with the solid and cystic wall
suprasellar area. What is the most likely diagnosis? components. There is significant morbidity
A. Intracranial aneurysm (panhypopituitarism, growth failure, visual loss)
B. Pineoblastoma associated with these tumors and their therapy owing
C. Mennigioma to the anatomic location.
D. Chiasmal glioma
E. Craniopharygioma
591 A 10 year old male was brought to the emergency The clinical diagnosis of poststreptococcal TIMOTHY TANG MIDTERM 1
room due to red-brown urine. Patient had a history glomerulonephritis is quite likely in a child presenting LEE SAY, MD EXAM - FEB
of sore throat 2 weeks prior. On physical with acute nephritic syndrome, evidence of recent (TOP 4 - AUG 2014
examination, patient was hypertensive, febrile, and streptococcal infection, and a low C3 level. On renal 2013 MED
with bipedal edema. Urinalysis showed ph 6.0, biopsy, Granular IgG, C3 on immunofluorescence and BOARDS;
specific gravity 1.30, albumin 3+, sugar (-), RBC subepithelial humps on electron microscopy will be TOPNOTCH MD)
40/hpf, WBC 0-2/hpf , RBC casts +. ASO titers were seen.
elevated. What are the expected biopsy findings
that with differentiate it from other causes of acute
glomerulonephritis?
A. Crescents on light microscopy and absence of
deposits on immunofluorescence and light
microscopy
B. Linear IgG, C3 on immunofluorescence, Positive
anti-GBM antibody
C. Diffuse mesangial IgA on immunofluorescence,
mesangial deposits in electron microscopy,
elevated seum IgA
D. Low serum C3, Granular IgG, C3 on
immunofluorescence, Subepithelial humps on
electron microscopy
E. Normal microscopy and Immunofluorescence
with effacement of foot processes on electron
microscopy

TOPNOTCH MEDICAL BOARD PREP PEDIATRICS SUPEREXAM Page 80 of 97


For inquiries visit www.topnotchboardprep.com.ph or email us at topnotchmedicalboardprep@gmail.com
TOPNOTCH MEDICAL BOARD PREP PEDIATRICS SUPEREXAM
For inquiries visit www.topnotchboardprep.com.ph or email us at topnotchmedicalboardprep@gmail.com
Item QUESTION EXPLANATION AUTHOR TOPNOTCH
# EXAM
592 A 1 year old female was brought to the emergency Kawasaki disease (KD), formerly known as TIMOTHY TANG MIDTERM 1
room due to 1 week history of fever. On physical mucocutaneous lymph node syndrome and infantile LEE SAY, MD EXAM - FEB
examination, PR 145, RR 32, temp 38.2. There was polyarteritis nodosa, is an acute febrile vasculitis of (TOP 4 - AUG 2014
generalized maculopapular rash, erythema of the childhood. Fever is characteristically high (104°F or 2013 MED
oral and pharyngeal mucosa, the tongue was higher), remittent, and unresponsive to antibiotics. BOARDS;
swollen, lips were dry and cracked, and cervical The duration of fever without treatment is generally TOPNOTCH MD)
lymphadenopathy. This disease entity is of clinical 1–2 wk, but it may persist for 3–4 wk. Prolonged fever
significance due to its predilection to? is prognostic for the development of coronary artery
A. Renal arteries disease. In addition to fever, the five characteristic
B. Aorta features of Kawasaki disease are: bilateral bulbar
C. Coronary arteries conjunctival injection, usually without exudate;
D. Cerebral arteries erythema of the oral and pharyngeal mucosa with
E. Pulmonary arteries strawberry tongue and dry, cracked lips, and without
ulceration; edema and erythema of the hands and feet;
rash of various forms (maculopapular, erythema
multiforme, or scarlatiniform) with accentuation in
the groin area; and nonsuppurative cervical
lymphadenopathy, usually unilateral, with node size of
≥1.5 cm. Perineal desquamation is common in the
acute phase. Periungual desquamation of the fingers
and toes begins 1–3 wk after the onset of illness and
may progress to involve the entire hand and foot.
Cardiac involvement is the most important
manifestation of Kawasaki disease. Myocarditis,
manifested as tachycardia out of proportion to fever
occurs in at least 50% of patients.
593 A 2 year old was admitted at the pediatric ICU due The surfaces of burns or wounds are frequently TIMOTHY TANG MIDTERM 1
to burn injuries to the trunk and both lower populated by Pseudomonas and other gram-negative LEE SAY, MD EXAM - FEB
extremities. On the 5th day at the ICU, patient was organisms. The characteristic skin lesions of (TOP 4 - AUG 2014
noted to have pink macules which later progressed Pseudomonas, ecthyma gangrenosum, whether 2013 MED
to hemorrhagic nodules and eventually to ulcers caused by direct inoculation or metastatic secondary BOARDS;
with ecchymotic and gangrenous centers with to septicemia, begin as pink macules and progress to TOPNOTCH MD)
eschar formation, surrounded by an intense red hemorrhagic nodules and eventually to ulcers with
areola. If you are entertaining an infection caused ecchymotic and gangrenous centers with eschar
by pseudomonas aeruginosa, what antibiotic has formation, surrounded by an intense red areola.
coverage for this organism? Antibiotics with Pseudomonas coverage are
A. Ciprofloxacin Ceftazidime, Ciprofloxacin, Tobramycin, Gentamicin,
B. Ceftriaxone Aztreonam, and Amikacin.
C. Trimethoprim-Sulfamethoxazole
D. Cloxacillin
E. Doxycycline
594 A 17 year old female was rushed to the emergency Cutaneous lesions in Stevens-Johnson syndrome TIMOTHY TANG MIDTERM 1
room due to multiple eythematous papules on the generally consist initially of erythematous macules LEE SAY, MD EXAM - FEB
trunk and upper extremities. Patient had was that rapidly and variably develop central necrosis to (TOP 4 - AUG 2014
recently given Ciprofloxacin. On examination, there form vesicles, bullae, and areas of denudation on the 2013 MED
papules with an erythematous outer border, an face, trunk, and extremities. The skin lesions are BOARDS;
inner pale ring, and a dusky purple to necrotic typically more widespread than in EM and are TOPNOTCH MD)
center noted on the trunk and upper extremities as accompanied by involvement of two or more mucosal
well as on the oral cavity. What is the most likely surfaces, namely the eyes, oral cavity, upper airway or
diagnosis? esophagus, gastrointestinal tract, or anogenital
A. Bullous pemphigoid mucosa. A burning sensation, edema, and erythema of
B. Bullous drug eruption the lips and buccal mucosa are often the presenting
C. Allergic vasculitis signs, followed by development of bullae, ulceration,
D. Stevens-Johnson syndrome and hemorrhagic crusting.
E. Pemphigus vulgaris
595 A 24 year old primigravid gave birth to a live term Children with a high lesion have a double-barrel TIMOTHY TANG MIDTERM 1
baby boy. On physical examination, the child was colostomy performed. This effectively separates the LEE SAY, MD EXAM - FEB
noted to have imperforate anus. Patient was fecal stream from the urinary tract. It also allows the (TOP 4 - AUG 2014
observed for 24 hours and there was no noted performance of an augmented pressure colostogram 2013 MED
passage of meconium stained urine or the presence before repair to identify the exact position of the distal BOARDS;
of meconium at the median raphe. At the 24th hour rectum and the fistula. The definitive repair or TOPNOTCH MD)
of life a, cross-table lateral x-ray was done and posterior sagittal anorectoplasty (PSARP) is
showed that the marker on the skin was 3.5 cm performed at about 1 yr of age.
from the rectum. What is the definitive
management for this type of impeforate anus?
A. Perform colostomy immediately
B. Simple dilatation with Hegar dilators
C. Posterior sagittal anorectoplasty
D. Swenson's pull-through procedure
E. Kasai procedure
596 An 11- month old infant was brought to you clinic Acute Otitis Media TIMOTHY TANG MIDTERM 1
due to fever irritability. On physical examination, First-line: High-dose amoxicillin (80–100 mg/kg/day). LEE SAY, MD EXAM - FEB
PR 100, RR 25, temp 37.9. The nasal turbintaes Alt for penicillin allergy: Cefuroxime, cefdinir, (TOP 4 - AUG 2014
were erythematous and congested without cefprozil, azithromycin 2013 MED
purulent discharge. On otoscopy, the tympanic Persistent otitis media (after 3 days): BOARDS;
membrane was bulging, amber-clored, and with air Amoxicillin/clavulanic acid, cefuroxime, or ceftriaxone TOPNOTCH MD)
bubbles behind the tympanic membrane. If you are (IM/IV)
considering acute otitis media, what is the dose of
amoxicillin that you will give to your patient?
A. 10-15 mg/kg/day
B. 30-40 mg/kg.day
C. 40-60 mg/kg/day
D. 60-80 mg/kg/day
E. 80-100 mg/kg/day

TOPNOTCH MEDICAL BOARD PREP PEDIATRICS SUPEREXAM Page 81 of 97


For inquiries visit www.topnotchboardprep.com.ph or email us at topnotchmedicalboardprep@gmail.com
TOPNOTCH MEDICAL BOARD PREP PEDIATRICS SUPEREXAM
For inquiries visit www.topnotchboardprep.com.ph or email us at topnotchmedicalboardprep@gmail.com
Item QUESTION EXPLANATION AUTHOR TOPNOTCH
# EXAM
597 A 7 month old girl was brought to the emergency Vitamin A deficiency in children in developing TIMOTHY TANG MIDTERM 1
room due to high fever. Patient had 4 days history countries has long been known to be associated with LEE SAY, MD EXAM - FEB
of fever (38C) followed by coryza, cough and increased mortality from a variety of infectious (TOP 4 - AUG 2014
maculopapular rash which started on the face and diseases, including measles. Several randomized 2013 MED
later spred to the trunk and extremities. What controlled trials of vitamin A therapy in the BOARDS;
vitamin deficiency is associated with increased developing world and the United States have TOPNOTCH MD)
mortality from this disease entity? demonstrated reduced morbidity and mortality from
A. Vitamin K measles. The American Academy of Pediatrics
B. Vitamin A suggests vitamin A therapy for selected patients with
C. Vitamin D measles.
D. Vitamin B1
E. Vitamin B6
598 On physical examination, the neonate was noted to Injury to the brachial plexus may cause paralysis of TIMOTHY TANG MIDTERM 1
have an absent Moro and Biceps reflex on the left the upper part of the arm with or without paralysis of LEE SAY, MD EXAM - FEB
arm. The left arm was also noted to be abducted, the forearm or hand or, more commonly, paralysis of (TOP 4 - AUG 2014
internally rotated, and the forearm was pronated. the entire arm. In Erb-Duchenne paralysis, the injury 2013 MED
What birth injury does the patient most likely is limited to the 5th and 6th cervical nerves. The BOARDS;
have? characteristic position consists of adduction and TOPNOTCH MD)
A. Erb-Duchenne paralysis internal rotation of the arm with pronation of the
B. Klumpke paralysis forearm. Power to extend the forearm is retained, but
C. Horner syndrome the biceps reflex is absent; the Moro reflex is absent
D. Duchenne muscular dystrophy on the affected side.
E. Myasthenia gravis
599 Which of the following may cause a reactivation of Measles infection is known to suppress skin test TIMOTHY TANG MIDTERM 1
pulmonary tuberculosis? responsiveness to purified tuberculin antigen. There LEE SAY, MD EXAM - FEB
A. Rubella may be an increased rate of activation of pulmonary (TOP 4 - AUG 2014
B. Varicella tuberculoses in populations of individuals infected 2013 MED
C. Mumps with Mycobacterium tuberculosis. BOARDS;
D. Measles TOPNOTCH MD)
E. Herpes

600 A 2 year old female was brought for consult due to Herpangina is characterized by sudden onset of fever, TIMOTHY TANG MIDTERM 1
a 2 day history of fever. Patient was also noted to sore throat, dysphagia, and lesions in the posterior LEE SAY, MD EXAM - FEB
have decreased appetite and was irritable. There pharynx. Temperatures can range from normal to (TOP 4 - AUG 2014
was no accompanying cough, colds, or rashes. 41°C (106°F); fever tends to be greater in younger 2013 MED
Mother gave paracetamol to the patient which patients. Characteristic lesions, present on the BOARDS;
provided temporary lysis of fever. On physical anterior tonsillar pillars, soft palate, uvula, tonsils, TOPNOTCH MD)
examination, PR 110, RR 24, temp 39.5C. There posterior pharyngeal wall, and, occasionally, the
were noted multiple, small vesicles and ulcers that posterior buccal surfaces, are discrete 1–2 mm
are surrounded by erythematous rings on the soft vesicles and ulcers that enlarge over 2–3 days to 3–4
palate, uvula, and posterior pharyngeal wall. There mm and are surrounded by erythematous rings that
were no noted skin lesions, and cardiac, pulmonary, vary in size up to 10 mm. Typically about 5 lesions are
and abdominal examinations were unremarkable. present, with a range of 1 to >15. The remainder of the
What is the most common etiologic agent that pharynx appears normal or minimally erythematous.
causes this type of infection? Fever generally lasts 1–4 days, and resolution of
A. Coxsackie A virus symptoms occurs in 3–7 days. A variety of
B. Coxsackie B virus enteroviruses can cause Herpangina, although
C. Herpes simplex virus coxsackie A viruses are implicated most often.
D. Epstein-Barr virus
E. Cytomegalovirus
601 What congenital anomaly is associated with growth Trisomy 21 (down syndrome) is characetrized by the RACHELLE FINAL EXAM -
and mental retardation, abnormal facial features ff: hypotonia, flat face, upward and slanted palpebral MENDOZA, MD FEB 2013
like low set ears, hypotonia and an endocardial fissures and epicanthic folds, speckled irises (TOP 9 - AUG
cushion defect? (Brushfield sports);varying degrees of mental and 2012 MED
A. Velocardiofacial syndrome growth retardation;dysplasia of the pelvis, cardiac BOARDS;
B. Down syndrome malformations (endocardial cushion defect) and TOPNOTCH MD)
C. Turner syndrome simian crease;short, broad hands, hypoplasia of
D. VATER syndrome middle phalanx of 5th finger, duodenal atresia, and
high arched palate. (Nelson's 11th ed)
602 The immediate postnatal changes in a term Foramen ovale functionally closes immediately after RACHELLE FINAL EXAM -
newborn includes the following, EXCEPT birth. MENDOZA, MD FEB 2013
A. Decrease in pulmonary vascular resistance (TOP 9 - AUG
B. Decrease in right to left shunting via ductus 2012 MED
arteriosus BOARDS;
C. Increase in venous return to the left atrium TOPNOTCH MD)
D. Increase right to left shunting via foramen ovale
603 A 14-year old male is expected to have the 14-16 year old adolescents (middle adolescence) has RACHELLE FINAL EXAM -
following psychologic and mental characteristics, the following characteristics: emergence of abstract MENDOZA, MD FEB 2013
EXCEPT: thought (formal operations), may perceive future (TOP 9 - AUG
A. More stable body image implications, but may not apply in decision-making, 2012 MED
B. Intense peer group involvement questioning more, Concern with attractiveness, BOARDS;
C. Initiation of relationships and sexual activity Increasing introspection, Conflicts over control and TOPNOTCH MD)
D. Emergence of abstract thought (formal independence, Struggle for acceptance of greater
operations) autonomy, Intense peer group involvement,
E. No exception Preoccupation with peer culture, Peers provide
behavioral example, Testing ability to attract partner,
Initiation of relationships and sexual activity,
Questions of sexual orientation, Gauging skills and
opportunities. (Nelson's 11th ed)
604 The following genetic conditions predispose a child The following are genetic conditions that may RACHELLE FINAL EXAM -
to develop leukemia, EXCEPT: predispose a child to develop leukemia: Down MENDOZA, MD FEB 2013
A. Down syndrome syndrome, Fanconi syndrome, Bloom syndrome, (TOP 9 - AUG
B. Fanconi syndrome Diamond-Blackfan anemia, Schwachman syndrome, 2012 MED
C. Severe combined immune deficiency Klinefelter syndrome, Turner syndrome, BOARDS;
D. Klinefelter syndrome Neurofibromatosis type 1, Ataxia-telangiectasia, TOPNOTCH MD)
E. None of the above. Severe combined immune deficiency, Paroxysmal

TOPNOTCH MEDICAL BOARD PREP PEDIATRICS SUPEREXAM Page 82 of 97


For inquiries visit www.topnotchboardprep.com.ph or email us at topnotchmedicalboardprep@gmail.com
TOPNOTCH MEDICAL BOARD PREP PEDIATRICS SUPEREXAM
For inquiries visit www.topnotchboardprep.com.ph or email us at topnotchmedicalboardprep@gmail.com
Item QUESTION EXPLANATION AUTHOR TOPNOTCH
# EXAM
nocturnal hemoglobinuria, Li-Fraumeni syndrome
(Nelson's 11th ed)

605 The following conditions will give rise to a state of Congenital adrenal hyperplasia has several forms. RACHELLE FINAL EXAM -
hyponatremia, EXCEPT: The most common form (21 b-hydroxylase deficiency) MENDOZA, MD FEB 2013
A. SIADH leads to increased mineralocorticoid synthesis, (TOP 9 - AUG
B. Acute Renal Failure leading to sodium retention and hypertension. 2012 MED
C. Congenital Adrenal Hyperplasia BOARDS;
D. Cystic Fibrosis of the Pancreas TOPNOTCH MD)
E. None of the above

606 A 7 year old male was noted to have fever of five In dengue fever, pancytopenia may occur after the 3–4 RACHELLE FINAL EXAM -
days duration, with temperature ranging from 39 to days of illness. Neutropenia may persist or reappear MENDOZA, MD FEB 2013
40 C . It is accompanied by frontal headache, joint during the latter stage of the disease and may (TOP 9 - AUG
pains and generalized malaise. Two days after fever continue into convalescence with white blood cell 2012 MED
has resolved, maculopapular erythematous rashes counts of <2,000/mm3. Platelets rarely fall below BOARDS;
appear at the lower and upper extremities in glove 100,000/mm3. Venous clotting, bleeding and TOPNOTCH MD)
and stocking pattern. The following describes the prothrombin times, and plasma fibrinogen values are
possible laboratory findings, EXCEPT: within normal ranges. The tourniquet test result may
A. Neutropenia be positive. Mild acidosis, hemoconcentration,
B. Low platelet count increased transaminase values, and hypoproteinemia
C. Hemoconcentration may occur during some primary dengue virus
D. Prolonged PT and PTT infections. The electrocardiogram may show sinus
E. Increased transaminase values bradycardia, ectopic ventricular foci, flattened T
waves, and prolongation of the P-R interval.
607 A 3-year old male was brought to the OPD due to Hemolytic uremic syndrome (HUS) is characterized by RACHELLE FINAL EXAM -
oliguria, pallor, lethargy, edema and generalized the acute onset of microangiopathic hemolytic anemia, MENDOZA, MD FEB 2013
weakness 2 weeks after an episode of bloody renal injury, and a low platelet count. The following (TOP 9 - AUG
diarrhea. Laboratory exams revealed hemoglobin are both present at some time during the illness: 1. 2012 MED
is 6 mg/dl, platelet is 35,000/mm3, creatinine is anemia (acute onset) with microangiopathic changes BOARDS;
2mg/dl, hematuria and proteinuria. Blood (i.e., schistocytes, burr cells, or helmet cells) on TOPNOTCH MD)
peripheral smear revealed segmented RBCs, peripheral blood smear; and 2. renal injury (acute
Coomb's test is negative. The most likely diagnosis onset) evidenced by either hematuria, proteinuria, or
is: elevated creatinine level (i.e., ≥1.0 mg/dL in a child
A. Hemolytic-Uremic Syndrome younger than 13 yr or ≥1.5 mg/dL in a person 13 yr or
B. Idiopathic Thrombocytopenic Purpura older or ≥50% increase over baseline).
C. Thrombotic Thrombocytopenic Purpura
D. Neprotic-Nephritic Syndrome
E. Acute Post-Streptococcal Glomerulonephritis
608 A 15-year old female complained of pharyngitis, Acute meningococcemia initially may mimic viral RACHELLE FINAL EXAM -
fever, muscle pain, body weakness, vomiting, illness with pharyngitis, fever, myalgias, weakness, MENDOZA, MD FEB 2013
watery diarrhea and headache. She was given vomiting, diarrhea, and/or headache. A (TOP 9 - AUG
supportive medications by her mother. After 3 maculopapular rash is evident in about 7% of cases, 2012 MED
days, she became dyspneic, lethargic and irritable. typically before more serious signs develop. Limb BOARDS;
In the ER, she was found to be hypotensive and pain, myalgias, or refusal to walk occur in many cases TOPNOTCH MD)
there was note of several purpuric rashes all over and is the primary complaint in 7% of otherwise
body.She then had a generalized seizure, lost clinically unsuspected cases. Cold hands or feet and
consiousness and proceeded to coma. The most abnormal skin color are also early signs. In fulminant
likely diagnosis is: meningococcemia cases, the disease progresses
A. Henoch-Schönlein purpura rapidly over hours to septic shock characterized by
B. Toxic shock syndrome prominent petechiae and purpura (purpura
C. Meningococcemia fulminans), hypotension, DIC, acidosis, adrenal
D. Idiopathic thrombocytopenic purpura hemorrhage, renal failure, myocardial failure, and
E. Subsclerosing panencephalitis coma
609 An 8-year old male had remittent fever, with Tmax The patient has kawasaki disease. In addition to fever, RACHELLE FINAL EXAM -
at 40 C, accompanied by maculopapular rashes with the five characteristic features of Kawasaki disease MENDOZA, MD FEB 2013
accentuation on the groin. He was brought to a are: bilateral bulbar conjunctival injection, usually (TOP 9 - AUG
pediatrician and was diagnosed with measles. He without exudate; erythema of the oral and pharyngeal 2012 MED
was advised Vit.A supplementation and supportive mucosa with strawberry tongue and dry, cracked lips, BOARDS;
treatment. Fever, however, persisted for 6 more and without ulceration; edema and erythema of the TOPNOTCH MD)
days. On physical exam, the patient developed hands and feet; rash of various forms (maculopapular,
bilateral bulbar conjunctival injection without erythema multiforme, or scarlatiniform) with
exudate, erythema of the oral and pharyngeal accentuation in the groin area; and nonsuppurative
mucosa with strawberry tongue and dry, cracked cervical lymphadenopathy, usually unilateral, with
lips, edema and erythema of the hands and feet, still node size of ≥1.5 cm. Perineal desquamation is
with maculopapular rash and unilateral cervical common in the acute phase. Periungual desquamation
lymphadenopathy. There was periungual of the fingers and toes begins 1–3 wk after the onset of
desquamation of the fingers and toes. What is the illness and may progress to involve the entire hand
most appropriate treatment for this patient? and foot. Treatment involves high dose aspirin and
A. IV fluids with electrolyte replacement IVIG.
B. Aminoglycosides
C. 3rd cephalosporin
D. Intravenous immunoglobulin
E. Supportive treatment
610 A 1-year old old male comes in for a well-baby visit. The missed dose of hepatitis B may be given along RACHELLE FINAL EXAM -
He had BCG, 3 doses of DPT & OPV and 2 doses of with measles vaccine. MENDOZA, MD FEB 2013
hepatitis B vaccine. What can you give him for this (TOP 9 - AUG
particular visit? 2012 MED
A. Measles and 3rd dose of hepatitis vaccines BOARDS;
B. Measles vaccine TOPNOTCH MD)
C. 3rd dose of hepatitis B
D. Varicella vaccine
TOPNOTCH MEDICAL BOARD PREP PEDIATRICS SUPEREXAM Page 83 of 97
For inquiries visit www.topnotchboardprep.com.ph or email us at topnotchmedicalboardprep@gmail.com
TOPNOTCH MEDICAL BOARD PREP PEDIATRICS SUPEREXAM
For inquiries visit www.topnotchboardprep.com.ph or email us at topnotchmedicalboardprep@gmail.com
Item QUESTION EXPLANATION AUTHOR TOPNOTCH
# EXAM
611 A woman was noted to have a large volume of Down syndrome occurs in 20–30% of patients with RACHELLE FINAL EXAM -
amniotic fluid at the time of her delivery of her duodenal atresia. Other congenital anomalies that are MENDOZA, MD FEB 2013
child. At 6 hours of age, her baby begins associated with duodenal atresia include malrotation (TOP 9 - AUG
regurgitating small amounts of mucus and bile- (20%), esophageal atresia (10–20%), congenital heart 2012 MED
stained fluid. P.E. is normal. Abdominal x-ray disease (10–15%), and anorectal and renal anomalies BOARDS;
obtained showed a “double-bubble” sign. This (5%). TOPNOTCH MD)
condition is most commonly associated with:
A. Edward syndrome
B. Patau syndrome
C. Down syndrome
D. Li-Fraumeni syndrome
E. Fragile X syndrome
612 A preterm infant, 2-hr old, was observed to have Administration of betamethasone to women 48 hr RACHELLE FINAL EXAM -
shallow breathing at 65 breaths/minute, with before the delivery of fetuses between 24 and 34 wk MENDOZA, MD FEB 2013
grunting and cyanosis. CXR shows fine reticular of gestation significantly reduces the incidence, (TOP 9 - AUG
granularity of the parenchyma and air mortality, and morbidity of RDS. Administration of a 2012 MED
bronchograms. Which of the following should have 1st dose of surfactant into the trachea of symptomatic BOARDS;
been done to prevent this condition? premature infants immediately after birth is also an TOPNOTCH MD)
A. Administration of betamethasone to the mother effcetive preventive measure. In timing cesarean
48 hr before delivery section or induction of labor, estimation of fetal head
B. Administration of 1st dose of surfactant into the circumference by ultrasonography and determination
trachea of premature infant immediately after birth of the lecithin concentration in amniotic fluid by the
C. Determination of lecithin: sphingomyelin ratio in lecithin: sphingomyelin ratio decrease the likelihood
the amniotic fluid of delivering a premature infant.
D. All of the above
E. A and C only
613 The WHO recommendation states that breastmilk After 6 months of age, complementary feeding should RACHELLE FINAL EXAM -
should be the infant’s sole source of nutrition up to be started in order to achieve complete nutrition. MENDOZA, MD FEB 2013
about: (TOP 9 - AUG
A. 2 months of age 2012 MED
B. 6 months of age BOARDS;
C. 12 months of age TOPNOTCH MD)
D. 2 years of age
E. 3 years of age

614 A newborn was delivered from a 32 year old G3P3 APGAR scoring is as follows (0, 1, 2) RACHELLE FINAL EXAM -
(3003). Within sixty seconds after delivery, he was Heart rate: Absent, Below 100, Over 100 MENDOZA, MD FEB 2013
crying loudly, coughing upon sunctioning, with pink Respiratory effort: Absent; Slow, irregular; Good, (TOP 9 - AUG
body and blue extremities. There was some flexion crying 2012 MED
of extremities and the heart rate was 120 Muscle tone: Limp; Some flexion of extremities; Active BOARDS;
beats/min. What is the APGAR score? motion TOPNOTCH MD)
A. 6 Response to catheter in nostril (tested after
B. 7 oropharynx is clear): No response; Grimace; Cough or
C. 8 sneeze
D. 9 Color: Blue, pale; Body pink, extremities blue;
E. Cannot be determined Completely pink
615 A 4-year old boy was brought to your clinic because Definition of a positive tuberculin skin test for RACHELLE FINAL EXAM -
he was exposed to his grandmother who was children varies according to risk. INDURATION ≥5 MENDOZA, MD FEB 2013
coughing out blood for two months already. He has MM: 1.Children in close contact with known or (TOP 9 - AUG
good weight gain and appetite, and was generally suspected contagious people with tuberculosis 2012 MED
asymptomatic. A Mantoux tuberculin test. Which of disease. 2.Children suspected to have tuberculosis BOARDS;
the following should be the minimum size of disease: Findings on chest radiograph consistent with TOPNOTCH MD)
induration for this child to be positive for TB active or previously tuberculosis disease and clinical
infection? evidence of tuberculosis disease. 3.Children receiving
A. 5 mm immunosuppressive therapy or with
B. 10 mm immunosuppressive conditions, including HIV
C. 15 mm infection.
D. 20 mm INDURATION ≥10 MM: Children at increased risk of
E. 25 mm disseminated tuberculosis disease: 1.Children younger
than 4 yr of age. 2.Children with other medical
conditions, including Hodgkin disease, lymphoma,
diabetes mellitus, chronic renal failure, or
malnutrition. 3. Children born in high-prevalence
regions of the world. 4.Children frequently exposed to
adults who are HIV infected, homeless, users of illicit
drugs, residents of nursing homes, incarcerated or
institutionalized, or migrant farm workers. 5.Children
who travel to high-prevalence regions of the world.
INDURATION ≥15 MM: Children 4 yr of age or older
without any risk factors.
616 A 12-year old male with diagnosed with Anaphylactic shock is an example of distributive RACHELLE FINAL EXAM -
streptococcal pharyngitis and amoxicillin was shock, which is caused by abnormalities of vasomotor MENDOZA, MD FEB 2013
started. One hour after drug intake, he complained tone. There will be loss of venous capcitance (TOP 9 - AUG
of tingling sensation around his mouth, difficulty of decreases preload, loss of arterial capcitance, leading 2012 MED
swallowing and developed hoarse voice. At the ER, to decreased afterload or systemic blood pressure. BOARDS;
there was generalized urticaria, puffy eyelids, TOPNOTCH MD)
PR=130/min, RR=32/min, BP=70/30 and T=
37.2˚C.The patient is experiencing this type of
shock:
A. Cardiogenic
B. Hypovolemic
C. Distributive
D. Septic
E. Obstructive

TOPNOTCH MEDICAL BOARD PREP PEDIATRICS SUPEREXAM Page 84 of 97


For inquiries visit www.topnotchboardprep.com.ph or email us at topnotchmedicalboardprep@gmail.com
TOPNOTCH MEDICAL BOARD PREP PEDIATRICS SUPEREXAM
For inquiries visit www.topnotchboardprep.com.ph or email us at topnotchmedicalboardprep@gmail.com
Item QUESTION EXPLANATION AUTHOR TOPNOTCH
# EXAM
617 Which of the following is the most important The critical information for appropriate treatment of RACHELLE FINAL EXAM -
procedure in the diagnosis of infective infective endocarditis is obtained from blood cultures. MENDOZA, MD FEB 2013
endocarditis? All other laboratory data are secondary in importance. (TOP 9 - AUG
A. Complete blood count (Nelson's, 11th ed) 2012 MED
B. Echocardiography BOARDS;
C. Erythrocyte sedimentation rate TOPNOTCH MD)
D. Blood culture
E. Rheumatoid factor
618 A 9-month old male came to your clinic for Infants born to HBsAg-positive women should receive RACHELLE FINAL EXAM -
scheduled follow up. He was born from a HBsAg vaccine at birth, 1–2 mo, and 6 mo of age, with the first MENDOZA, MD FEB 2013
positive mother, and already received HBIg and dose accompanied with HBIG. Postvaccination testing (TOP 9 - AUG
complete hepatitis B vaccination (3 doses). You for HBsAg and anti-HBs should be done at 9–18 mo. If 2012 MED
obtained blood for immunologic studies. Result is the result is positive for anti-HBs, the child is immune BOARDS;
negative for both HBsAg and anti-HBs. What should to HBV. If the result is positive for HBsAg only, the TOPNOTCH MD)
be your next step? parent should be counseled and the child evaluated by
A. Counsel parents and refer to pediatric a pediatric gastroenterologist. If the result is negative
gastroenterologist for both HBsAg and anti-HBs, a 2nd complete hepatitis
B. Give 4th dose of hepatitis B vaccine B vaccine series should be administered, followed by
C. Start lamivudine therapy testing for anti-HBs to determine if subsequent doses
D. Start a 2nd complete hepa B vaccine series are needed. Administration of four doses of vaccine is
E. Repeat blood exam after 3 months permissible when combination vaccines are used after
the birth dose; this does not increase vaccine
reactogenicity. (Nelson's 11th ed)
619 A 3-year old female presents with fever, difficulty Viruses most commonly cause croup, the most RACHELLE FINAL EXAM -
breathing and barking cough. The symptoms are common form of acute upper respiratory obstruction. MENDOZA, MD FEB 2013
more apparent when the patient is lying down and Most patients have an upper respiratory tract (TOP 9 - AUG
about to sleep. On physical exam, there is apparent infection with some combination of rhinorrhea, 2012 MED
respiratory distress, tachypnea, alar flaring, pharyngitis, mild cough, and low-grade fever for 1–3 BOARDS;
inspiratory stridor and fever at 38.3oC. The most days before the signs and symptoms of upper airway TOPNOTCH MD)
likely diagnosis is: obstruction become apparent. The child then develops
A. Laryngotracheobronchitis the characteristic “barking” cough, hoarseness, and
B. Epiglottitis inspiratory stridor. The low-grade fever may persist,
C. Bacterial tracheitis although temperatures may reach 39–40°C (102.2–
D. Retropharyngeal abscess 104°F); some children are afebrile. Symptoms are
E. Acute bronchitis characteristically worse at night and often recur with
decreasing intensity for several days and resolve
completely within a wk.
620 Which of the following is NOT true in Nephrotic Because of renal protein loss and hypoalbuminemia, RACHELLE FINAL EXAM -
Syndrome? there is reduced intravascular volume. This MENDOZA, MD FEB 2013
A. Elevated serum cholesterol stimulates the rennin – angiotensisn system, resulting (TOP 9 - AUG
B. 85% is due to minimal change disease in enhanced renal sodium reabsorption. Eventually, 2012 MED
C. Reduced sodium reabsorption by the kidney the sodium partially contributes to the edema. The BOARDS;
D. Elevated triglycerides four symptoms of nephrotic syndrome is proteinuria, TOPNOTCH MD)
E. None of the above hypoalbuminemia, hyperlipidemia, edema.

621 A male infant was born at 32 weeks AOG via VON ANDRE DIAGNOSTIC
cesarian section because of bleeding from placenta MEDINA, MD EXAM - AUG
previa. Soon after birth, he developed respiratory (TOP 4 - FEB 2012
distress requiring supplemental oxygen and 2012 MED
mechanical ventilation. Chest x-ray shows BOARDS;
decreased lung volumes and a diffuse ground glass TOPNOTCH MD)
pattern with air bronchograms. Which of the
following is the most likely diagnosis?
A. persistent pulmonary hypertension of the
newborn
B. deficient surfactant
C. fluid retention in the lungs
D. bronchopulmonary dysplasia
E. congenital heart disease
622 You are called to the newborn nursery to evaluate a Trisomy 18 VON ANDRE DIAGNOSTIC
1 day-old female infant with unusual physical MEDINA, MD EXAM - AUG
findings. On examination, you note that the (TOP 4 - FEB 2012
neonate's hands are clenched with overlapping 2012 MED
digits and her lower extremities are extended and BOARDS;
crossed. You also note the presence of a rocker TOPNOTCH MD)
bottom feet and delicate, small facial features.
Which of the following chromosomal abnormalities
is the most likely cause of the patient's features?
A. Trisomy 13
B. Trisomy 18
C. Trisomy 21
D. Deletion on chromosome 7
E. Absence of a region on paternally derived
chromosome 15
623 An 8 year old girl presents with sore throat, fever VON ANDRE DIAGNOSTIC
and a rough sandpaper-like rash over her trunk and MEDINA, MD EXAM - AUG
extremities. A throat culture is positive for group A (TOP 4 - FEB 2012
beta hemolytic streptococcus. Treatment of her 2012 MED
infection with antibiotics will prevent which of the BOARDS;
following complications? TOPNOTCH MD)
A. Reactive arthritis
B. Rheumatic Fever
C. Pneumonia
D. RPGN
E. Meningitis

TOPNOTCH MEDICAL BOARD PREP PEDIATRICS SUPEREXAM Page 85 of 97


For inquiries visit www.topnotchboardprep.com.ph or email us at topnotchmedicalboardprep@gmail.com
TOPNOTCH MEDICAL BOARD PREP PEDIATRICS SUPEREXAM
For inquiries visit www.topnotchboardprep.com.ph or email us at topnotchmedicalboardprep@gmail.com
Item QUESTION EXPLANATION AUTHOR TOPNOTCH
# EXAM
624 An 18 year old boy, college sophomore, presents Vitamin A is proven as a beneficial supportive VON ANDRE DIAGNOSTIC
with high fever, headache, cough, conjunctivitis and treatmennt in measles. Koplik spots are transient and MEDINA, MD EXAM - AUG
a diffuse macular rash over his trunk and face. He is by the time the rash is present it can be no longer (TOP 4 - FEB 2012
unsure of his immunization status. You suspect appreciated. Bacterial pneumonia is the MC of 2012 MED
measles infection. Which of the following is correct moratality. BOARDS;
regarding this diagnosis? TOPNOTCH MD)
A. Koplik spots would likely be present upon
examination
B. Vitamin A may improve his outcome
C. Mortality is most commonly caused by measles
encephalitis
D. Diagnosis is based on culture and direct
fluorescent antigen testing
E. Corticosteroids will decrease sympotoms and
improve outcome
625 You are called to the nursery to evaluate a male Tricuspid atresia is the only cause of cyanosis in the VON ANDRE DIAGNOSTIC
newborn with cyanosis. On auscultation, you hear a newborn period that manifests with left axis deviation MEDINA, MD EXAM - AUG
single S2 but no murmur. Pulse oximetry shows an and left ventricular hypertrophy on ECG. (TOP 4 - FEB 2012
oxygen saturation of 72 % in room air. An 2012 MED
electrocardiogram reveals left axis deviation and BOARDS;
left ventricular hypertrophy. What is his likely TOPNOTCH MD)
diagnosis?
A. Tetralogy of Fallot
B. Transposition of great arteries
C. Truncus arteriosus
D. Total anomalous pulmonary venous
connection
E. Tricuspid atresia with intact ventricular
septum
626 A 3 year old boy is brought to the office by his This patients constipation is most likely functional or VON ANDRE DIAGNOSTIC
parents who are concerned because he has hard, non-organic. Functional constipation results from an MEDINA, MD EXAM - AUG
painful stools. For the past 4 months, their son inappropriate constriction of external anal sphincter . (TOP 4 - FEB 2012
defecates evry 3-4 days and cries during stooling. Most commonly, toddlers retain stool PURPOSELY 2012 MED
The resulting stool is very hard. PE of the child is because oftraumatic event, such as PAINFUL diaper BOARDS;
nomal. Which of the ollowing is correct regarding rash, painful diarrhea, or even physical abuse. TOPNOTCH MD)
his constipation?
A. His constipation is unlikely to lead to
encopresis
B. A barium enema should be ordered to evaluate
for Hirschprung's disease
C. His constipation likely resulted from a
traumatic triggering event, such as severe, painful
diaper rash or painful diarrhea
D. Abdominal radiographs should be ordered to
evaluate for an underlying organic cause of his
constipation
E. The parents should be instructed to encourage
their son to drink juice with each meal, and the boy
should be reevaluated in 3-4 months
627 A 5 week old infant has been brought to the clinic this patient has cholestasis, or retention of bile in the VON ANDRE DIAGNOSTIC
due to jaundice. He was born full term, weighing 8 liver, manifested by elevated conjugated (direct MEDINA, MD EXAM - AUG
pounds and 9 ounces and he had an uncomplicated bilirubin).in addition, the findings of hepatomegaly (TOP 4 - FEB 2012
delivery and neonatal course. Today, his weight is 9 and poor growth support the diagnosis. MC causes of 2012 MED
pounds, 1 ounce, and his vital signs are normal. On cholestatsis include biliary atresia and choledochal BOARDS;
examination, his liver is enlarged and 4 cm below cyst. TOPNOTCH MD)
the right costal margin. jaundice is present.
Laboratory evaluation reveals a total bilirubin of
12.9 mg/dL with a direct component of 5.9 mg/dL.
Which of the following is the next most appropriate
next step?
A. begin phototherapy to treat jaundice
B. refer the patient for a liver transplant
C. begin ursodeoxhycholic acid to enhance bile
flow
D. order an urgent abdominal ultrasound and
radionucleotide imaging study of the liver
E. reassure the parents that no treatment is
required as he likely has neonatal hepatitis
628 A previously well 8 month old infant has had a bronchiolitis VON ANDRE DIAGNOSTIC
runny nose and has been sneezing and coughing for MEDINA, MD EXAM - AUG
2 days. 5 hours ago, his cough became worse. On (TOP 4 - FEB 2012
P.E, he is in moderate respiratory distress with 2012 MED
nasal flaring, hyperexpansion of the chest, and BOARDS;
easily audible wheezing without rales. What is the TOPNOTCH MD)
most likely diagnosis?
A. Bronchial asthma
B. Bronchiolitis
C. Viral croup
D. Epiglottitis
E. Diphtheria

TOPNOTCH MEDICAL BOARD PREP PEDIATRICS SUPEREXAM Page 86 of 97


For inquiries visit www.topnotchboardprep.com.ph or email us at topnotchmedicalboardprep@gmail.com
TOPNOTCH MEDICAL BOARD PREP PEDIATRICS SUPEREXAM
For inquiries visit www.topnotchboardprep.com.ph or email us at topnotchmedicalboardprep@gmail.com
Item QUESTION EXPLANATION AUTHOR TOPNOTCH
# EXAM
629 A deficiency of this trace element is associated with Zinc VON ANDRE DIAGNOSTIC
skin ulcers, reduced immune response and MEDINA, MD EXAM - AUG
hypogonadal dwarfism: (TOP 4 - FEB 2012
A. Chromium 2012 MED
B. Cobalt BOARDS;
C. Iron TOPNOTCH MD)
D. Magnesium
E. Zinc

630 Early diagnosis of cerebral palsy is important VON ANDRE DIAGNOSTIC


because it permits: MEDINA, MD EXAM - AUG
A. Genetic counseling to prevent subsequent cases (TOP 4 - FEB 2012
B. Treatment of underlying lesion and prevention 2012 MED
of progression BOARDS;
C. Guidance that may minimize or prevent TOPNOTCH MD)
secondary physical and emotional problems
D. Prophylactic anticonvulsant treatment before
onset of seizures
E. Avoidance of unrealistic expectations
631 The highest death rate in the pediatric age group Mortality is highest in the first year of life VON ANDRE DIAGNOSTIC
occurs in: MEDINA, MD EXAM - AUG
A. First month of life (TOP 4 - FEB 2012
B. Between 2 and 12 months 2012 MED
C. Between 2 and 4 years of life BOARDS;
D. Just before puberty TOPNOTCH MD)
E. During adolescence

632 Which of the following is most likely to occur as an Chorea can stand alone in the diagnosis of Acute VON ANDRE DIAGNOSTIC
isolated manifestation of acute rheumatic fever? rheumatic fever MEDINA, MD EXAM - AUG
A. Arthritis (TOP 4 - FEB 2012
B. Carditis 2012 MED
C. Chorea BOARDS;
D. Erythema marginatum TOPNOTCH MD)
E. Fever

633 Which malignancy is uncommon in children under Osteosarcoma is usually seen in puberty of eardly VON ANDRE DIAGNOSTIC
5 years old? adolescence period. MEDINA, MD EXAM - AUG
A. Retinoblastoma (TOP 4 - FEB 2012
B. Neuroblastoma 2012 MED
C. Osteosarcoma BOARDS;
D. Leukemia TOPNOTCH MD)
E. Wilm's tumor

634 Rectal prolapse in an infant associated with failure Cystic fibrosis in children with rectal prolapse and VON ANDRE DIAGNOSTIC
to thrive is highly suggestive of? failure to thrive is common. MEDINA, MD EXAM - AUG
A. Crohn's disease (TOP 4 - FEB 2012
B. Functional constipation 2012 MED
C. Ehler-Danlos syndrome BOARDS;
D. Intussusception of the colon TOPNOTCH MD)
E. Cystic Fibrosis

635 A 10 year old girl presents for evaluation of fatigue, This is a case of dermatomyositis. Calcinosis or VON ANDRE DIAGNOSTIC
diminished appetite, and weakness. On physical calcium deposition, in msucle, fascia and subcutanous MEDINA, MD EXAM - AUG
examination, a periorbital violaceous heliotrope tissue occurs in 70 % OF CASES. (TOP 4 - FEB 2012
rash is evident. Which of the following statements 2012 MED
is most accurate regarding the probable diagnosis? BOARDS;
A. Children with this diagnosis typically present TOPNOTCH MD)
with distal muscle weakness with an ascending
pattern
B. This patient has a 25 % likelihood of developing
a subsequent malignancy
C. Steroids are contraindicated
D. The clinical course may be complicated by
calcium deposition in the muscle, fascia, and
subcutaneous tissue
E. This patients's disease is more common in
males than in females
636 A 12 year old boy presents with severe arthritis of Late onset pauciarticular JRA is male predominant and VON ANDRE DIAGNOSTIC
the hips and sacroiliac joints. Laboratory studies is almost presents in children older than 8 years old. MEDINA, MD EXAM - AUG
reveal that the patient is HLA-B27 positive. Which (TOP 4 - FEB 2012
of the following is the most likely diagnosis? 2012 MED
A. Early onset pauciarticular juvenile rheumatoid BOARDS;
arthritis TOPNOTCH MD)
B. Late onset pauciarticular juvenile rheumatoid
arthritis
C. Rheumatoid factor-negative polyarticular JRA
D. Rheumatoid factor-positive polyarticular JRA
E. Systemic-onset JRA

TOPNOTCH MEDICAL BOARD PREP PEDIATRICS SUPEREXAM Page 87 of 97


For inquiries visit www.topnotchboardprep.com.ph or email us at topnotchmedicalboardprep@gmail.com
TOPNOTCH MEDICAL BOARD PREP PEDIATRICS SUPEREXAM
For inquiries visit www.topnotchboardprep.com.ph or email us at topnotchmedicalboardprep@gmail.com
Item QUESTION EXPLANATION AUTHOR TOPNOTCH
# EXAM
637 A 7 month old male infant is brought to your clinic Celiac disease may presnt with vomiting, bloating, VON ANDRE DIAGNOSTIC
with a 1 month long history of fussiness, foul-smelling stools, or failure to thrive. MEDINA, MD EXAM - AUG
intermittent vomiting, and nonbloody, foul- (TOP 4 - FEB 2012
smelling stools. He has not gained weight since his 2012 MED
6 month health maintenance examination. His diet BOARDS;
consists of formula and wheat cereal that was TOPNOTCH MD)
introduced at 6 months of age. Which of the
following is the most likely diagnosis?
A. lactase deficiency
B. celiac disease
C. crohn's disease
D. gastroesophageal reflux disease
E. cow's milk protein intolerance
638 When does physiologic anemia of TERM infants 8-12 weeks: TERM. 6-8 weeks: PRETERM VON ANDRE DIAGNOSTIC
occur? MEDINA, MD EXAM - AUG
A. 3-6 weeks of age (TOP 4 - FEB 2012
B. 6-8 weeks of age 2012 MED
C. 8-12 weeks of age BOARDS;
D. 12-16 weeks of age TOPNOTCH MD)
E. At birth

639 A newborn in the delivery room cried vigorously, APGAR of 9/10 VON ANDRE DIAGNOSTIC
with regular respirations, pinkish body and bluish MEDINA, MD EXAM - AUG
extremities, active, with a heart rate of 146/min. (TOP 4 - FEB 2012
What is the APGAR score? 2012 MED
A. 6 BOARDS;
B. 7 TOPNOTCH MD)
C. 8
D. 9
E. 10
640 A 2 year old patient was doagnosed to have a VON ANDRE DIAGNOSTIC
hydrocephalus. He was operated and MEDINA, MD EXAM - AUG
ventriculoperitoneal shunt was applied. After 1 (TOP 4 - FEB 2012
year, infection of the shunt was noted. What is the 2012 MED
most common causative organism for this kind of BOARDS;
infection? TOPNOTCH MD)
A. Streptococcus pyogenes
B. Staphylococcus epidermidis
C. Staphylococcus aureus
D. Pseudomonas aeruginosa
E. E. coli
641 A 4 year old boy was brought to ER due to VWD is the most common inherited bleeding disorder. LITO JAY DIAGNOSTIC
uncontrolled bleeding. Upon history, you learned Usual clinical picture is a child who undergone a MACARAIG, MD EXAM - AUG
that few hours PTC, patient had 2 of his incisors certain procedure and had uncontrollable bleeding. (TOP 8 - FEB 2013
extracted and bleeding was not controlled. Another clue is the prolonged bleeding time with 2013 MED
Hematologic exams were normal aside from normal platelet count. BOARDS;
prolong bleeding time. What is your primary TOPNOTCH MD)
consideration?
A. Von Willebrand Disease
B. Hemophilia A
C. Hemophilia B
D. DIC
E. None of the above
642 The ability to copy forms develops in a regular LITO JAY DIAGNOSTIC
order. Which of the following is the correct MACARAIG, MD EXAM - AUG
sequence? (TOP 8 - FEB 2013
A. Copy a square, a cross, a circle 2013 MED
B. Copy a square, a circle, a cross BOARDS;
C. Copy a cross, a circle, a square TOPNOTCH MD)
D. Copy a circle, a square, a cross
E. Copy a circle, a cross, a square
643 Which of the following would indicate Tanner stage LITO JAY DIAGNOSTIC
3 sexual development in a female child? MACARAIG, MD EXAM - AUG
A. Acne (TOP 8 - FEB 2013
B. Breast and papilla elevated as a small mound 2013 MED
C. Darkly pigmented, slightly curly pubic hair BOARDS;
D. Fine hair on upper lip TOPNOTCH MD)
E. Menstruation

644 An 8 month old child is expected to have what Age in months - 6 months = expected number of teeth LITO JAY DIAGNOSTIC
number of teeth/tooth? MACARAIG, MD EXAM - AUG
A. 1 (TOP 8 - FEB 2013
B. 2 2013 MED
C. 3 BOARDS;
D. 4 TOPNOTCH MD)
E. None

645 You went on a night duty as a ward resident-on- LITO JAY DIAGNOSTIC
duty and you saw an interesting case of an infant MACARAIG, MD EXAM - AUG
with low-set ears whose laboratory examinations (TOP 8 - FEB 2013
showed hypocalcemia, absence of a thymic shadow 2013 MED
on chest roentgenogram, and persistent BOARDS;
candidiasis. This is most probably a case of? TOPNOTCH MD)
A. an autoimmune disease
B. Di George sundrome
C. Idiopathic hypoparathyroidism
D. Renal failure
TOPNOTCH MEDICAL BOARD PREP PEDIATRICS SUPEREXAM Page 88 of 97
For inquiries visit www.topnotchboardprep.com.ph or email us at topnotchmedicalboardprep@gmail.com
TOPNOTCH MEDICAL BOARD PREP PEDIATRICS SUPEREXAM
For inquiries visit www.topnotchboardprep.com.ph or email us at topnotchmedicalboardprep@gmail.com
Item QUESTION EXPLANATION AUTHOR TOPNOTCH
# EXAM
E. Severe combined immunodeficiency

646 Which of the following organism is the most LITO JAY DIAGNOSTIC
frequent cause of neonatal meningitis? MACARAIG, MD EXAM - AUG
A. Group B strep (TOP 8 - FEB 2013
B. E. coli 2013 MED
C. Listeria monocytogenes BOARDS;
D. H. influenza B TOPNOTCH MD)
E. Strep pneumoniae

647 At your clinic in the province, a mother brought her the rasher after fever indicates Roseola, locally known LITO JAY DIAGNOSTIC
3 year old child with maculopapular rashes on as tigdas-hangin MACARAIG, MD EXAM - AUG
trunk and limbs. Mother told you that 2 days prior (TOP 8 - FEB 2013
the rash, her child had fever of 38-39 degrees 2013 MED
Centigrade. With this history, you know this is a BOARDS;
case of? TOPNOTCH MD)
A. Measles
B. German Measles
C. Roseola
D. Rubella
E. Rubeola
648 Another 3 year old patient was brought tou you due German Measles (rubella) LITO JAY DIAGNOSTIC
to rashes. This time, it appeared during the peak of MACARAIG, MD EXAM - AUG
fever spikes and your PE showed unilateral (TOP 8 - FEB 2013
posterior lymphadenopathy. What is your 2013 MED
diagnosis? BOARDS;
A. Roseola TOPNOTCH MD)
B. Rubeola
C. Rubella
D. Fifth disease
E. Infectious mononucleosis
649 Your last patient that day was a 7 year old female LITO JAY DIAGNOSTIC
who was also brought for consult due to rashes. MACARAIG, MD EXAM - AUG
This was accompanied by cough, coryza and red (TOP 8 - FEB 2013
eyes. Your initial impression should be? 2013 MED
A. Roseola BOARDS;
B. Rubeola TOPNOTCH MD)
C. Rubella
D. Fifth disease
E. Infectious mononucleosis
650 A 2 year old child was brought for consult due to LITO JAY DIAGNOSTIC
poor feeding. Mother told you that patient had been MACARAIG, MD EXAM - AUG
experiencing high grade fever for the past 6 days. (TOP 8 - FEB 2013
PE showed erythematous eyes, unilateral 2013 MED
lymphadenopathy, strawberry tongue, edematous BOARDS;
hands and feet with desquamating rash. What is the TOPNOTCH MD)
most appropriate treatment?
A. Immunoglobulins
B. IV hydrocortisone
C. Aspirin
D. A and B
E. A and C
651 A 16 year old male was brought to ER due to severe LITO JAY DIAGNOSTIC
abdominal pain and tea-colored urine. History MACARAIG, MD EXAM - AUG
revealed Viral pneumonia a week PTC. PE showed (TOP 8 - FEB 2013
macular rashes on lower extremities. What is your 2013 MED
impression? BOARDS;
A. Henoch Schonlein Purpura TOPNOTCH MD)
B. Post-strep Glomerulonephritis
C. IgA nephropathy
D. Berger's disease
E. Buerger's disease
652 Erythema toxicum is? LITO JAY DIAGNOSTIC
A. More common among term than premature MACARAIG, MD EXAM - AUG
infants (TOP 8 - FEB 2013
B. Usually associated with fever and a general 2013 MED
toxic state BOARDS;
C. Uncommon before the 5th day of life TOPNOTCH MD)
D. Usually associated with an elevated peripheral
WBC count
E. Manifested by a papulo-vesicular rash

TOPNOTCH MEDICAL BOARD PREP PEDIATRICS SUPEREXAM Page 89 of 97


For inquiries visit www.topnotchboardprep.com.ph or email us at topnotchmedicalboardprep@gmail.com
TOPNOTCH MEDICAL BOARD PREP PEDIATRICS SUPEREXAM
For inquiries visit www.topnotchboardprep.com.ph or email us at topnotchmedicalboardprep@gmail.com
Item QUESTION EXPLANATION AUTHOR TOPNOTCH
# EXAM
653 An infant born at 39 weeks of gestation and LITO JAY DIAGNOSTIC
weighing 2000 grams should be classified as? MACARAIG, MD EXAM - AUG
A. Low birth weight (TOP 8 - FEB 2013
B. premature 2013 MED
C. Small for gestational age BOARDS;
D. A and C TOPNOTCH MD)
E. B and C

654 The newborn with cytomegalovirus (CMV) LITO JAY DIAGNOSTIC


infection at birth most commonly presents with? MACARAIG, MD EXAM - AUG
A. hepatosplenomegaly (TOP 8 - FEB 2013
B. hepatitis 2013 MED
C. thrombocytopenia BOARDS;
D. Cerebral calcifications TOPNOTCH MD)
E. No symptoms

655 Factors that are more characteristic of childhood- LITO JAY DIAGNOSTIC
onset Asthma than adult-onset asthma? MACARAIG, MD EXAM - AUG
A. A ratio of 2:1 females to males (TOP 8 - FEB 2013
B. An asscociation with history of eczema 2013 MED
C. Normal levels of IgE BOARDS;
D. Lack of response to provocation with inhaled TOPNOTCH MD)
antigens
E. Lack of seasonal variation in severity of
symptoms
656 Hemolytic anemia in premature infants is being Hemolytic anemia= Vit. E. Hemorrhagic disease of the LITO JAY DIAGNOSTIC
linked to lack or deficiency of newborn=Vit. K. MACARAIG, MD EXAM - AUG
A. Vitamin C (TOP 8 - FEB 2013
B. Vitamin D 2013 MED
C. Vitamin E BOARDS;
D. Vitamin K TOPNOTCH MD)
E. Vitamin A

657 You are given a case of 2 year-old female suffering LITO JAY DIAGNOSTIC
from loose watery diarrhea of 5-6 episodes per day. MACARAIG, MD EXAM - AUG
Condition started just a day prior to consult and no (TOP 8 - FEB 2013
other symptoms like fever or vomiting noted. PE 2013 MED
showed weak-looking child, eager to drink, and skin BOARDS;
draws back slowly. According to IMCI, what is the TOPNOTCH MD)
best mangement?
A. Give probiotics
B. Give ORS
C. Start IV fluids at 75ml/kg
D. Observe
E. None of the above
658 According to IMCI, every measles patient or LITO JAY DIAGNOSTIC
suspected measles patient should be given MACARAIG, MD EXAM - AUG
A. Vitamin A (TOP 8 - FEB 2013
B. Pyridoxine 2013 MED
C. Vitamin C BOARDS;
D. Zinc TOPNOTCH MD)
E. Active immunization

659 The most common Nephrotic Syndrome in children LITO JAY DIAGNOSTIC
is? MACARAIG, MD EXAM - AUG
A. Lipoid Nephrosis (TOP 8 - FEB 2013
B. Minimal change disease 2013 MED
C. Membranoproliferative glomerulonephritis BOARDS;
D. A or B TOPNOTCH MD)
E. A or C

660 A 5 year old female was brought to your clinic due LITO JAY DIAGNOSTIC
to rashes on vulval area. Patient's mother noticed MACARAIG, MD EXAM - AUG
her to keep on scratching her pudendum especially (TOP 8 - FEB 2013
at night. What is your differential diagnosis? 2013 MED
A. pinworm BOARDS;
B. Enterobius infection TOPNOTCH MD)
C. candidiasis
D. A or B
E. None of the above
661 A newborn infant is noted to have acrocyanosis, HR The Apgar score is determined by evaluating the HAZEL KAREN MIDTERM 2 -
of 90/min, weak passive tone, with active newborn baby on five simple criteria on a scale from RAZ, MD (TOP 6 - AUG 2013
withdrawal and good cry. What is the apgar score? zero to two, then summing up the five values thus FEB 2013 MED
A. 5 obtained. The resulting Apgar score ranges from zero BOARDS;
B. 6 to 10. The five criteria are summarized using words TOPNOTCH MD)
C. 7 chosen to form an
D. 8 acronym (Appearance, Pulse, Grimace, Activity, Respir
E. 9 ation).

662 Described as a lacy, reticulated vascular pattern When a newborn infant is exposed to low HAZEL KAREN MIDTERM 2 -
over most of the body when the the baby is cooled? environmental temperatures, an evanescent, lacy, RAZ, MD (TOP 6 - AUG 2013
A. Milia reticulated red and/or blue cutaneous vascular FEB 2013 MED
B. Erythema toxicum pattern appears over most of the body surface. This BOARDS;
C. Hemangioma vascular change represents an accentuated TOPNOTCH MD)
D. Cutis marmorata physiologic vasomotor response that disappears with
E. Nevus sebaceous increasing age, although it is sometimes discernible
even in older children.

TOPNOTCH MEDICAL BOARD PREP PEDIATRICS SUPEREXAM Page 90 of 97


For inquiries visit www.topnotchboardprep.com.ph or email us at topnotchmedicalboardprep@gmail.com
TOPNOTCH MEDICAL BOARD PREP PEDIATRICS SUPEREXAM
For inquiries visit www.topnotchboardprep.com.ph or email us at topnotchmedicalboardprep@gmail.com
Item QUESTION EXPLANATION AUTHOR TOPNOTCH
# EXAM
663 Included in newborn screening in the Philippines, Disorders screened in the Philippines includes the HAZEL KAREN MIDTERM 2 -
except? above plus G6PD Deficiency and congenital RAZ, MD (TOP 6 - AUG 2013
A. MSUD hypothyroidism. Newborn screening is ideally done FEB 2013 MED
B. CAH on the 48th - 72nd hour of life. However, it may also BOARDS;
C. PKU be done after 24 hours from birth. TOPNOTCH MD)
D. Galactosemia
E. None of the above

664 True of physiologuc jaundice, except? Physiologic jaundice appears on the 2nd -3rd DOL and HAZEL KAREN MIDTERM 2 -
A. Appears on the 1st DOL peaks at 5th DOL. RAZ, MD (TOP 6 - AUG 2013
B. Peak bilirubin 10 - 12 mg/dL (term) FEB 2013 MED
C. Rate of bilirubin rise < 5mg/dL BOARDS;
D. Treatment is exposure to UV light TOPNOTCH MD)
E. None

665 Live attenuated vaccines except? Salk polio vaccine is inactivated whole virus, while HAZEL KAREN MIDTERM 2 -
A. MMR Sabin polio vaccine is live attenuated. RAZ, MD (TOP 6 - AUG 2013
B. BCG FEB 2013 MED
C. Yellow fever BOARDS;
D. Polio Salk TOPNOTCH MD)
E. Influenza

666 Which of the following is/are contraindications for Contraindications for vaccination includes fever, HAZEL KAREN MIDTERM 2 -
immunization? moderate - serious illness, egg allergy. RAZ, MD (TOP 6 - AUG 2013
A. Mild acute illness in an otherwise well child FEB 2013 MED
B. Prematurity BOARDS;
C. Allergy to egg protein TOPNOTCH MD)
D. A and b
E. AOTA

667 How should Hepatitis B vaccine be given for routine Hepatitis B vaccine is given at birth and a total of 3 HAZEL KAREN MIDTERM 2 -
vaccination? doses os given by 18 mos of age. All children not RAZ, MD (TOP 6 - AUG 2013
A. Infant of Hepa B positive mother should receive immunized or does not have booster, must be started FEB 2013 MED
1st doe of HBV plus Hepa B Ig at 2 different sites on the regimen. BOARDS;
within 12 hours of birth TOPNOTCH MD)
B. Total of 6 does by 18 mos of age
C. all children not immunized need not begin
vaccination due to acquired immunity
D. b and c
E. none
668 Infant presented with barking cough, hoarseness Croup (or laryngotracheobronchitis) is a respiratory HAZEL KAREN MIDTERM 2 -
and inspiratory stridor which was relieved by condition that is usually triggered by an acute viral RAZ, MD (TOP 6 - AUG 2013
administration of nebulized epinephrine. What is infection of the upper airway, usually parainfluenza FEB 2013 MED
the causative agent? The infection leads to swelling inside the throat, which BOARDS;
A. RSV interferes with normal breathing and produces the TOPNOTCH MD)
B. H. influenzae type b classical symptoms of a "barking" cough, stridor, and
C. Parainfluenza virus hoarseness.
D. Influenza
E. Strep. pneumoniae
669 A 2 year old child presents to the ER with high Bacterial tracheitis is a bacterial infection of HAZEL KAREN MIDTERM 2 -
fever, respiratory distress and brassy cough. CXR the trachea and is capable of producing airway RAZ, MD (TOP 6 - AUG 2013
was done showing subglottic narrowing with obstruction.One of the most common causes FEB 2013 MED
ragged tracheal air column. Diagnosis? is Staphylococcus aureus and often follows a recent BOARDS;
A. Croup viral upper respiratory infection. It is the most TOPNOTCH MD)
B. Laryngomalacia serious in young children, possibly because of the
C. Laryngitis relatively small size of the trachea that gets easily
D. Bronchitis blocked by swelling. The most frequent sign is the
E. Bacterial tracheitis rapid development of stridor.
670 Most common causative agent of external otitis? Otitis externa (also known as "External otitis" and HAZEL KAREN MIDTERM 2 -
A. S. aureus "Swimmer's ear") is an inflammation of the outer ear RAZ, MD (TOP 6 - AUG 2013
B. S.pneumoniae and ear canal. Most commonly caused by FEB 2013 MED
C. Parainfluenza Pseudomonas aeruginosa. BOARDS;
D. Pseudomonas TOPNOTCH MD)
E. H. influenza type b

671 True of murmurs in children? Innocent murmurs in children are physiologic, usually HAZEL KAREN MIDTERM 2 -
A. Absence of murmurs rules out congenital heart grade 2/6 or less. Absence of murmurs does not rule RAZ, MD (TOP 6 - AUG 2013
defects out CHD. All diastolic murmurs are pathologic. FEB 2013 MED
B. All diastolic murmurs are pathologic BOARDS;
C. Physiologic murmurs have a grade of 3/6 or TOPNOTCH MD)
greater
D. A and c only
E. All of the above
672 True of sources of Infective endocarditis and its Streptococcus viridans is the most common cause of HAZEL KAREN MIDTERM 2 -
causative agent, except? infective endocarditis after dental procedures. RAZ, MD (TOP 6 - AUG 2013
A. S. mutans : dental procedures FEB 2013 MED
B. Fungi : open heart surgery BOARDS;
C. Staph. epidermidis : indwelling catheter TOPNOTCH MD)
D. S. aureus : previously normal heart
E. None

TOPNOTCH MEDICAL BOARD PREP PEDIATRICS SUPEREXAM Page 91 of 97


For inquiries visit www.topnotchboardprep.com.ph or email us at topnotchmedicalboardprep@gmail.com
TOPNOTCH MEDICAL BOARD PREP PEDIATRICS SUPEREXAM
For inquiries visit www.topnotchboardprep.com.ph or email us at topnotchmedicalboardprep@gmail.com
Item QUESTION EXPLANATION AUTHOR TOPNOTCH
# EXAM
673 Patient presented with conjunctivitis, Kawasaki disease s an autoimmune disease in which HAZEL KAREN MIDTERM 2 -
polymorphous exanthems, palpale lymph nodes > the medium-sized blood vessels throughout the body RAZ, MD (TOP 6 - AUG 2013
1.5 cm diameter, strawberry tongue and swelling of become inflamed. It is largely seen in children under FEB 2013 MED
hands, diagnosis? five years of age. It affects many organ systems, BOARDS;
A. Scarlet fever mainly those including the blood vessels, skin, mucous TOPNOTCH MD)
B. SLE membranes, and lymph nodes; however its rare but
C. Kawasaki most serious effect is on the heart where it can cause
D. Henoch - Scholein Purpura fatal coronary artery aneurysms in untreated children.
E. Rheumatic fever
674 An 8 year old boy with a history of sore throat is Henoch–Schönlein purpura is a disease of the skin and HAZEL KAREN MIDTERM 2 -
seen with a palpable rash on the legs and buttocks other organs that most commonly affects children. RAZ, MD (TOP 6 - AUG 2013
associated with abdominal pain and diarrhea. On With kidney involvement, there may be a loss of small FEB 2013 MED
PE, spleen and liver were enlarged. Urinalysis amounts of blood and protein in the urine, but this BOARDS;
showed microscopic hematuria. CBC showed usually goes unnoticed; in a small proportion of cases, TOPNOTCH MD)
thrombocytopenia. what is the cause of this the kidney involvement proceeds to chronic kidney
patient's hematuria? disease. HSP is often preceded by an infection, such
A. Lupus nephritis as pharyngitis.
B. Membranoproliferative glomerulonephritis
C. Post - streptococcal glomerulonephritis
D. Henoch Schonlein Purpura nephritis
E. none
675 Type of leukemia most responsive to treatment? Acute lymphoblastic leukemia (ALL) is a form HAZEL KAREN MIDTERM 2 -
A. ALL of leukemia, or cancer of the white blood RAZ, MD (TOP 6 - AUG 2013
B. AML cells characterized by excess lymphoblasts. The FEB 2013 MED
C. CLL earlier acute lymphocytic leukemia is detected, the BOARDS;
D. CML more effective the treatment. TOPNOTCH MD)
E. All of the above

676 Treatment of pediatric patient with primary TB? Primary TB is treated with 2 months of INH, HAZEL KAREN MIDTERM 2 -
A. 2 HRZE/ 4HR rifampicin and pyrazinamide plus additional 6 months RAZ, MD (TOP 6 - AUG 2013
B. 2 HRZ/4HR of INH + rifampicin only. 9 months INH is for FEB 2013 MED
C. 9 INH treatment of latent TB. BOARDS;
D. 2HRZES/1HRZ/5HR TOPNOTCH MD)
E. None of the above

677 A 9 month old infant is brought to the ER because of Rubella - german measles, rubeola - measles, HAZEL KAREN MIDTERM 2 -
development of rash at the trunk area. History erythema infectiosum - 5th disease, varicella - chicken RAZ, MD (TOP 6 - AUG 2013
showed a 3-day fever of about 39 degrees. The pox FEB 2013 MED
rashes appeared after the resolution of fever. BOARDS;
Diagnosis? TOPNOTCH MD)
A. Rubella
B. Rubeola
C. Roseola
D. Erythema infectiosum
E. Varicella
678 Infection with EBV is associated with what type of EBV is associated with particular forms of cancer, such HAZEL KAREN MIDTERM 2 -
malignancy? as Hodgkin's lymphoma, Burkitt's RAZ, MD (TOP 6 - AUG 2013
A. Hepatocellular carcinoma lymphoma, nasopharyngeal carcinoma, and conditions FEB 2013 MED
B. Osteosarcoma associated with human immunodeficiency virus (HIV) BOARDS;
C. Squamous cell CA of the esophagus such as hairy leukoplakia and central nervous TOPNOTCH MD)
D. Burkitt's lymphoma system lymphomas.
E. Non - hodgkin's lymphoma

679 Contraindication for gastric lavage? Lavage is contraindicated when patients have a HAZEL KAREN MIDTERM 2 -
A. coma compromised, unprotected airway and in patients at RAZ, MD (TOP 6 - AUG 2013
B. seizures risk of gastrointestinal hemorrhage or perforation. FEB 2013 MED
C. Depressed gag reflex Relative contraindications include when the poisoning BOARDS;
D. Changes in sensorium is due to a corrosive substance , hydrocarbons , or for TOPNOTCH MD)
E. AOTA poisons that have an effective antidote.

680 Best clinical indicator for measure or under or BMI has been used by the WHO as the standard for HAZEL KAREN MIDTERM 2 -
overweight infants recording obesity statistics since the early 1980s. RAZ, MD (TOP 6 - AUG 2013
A. Height FEB 2013 MED
B. Weight BOARDS;
C. BMI TOPNOTCH MD)
D. Bone age
E. Weight for height

681 A 5 yo boy came to ED with complaints of Answer: C. Henoch-Schönlein Purpura (pp. 198, MICHELLE JAY MIDTERM 1 -
intermittent abdominal pain associated with loose Pediatrics: USMLE Step 2 CK, 2008-2009 edition). FRANCISCO, MD AUG 2013
stools. He has recently recovered from a viral upper SIMILAR TO PREVIOUS BOARD EXAM (TOP 9 - FEB
respiratory infection. Physical examination findings CONCEPT/PRINCIPLE 2013 MED
revealed maculopapular lesions on the legs and BOARDS;
buttocks. CBC, coagulation studies, and electrolytes TOPNOTCH MD)
are normal. Urinalysis revealed RBC = 6-8/hpf,
WBC = 2-4/hpf, (+) occasional cylindruria, (+)
albuminuria. Which of the following is the most
likely diagnosis to the case presented?
A. Kawasaki Disease
B. Guillain-Barré Syndrome
C. Henoch-Schönlein Purpura
D. Waardenburg Syndrome

TOPNOTCH MEDICAL BOARD PREP PEDIATRICS SUPEREXAM Page 92 of 97


For inquiries visit www.topnotchboardprep.com.ph or email us at topnotchmedicalboardprep@gmail.com
TOPNOTCH MEDICAL BOARD PREP PEDIATRICS SUPEREXAM
For inquiries visit www.topnotchboardprep.com.ph or email us at topnotchmedicalboardprep@gmail.com
Item QUESTION EXPLANATION AUTHOR TOPNOTCH
# EXAM
682 A 14 yo girl who has not achieved menarche Answer: C. SMR III (pp. 285 and 286, Pediatrics: MICHELLE JAY MIDTERM 1 -
presents to your clinic with her concerned mother. USMLE Step 2 CK, 2008-2009 edition). FRANCISCO, MD AUG 2013
The mother is afraid that her daughter is not (TOP 9 - FEB
“normal.” On physical examination, the patient 2013 MED
appears well-nourished and is in the 50th BOARDS;
percentile for height and weight. Her breast TOPNOTCH MD)
examination shows the areolar diameter to be
enlarged, but there is no separation of contours.
Her pubic hair is increased in amount and is curled
but is not course in texture. In order for you to
classify the time, course and progress of the
patient’s changes during puberty, you will
categorize her to what Sexual Maturity Rating
(SMR)?
A. SMR I
B. SMR II
C. SMR III
D. SMR IV
683 A child with unexplained bone pain, easy Answer: A. Gaucher Disease MICHELLE JAY MIDTERM 1 -
bruisability, and hepatomegaly was suspected to Notes: FRANCISCO, MD AUG 2013
have lipid storage disease. Enzyme assay of • Gaucher – B-glucosidase defect (TOP 9 - FEB
cultured fibroblasts showed defects in B- • Niemann-Pick – Acid sphingomyelinase defect 2013 MED
glucosidase activity. Which of the following • Tay-Sachs – B-hexosaminidase defect BOARDS;
diseases does he have? • Sandhoff – B-hexosaminidase defect TOPNOTCH MD)
A. Gaucher Disease
B. Niemann-Pick Disease
C. Tay-Sachs Disease
D. Sandhoff disease
684 A mother is concerned about her daughter’s weight. Answer: C. Assure her that weight for age is normal MICHELLE JAY MIDTERM 1 -
The 9 yo girl weighs 26 kg. Nutritional history and continue with her diet FRANCISCO, MD AUG 2013
reveals a well-balanced diet. What advice will you Notes: (TOP 9 - FEB
give? • Weight is the best index of growth and nutrition 2013 MED
A. Enroll her daughter on a weight loss program • Mnemonics: BOARDS;
B. Prescribe multivitamins and mineral o Infants <6 mos: Wt (gm) = age (mos) x 600 + BW TOPNOTCH MD)
supplements o 6-12 mos: Wt (gm) = age (mos) x 500 + BW
C. Assure her that weight for age is normal and o >2 yo: Wt (kg) = age (years) x 2 + 8
continue with her diet
D. Tell her to wait for her menarche and then you
will reassess
685 A child was brought to the Emergency Room due to Answer: A. Volvulus MICHELLE JAY MIDTERM 1 -
severe abdominal pain, nausea with gradual Notes: The radiographic finding of volvulus has been FRANCISCO, MD AUG 2013
distention. The PGI had a difficulty in inserting the described as the kidney-bean shape, coffee-bean (TOP 9 - FEB
Nasogastric tubing. Few hours after, patient ha d shape, bent inner tube shape, ace of spades or ‘Omega- 2013 MED
several episodes of vomiting. A plain abdominal x- loop sign’ BOARDS;
ray demonstrates a huge air-filled distended bowel TOPNOTCH MD)
like the shape of an inverted U, with the convexity
of the U facing the right upper abdominal quadrant.
What is the most likely diagnosis?
A. Volvulus
B. Intussusceptions
C. Pyloric Stenosis
D. Meckel’s Diverticulum
686 A toddler was brought to your clinic for multiple Answer: B. Continue feeding and follow-up in 5 days MICHELLE JAY MIDTERM 1 -
bouts of watery, non-bloody diarrhea without Notes: The patient has AGE/AID with SOME FRANCISCO, MD AUG 2013
vomiting. He is tachycardic, with weak pulses, dehydration. The appropriate management is choice (TOP 9 - FEB
reduced skin turgor and sunken eyes. He is B. Cotrimoxazole may be given if with blood in stool. 2013 MED
lethargic but irritable to touch. He gulps vigorously BOARDS;
the oral rehydrating solution offered to him. Upon TOPNOTCH MD)
your assessment based on IMCI, which of the
following management plans is the most
appropriate in this patient?
A. Give fluid and food and follow-up in 5 days if not
improving
B. Continue feeding and follow-up in 5 days
C. Refer urgently to a hospital
D. Treat for 5 days Cotrimoxazole and follow-up
after 2 days
687 Upon physical examination of a newborn, an Answer: D. A transient and harmless condition MICHELLE JAY MIDTERM 1 -
extraordinary division of the body from the Notes: FRANCISCO, MD AUG 2013
forehead to the pubis into red and pale halves was • Mottling - general circulatory instability, related to (TOP 9 - FEB
noticed. This is most likely transient fluctuation in skin temperature 2013 MED
A. Caused by general circulatory instability • Pallor – due to anemia, asphyxia, shock, or edema BOARDS;
B. Related to a transient fluctuation in skin • Harlequin Color Change – a transient and harmless TOPNOTCH MD)
temperature condition (extraordinary division of the body from the
C. Due to anemia, asphyxia, shock or edema forehead to the pubis into red and pale halves)
D. A transient and harmless condition
688 A newborn was brought to the ER for seizures. She Answer: A. Penicillin MICHELLE JAY MIDTERM 1 -
was born at home and a sharpened bamboo stick Notes: Neonatal tetanus is an acute spastic paralystic FRANCISCO, MD AUG 2013
was used to cut the umbilical cord. The mother illness caused by the neurotoxin (Tetanospasmin) (TOP 9 - FEB
never had prenatal check-up and her immunization produced by Clostridium tetani. In infants causes 2013 MED
status was unknown. Your diagnosis is Neonatal progressive difficulty in feeding, associated hunger BOARDS;
Tetanus. The drug of choice is most likely and crying. In older children, causes trismus, sardonic TOPNOTCH MD)
A. Penicillin smile. Treatment includes Human Tetanus Ig, PCN G
B. Metronidazole (DOC), surgery.
C. Ceftriaxone
D. Chloramphenicol
TOPNOTCH MEDICAL BOARD PREP PEDIATRICS SUPEREXAM Page 93 of 97
For inquiries visit www.topnotchboardprep.com.ph or email us at topnotchmedicalboardprep@gmail.com
TOPNOTCH MEDICAL BOARD PREP PEDIATRICS SUPEREXAM
For inquiries visit www.topnotchboardprep.com.ph or email us at topnotchmedicalboardprep@gmail.com
Item QUESTION EXPLANATION AUTHOR TOPNOTCH
# EXAM
689 Dr. B. Aguhan, a first year pediatric resident is Answer: A. Apt test on the blood in the stool. MICHELLE JAY MIDTERM 1 -
called to the newborn nursery to evaluate a 2-day- Notes: The most common cause of blood in the stool of FRANCISCO, MD AUG 2013
old who has passed a bloody stool. The patient was a newborn is swallowed maternal blood. The Apt test (TOP 9 - FEB
born by NSVD to a 34 yo primigravida. The birth helps distinguish adult haemoglobin from fetal 2013 MED
weight was 3.7 kg, and Apgar Score of 8,9. The haemoglobin. It is easier and less expensive than BOARDS;
mother had good prenatal care throughout the electrophoresis. If adult Hb is detected, the blood TOPNOTCH MD)
entire pregnancy. She denied using tobacco, swallowed is maternal blood. If fetal Hb is found, a
alcohol, or drugs during her pregnancy. The patient search begins for the cause of the bleeding.
has been sleeping, feeding, defecating, and
urinating well since birth. Vital signs are stable, and
the rest of the PE findings are within normal limits.
Which of the following tests would best determine
the cause of the bloody stool?
A. Apt test on the blood in the stool
B. Hemoglobin electrophoresis of the blood from
the stool
C. Barium enema
D. Methylene blue stain in the stool
690 Five days ago, a 10 yo child had an upper Answer: D. Reye Syndrome MICHELLE JAY MIDTERM 1 -
respiratory infection and was given symptomatic Notes: Reye Syndrome is a secondary mitochondrial FRANCISCO, MD AUG 2013
treatment and an antipyretic (i.e., Aspirin for fever). hepatopathy that, in genetically susceptible (TOP 9 - FEB
She had seemingly recovered but now presents individuals, has a very high association with the 2013 MED
with fever, protracted vomiting, and lethargy. ingestion of aspirin-containing medicines during BOARDS;
Physical examination reveals mild hepatomegaly. influenza-like illness or varicella. It has a biphasic TOPNOTCH MD)
Serum analysis found total bilirubin, serum course. After the child seems to have recovered, the
transaminases, and serum ammonia to be symptoms of vomiting, lethargy, and confusion
increased. CSF opening pressure was 210 mmHg. appears and progress quickly. Liver enzymes and the
CSF cell count & differential count and remaining serum ammonia level are elevated. Treatment
parameters were normal. Which of the following is requires early recognition and control of increased
the most likely diagnosis? intracranial pressure.
A. Hepatits A Virus (HAV)
B. Drug-induced Hepatitis
C. Gilbert Syndrome
D. Reye Syndrome
691 During examination of a 2-month old infant, you Answer: B MICHELLE JAY MIDTERM 1 -
note that the infant’s umbilical cord is still firmly Delayed separation (after 1 month) characteristic of FRANCISCO, MD AUG 2013
attached. This delayed cord separation is suggestive LAD type 1; px will have overwhelming bact.infection. (TOP 9 - FEB
of which of the following? Omphalocele- abdominal wall defect w/ protrusion of 2013 MED
A. Occult omphalocele intestine BOARDS;
B. Leukocyte Adhesion deficiency Umbilical granuloma forms after cord is separated. TOPNOTCH MD)
C. IgG subclass deficiency Persistent urachus- urine coming out of umbilicus.
D. Umbilical granuloma
E. Persistent urachus
692 The age that a patient most likely has when she can MICHELLE JAY MIDTERM 1 -
dress with help, ride a tricycle, knows her own age FRANCISCO, MD AUG 2013
and can speak in short sentences is (TOP 9 - FEB
A. 1 year 2013 MED
B. 2 years BOARDS;
C. 3 years TOPNOTCH MD)
D. 4 years

693 A 2-yr.old boy is brought to the E.R. w/ complaint Answer: C MICHELLE JAY MIDTERM 1 -
of fever x 6 days and development of a limp. P/E Other S/Sx can be asso.w/ many FRANCISCO, MD AUG 2013
reveals an erythematous exanthem over his body, conditions.Conjunctivitis is suggestive of (TOP 9 - FEB
ocular conjunctivitis, dry, cracked lips, red throat Kawasaki.Fissured lips could occur due to 2013 MED
and cervical lymphadenopathy, (+) grade 2/6 dehydration from fever. Nuetrophilic predominance BOARDS;
systolic ejection murmur at LLSB. WBC Count & and inc.ESR is common to all but inc.plt within the TOPNOTCH MD)
differential show predominance of neutrophils w/ constellation of these symptoms is seen only in
increased platelets on smear. Your impression is Kawasaki.
A. Scarlet fever
B. Rheumatic fever
C. Kawasaki disease
D. Juvenile rheumatoid arthritis
E. Infectious mononucleosis
694 A 16-year old male patient, heartbroken from a Answer: D MICHELLE JAY MIDTERM 1 -
recent breakup tried to commit suicide by taking an Compounds not adsorbed: alcohols, acids, ferrous FRANCISCO, MD AUG 2013
unknown quantity of unknown material he found at sulphate, strong bases, cyanide, lithium & potassium. (TOP 9 - FEB
home. Patient was then brought to the E.R within Activated charcoal dec.half-life & inc.clearance of 2013 MED
30 mins. from ingestion. For which of the following drugs w/ enterohepatic circulation. BOARDS;
household substances/medications should TOPNOTCH MD)
activated charcoal be given as part of emergency
center treatment?
A. Liquid sosa
B. Ethylene glycol
C. Zonrox
D. Phenobarbital
E. Lithium

TOPNOTCH MEDICAL BOARD PREP PEDIATRICS SUPEREXAM Page 94 of 97


For inquiries visit www.topnotchboardprep.com.ph or email us at topnotchmedicalboardprep@gmail.com
TOPNOTCH MEDICAL BOARD PREP PEDIATRICS SUPEREXAM
For inquiries visit www.topnotchboardprep.com.ph or email us at topnotchmedicalboardprep@gmail.com
Item QUESTION EXPLANATION AUTHOR TOPNOTCH
# EXAM
695 A 3-day old infant with a single 2nd heart sound has Answer: B MICHELLE JAY MIDTERM 1 -
had progressively deepening cyanosis since birth Transposition of the great vessels w/ an intact FRANCISCO, MD AUG 2013
but no respiratory distress. Chest radiography ventricular septum presents w/ early cyanosis, a (TOP 9 - FEB
demonstrates no cardiomegaly and normal normal-sized heart (classic egg on string in xray). 2013 MED
pulmonary vasculature. ECG shows an axis of 1200 Normal or slightly inc.pulmo.vascular markings and r BOARDS;
and RV prominence. Which of the following is most axis deviation w/ RVH in ECG. TOPNOTCH MD)
likely responsible for the cyanosis?
A. Tetralogy of Fallot
B. Transposition of the great vessels
C. Tricuspid atresia
D. TAPVR
696 A 19-yr old primi develops toxemia in her last Answer: A MICHELLE JAY MIDTERM 1 -
trimester of pregnancy and during the course of her An infant of 2100 gms is considered SGA. Pregnancy- FRANCISCO, MD AUG 2013
labor ,is treated with MgSO4. At 38 weeks AOG, she induced hypertension can produce a dec.in (TOP 9 - FEB
delivers a 2100gms infant with APGAR score of 1 at uteroplacental blood flow and areas of placental 2013 MED
1 min. And 5 at 5 mins. At 18 hrs of age, hematocrit infarction. This can result in fetal nutritional BOARDS;
was 79%, plt.ct. 100,000/uL, glucose 38 mg/dl, Mg deprivation and intermittent fetal hypoxemia, w/ a TOPNOTCH MD)
2.5 mEq/L, Ca 8.7 mg/dl. Soon after, infant has a dec.in glycogen storagemand relative erythrocytosis.
generalized convulsion. The most likely cause is
A. Polycythemia
B. Hypoglycaemia
C. Hypocalcemia
D. Hypermagnesemia
E. Thrombocytopenia
697 A 1-yr old boy presents w/ complaint from his Answer: D MICHELLE JAY MIDTERM 1 -
parents of “not developing normally”. He was a Hurler syndrome other features: umbilical hernia, FRANCISCO, MD AUG 2013
product of uneventful pregnancy and delivery, and kyphoscoliosis, deafness, cloudy corneas and claw (TOP 9 - FEB
reportedly normal at birth. His previous health care hand deformity. 2013 MED
provider noted developmental delay, and child BOARDS;
seemed to have enlarged spleen & liver. On P/E you TOPNOTCH MD)
notice that child has short stature, macrocephaly,
hirsutism and coarse facies w/ decreased joint
mobility which you suspect to be
A. Beckwith-Wiedemann syndrome
B. Crouzon syndrome
C. Edwards syndrome
D. Hurler syndrome
698 A 15-yr old girl is seen in your clinic with a Answer: D MICHELLE JAY MIDTERM 1 -
sprained ankle, which occurred the previous day The patient could have any number of FRANCISCO, MD AUG 2013
while she was exercising in her room. You realize problems.However, s close look at her weight in (TOP 9 - FEB
that you have not seen her for quite some time and comparison to previous ones is in order: she has all of 2013 MED
begin to expand your P/E beyond her ankle. You the physical exam findings seen in bulimia. BOARDS;
find relatively minimal swelling on her reddened, TOPNOTCH MD)
irritated uvula. She seems to be somewhat hirsute
on her arms and legs but has thinning of hair on her
head, resting HR of 60 bpm, and her oral temp.is
96%F. Further questioning suggests that she has
developed secondary amenorrhea. Your next step
in management would be
A. HIV testing
B. Radiograph of ankle
C. Thyroid function panel
D. Comparison of current and past weights
E. Pregnancy testing
699 Two weeks after a viral syndrome, a 2-yr old child Answer: C MICHELLE JAY MIDTERM 1 -
develops bruising and generalized petechiae, more ITP is the most common form of thrombocytopenic FRANCISCO, MD AUG 2013
prominent over the legs. No hepatomegaly or LN purpura. In most cases, it is preceeded by a viral (TOP 9 - FEB
enlargement is noted. P/E is unremarkable. Labs infection. No diagnostic test solely identifies this 2013 MED
show normal Hgb, Hct, WBC Ct.and differential. disease; exclusion is necessary. BOARDS;
Platelet count is 15,000/uL. Your impression is TOPNOTCH MD)
A. Von Willebrand disease
B. Acute Leukemia
C. Idiopathic thrombocytopenic purpura
D. Aplastic anemia
E. Thrombotic thrombocytopenic purpura
700 A 17-yr old is brought to the ER by the parents with Answer: B MICHELLE JAY MIDTERM 1 -
complaint of coughing up blood. He is stabilized Goodpasture disease has both lung & renal FRANCISCO, MD AUG 2013
and his haemoglobin and hematocrit levels are in involvement. Kidney biopsy and finding (TOP 9 - FEB
the safe range. During confinement, patient autoantibodies to glomerular BM. 2013 MED
developed hematuria and proteinuria. You suspect BOARDS;
patient to have TOPNOTCH MD)
A. Hemolytic uremic syndrome
B. Goodpasture syndrome
C. Nephrotic syndrome
D. Post-streptococcal glomerulonephritis
E. Renal vein thrombosis

TOPNOTCH MEDICAL BOARD PREP PEDIATRICS SUPEREXAM Page 95 of 97


For inquiries visit www.topnotchboardprep.com.ph or email us at topnotchmedicalboardprep@gmail.com
TOPNOTCH MEDICAL BOARD PREP PEDIATRICS SUPEREXAM
For inquiries visit www.topnotchboardprep.com.ph or email us at topnotchmedicalboardprep@gmail.com

Item # ANSWER 88 C 176 C 264 D 352 C


1 B 89 B 177 B 265 E 353 C
2 C 90 D 178 D 266 C 354 D
3 C 91 B 179 B 267 C 355 C
4 B 92 C 180 A 268 A 356 B
5 E 93 D 181 C 269 B 357 B
6 C 94 C 182 A 270 B 358 D
7 B 95 D 183 C 271 C 359 C
8 C 96 A 184 A 272 D 360 D
9 D 97 C 185 D 273 A 361 E
10 B 98 D 186 C 274 E 362 C
11 E 99 E 187 D 275 C 363 A
12 C 100 B 188 A 276 E 364 A
13 A 101 C 189 C 277 A 365 D
14 C 102 B 190 D 278 D 366 B
15 D 103 B 191 C 279 D 367 A
16 C 104 D 192 D 280 B 368 D
17 A 105 E 193 E 281 A 369 C
18 D 106 E 194 B 282 C 370 A
19 A 107 D 195 C 283 E 371 B
20 B 108 C 196 D 284 C 372 B
21 C 109 A 197 A 285 B 373 B
22 D 110 C 198 C 286 D 374 A
23 E 111 C 199 D 287 A 375 A
24 C 112 A 200 E 288 E 376 D
25 A 113 D 201 C 289 B 377 B
26 E 114 B 202 C 290 C 378 B
27 B 115 C 203 A 291 B 379 C
28 C 116 C 204 D 292 C 380 C
29 C 117 A 205 E 293 E 381 D
30 D 118 D 206 D 294 C 382 C
31 C 119 A 207 B 295 D 383 B
32 E 120 E 208 A 296 A 384 D
33 B 121 C 209 B 297 B 385 B
34 C 122 A 210 B 298 E 386 E
35 A 123 C 211 B 299 D 387 A
36 A 124 D 212 C 300 A 388 B
37 A 125 E 213 E 301 A 389 A
38 D 126 A 214 D 302 C 390 E
39 D 127 B 215 D 303 B 391 D
40 A 128 A 216 A 304 A 392 A
41 A 129 D 217 C 305 A 393 B
42 D 130 B 218 C 306 E 394 E
43 E 131 C 219 B 307 C 395 C
44 A 132 D 220 D 308 C 396 B
45 E 133 D 221 B 309 C 397 C
46 B 134 C 222 B 310 C 398 A
47 A 135 A 223 C 311 B 399 B
48 D 136 B 224 A 312 A 400 D
49 C 137 A 225 D 313 B 401 C
50 C 138 E 226 D 314 A 402 C
51 D 139 D 227 A 315 C 403 D
52 C 140 A 228 D 316 E 404 C
53 B 141 D 229 C 317 B 405 D
54 B 142 B 230 D 318 A 406 A
55 A 143 A 231 C 319 B 407 B
56 C 144 E 232 A 320 D 408 C
57 C 145 C 233 B 321 C 409 B
58 B 146 E 234 D 322 A 410 D
59 C 147 B 235 B 323 D 411 A
60 B 148 B 236 A 324 E 412 E
61 A 149 D 237 A 325 B 413 E
62 E 150 A 238 B 326 B 414 B
63 D 151 C 239 A 327 D 415 D
64 C 152 B 240 A 328 A 416 D
65 B 153 D 241 B 329 E 417 D
66 C 154 E 242 C 330 D 418 D
67 E 155 B 243 D 331 C 419 E
68 E 156 D 244 A 332 B 420 A
69 C 157 E 245 A 333 A 421 D
70 D 158 C 246 E 334 B 422 A
71 D 159 D 247 B 335 C 423 B
72 E 160 E 248 A 336 D 424 B
73 C 161 E 249 E 337 D 425 D
74 B 162 B 250 A 338 A 426 D
75 E 163 C 251 B 339 C 427 A
76 D 164 E 252 D 340 D 428 C
77 C 165 E 253 C 341 C 429 D
78 C 166 B 254 B 342 C 430 C
79 C 167 E 255 A 343 A 431 D
80 C 168 B 256 B 344 D 432 C
81 B 169 D 257 B 345 A 433 B
82 A 170 E 258 B 346 C 434 C
83 C 171 B 259 C 347 B 435 A
84 D 172 D 260 A 348 C 436 E
85 B 173 A, D, E 261 C 349 B 437 C
86 A 174 E 262 D 350 D 438 A
87 D 175 B 263 A 351 D 439 B
TOPNOTCH MEDICAL BOARD PREP PEDIATRICS SUPEREXAM Page 96 of 97
For inquiries visit www.topnotchboardprep.com.ph or email us at topnotchmedicalboardprep@gmail.com
TOPNOTCH MEDICAL BOARD PREP PEDIATRICS SUPEREXAM
For inquiries visit www.topnotchboardprep.com.ph or email us at topnotchmedicalboardprep@gmail.com
440 B 529 D 618 D
441 D 530 A 619 A
442 D 531 B 620 C
443 B 532 C 621 B
444 C 533 A 622 B
445 D 534 C 623 B
446 B 535 A 624 B
447 C 536 D 625 E
448 B 537 D 626 C
449 B 538 B 627 D
450 D 539 C 628 B
451 B 540 A 629 E
452 A 541 E 630 C
453 B 542 C 631 A
454 B 543 B 632 C
455 C 544 A 633 C
456 A 545 B 634 E
457 B 546 E 635 D
458 C 547 E 636 B
459 C 548 C 637 B
460 B 549 A 638 C
461 A 550 B 639 D
462 C 551 D 640 B
463 B 552 E 641 A
464 A 553 C 642 E
465 E 554 D 643 C
466 A 555 C 644 B
467 D 556 A 645 B
468 C 557 B 646 A
469 C 558 E 647 C
470 C 559 D 648 C
471 A 560 D 649 B
472 B 561 A 650 E
473 B 562 A 651 A
474 A 563 C 652 A
475 E 564 B 653 D
476 A 565 A 654 E
477 A 566 C 655 B
478 C 567 A 656 C
479 B 568 B 657 B
480 B 569 B 658 A
481 D 570 A 659 D
482 A 571 B 660 D
483 D 572 A 661 C
484 B 573 C 662 D
485 D 574 A 663 E
486 A 575 D 664 A
487 C 576 C 665 D
488 C 577 A 666 C
489 D 578 C 667 A
490 B 579 D 668 C
491 C 580 B 669 E
492 A 581 A 670 D
493 B 582 B 671 B
494 D 583 B 672 A
495 D 584 D 673 C
496 A 585 C 674 D
497 A 586 C 675 A
498 B 587 D 676 B
499 B 588 B 677 C
500 A 589 B 678 D
501 D 590 E 679 E
502 B 591 D 680 C
503 E 592 C 681 C
504 A 593 A 682 C
505 C 594 D 683 A
506 D 595 C 684 C
507 A 596 E 685 A
508 C 597 B 686 B
509 C 598 A 687 D
510 B 599 D 688 A
511 E 600 A 689 A
512 A 601 B 690 D
513 A 602 D 691 B
514 C 603 A 692 C
515 A 604 E 693 C
516 A 605 C 694 D
517 606 D 695 B
518 C 607 A 696 A
519 D 608 C 697 D
520 D 609 D 698 D
521 A 610 A 699 C
522 D 611 C 700 B
523 A 612 D
524 B 613 B
525 C 614 C
526 A 615 A
527 C 616 C
528 A 617 D
TOPNOTCH MEDICAL BOARD PREP PEDIATRICS SUPEREXAM Page 97 of 97
For inquiries visit www.topnotchboardprep.com.ph or email us at topnotchmedicalboardprep@gmail.com

You might also like